153
Scripts & Answer Key Insider: The Super Guide

Insider: The Super Guide Scripts & Answer Key

  • Upload
    others

  • View
    9

  • Download
    0

Embed Size (px)

Citation preview

Page 1: Insider: The Super Guide Scripts & Answer Key

Scripts & Answer Key

I n s i d e r : T h e S u p e r G u i d e

Page 2: Insider: The Super Guide Scripts & Answer Key

P A R T ABasic ComprehensionQuestions

Referent Questions

Basic Drills _ p.25

1. Both the pronoun “them” and its antecedent “particles”

function as objects in their sentences.

2. The previous sentence discusses “isolating California.”

This concept matches up logically with the idea of a divide.

Vocabulary Questions

Basic Drills _ p.27

1. One context clue is the phrase “no matter what other

precepts they adhere to.” This indicates that a “precept”

and a “tenet” are the same thing. Therefore, if you know

the word “precept,” you can infer the meaning of “tenet.”

2. The statement “the race was on to build” a perpetual

motion machine provides a strong context clue indicating

that “endeavor” must be similar in meaning to “attempt.”

3. The diplomat is described as “oafish and uncultured.”

Since these two adjectives are linked by the conjunction

“and,” they must have relatively similar meanings.

Fact Questions

Basic Drills _ p.29

1. The key words “discovery” and “forgotten” help the

reader find the answer to the question.

2.

3.

Negative Fact Questions

Basic Drills _ p.31~32

1.

2.Most landforms that are now seen owe their

existence to glacial deposition. These formations

occurred as glaciers moved south and pushed walls

of rocks, soil, and other rubble ahead of them. At

some point, however, these glaciers could no longer

move south as they reached a point of stability,

melting as fast as they were growing. They would

then begin to retreat, and, in doing so, they would

In 1856, the United States Supreme Court ruled that

the African-American slave Dred Scott could not gain

freedom in a state that prohibited slavery, and this

case had many damaging implications for slaves,

African Americans, and the country in general. First, it

stated that any person of African descent could never

be a citizen, whether he was free or not. Further, it

claimed that since slaves were property, they could

not be taken from their owners, even if these owners

lived in a free state. The decision, in principle

anyway, allowed for slavery to be legal in any state in

the US. In effect, it further divided the free states of

the North from the slave states of the South, and it

moved both sides closer to the American Civil War.

Research indicates that the human brain is

constantly being tricked by commercial

advertisements. Advertising seems to bypass the

logical parts of the brain and instead targets less

rational and more emotional parts of the brain. It

appears that most ads concentrate on the portion of

the brain that feels pleasure. For instance, a

television commercial showing a happy family joined

together at dinner may elicit a pleasurable response

from someone who views it. This pleasure will cause

the viewer to keep watching the commercial in hopes

of maintaining the pleasant feeling. As a result, he

will begin to see a particular product featured in the

commercial, like a food item or beverage. Eventually,

the viewer unconsciously begins to associate the

featured product with the pleasurable response to the

advertisement, and he will be inclined to buy the

product because of this association.

showed the potential of penicillin as a powerful killer

of bacteria. Unfortunately, it was extremely difficult to

produce any useful quantities of the mold at the time.

Although he is not given much credit in medical

history, Ernest Duchesne, a French medical student,

was actually the person who first discovered

penicillin in 1896. Duchesne discovered that a certain

type of mold could destroy potentially harmful

bacteria. However, it was not immediately clear to

other researchers that the mold was responsible for

killing the bacteria; thus, the discovery remained

more or less forgotten. It was not until 1939 that a

team of scientists from Oxford University finally

Chapter 1

READING Section

A2 Insider: The Super Guide

Page 3: Insider: The Super Guide Scripts & Answer Key

3.

Sentence Simplification Questions

Basic Drills _ p.34

1. Kinesiologists, or coaches with special knowledge of

kinesiology, can predict the complications that may arise

from altering normal movements and thus avoid muscle

strains, torn rotator cuffs, ligament damage, and

herniated tissue.

2. Answer choice (A) states that kinesiologists can solve

problems caused by injuries. The original sentences says

that they can avoid these problems.

Answer choice (C) says that coaches should consult

with a kinesiologist. This idea is never discussed in the

original sentence. Answer choice (D) does not discuss

how kinesieologists can help an athlete avoid injuries.

Question Practice 1 _ p.35~37

1. (D) – Read the sentence carefully in order to gather a

contextual clue for the vocabulary word’s meaning. The

sentence states that animal therapy “has not normally

been considered worthy of true scientific investigation.”

This suggests that animal therapy is not a part of

acceptable science.

2. (C) – By scanning the paragraph for the key words

“maximum relaxation” you should be able to find that the

correct answer choice is directly stated in the paragraph.

3. (D) – The sentence states that epinephrine was released.

Since you know that epinephrine is a “hormone secreted

during stress,” you can infer that “released” and “secreted”

have similar meanings.

4. (A) – The idea of “witnessing significant differences in

allowing patients to interact with dogs and people” from

the original sentence is summarized as “Their

observations” in the correct answer choice. The idea that

researchers “encouraged doctors to consider this type of

therapy” is paraphrased as “recommend animal therapy

and traditional therapy for recovering patients” in the

correct answer choice.

5. (D) – Follow the progression of ideas within the passage.

The pro-form is used at the very end of the paragraph, just

after a discussion of how dogs can detect when a person

is about to have a seizure. Thus, the pro-form “these

episodes” most logically refers to seizures.

6. (B) – The best way to answer this question is to eliminate

any answer that contradicts the passage. The passage

clearly states that dogs can be used to warn their owners

that they are about to have a seizure. It does not state that

the dogs can prevent seizures.

7. (A) – Read both the question and the answer choices

carefully for this question. The incorrect answer choices

The extinction of megafauna, or large ancient

mammals, occurred about the same time as the

appearance of man, so it has been theorized that

humans were responsible for this extinction. Some

scientists suggest that humans may have over-

hunted these animals and driven them to extinction.

There is no doubt that these animals were a valuable

source of food for early man; there is ample

archaeological evidence for human consumption of

them. For example, bones have been found with

marks indicating that stone tools were used to

remove meat from them. Those in support of the

human extinction theory also point to evidence

indicating that humans caused the extinction of the

giant lemur in Madagascar and the moa in New

Zealand. They claim that similar extinctions could

have been perpetrated by humans in prehistoric

times.

Some scientists point to climate change as the most

likely factor in the disappearance of these animals.

During the Ice Ages, Siberia had more of a steppe-

like climate with expansive grasslands, perfect for

supporting large, grass-eating mammals like the

mammoth. Unfortunately, as the climate changed

after the Ice Age, the grasslands were replaced by

swampy tundra, causing many populations of

megafauna to starve to death. Though humans may

have contributed to some degree to the extinction of

these animals, it seems more likely that they were

already on the decline due to a loss of their main

food supply.

Other scientists propose a third possible explanation.

They theorize that these mammals, especially in

North America, probably succumbed to disease

carried by some microbe, which they called a

"hyperdisease" because it was able to jump across

species boundaries. Unfortunately, though the

hypothesis may be interesting, the scientists who

support it have found no proof that such a disease

actually occurred. It seems, then, that it will still be

some time before there will be a universally accepted

explanation for the megafauna extinction.

leave behind the debris that they had shoved in front

of them. In some places, these hills of debris were up

to several hundred feet high. The long hill that

marked the southernmost extent of a glacier is called

the terminal moraine. Today, some can still be seen

running across parts of the US. Perhaps the greatest

example of one is the island of Long Island, which

lies just east of New York City.

Scripts & Answer Key A3

RE

AD

ING

Page 4: Insider: The Super Guide Scripts & Answer Key

misstate information from the paragraph. For example,

answer choice (D) states that the dogs warn others before

their owner has a seizure. The passage clearly states that

the dogs warn their owners. However, the last sentence of

the paragraph clearly states that researchers do not

understand how the dogs are able to predict seizures. In

other words, they are able to help their owners in a way

that cannot really be explained.

8. (D) – The passage clearly states that “Even severely

emotionally withdrawn children gained benefits from this

animal therapy.”

Question Practice 2 _ p.38~40

1. (D) – The sentence states that the term was “first coined.”

The word “first” provides a strong context clue to the

meaning of “coined.”

2. (D) – Read the entire paragraph and pay attention to

contextual clues. The paragraph states that “several

theories have been postulated.” The next sentence reads,

“One theory suggests that the British desired to distinguish

their pronunciation from French pronunciation.” Both

“postulated” and “suggests” apply to theories, so the two

words are most likely similar in meaning.

3. (A) – The highlighted sentence contains three key ideas:

there were multiple dialects spoken, they gradually merged

into one dialect, and this single dialect was understood by

all. Understanding the relationship between these three

ideas is the best way to select the correct answer choice.

The incorrect choices either distort or omit information

regarding the relationship between these ideas.

4. (C) – Skim the passage for the key phrase “Black Plague.”

The next sentence clearly states that “this epidemic forced

many people from different regions to migrate into big

cities.” The phrase “metropolitan areas” in the answer

choice is a paraphrase of “big cities.”

5. (A) – The sentence states that “it is pronounced as the

vowel sound in ‘eye.’” Since “it” must be pronounced, (A)

is the only logical answer.

6. (D) – The answer to this question is a detail that supports

a central idea of the paragraph. The correct answer can be

easily obtained by skimming the paragraph. The last

sentence discusses how “people in Scotland commonly

use Middle English pronunciation for many vowel-sounds.”

7. (C) – A key context clue is that the Great Vowel Shift

“inundated” English with many perplexing spelling

conventions.

8. (B) – The standardized pronunciation of English is

discussed in paragraph 2. The creation of diphthongs is

discussed in paragraph 3. The creation of the silent “e” is

discussed in paragraph 4.

Inferencing Reading &Reading to Learn Questions

Inference Questions

Basic Drills _ p.43

1.

2. The passage states that the Napa Valley has very many

different types of soils. It also states that different soils

support different types of grapes. Therefore, you can

infer that the presence of a wide variety of soils helps in

growing a wide variety of grapes.

Rhetorical Purpose Questions

Basic Drills _ p.45

1. The first sentence of the paragraph clearly indicates that

the main focus of the paragraph will be that dependency

on cotton affected farmers outside of the United States

as well.

2. The first sentence of the paragraph introduces the idea

of “developments in technology” and how these

developments were used to increase productivity in

fishing. Later in the paragraph the author states that

“there were countless numbers of fish or, at least, so it

seemed.” This suggests that the author will discuss

evidence that proved fishermen were wrong to assume

their activities were not harmful.

Sentence Insertion Questions

Basic Drills _ p.47

1. The insertion sentence states that “supplements contain

many times the recommended daily levels of vitamins

and minerals.” This matches up with the idea that “taking

too many supplements can even be harmful.”

2. The word “consequently” in the insertion sentence

indicates that slow and one-sided communications were

the result of something. The idea that telegraphs could

only send one signal at a time completes the cause and

effect relationship.

Although the use of candles can be traced back to

about 400 AD, they did not become widely used until

the 14th Century. The best candles were made of

beeswax, but these were generally confined to use in

religious rituals in churches. Most average people of

the time preferred to use inexpensive tallow candles

made from animal fat. Tallow candles may have

been cheap, but they tended to be smoky and smelly

and dripped profusely while giving very weak light.

Chapter 2

A4 Insider: The Super Guide

Page 5: Insider: The Super Guide Scripts & Answer Key

RE

AD

ING

Scripts & Answer Key A5

Prose Summary Questions

Basic Drills _ p.49~50

1. Answer choice (A) is discussed in the second paragraph.

Answer choice (C) is discussed in the third paragraph.

Answer choice (D) is discussed in the first paragraph.

2. Answer choices (B) and (F) can be eliminated because

they discuss minor details from the passage.

3. Answer choice (E) can be eliminated because it does not

follow the focus of the introductory sentence, which is

how family structures are determined, not how they are

changing.

Categorization Questions

Basic Drills _ p.52~53

Question Practice 1 _ p.54~56

1. (B) – The description of the Monroe Doctrine as “a

remarkable declaration” provides one context clue for this

question. In addition, the flow of ideas in the paragraph

centers around events that weakened European nations

and allowed the US to issue the declaration. This also

makes (B) the most logical choice.

2. (C) – The paragraph states that European nations were in

a “poor position to press their colonial rights” due to the

Napoleonic Wars. It also states that several former

colonies had already declared independence. This clearly

suggests that European nations lost control of their

colonies due to the Napoleonic Wars.

3. (A) – The paragraph states that England, which had

banned the slave trade, had little economic incentive for

continued colonization. On the other hand, the slave

colonies of Spain and France generated much income.

Therefore, it is reasonable to assume that only colonies

that utilized slavery were economically viable.

4. (B) – The preceding sentence states that the US would not

break from its policy of non-intervention for 100 years. The

mention of World War II serves to reemphasize this idea.

5. The sentence should be placed at the first square. – All of

the following sentences explain the reasons the British had

for endorsing the Monroe Doctrine.

6. (B), (E), and (F) – Answer choices (A) and (C) are relatively

minor details from the passage. Answer choice (D)

misstates information from the passage; the isolation of

the US is never discussed as a drawback.

(F) Such stress can be intensified by using percussive

instruments, such as drums, or special techniques like

hammering down on a piano chord. (B) Some musical

researchers believe that all ideas of rhythm stem from

the natural rhythms that people find around them, like

the sound of a heartbeat, the rhythm of walking, the

breaking of waves, or anything else that keeps a

constant tempo. Tempo is defined as the number of

beats per minute, and it is usually a consistent pattern of

interchanging accentuated and non-accentuated beats.

There is some disagreement about the importance of

rhythm in music. In most Western classical music,

rhythm plays a subordinate role because it does not

have the same emotional impact as the elements linked

with melody. (A) However, in the music of non-Western

cultures, rhythm tends to be the most dominant element

and is what most people of these cultures primarily

notice.Music can have a powerful emotional effect on people.

Music does this by making use of notes, or individual

sounds that act in relation to one another. The

organization of these notes has two main aspects:

melody and harmony. There is also a third aspect to

music: rhythm, although it doesn’t have to do with the

organization of particular notes. Musicians will

manipulate these elements in order to invoke an

emotional response from the listener.

The first and most important element of music is melody.

This is the organization of sounds in a specific manner.

Usually, this succession of notes is heard as some sort

of unit. Melody is usually the most significant component

of a musical piece because it acts as the most distinctive

feature that allows someone to identify that piece. (C) It is

a group or pattern of sounds that determines melody,

and this pattern will occur several times within a piece,

giving the entire musical composition some sense of

unity. (E) The use of melody is what gives most music its

emotional appeal to listeners.

Harmony, the next important aspect of music, works

within the central melody of a musical piece. Harmony is

simply the playing of two or more sounds

simultaneously. Some notes, when played together, can

produce an emotional tension within the listener, while

other combinations complement each other and relieve

this tension. Most chords, or groups of notes played

together, are formed in certain patterns to produce

different emotional effects. The continuous play between

tension and resolution gives music most of its emotional

impact.

The final main aspect of music is rhythm. When certain

patterns of stressed and unstressed beats regularly

occur in a melody, they produce a particular rhythm.

Page 6: Insider: The Super Guide Scripts & Answer Key

Question Practice 2 _ p.57~59

1. (A) – The author is making an analogy between sculptors

and film directors. The purpose of this analogy is to

illustrate how important choosing the correct type of stone

is in sculpture.

2. (B) – The main idea of the paragraph is that soft rocks are

easier to work with than hard rocks. Answer choice (B) fits

most logically with this idea.

3. (B) – This inference can be easily made by examining the

unstated connections between statements in the

paragraph. The paragraph states that metamorphic rocks

are soft and easier to carve. It also states that marble is

preferred among sculptors for this reason. Therefore,

marble must be a metamorphic rock.

4. (D) – The passage states that acid rain damages

sculptures. It also states that acid rain is becoming more

common in urban areas. Therefore, you can infer that

sculptures in city parks are easily damaged and thus

becoming less common.

5. The sentence should be placed at the fourth square. –

The phrase “extensive weathering” in the insertion

sentence is a key context clue. “Extensive weathering” is

the same thing as “erosion.” Therefore, the fourth square

is the most logical place for the sentence.

6. (A) Often soft and easily carved (Metamorphic)

(B) May require special carving tools (Igneous)

(C) Can be difficult to obtain (X)

(D) Not badly damaged by acid rain (Igneous)

(E) Sculptures tend to be somewhat lifeless (X)

(F) Can be used when detail is necessary (Metamorphic)

(G) Few color choices (Igneous)

Start by eliminating the two answer choices that are

obviously not discussed in the passage. Then try to match

the remaining answer choices with the correct type of

rock. Most of the answer choices can be matched by

remembering that the key difference between igneous and

metamorphic rocks is that igneous rocks are much harder.

Therefore, they are not easily damaged by acid rain and

require special carving tools.

P A R T BEarth Sciences

Guided Reading 1 _ p.64~65

1. (A) – The phrase “land features” is the subject of the

preceding clause in the sentence.

2. (A) – The second paragraph states that rain forms

“indentations and grooves” in immature karst formations.

“Indentations and grooves” are types of wear patterns.

3. (A) – The passage clearly states that mature karst

formations are characterized by extensive cave

complexes. It also states that the karst formations in New

Mexico and Arizona have some of the most extensive cave

complexes in the world. Therefore they must be mature

karst formations.

4. (D) – Paragraph 4 clearly states that the greater the acidity

of the rainwater, the faster karst formation takes place.

Guided Reading 2 _ p.66~67

1. (C) – The following sentences in the paragraph state that

crops and animals die as a result of nuclear winter.

Therefore, “extinction” is the only logical answer choice.

2. (A), (B), and (C) – The introductory sentence of the

summary clearly indicates that the correct answer choices

should deal with the potential causes of nuclear winter and

its effects. Answer choice (D) does not follow this focus.

Answer choices (E) and (F) contain ideas that are not

stated in the passage.

Unguided Reading _ p.68~70

1. (A) – A clear context clue comes early in the paragraph

when the author states that the team discovered “more

than they could have anticipated.” This clearly suggests

that some of the discoveries were accidental.

2. (B) – The original sentence states that “particles are

constantly bouncing back and forth,” meaning they are

propelled from one pole to another. Another key phrase

from the original sentence is that “the magnetic field

causes these particles to be deflected,” meaning they are

repelled.

3. (A) – Working backward from the pronoun clearly

indicates that “electronic components” is the correct

antecedent for “them.”

Chapter 1

A6 Insider: The Super Guide

Page 7: Insider: The Super Guide Scripts & Answer Key

4. (D) – The passage clearly states that digitization of

electronic components (which is a relatively new

technology) has made satellites more vulnerable to

radiation.

5. (C) – The passage states that the safe zone is located

between the lower and upper belts. Therefore, answer (C)

is clearly an incorrect statement.

6. (C), (D), and (E) – The introductory sentence of the

summary clearly indicates that the focus of the summary

should be the causes and makeup of the radiation belts.

Answer choices (A), (B), and (F) do not follow this focus.

Building Your Reading Skills _ p.71

Exercise 1 – Tracking Multiple Topics

| Vocabulary Review | _ p.72

1. porous 2. geomagnetic

3. fissures 4. subterranean

5. aurora 6. bedrock

7. sediment 8. solar winds

9. erosire 10. ozone

| iBT Practice | _ p.73~77

1. Vocabulary Question – (C)

Answer choices (A) and (B) are illogical in the context of

the passage and can be eliminated. Similarly, since the

effect of global warming on thermohaline circulation is only

a theory, answer choice (C) does not make sense in the

overall context of the passage.

2. Referent Question – (A)

Similar to the phrase “this oceanic motion” earlier in the

sentence, “this activity” is a pro-form used to refer to

thermohaline circulation.

3. Fact Question – (D)

This is clearly stated in the last sentence of paragraph 1.

4. Vocabulary Question – (B)

The focus of paragraph 2 is what causes thermohaline

circulation. Since “catalyst” appears in the topic sentence

of the paragraph, this provides a strong context clue to its

meaning.

5. Referent Question – (D)

Both “seawater” and the pronoun “it” are the subjects of

their respective clauses in the sentence.

6. Negative Fact Question – (A)

Answer choice (A) is contradicted by the passage.

Paragraph 2 clearly states that upwelling occurs in the

North Pacific and Indian oceans.

7. Vocabulary Question – (B)

A key context clue in the sentence is that melting ice caps

would “dilute nearby salt water.” Since “dilute” means to

make something less concentrated, it makes sense that

“diminish” would have a somewhat similar meaning.

8. Fact Question – (C)

By skimming the paragraph for the word “chimneys”, you

should quickly be able to find the statement that “These

chimneys are actually immense columns of cold water that

shoot down to the ocean floor.”

9. Fact Question – (D)

By skimming the paragraph for the phrase “gyre of activity”

you should quickly be able to establish the evidence for

this comes from “photographs recently taken from

satellites.”

10. Vocabulary Question – (A)

A key context clue is the contrast of ideas in the sentence.

Some scientists warn of a problem, while others reject it.

11. Sentence Simplification Question – (B)

While the correct answer choice omits minor details, it

retains all of the key ideas from the original sentence.

12. Inference Question – (A)

The passage discusses several serious consequences of a

halt in thermohaline circulation, such as increasingly severe

hurricanes and floods. However, these consequences are

discussed using the subjunctive mood, suggesting that

they are uncertain.

13. Sentence Insertion Question – The sentence should go

at the third square.

The insertion sentence discusses a possible consequence

of a shutdown in thermohaline circulation, but it is a

consequence that is unrelated to the weather. Therefore, it

would be illogical to place the insertion sentence at the first

or second square because all of these sentences discuss

the possible effects on the weather.

14. Prose Summary Question – (A), (C), (F)

The main focus of the introductory sentence is the role of

thermohaline circulation in redistributing heat and the

possible effects of global warming on the system. Only

answer choices (A), (C), and (F) address these concepts.

Scripts & Answer Key A7

RE

AD

ING

Inner Belt

L Mostly protonsL Caused by cosmic

rays reflected off

the atmosphere

Controlled

by earth’s

magnetic

field

L Mix of particlesL Has several

causes

Outer Belt

Page 8: Insider: The Super Guide Scripts & Answer Key

Natural Sciences

Guided Reading 1 _ p.80~81

1. (B) – Scan for the word “taurine” in the paragraph. Then

read the sections around this word. The paragraph clearly

states that taurine can only be found in meat.

2. (A) – The main focus of the paragraph is how being

carnivorous has affected the development of cats.

Therefore, (A) is the most logical answer.

3. (D) – This is directly contradicted by the passage. The

passage states that cats hunt at night because they do not

perspire.

4. The sentence should be placed at the second square. –

Key words in the insertion sentence are “enzymes” and

“sugars.” The sentence before the second square states

that cats lack the enzymes to process plant carbohydrates

and glucose. Glucose is a form of sugar.

Guided Reading 2 _ p.82~83

1. The correct answer is (B). – The paragraph states that

“Until recent decades, when the development of genetic

engineering opened up a new world of possibilities, such

manipulation was accomplished exclusively through two

methods: selective breeding and cross breeding.” The

phrase in bold clearly implies that improvements have

been made in the methods of species manipulation.

2. (A) More reliable in terms of results (Selective Breeding)

(B) Only possible with certain animals (Cross Breeding)

(C) Produces more robust offspring (Cross Breeding)

(D) Can lead to species extinction (X)

(E) Can produce undesirable weaknesses within

aspecies (Selective Breeding)

(F) Most common form of species manipulation

(Selective Breeding)

Species extinction is never discussed in the passage, so

this answer choice can easily be eliminated. The passage

states that selective breeding can cause genetic defects in

a species (i.e. produce undesirable weakness), achieve

fairly predictable results (i.e. is reliable), and is the most

common form of species manipulation. The passage

states that cross breeding produces stronger, healthier

(i.e. more robust) offspring and that only closely related

species may be cross bred.

Unguided Reading _ p.84~86

1. (B) – The first sentence states that flowering plants are the

“preeminent” form of vegetation. The next sentence states

that they are the “dominant” form. These two words must

have roughly similar meanings. Therefore (B) is the most

logical choice. ** Note: In certain situations, “preeminent”

can be similar in meaning to “prestigious,” but in the

context of the passage this would not make sense.

2. (A) – The passage clearly states that animals “proceed to

scatter the pollen among distant flowering plants and drop

the seeds in their excrement.”

3. (C) – If cross pollination is advantageous because it

increases genetic diversity, then it is only logical to assume

that self-fertilization leads to a lower level of genetic

diversity.

4. (D) – This is clearly contradicted by the passage.

Paragraph 3 states that the pollen grains of flowering

plants contain fewer cells.

5. The sentence should be placed at the fourth square. – If

it is placed in any other location, it interrupts the

explanation of the structure of flowering plants and the

advantages this structure gives.

6. (B), (C), (D) – The main focus of the introductory sentence

is the anatomical features of flowering plants. Only (B), (C),

and (D) specifically discuss such features.

Building Your Reading Skills _ p.87

Exercise 1 – Identifying the Major Topics of the Passage

Exercise 2 – Identifying Key Ideas in the Paragraph

Another advantageous feature of flowering plants is their

ability to fertilize seeds more quickly than non-flowering

plants. Flowering plants accomplish this by having a

reduced number of cells in both their male and female

pollen grains. Flowering plants can then produce sex

cells more quickly, and the fewer number of cells means

that the pollen grains will become seeds more quickly,

too. The quick production of fertilized seeds also means

that seeds can be dispersed quickly and in greater

quantities than in non-flowering plants. Because of this

relatively short time – non-flowering plants can, in

The preeminent form of vegetation on land is the family

of plant species that generates flowers. Flowering plants

have become the dominant form of land vegetation

because they possess various features in their

anatomical structures that provide them with many

reproductive advantages over non-flowering plants.

These advantages include dispersal mechanisms that

allow for more active and widespread fertilization, pollen

grains that allow them to fertilize seeds more quickly

than non-flowering plants can, and a physiological

structure that prevents self-fertilization and allows

flowering plants to achieve more genetic diversity.

Chapter 2

A8 Insider: The Super Guide

Page 9: Insider: The Super Guide Scripts & Answer Key

Scripts & Answer Key A9

RE

AD

ING

| Vocabulary Review | _ p.88

1. gene pool 2. pathogen

3. omnivores 4. vertebrates

5. microbe 6. physiological

7. sterile 8. enzymes

9. regurgitate 10. procreation

| iBT Practice | _ p.89~93

1. Vocabulary Question – (A)

There are two context clues in the sentence. First, the

word “innate” means inherent or basic. Second, the

sentence states that innate immune systems are intrinsic in

all organisms.

2. Referent Question – (D)

The sentence states that “In addition to such biological

material there are other barriers…” Thus, whatever is

referred to by “such biological material” must act as a

barrier inside the body. Only mucus (answer choice D)

does this.

3. Fact Question – (B)

The paragraph states that leukocytes, which are contained

in the blood, start biochemical reactions.

4. Vocabulary Question – (D)

The sentences states that T-cells “mediate” the responses

of the two immune systems to more effectively kill

infections. If there are responses from two immune

systems, it makes sense that these responses must be

coordinated.

5. Inference Question – (B)

The paragraph states that the adaptive immune system

learns to identify the cells from different types of infections

and that this makes it more resistant to recurring

infections. This clearly implies that the adaptive immune

system is more effective in the long term.

6. Vocabulary Question – (C)

A key context clue in the sentence is the statement that B-

cells will duplicate.

7. Referent Question – (A)

Both the pronoun and its antecedent function as objects in

the sentence. In addition, the idea that B-cells mark

antigens for attack by leukocytes matches up with the idea

that “them” and microbes are easy targets.

8. Fact Question – (B)

The paragraph clearly states that B-cells produce

antibodies.

9. Vocabulary – (A)

A key context clue is that the key idea in the sentence is

the idea of a fetus. Since a fetus must be conceived, this is

the most logical choice.

10. Fact Question – (D)

The paragraph clearly states that antibodies are gained

through the mother’s breast milk and nutrients passed

through the placenta.

11. Inference Question – (C)

This is the main focus of the paragraph. The paragraph

states that it is interesting that inception is possible among

such an aggressive immune system. It then goes on to

explain possible reasons why the fetus is ignored by the

immune system. The clear assumption is that the immune

system should be expected to attack the fetus, since it

does not share the mother’s exact DNA.

12. Sentence Insertion Question – The sentence should be

placed at the first square.

In any other location, it breaks up the natural flow of ideas

that exists between the sentences explaining the adaptive

immune system.

13. Categorization Question

(A) Antigens attach to specific cells (Adaptive)

(B) Responsible for creating pathogens (X)

(C) Works like a vaccination (Adaptive)

(D) Includes sneezing and coughing (Innate)

(E) Causes inflammations in tissue (Innate)

(F) T-cells seek and destroy cells (Adaptive)

(G) Uses cellular cooperation and biochemical reactions

(Both)

(H) Found in all organisms (Innate)

Try to match the most obvious answer choices first. The

passage clearly states that all organisms have an innate

immune system, and, if you have understood the

explanation of the adaptive immune system, it should be

clear that it works like a vaccination. You might need to

skim the passage to find the other answer choices. Use

the organization of the passage to help you. Anything

relating to the innate immune system should be discussed

in the first half of the passage, and anything relating to the

adaptive immune system should be found in the second

half of the passage.

contrast, take up to a year to fertilize the egg cells of

their species – flowering plants both set out their seeds

sooner than most other plants and produce fertilized

seeds at a quicker rate. The speed of these functions

allows flowering plants to set up annual life-cycles and

find a wider variety of habitats in which to establish

themselves, claiming most of the fertile soil and

resources before non-flowering plants can.

Page 10: Insider: The Super Guide Scripts & Answer Key

Physical Sciences

Guided Reading 1 _ p.96~97

1. (D) – The passage clearly states that “the inductor causes

the flowing electrical energy to create a magnetic field.”

2. (A), (B), (F) – The introductory sentence gives a clear

indication of what the focus of the summary should be.

Based on this, (E) can be eliminated as a possible answer

choice. Answer choices (C) and (D) can be eliminated

because they discuss relatively minor details.

Guided Reading 2 _ p.98~99

1. (C) – A key idea from the question is “the limitations of

optical telescopes.” Skim the paragraph until you find a

similar idea. The paragraph states that “there are limits to

human vision, and this in turn limits the capabilities of

optical telescopes.” At this point you should start reading

the paragraph more carefully. Answer choices (A), (B), and

(D) are clearly discussed as limitations to optical

telescopes.

2. (B) – The last sentence in paragraph 2 states that “For

telescopes to be more effective, they would have to

overcome these hindrances.” This gives a clear context

clue concerning the author’s purpose in discussing the

limitations of light telescopes.

3. (A) – The passage clearly states that “scientists need to

place special telescopes on satellites in order to detect

these most powerful frequencies.”

4. The sentence should be placed at the second square. –

The phrase “these celestial bodies” refers to stars and

nebula. This indicates that the second square is the most

logical place for the insertion sentence.

Unguided Reading _ p.100~102

1. (C) – Answer choices (A) and (B) change the meaning of

the sentence in important ways. Answer choice (D) omits

important ideas from the original sentence because it does

not discuss the difference between the traditional definition

of acids and bases and the new understanding scientists

have of these materials.

2. (B) – The context of the sentence clearly indicates that

answer choices (A) and (D) are incorrect. Furthermore,

while the sentence does discuss oxygen and hydrogen

atoms, (C) does not make logical sense if it replaces “they”

in the sentence.

3. (A) – The final sentence in paragraph 2 states that “The

lasting importance of this definition was that it showed that

acids and bases exchanged positively charged protons;

this detail would be the foundation of a more

comprehensive understanding of chemicals and reactions.”

4. (C) – The paragraph states that the Bronsted-Lowery

definition stated that “acids were chemicals that donated

protons” and that “bases were chemicals that accepted

these protons.” Since the definition indicates what acids

and bases do in a reaction, it indicates the role of each

substance.

5. (D) – A key context clue in the sentence is the phrase

“only an electron must be present.” This means a proton is

not required.

6. (A) Base receives protons in a reaction (Bronsted-

Lowery)

(B) All acids contain oxygen (X)

(C) Only electrons needed for reaction (Lewis)

(D) Chemical can be a base or an acid, depending on

reaction (Both)

(E) Developed upon the ideas of Arrhenius (Both)

(F) Sub-atomic particles can operate as acids or bases

(Lewis)

The passage clearly states that under both definitions any

substance can be an acid or a base. This allows you to

easily identify which answer choice should not be used.

The final paragraph specifically states that Lewis expanded

on the Bronsted-Lowery definition by stating that it is

electrons, not protons, that are traded during an acid-base

reaction and that even sub-atomic particles can be

considered acids or bases.

Building Your Reading Skills _ p.103

Exercise 1 – Picking Out Transitional Phrases

Speculation regarding the exact nature of acids and

bases began in the late 19th century, when chemists

were mixing various chemicals in water solutions and

observing energetic reactions. Early on, most chemists

focused on the specific nature of acids: some scientists

believed that all acids contained oxygen atoms, while

others believed that they all contained hydrogen. These

incorrect theories were seriously considered during their

time, but then, in 1903, a chemist named Svante

Arrhenius established the basis for the modern-day

definition of acids and bases. He believed that acids

were chemicals that gave off protons after they dissolved

in water and that bases always released hydroxide ions

to interact with acids. He also thought that salt and water

were the byproducts of acid-base reactions. The lasting

importance of this definition was that it showed that acids

and bases exchanged positively charged protons; this

detail would be the foundation of a more comprehensive

understanding of chemicals and reactions.

Chapter 3

A10 Insider: The Super Guide

Page 11: Insider: The Super Guide Scripts & Answer Key

Exercise 2 – Identifying the Main Idea and Key Details

| Vocabulary Review | _ p.104

Exercise 1

1. (C) 2. (B)

3. (C) 4. (D)

5. (A)

Exercise 2

1. circuit

2. electromagnetic spectrum

3. proton

4. nebula

5. electron

| iBT Practice | _ p.105~109

1. Referent Question – (B)

Based on the context of the sentence, (B) is the only

logical answer choice.

2. Inference Question – (B)

The paragraph states that developing an atomic model

was “a process of trial and error.” This clearly implies that

many early atomic theories were disproved.

3. Vocabulary Question – (B)

There are no particularly strong context clues that would

allow you to determine the meaning of “nebulous.” Answer

choice (D) could easily be eliminated because it would be

illogical for a limited substance to saturate all things.

4. Referent Question – (A)

The clause “which had negative charges that caused them

to glow” modifies the word “particles”. Therefore, (A) is the

most logical answer choice.

5. Fact Question – (A)

The paragraph states that Thomson discovered objects

that he called “corpuscles.” Later, the paragraph states

that these corpuscles came to be known as electrons.

6. Vocabulary Question – (C)

The phrase “could not pass through” in the sentence

provides a clear context clue.

7. Fact Question – (D)

This is stated directly in the paragraph.

8. Vocabulary Question – (B)

A key context clue is the phrase “correct many of its

inaccuracies.”

9. Fact Question – (A)

The paragraph clearly states that Rutherford concluded

that atoms “had concentrated centers” that were relatively

dense.

10. Fact Question – (D)

The paragraph clearly states that “only certain amounts of

electrons could be placed in certain orbits.” Saying that

“electrons occupy orbits according to specific criteria” is a

generalized way of stating this.

11. Inference Question – (C)

The paragraph states that further improvements were

made to Rutherford’s model. If it was possible to make

improvements to his model, it must have contained some

inaccuracies.

12. Sentence Insertion Question – The sentence should be

placed at the fourth square.

The word “thus” is a clear context clue that the insertion

sentence is drawing a conclusion based on a previous

statement. This means that the first and second squares

are illogical placements for the sentence. The insertion

sentence says that atoms have positive charges in a

negative shell. This clearly matches with the statement in

the passage that electrons are on the outside of the atom.

13. Categorization Question

(A) Experiments with gold foil (Rutherford)

(B) Atom doesn’t have a central cluster (Plum pudding)

(C) Quantum forces in electron orbits (X)

(D) Experiments with cathode rays (Plum pudding)

(E) Electron’s orbit similar to planet’s (Rutherford)

(F) Electrons held steady by a charged field (Plum

pudding)

If you remember some of the key differences between the

plum pudding model and later atomic models, you should

be able to determine that atoms lack a nucleus (central

cluster) in the plum pudding model and that in the

Rutherford model, electrons orbit the nucleus. You may

need to skim the passage to find the other answer

choices. Use the organization to help you. The plum

pudding model is discussed in the first part of the

passage, and later atomic models are discussed

afterwards.

Gilbert Lewis would further expand upon this definition in

1923 with the Lewis definition. Lewis also developed

upon Arrhenius’ ideas, but, instead of focusing on the

positively charged protons, he emphasized the activity of

negatively charged electrons in an acid-base reaction.

He defined a base as any material that gives electrons

and an acid as any material that receives these electrons

in a chemical reaction. This was significant because it

showed that a proton isn’t imperative for initiating a

chemical reaction: only an electron must be present.

Thus, not only do many more substances qualify as

acids or bases in a chemical reaction, but even atoms

and sub-atomic particles qualify as acids and bases, so

long as they give or receive electrons.

Scripts & Answer Key A11

RE

AD

ING

Page 12: Insider: The Super Guide Scripts & Answer Key

Social Sciences

Guided Reading 1 _ p.112~113

1. (B) – In order to answer this question correctly, you must

read this paragraph carefully. The paragraph states that

“there are many competing theories about the nature of

dreams.” From this, you can infer that Freud’s theories are

not universally accepted.

2. (C) – Paragraph 2 explains Freud’s theory on why dreams

contain so much symbolic content. In the last sentence,

the author explains that “the conscious mind remains alert

enough to monitor the unconscious; thus, the

unconsciousness expresses its wishes by disguising its

forbidden content as symbols that are not mentally or

socially prohibited.”

3. (B) – The highlighted sentence emphasizes the same point

twice: Freud didn’t determine the symbolism; the dreamer

did. The other idea in this sentence is that free association

was simply the technique that allowed the patient to do

this. Only the correct answer choice accurately relates

these ideas. The incorrect answer choices either indicate

that Freud himself forcibly interpreted the symbols or they

omit information.

4. The sentence should be placed at the third square. – The

pro-form provides an important clue in determining where

to place the insertion sentence, as “such manifestations”

refers to the way sexual desires manifested in dreams.

Guided Reading 2 _ p.114

1. (C) – Paragraph 3 directly states that “both the House and

the Senate have the power to draft legislation.”

2. (A) Can create legislation (Both)

(B) More likely to compromise on agendas (Senate)

(C) Can appoint officers to federal positions (X)

(D) Two officials from each state (Senate)

(E) All officials elected every two years (House)

(F) Officials represent narrow, local interests (House)

The easiest way to classify the answer choices correctly is

to recall the central idea of the passage, which is that the

stability of the Senate and its tendency to compromise is

partially due to smaller size and longer terms. Based on

this knowledge, you know that the Senate has only two

officials to represent each state and that they serve six

years; in contrast, hundreds of members of the House of

Representatives are elected every two years and they

represent smaller, local areas. Other choices should be

more obvious: both bodies can make laws, and the power

to appoint federal officers belongs to the President, not

Congress.

Unguided Reading _ p.116~118

1. (A) – It is important to read the vocabulary word in the

context of the sentence. When discussing the methods of

early philosophers, the author explains that “their studies

were rudimentary in comparison with contemporary

studies.” Logically, later scientists would conduct more

advanced studies, so, by comparison, early philosophers

conducted crude scientific studies.

2. (C) – This paragraph introduces early philosophical ideas

by explaining that they were not completely separate from

early scientific studies. In contrast to this, the paragraph

continues to explain that modern science stands on its

own as a field of study, while “most modern philosophy

deals with abstract matters like existence, morals, and

thought.” From these two statements you can infer that

modern philosophy does not study the external world.

3. (D) – By reviewing the paragraph before you answer the

question, you should be able to find the answer easily. The

paragraph clearly states “Plato maintains that all physical

things, both creatures and objects, are based on perfect

ideas. For instance, the perfect idea of a horse exists in a

non-physical world, and horses in the physical world are

just imperfect versions of this idea.” Clearly the example of

the horse is being used to better explain Plato’s rather

complex ideas.

4. (B) – By skimming the paragraph you should easily be able

to determine that Aristotle “argues that the physical world

is the only true reality, the only source of knowledge.”

5. (A) – The paragraph states that “Rather than employing

mere intuition, these scientists base all of their knowledge

on their observations of the physical world, as Aristotle

proposed.” Therefore, intuition is not one of the practices

modern scientists derived from ancient philosophers.

6. (A) Claimed that physical world is imperfect (Plato)

(B) Saw science and philosophy as separate entities (X)

(C) Believed that ideas exist outside of human thought

(Plato)

(D) Believed that things change to fulfill purpose

(Aristotle)

(E) Claimed that logic is unnecessary for understanding

(X)

(F) Argued that ideas manifest in the physical world

(Plato)

(G) Argued that knowledge comes from observation

(Aristotle)

Remember that Plato believed in ideas that existed outside

of the physical world. He also believed that these ideas

Chapter 4

A12 Insider: The Super Guide

Page 13: Insider: The Super Guide Scripts & Answer Key

Scripts & Answer Key A13

RE

AD

ING

were perfect and they manifested themselves imperfectly in

the physical world. Keeping this in mind, use the process of

elimination to associate the correct choices with Aristotle,

who believed in observation and the importance of change.

Skim the paragraph if you need to determine which choices

are unnecessary, although it should be obvious that neither

philosopher dismissed logic’s importance or considered

science as separate from philosophy.

Building Your Reading Skills _ p.119

1. (C) 2. (A)

| Vocabulary Review | _ p.120

Exercise 1

1. (D) 2. (A)

3. (B) 4. (G)

5. (I) 6. (J)

7. (F) 8. (C)

9. (H) 10. (E)

Exercise 2

1. Psychoanalyst: With the help of his psychoanalyst, the

boy was able to overcome his fear of flying.

2. Agenda: I never feel like I can completely trust Kelly

because she always seems to have a hidden agenda.

3. Intuition: Through many years of detective work, the

police officer had learned to trust his feelings and follow

his intuition.

4. Legislation: The politicians were so busy arguing with

each other this year that they did not succeed in passing

a single piece of important legislation.

5. Dissident: The cruel dictator arrested anyone he

considered to be a dissident or an enemy of his

government.

| iBT Practice | _ p.121~125

1. Referent Question – (B)

Both the pronoun “they” and its antecedent “autocratic

governments” function as subjects for clauses within the

sentence. In addition, the word “although” in the sentence

indicates that there will be a contrast of ideas. Although

autocratic governments were most common in civilizations

that predate democracy, they still persist today.

2. Vocabulary Question – (D)

Read the sentence for a context clue to what the

vocabulary word means. The sentence discusses “the use

of social institutions to disseminate propaganda around

the country.”

3. Fact Question – (C)

Scan the paragraph for any mention of the Romans. It

should then be easy to find the statement that “Imperial

Rome controlled its economy through building trade routes

and issuing coins to standardize trade.”

4. Rhetorical Purpose Question – (A)

The first sentence directly states the central idea of the

paragraph: “In autocracies . . . the ruler and his supporters

come to power either through bloody conflict or amidst

harsh crises like famine or economic depression.” The

central idea is that autocracies gain power through

exploiting crises. (A) is the only answer choice that is

relevant to this central idea.

5. Vocabulary Question – (C)

Pay close attention to the sentence in which the

vocabulary word appears, for a later word renames it. The

sentence mentions that, with the invention of radio,

“people wouldn’t have to settle for a mere facsimile of the

ruler’s image; they could listen to his very voice instead of

gazing at his visual copy.” Both “facsimile” and “copy”

describe replicated images of the ruler; the two words are

synonyms with “replica,” which is logically the correct

choice.

6. Fact Question – (C)

Scan the paragraph for any mention of the Soviet Union. It

should then be easy to find the statement that the Soviet

Union “took over all Russian newspapers and radio

stations in order to pass off selective or false information

as actual news.”

7. Negative Fact Question – (D)

The paragraph discusses the use of radio to “broadcast

their voices throughout entire nations and enchant them

with charismatic speeches,” control “official

communications, making government action like military

responses much more effective,” and “control the flow of

information to its citizens.” Therefore, the only use of radio

that is not discussed in the paragraph is issuing threats to

political opponents.

8. Inference Question – (D)

The answer is strongly implied by the final, key sentence:

“Because these advancements in technology are so

powerful and threatening, autocratic governments feel

compelled to utilize them before private citizens can.”

Logically, an autocratic government wants to have control

of this dangerous technology because if they don’t then it

can be used to take power from them, just as they have

used this technology to take power from their own citizens.

9. Fact Question – (B)

The discussion of the frumentarii is an example of the

paragraph’s central idea, which is how autocratic militaries

and secret police are used to suppress citizens. Knowing

the central idea should automatically eliminate answer

Page 14: Insider: The Super Guide Scripts & Answer Key

Archaeology & Anthropology

Guided Reading 1 _ p.128~129

1. (D) – The paragraph clearly states that uncontacted

peoples fiercely guard their isolation. Since it also states

that the Huaorani are an uncontacted people, you can

assume that they oppose contact with the outside world.

2. (D) – The paragraph clearly states that “A man may only

marry the daughters of his paternal aunts or those of his

paternal uncles.”

3. (B) – The sentence before the example states that the

Huaorani believe that every animal possesses a spirit and

that they respect and honor this spirit. The sentence that

follows the example states that the ritual is meant to pay

respect to the animal’s spirit. Clearly, the purpose of the

example is to illustrate this respect.

4. (C) – Shamanism is discussed in paragraph 4 and

polygamy and interbreeding are discussed in paragraph 3.

Guided Reading 2 _ p.130~131

1. The sentence should be placed at the second square. –

The preceding sentence states that the yield of Greek

masterpieces was less than expected. This clearly

connects to the insertion statement which says that only

10% of the papyri were literature.

2. (A), (C), and (F) – The introductory sentence clearly

indicates that the focus of the summary will be the reasons

why Oxyrhnchus is an abundant source of papyri. Answer

choices (B) and (E) do not directly address this concept.

Answer choice (D) is not discussed in the passage.

Unguided Reading _ p.132~134

1. (D) – Key phrases from the original sentence are “ancient

belief system and religious traditions” and “a kind of pre-

Christian mysticism”. These ideas are simplified in the

correct answer choice and stated as “The primitive

religious practices of the Berbers.”

2. (A) – The fact that the “assimilation” is linked to the word

“intermarriage” by the conjunction “and” suggests that it

must be a somewhat similar or related idea. That allows

you to eliminate answer choices (B) and (C), and strongly

suggests that (A) is the correct answer.

3. (C) – The final sentence in paragraph 3 clearly states that

“a number of Berber tribes retreated to safety and freedom

in the mountains, beyond the control of the invading

Arabs.”

Chapter 5

A14 Insider: The Super Guide

choices (C) and (D), which don’t address the suppression

of citizens. For the correct answer, skim the paragraph for

this sentence: “Secret police like the Roman frumentarii

and the Nazi Gestapo were instrumental in pervasive

surveillance, terrorizing citizens so that many wouldn’t

even say anything that might be treasonous.”

10. Inference Question – (A)

The following sentence provides an important clue as to

which answer choice is correct: “The charges are usually

exaggerated or fabricated in these spectacles, but the

public watches dissenters pay dearly for opposing the

regime.” This emphasizes the role of the spectators, who

will see the price for challenging the government’s power.

11. Negative Fact Question – (B)

This question can be tricky because it is a large question

that covers information discussed throughout the entire

reading, but, at the same time, the answer choices are

specific details. You must understand the central ideas

within the passage in order to answer this question

correctly. An autocratic government uses three methods

to control its people: exploiting hardships and economic

situations, controlling various media and information, and

employing violence to suppress opponents. Each answer

choice corresponds to one of these central ideas except

medicine. It is never mentioned in the passage, so it is the

correct answer.

12. Sentence Insertion Question – The sentence should be

placed at the fourth square.

There are two ideas being expressed in the insertion

sentence: new technology gives the government greater

power, and it also threatens the government. The insertion

sentence should act as a transition between a discussion

of radio and the reasons why autocracies must control

technology in general. First, the discussion of new

technologies should follow a full discussion of radio.

Second, the idea that this technology threatens the

government directly connects to the last sentence, which

explains why the government must control such

technology. The fourth block marks the space where such

a transition between these two ideas belongs.

13. Prose Summary Question – (A), (B), (D)

The introductory sentence mentions three specific kinds of

autocratic control: economic, social, and political. The way

to correctly answer this question is to select the answer

choices that mention the central ideas and describe one of

these kinds of control. Choice (A) directly addresses

political control through violence, choice (B) discusses how

autocracies consolidate economic control, and choice (D)

discusses social control through the control of technology.

The other answer choices either address minor details and

specific examples from the passage or are incorrect.

Page 15: Insider: The Super Guide Scripts & Answer Key

4. (B) – The clearest context clue for this vocabulary question

is the main idea of the passage, which is that Berbers

were the original inhabitants of Northern Africa and that

their culture was replaced during the Arab invasion.

5. (C) – The general flow of ideas in the paragraph suggests

that (C) is the correct answer. The paragraph states that

Berbers face prejudice in their society and that many

identify themselves as Arabs. The implied idea is that they

do not identify themselves as Berbers because of this

prejudice.

6. (A) Reside in small villages in the mountains (Modern

Berbers)

(B) Revere the spirits of their ancestors (Ancient Berbers)

(C) Widespread religious influence (Ancient Berbers)

(D) Descended from the Egyptians and Phoenicians (X)

(E) Associated with poor, primitive culture (Modern

Berbers)

(F) Prejudicial toward Arab population (X)

(G) Forced to fight oppression (Modern Berbers)

The two incorrect answer choices can be easily eliminated

because they clearly contradict ideas contained in the

passage. You know that the ancient Berbers had

widespread religious influence because they influenced the

regions of the Egyptians and the Greeks. You also know

that they prayed to their ancestors for spiritual guidance,

so they revered the spirits of their ancestors. You know

that modern Berbers face prejudice in modern Moroccan

society (i.e they are oppressed), and the first paragraph

clearly states that they are associated with poverty and

looked upon as a primitive culture.

Building Your Reading Skills _ p.135

Exercise 1 – Finding the Main Topic of Discussion

Exercise 2 – Identifying Key Concepts and Examples

| Vocabulary Review | _ p.136

1. native tongue 2. polygamy

3. taboos 4. shaman

5. descent 6. animistic

7. rite of passage 8. deities

9. kin 10. excavation

| iBT Practice | _ p.137~141

1. Sentence Simplification Question – (B)

The concept that tattoos have a “rebellious attraction”

summarizes much of the information in the original

sentence.

2. Rhetorical Purpose Question – (A)

The origin of the tattoos among older cultures is the main

topic of the passage, so this answer choice is most logical.

3. Fact Question – (B)

The last sentence in paragraph 2 states that some

researchers think that tattoos were used as a “therapeutic

treatment for bones.”

4. Inference Question – (D)

The second paragraph lists a large number of places

where tattoos have been found on early skeletons. This

suggests that tattoos did not originate in one single place.

5. Referent Question – (C)

The main focus of the sentence is the uses of tattoos in

different cultures. Therefore answer choice (C) is the most

logical choice.

6. Fact Question – (A)

The last sentence in paragraph three states that Roman

One major aspect of Berber culture that has been lost

over the ages is its ancient belief system and religious

traditions - a kind of pre-Christian mysticism - practiced

before the mass conversion of Berbers to Islam. The

ancient Berbers considered the spirits of ancestors to be

gods and worshipped them as such. One common

practice was to pray to the spirit of one’s ancestor in

search of spiritual guidance, then sleep in the ancestor’s

tomb and await a response in the form of a dream. In

addition to the spirit world, the ancient Berber religion

involved the worship of stars, the most important of

which was the sun. Berber mythology influenced and

informed many other religions as well, including the

ancient Egyptians and Greeks. Perhaps those most

influenced were the Phoenicians, who adopted the

Berber god Amon into their own belief system, uniting

him with their highest deity Baal.

The term “Berber” has long been used as a general

name for the small population of North Africans who still

speak the languages of the ancient Berber tribes. They

are associated with poverty and peasantry, and

commonly looked upon as simply a small, primitive

culture on the fringes of a predominantly Arab

population. This ethnic minority, however, represents the

oldest known inhabitants of North Africa, with evidence

dating their presence in the region to as far back as

9,000 B.C., though the actual time or place of their origin

is too difficult to determine. In ancient times, they were

feared by the Romans, as well as the Greeks and

Egyptians. At one point, their domain was so vast -

covering North Africa from Morocco to Egypt - that the

Romans feared they would become the next Carthage.

Over the years, however, successive waves of invaders

and foreign settlers resulted in the gradual

disappearance of Berber culture and identity.

Scripts & Answer Key A15

RE

AD

ING

Page 16: Insider: The Super Guide Scripts & Answer Key

soldiers began to get tattoos after witnessing the ferocity

of British tribes.

7. Negative Fact Question – (D)

The use of tattoos to indicate one’s status as a criminal is

not discussed until paragraph 5, when it is discussed in

relation to more modern culutres.

8. Vocabulary Question – (D)

The following sentence states that young girls were

tattooed when they reached puberty. Since puberty marks

the beginning of physical adulthood, this provides a clear

context clue.

9. Fact Question – (C)

The paragraph clearly states that “adolescent male was

expected to endure the long, agonizing process of a full

body tattoo in order to be accepted as a man.”

10. Inference Question – (B)

The paragraph states that tattoos “expressed a link to the

spiritual world that was the foundation of most of these

cultures.” This clearly implies that tattoos were essential

parts of their rituals.

11. Vocabulary Question – (C)

The sentence states that prejudices and objections led

people to interdict the use of tattoos. This clearly suggests

that “interdict” means something similar to “prohibit” or

“stop”.

12. Inference Question – (D)

The first paragraph states that tattoos are commonly used

to identify with a particular group in modern society. The

fifth paragraph states that mysticism has been rejected by

modern society and that tattoos have been put to different

uses. This clearly implies that tattoos have lost their

ritualistic significance.

13. Sentence Insertion Question – The sentence should be

placed at the fourth square.

The word “arthritis” in the insertion sentence is a key

context clue. It clearly connects with the idea of

“therapeutic treatment for bones.”

14. Categorization Question

(A) Used to identify certain groups (Both)

(B) Used as a magic charm (Ancient/Tribal Cultures)

(C) Used to mark criminals and prisoners (Modern

Cultures)

(D) Used as a fashionable luxury (Modern Cultures)

(E) Used to remain unseen in battle (X)

(F) Used as a rite of passage (Ancient/Tribal Cultures)

(G) Used to identify arthritic joints in mummies (X)

Both incorrect answer choices are distractors. They

contain concepts discussed in the passage, but they

misstate those concepts. The passage states that in

modern cultures certain groups use tattoos as marks of

identification, and that in ancient cultures, they were used

to mark the passage into adulthood. Therefore, tattoos are

used to identify groups of people in both types of cultures.

The passage clearly states that ancient cultures used

tattoos as rites of passage into adulthood and that some

tribes used them as magical charms.

A16 Insider: The Super Guide

Page 17: Insider: The Super Guide Scripts & Answer Key

Economics

Guided Reading 1 _ p.144~145

1. (B) – The second paragraph clearly states that the price of

silver “fell dramatically.”

2. (A) – The paragraph states that the system of the gold

standard was “vulnerable” and goes on to discuss how

gold was hoarded following the stock market crash in

1929. This clearly suggests that the gold standard had

serious problems.

3. (C) – Answer choices (A) and (D) are clearly illogical in the

context of the sentence and can easily be eliminated.

Answer choice (B) fits logically into the sentence, but in the

context of the passage (C) makes more sense.

4. The sentence should go at the third square. – The

concept of “creditors” clearly matches up with the idea

that people needed to pay off debts.

Guided Reading 2 _ p.146~147

1. (A) – Answer choices (B), (C), and (D) misstate ideas from

the original sentence.

2. (A) It is the standard measurement of economic

progress. (GDP)

(B) Measures the costs against the benefits of economic

growth. (GPI)

(C) Determines which countries are developed,

developing, or underdeveloped. (X)

(D) Includes human and environmental factors in

calculating progress. (GPI)

(E) Includes factors such as literacy and life expectancy. (X)

(F) Measures only the market value of all goods and

services. (GDP)

(G) Focuses on the sustainability of progress. (GPI)

The key point of the passage is that GDP is not an

effective measurement of standard of living because it

does not account for the negative effects of economic

activity. Using this main idea to guide your thinking, most

of the answer choices should be obvious.

Unguided Reading _ p.148~150

1. (C) – Answer choice (C) is directly contradicted by the

paragraph. Stagflation results in high unemployment.

2. (D) – Since the paragraph clearly states that there is no

way to fix one part of stagflation without making the other

part worse, you can conclude that there is no ideal

solution for the problem.

3. (B) – The flow of the ideas in the paragraph, which states

that the steps by the government to fix the problem only

made it worse, and describes the government’s policies as

“extremist,” provide context clues for this question.

4. (B) – The paragraph clearly states that “Economic disaster

struck in the 1980s and 1990s, when much of the farmland

was confiscated by the government and redistributed

among wealthy elites and members of government.”

5. The sentence should be placed at the fourth square. –

The flow of ideas in the paragraph implies a clear cause

and effect relationship. Foreign aid was withdrawn because

the government’s poor management worsened the problem.

6. (B), (C), (E) – Answer choices (A) and (F) are relatively

minor ideas, and (D) is not strongly connected to the

introductory sentence of the summary.

Building Your Reading Skills _ p.151

Exercise 1

Exercise 2

One extreme example of stagflation presently taking

place is the collapse of the Zimbabwean economy.

Zimbabwe’s economic problems can be attributed to a

long history of (2) legislative efforts aimed at

reapportioning farmland among the native population.

Economic disaster struck in the 1980s and 1990s, when

much of the farmland was confiscated by the

government and redistributed among (1) wealthy elites

and members of government, leaving much of the

population without homes or land upon which to build

them. Displacing commercial farmers and giving away

their farmlands led to (3) the greatest famine the

country had ever seen. Food production declined

sharply, and certain imported foods were banned by the

government. Such extremist policies drew derision from

much of the world community.

The term “stagflation” was coined in 1965 by a senior

British finance minister to describe what was, at that

time, an unprecedented economic phenomenon. A

combination of the words “stagnation” and “inflation,” the

term is used to describe a period of sharp inflation, in

which the value of the currency is falling against sharply

rising prices, in addition to high unemployment or

economic recession. Before the 1960s, economists

theorized that this set of circumstances was impossible.

It was believed that during a recession, high

unemployment would curb demand and keep inflation at

bay. However, as stagflation began to emerge in various

economies around the world, these economists were

forced to change their way of thinking. Though

stagflation is indeed a rare phenomenon, it does occur

and brings with it a number of unique challenges.

Chapter 6

Scripts & Answer Key A17

RE

AD

ING

Page 18: Insider: The Super Guide Scripts & Answer Key

| Vocabulary Review | _ p.152

1. legal tender 2. growth rate

3. gross 4. panic

5. standard of living 6. crash

7. inflation 8. net

9. sustainability 10. consumption

| iBT Practice | _ p.153~157

1. Vocabulary Question – (B)

The rest of the paragraph lists a number of benefits that

offshore banks have. Therefore the flow of ideas in the

paragraph provides a context clue to this question.

2. Referent Question – (C)

The pro-form is the subject of a sentence. Out of the four

answer choices, only “offshore banks” works as a subject

in the paragraph. This makes (C) the most probable

answer.

3. Sentence Simplification Question – (A)

The correct answer choice omits some minor ideas from

the original sentence, but it correctly relates all of the key

ideas.

4. Fact Question – (C)

The paragraph clearly states that offshore banks offer

savings accounts with tax free interest.

5. Inference Question – (D)

The paragraph states that maintaining offshore accounts is

expensive, so it is logical to conclude that only the rich can

afford them.

6. Vocabulary Question – (B)

The paragraph states that offshore banks hold about one

trillion dollars that is used for tax evasion. This huge

amount provides a context clue to the meaning of

“exorbitant.”

7. Negative Fact Question – (C)

Tax evasion, drug smuggling, and money laundering are all

clearly discussed in paragraph 3.

8. Inference Question – (A)

This can be inferred from paragraph 3, where the possible

effects of offshore banks on developing countries are

discussed.

9. Referent Question – (D)

The sentence states that “they belong to suspected

criminals.” Therefore, “they” must refer to something that

criminals can own. In this context, answer choice (D) is the

only answer choice that makes logical sense.

10. Fact Question – (B)

This is directly stated at the beginning of paragraph 5.

11. Rhetorical Purpose Question – (B)

The relevance of this is explained in the next sentence,

which states that “despite the mistaken notion that they

are illegal, most business conducted in offshore banks is

actually legitimate and even helpful in its cooperation with

regulated banks.”

12. Sentence Insertion Question – The sentence should be

placed at the first square.

The phrase “such measures” logically connects to the idea

that “United States taxpayers must report all income to the

government, even interest from offshore accounts.”

13. Prose Summary Question – (B), (E), (F)

Answer choices (A), (C), and (D) all contain relatively minor

details from the passage.

A18 Insider: The Super Guide

Page 19: Insider: The Super Guide Scripts & Answer Key

Scripts & Answer Key A19

RE

AD

ING

History

Guided Reading 1 _ p.160~161

1. (B) – This answer is directly stated in the paragraph. The

paragraph states that “While Rome remained the capital of

the Western Empire, Constantinople was the capital of the

wealthier and more stable Eastern Empire. Thus, the

church based in Constantinople also saw itself as the

natural leader of the Christian world.”

2. (A) – A clear context clue comes in the statement that

these churches “fell in line and acknowledged the

leadership of Constantinople.” Clearly, falling in line must

be somewhat similar to acknowledging leadership.

3. (C) – This idea is clearly introduced in the first sentence of

the paragraph, which states that “The fall of the Western

Empire further complicated the issue.”

4. (B) – The paragraph states that the Church of

Constantinople lost much territory due to the rise of Islam.

It also states that after this, the Church of Rome began to

press its claims of preeminence more vigorously. The clear

implication is that the weakening of the Church of

Constantinople benefited the Church of Rome.

Guided Reading 2 _ p.162~163

1. (C) – The main point of the passage is that the terra-cotta

army was built to protect the emperor after his death.

Therefore answer choice (C) is contradicted by the main

point of the passage.

2. (B), (C), (F) – The introductory sentence of the summary

focuses on the idea of protecting the emperor after his

death. Answer choices (B) and (C) address how the

complex was designed to protect the emperor and answer

choice (F) discusses why the emperor felt such protection

was necessary. Answer choices (A), (D) and (E) all focus on

minor details from the passage.

Unguided Reading _ p.164~166

1. (C) – The paragraph provides several context clues. First,

the paragraph states that the territory of California was

largely ignored. The contrast created by the use of

“however” in the sentence also provides a context clue.

The Spanish did establish strongholds. However, their

involvement was marginal.

2. (D) – The pronoun refers to the Spanish, and the

passage’s main idea is an important clue. In the phrase

“their claim to the territory,” “territory” refers to “upper

California,” which is later stated in the sentence. Spain’s

possession of California is the main idea of the entire

passage, so “their” logically refers to the Spanish.

3. (C) – By skimming the paragraph for the key words“monasteries” and “Upper California” you should easily beable to find the statement that “Fearing that their claim tothe territory might be threatened, the Spanish sent monksto establish monasteries throughout Upper California.”

4. (A) – This idea is implied in one important sentence in theparagraph. The author states that while they wanted tospread Christianity, the “Spanish did, however, prospergreatly off of the fruits of their Native American pupils’labor, trading wine, oils, pelts, and produce for otherexpensive goods . . .” The phrase “expensive goods”indicates money being exchanged, implying that themonks made a monetary profit.

5. The sentence should be placed at the second square. –The phrase “their presence” at the beginning of theinsertion sentence is the main indication of where it shouldgo. This phrase refers to a plural noun. After quicklyscanning the paragraph, the only plural noun available is“useful bays present along the coast.” This makes sense:logically, the presence of useful bays provided enoughevidence that the land could be colonized.

6. (C), (D), (F) – These answer choices correctly address themain idea of the introductory sentence and passage, whichis that the settlement of California was the result of a longprocess. The correct answer choices all focus on importantsteps or developments that contributed to this process.

Building Your Reading Skills _ p.167

Exercise 1 – Identifying Logical Connections

Most colonial powers concentrated their efforts on what

is now the northeast United States, as well as Central

and South America, so the territory now known as

California went largely ignored. Spain laid claim to the

region, and they had already established strongholds in

Mexico and the southern US. However, their involvement

with it was marginal for many years. Hernán Cortés led

many expeditions along Baja, seeking a mythical island

full of treasures that he believed lay somewhere in the

Gulf of California, but he never reached the area now

known as California.

Juan Rodriguez Cabrillo was the first to travel farther

than Baja (then called “Lower California”), in effect

becoming the first European to lay eyes on the long

stretch of shoreline known today as the California coast.

Cabrillo spent little time on shore, but he did make an

important discovery at sea. Contrary to earlier beliefs, he

found that there were many useful bays along the coast.

Because of this, the area could support large-scale trade.

Cabrillo’s findings created a renewed interest in what

was known as “Upper California.” In 1602, Sebastián

Vizcaíno was commissioned by the King of Spain to map

the Upper California shoreline. Among the stops along

his route were Monterey Bay and San Diego.

Chapter 7

Page 20: Insider: The Super Guide Scripts & Answer Key

| Vocabulary Review | _ p.168

Exercise 1

1. meticulous 2. coerced

3. infiltrate 4. prominent

Exercise 2

1. preeminence 2. expedition

3. stronghold 4. forge

5. factions 6. conscripts

| iBT Practice | _ p.169~173

1. Fact Question – (C)

The answer to this question can best be obtained by

skimming the paragraph. In the middle there is a sentence

that directly states this: “The trials proceeded among the

devoutly Christian Puritans, who construed any unusual

religious practice or general behavior as witchcraft or devil

worship; any supposed witchcraft was punishable by

death.”

2. Referent Question – (A)

Consider the context of the sentence; the phrase reads

“they just suffered . . .” This implies that “they” have

feelings, so “people” is correct.

3. Sentence Simplification Question – (B)

This answer correctly summarizes the ideas of the long

highlighted sentence. The main idea is that a Puritan’s life

centered on worshipping God, which rephrases “exalting

God above all else.” The first part of the highlighted

sentence provides a long list of the ways Puritans achieved

this, but the items in this list are simply examples of their

rigorous standards of piety. The correct answer is thus a

paraphrase of the sentence in the passage.

4. Fact Question – (D)

The paragraph discusses how many elements led to girls

having hysterical imaginations and claiming that specters

attacked them. While it was insubstantial, “this spectral

evidence was enough to charge the accused person with

witchcraft.”

5. Inference Question – (B)

In order to determine the correct answer, it is important to

understand the ideas that the paragraph is discussing,

especially the reasons why the girls made the accusations.

The paragraph directly states that Puritan girls had very

few creative outlets and that almost everything they read

focused on the dangers of sin and witchcraft. It also states

that these ideas played a role in the accusations made by

the girls. Taken together, these facts clearly imply that the

imaginations of the girls and their lack of creative outlets

led to their accusations.

6. Vocabulary Question – (A)

Reading the full sentence should provide enough of a

contextual clue to the highlighted word’s meaning: “Many

settlers, including Puritans, moved further west,

encroaching upon Native American land, leading to bloody

conflicts.” The conflicts resulted because the settlers

intentionally moved west upon land that didn’t belong to

them. In other words, they intruded upon this land.

7. Negative Fact Question – (C)

Because the key phrase is highlighted, the answer can be

found by simply skimming over the sentence. The only

factor that isn’t even mentioned is a rival religion, whereas

“fighting, anarchy, and disasters like floods, famine, and

smallpox created an atmosphere of paranoia.”

8. Rhetorical Purpose Question – (B)

Because the key phrase is highlighted, the reason why

Samuel Parris is mentioned can be found in the sentence.

It reads, “When the controversial Samuel Parris became

the village minister, these tensions culminated in a definite

rift . . .” The word “tensions” is a direct clue, and Samuel

Parris is used as an example of how these tensions within

Salem grew and made the witch trials possible.

9. Negative Fact Question – (A)

Use a process of elimination to determine the answer. The

paragraph states that “no one was safe from the hysterical

girls’ accusations,” so hysterical girls were the accusers,

not the accused.

10. Vocabulary Question – (D)

The meaning of the highlighted word is “appease,” which

can be determined by the context of the paragraph. In

order to “maintain order in the colony,” the new governor

wanted to calm the people down by meeting some of their

demands, which included investigating the accusations of

witchcraft. He set up the special court to appease them,

the logical choice for “placate.”

11. Fact Question – (B)

The fact that the Samuel Parris scandal was forgotten in

the midst of the witch trials is directly stated in the

paragraph. Skim it closely until coming upon the sentence

that describes how “the hysteria drew attention away from

attempts to remove him as village minister.” This is how

Parris benefited from the trials.

12. Sentence Insertion Question – The sentence should be

placed at the third square.

The key to figuring this out is to understand whom “he”

refers to in the insertion sentence. The transitional phrase

mentions that he was a judge and successor to the

governorship; the only man who is identified as a chief

justice and the person who “became governor after Phips”

is William Stougher. Thus, the most logical place for the

insertion sentence is after a description of Stougher, which

provides a referent for “he.”

13. Prose Summary Question – (A), (C), (E)

A20 Insider: The Super Guide

Page 21: Insider: The Super Guide Scripts & Answer Key

Scripts & Answer Key A21

RE

AD

ING

The introductory sentence simply mentions that social and

cultural tensions contributed to the witch trials. The three

correct answers are all statements that paraphrase the key

ideas within the passage. Specifically, the internal strife

and bloodshed that caused paranoia (A), the exploitation

of the trials for political gain (C), and the suppression of

creative expression that caused the girls to make up

stories (E) are the main factors that made the witch trials

so deadly. The three incorrect answer choices either deal

with minor details or contain ideas that are not expressed

in the passage.

Art & Literature

Guided Reading 1 _ p.176~177

1. (A) – This question can be answered by skimming the

paragraph to find the word that is modified by

“inextricably.” This word is “connected.” In the next

sentence, the phrase “this connection was so inseparable”

refers back to this idea that the church and captivity

narratives were “inextricably connected,” so both

“inextricably” and “inseparable” describe this connection.

Thus, “inseparably” is an appropriate synonym.

2. (C) – Tracing the pronoun back to the referent is simple for

this question. Read the previous sentence: “According to

the church, the sins of greed and pride were especially

rampant.” The pronoun then appears: “They drove colonists

to want more . . .” Here, “they” obviously refers back to

the sins, the only plural noun in the previous sentence.

3. (C) – If the reader focuses on the main idea of the

passage, then the answer is obvious. The entire point of a

captivity narrative is to make the woman suffer and

renounce sin: “Deprived of material things and forced into

slavery, the childless, widowed captive turns to God for

protection and guidance.” The idea that this character

would give up Christianity directly contradicts the purpose

behind a captivity narrative.

4. (B) – This is clearly suggested by the last two sentences of

the paragraph, which state that “The message that the

Puritan church was trying to communicate through these

accounts of Indian captivity is clear: repent for your sins,

humble yourself before God, and submit to the authority of

the church. Then, God won’t force you to suffer the ordeal

of Indian captivity.”

Guided Reading 2 _ p.178~179

1. The sentence should be placed at the first square. – The

paragraph describes how Wagner constructed his

elaborate operas, and it lists several of the actions Wagner

undertook to achieve such large productions. For the

insertion sentence, the key word is “began.” This indicates

that Wagner’s writing the music was the first step he took

in creating the opera. Logically, this sentence should come

before the other sentences that describe Wagner’s

actions.

2. (A) Operas were usually no longer than two hours (Italian

Composers)

(B) Works were used for nationalism and later

propaganda (Wagner)

(C) Emphasized singer’s voice above music (Italian

Composers)

Chapter 8

Page 22: Insider: The Super Guide Scripts & Answer Key

(D) Works weren’t popular in the nineteenth century (X)

(E) Incorporated mythology into operas (Wagner)

(F) Influenced by the writings of Freud and Nietzsche (X)

(G) Constructed elaborate stage sets and costumes

(Wagner)

The best way to answer this question is to remember the

main points about Wagner’s operas. They included a lot of

national myths and epics, so he wanted long, loud, and

extravagant musical productions. As a result, later German

groups used his operas to promote nationalism. Then, by

process of elimination, pick out the details of Italian opera:

it’s no longer than two hours and would never use loud

music to drown out a singer. Obviously, both genres were

popular in the nineteenth century.

Unguided Reading _ p.180~182

1. (A) – The paragraph directly states that Wordsworth’s new

language “rejected the political and social values of the

Enlightenment.”

2. (C) – An important detail about the correct answer choice

is that it uses the French Revolution as an example while

the incorrect choices make the French Revolution the main

topic of the sentence. The main idea behind the

highlighted sentence is that the French Revolution was an

example of a conflict Wordsworth supported because he

thought that struggles like these were needed to empower

the lower classes. All of the other choices omit one or

more details of this idea.

3. (B) – Pay attention to the context in which this word

appears. According to the passage, Wordsworth

overwhelmingly associated innocence and purity with the

countryside, in contrast to the worries and corruption of

“technological progress, urban development, and cutthroat

politics” that he believed were exclusive to the city. By

associating abstract, pleasant qualities with the

countryside, the most logical answer is “peace.”

4. (D) – This is an important fact because it is the main focus

of the paragraph and one of the central ideas of the

passage. In addition, the wrong answer choices, (A), (B),

and (C), are all things that Wordsworth opposed.

5. (B) – The idea is never directly stated, but the idea that

new poetic language was written for the lower classes is

explained throughout the paragraph. One important

aspect of Wordsworth’s poetry is that he wanted to write

poems about common folk for common folk because they

couldn’t afford an education like that of the upper classes.

The fact that Wordsworth wanted less educated people to

read his poetry is key detail in this paragraph, as he

rejected the “typically complex poetic diction and

conventions that only the rich were able to learn” in favor

or a more common and easily grasped language.

6. (B), (C), (F) – The introductory sentence of the summary

clearly indicates that the focus of the summary should be

the connections between Wordsworth’s poems and the

beliefs and values of Romanticism. (A) can be eliminated

because it has no connection to this idea. (D) can be

eliminated because it only discusses Romanticism, not

Wordsworth’s poetry. (E) can be eliminated because it

incorrectly states information from the passage;

Wordsworth did not object to the French Revolution.

Building Your Reading Skills _ p.183

Exercise 1 – Identifying Organization

(2) If Romanticism advocated the celebration of

human individuality and nature, then the Enlightenment

advocated the study and control of both. Before

Romanticism, societies throughout Europe had

advanced science and centralized political authority in

order to create a paradigm for the modern nation-state,

but proponents of Romanticism believed that these

developments had ultimately oppressed and deprived

common people for the benefit of aristocrats and royalty.

Romantics reacted against such social standards;

sometimes reactions were violent, including multiple

revolutions and struggles for independence across

Europe that raged for over half a century. One example

of outright violence is the French Revolution, which

Wordsworth openly supported and believed to be

necessary in eradicating the reigning social and political

order; he believed that only through such struggles could

common people truly empower themselves.

(3) Common people, such as farmers, villagers, and

laborers in various industries, were the main subjects of

Wordsworth’s poems in Lyrical Ballads. Previously, most

poetry focused on people in higher classes, particularly

the noblemen, politicians, and royal families who were

usually located in urban areas. To counteract this trend,

Wordsworth emphasized not only the lower-classes but

untamed nature itself - farms, woods, and rustic

landscapes. He believed that the lower class individuals

experienced greater serenity in their natural environment

than urban denizens did in their environment.

Wordsworth believed that people who resided within the

unadulterated nature of the countryside were purer and

more innocent because they were less corrupted by

technological progress, urban development, and

cutthroat politics.

(1) Because of Wordsworth’s beliefs, Lyrical Ballads

contributed to the permanent transformation of poetic

language in English. Instead of writing with typically

complex poetic diction and conventions that only the rich

were able to learn, Wordsworth utilized the language of

A22 Insider: The Super Guide

Page 23: Insider: The Super Guide Scripts & Answer Key

| Vocabulary Review | _ p.184

Exercise 1

1. (C) 2. (A)

3. (A) 4. (C)

5. (A)

Exercise 2

1. (A) 2. (E)

3. (B) 4. (D)

5. (C)

| iBT Practice | _ p.185~189

1. Referent Question – (D)

This pro-form refers to the masterpieces that Renaissance

artists produced. The easiest way to determine this is to

read the sentence: “One of these is the statue of David.”

Logically, the statue is a masterpiece, so the pro-form that

identifies the statue also refers to masterpieces, which

were mentioned in the previous sentence.

2. Fact Question – (C)

The paragraph clearly states that although the statue was

originally intended for a cathedral, it was placed in front of

city hall in Florence because it was so popular.

3. Referent Question – (B)

The sentence states, “Michelangelo portrays the young

shepherd David, who seeks help from God in defending

Israel against the giant Goliath and ultimately slays him

with a sling and stone.” By understanding the main idea of

the paragraph, which is the importance of David and his

story to Renaissance culture, it is easy to see that “him”

refers to Goliath, whom David slew.

4. Vocabulary Question – (D)

In order to pick the correct answer, it is important to

understand the context in which the vocabulary word

appears. It is especially important to understand the

relationship between the church and Michelangelo. This

sentence best states the relationship: “Because the church

was Michelangelo’s primary patron, his statue naturally fell

into a long tradition of artwork that extolled Christian

beliefs.” Logically, because the church paid him,

Michelangelo would praise, or acclaim, their beliefs.

5. Rhetorical Purpose Question – (A)

This choice is the only answer choice that addresses the

main idea of both the paragraph and the entire passage.

The paragraph explains that so much art had religious

content because the Catholic church had so much power.

One sentence directly states this: “The Catholic church in

Rome was the most powerful social force in Medieval and

early Renaissance Europe, and it had enough material

wealth to finance all kinds of extravagant work, from

paintings to entire cathedrals.” The author mentions the

church’s power to explain how it influenced art.

6. Vocabulary Question – (C)

An important contextual clue for this question is the

sentence in which the vocabulary word appears. It reads,

“With the creation of David, Michelangelo emulated the

style of ancient Greek and Roman statues in order to relate

similar themes . . .” Naturally, if he wanted to produce

similar results, then he would use a similar style. In other

words, he imitated the style of these artists.

7. Negative Fact Question – (B)

The answer doesn’t relate to the main idea of the passage;

in fact, it directly contradicts the idea of Humanism.

Skimming the paragraph will reveal that complete

submission to religion isn’t mentioned, but understanding

the main idea behind humanism is more helpful: it regards

“human beings as powerful, dignified, and rational

creatures.” All of the other answer choices fit this definition,

so, by process of elimination, (B) clearly is the correct

answer choice.

8. Vocabulary Question – (B)

Look for words that the vocabulary word renames in the

paragraph. After the vocabulary word appears, the next

sentence reads, “David was an inspiration during such

conflicts for independence.” This rewords the previous

phrase, “struggled for autonomy.” Thus, autonomy and

independence are synonyms.

9. Sentence Simplification Question – (D)

The highlighted sentence paraphrases some key ideas

from this paragraph. The main ideas are that the human

body is the perfect art form because it was created by

God and that Renaissance artists wanted to respect this

perfection by depicting the body accurately. After carefully

reading the answer choices, it is clear that only (D)

contains a proper summary. The other answer choices

either omit or distort important information.

10. Fact Question – (C)

One way to answer this question is to skim the paragraph

for the correct answer. However, because there are so

many details, it helps to remember that Michelangelo

the lower classes for his poetry. He did this to both make

poetry accessible to everyone, not just for the upper

classes who could afford the best education, and to

express his own admiration for the subjects of his poetry.

As a result, writers of poetry after Wordsworth have

incorporated more common language and realistic

speech into their work in order to accurately portray their

subjects. Poetic language no longer needs to be

selectively complex or contain bombast; it can be simple

and still express the natural beauty of its subject.

Scripts & Answer Key A23

RE

AD

ING

Page 24: Insider: The Super Guide Scripts & Answer Key

wanted to show David as deep in thought, not in the

middle of bloodshed as previous sculptors did. Thus, “the

most visible detail that distinguishes this statue of David

from previous statues of David” is “the absence of

Goliath’s head, which David severs in triumph.”

11. Rhetorical Purpose Question – (A)

This answer paraphrases the central idea of the

paragraph, so it is important to understand the details

being discussed and why they are being discussed. The

author provides these details in order to show how

Michelangelo wanted David to embody rational humanist

qualities. The early sentence provides a major clue: “The

statue was designed to show David’s mental prowess as

well as his physical power.”

12. Sentence Insertion Question – The sentence should be

placed at the second square.

In order to place the insertion sentence in the correct

place, simply pay attention to the progression of ideas

within the paragraph. The insertion sentence mentions

how the statue was moved to a final location. Thus, the

sentence should come after a description of where the

statue was originally located. The second square follows

such a description, so the insertion sentence belongs here.

13. Prose Summary Question – (A), (B), (D)

The central focus of the introductory sentence is that

Michelangelo’s David is an excellent example of

Renaissance art. Thus, the choices that support this

concept should adequately paraphrase the main ideas

about the Renaissance and how David fulfills the

standards of art in this time period. Most importantly, the

Renaissance celebrated humanist beliefs (A) as well as

religious beliefs. Michelangelo’s David fulfills both of these

key requirements: David is a religious subject common in

the Renaissance (B), and the way that Michelangelo

portrays him emphasizes the humanist value of humans’

physical and mental potential (D). The other answer

choices only provide minor details about humanism, the

Renaissance, or Michelangelo, or they are incorrect.

A24 Insider: The Super Guide

Page 25: Insider: The Super Guide Scripts & Answer Key

Scripts & Answer Key A25

Main Idea Questions

Basic Drills _ p.200

1. (C) – Although the student mentions that he is on an

athletic scholarship, and that he plays on the football team,

this information is background information which explains

why he will miss some assignments.

2. There are two groups of words that are repeated or have

similar meanings. The first is words like: burglary, break-

ins, robberies, crime, and thieves. The other group

consists of words like: safeguard, protect, thief-proof, and

precaution. Based on these word groupings, you should

be able to figure out that the main point of the lecture has

to do with how to protect your home from thieves.

3. (C) – The notes provided for the lecture are organized in a

format that makes physical comparisons between humans

and chimps. Although genetics and evolution are mentioned

in the lecture, they are clearly not the main ideas.

3.

Professor (female): You know, much has been made

over the years about the apparent evolutionary

connection between the great apes, especially chimps,

and human beings. Even before genetic analysis, the

similarities were pretty obvious. And now we know that

the chimp genome and the human genome are about

98% identical. So the claim of an evolutionary link seems

pretty strong to me. But what I want to talk about today is

that other 2%, the anatomical differences it creates, and

why those differences exist.

Alright, first, and most obviously, humans have larger

brains than chimps, uh, about three times larger. Now,

humans need that big brain to make tools, for language

use, and for the advanced problem skills that are

characteristic of our species. Chimps, on the other hand,

are only opportunistic tool users, uh, they don’t make

their own tools. Their communicative abilities are limited

to grunts, growls, and the like ... and their problem

solving abilities are obviously far below our own.

Another important anatomical difference would be the

shape of the hands. Humans have long, thin fingers and

a long thumb. Chimps have long fingers, although not as

thin as ours, and a short thumb. Now, let’s look at how

each species uses its hands. Humans use their hands for

precision gripping and manipulation of objects. In fact, next

to our brains, you could say that our hands are our greatest

tool. Chimps do use their hands to grip things, but not

nearly with the precision that we do. In addition, chimps

walk on their hands a great deal. So their hands have to be

configured to support a lot more weight than ours do.

[ f Script ]

1.

Student (male): Excuse me, professor? Can I come in?

Professor (female): Sure, Michael. What can I do for you?

Student: Well, I don’t know if you know or not, but I’m on

an athletic scholarship. I’m on the football team. Anyway,

the point is we have to travel for a lot of away games, so

I’m going to be out of class a lot. I was just wondering

what I should do about getting the notes and the

assignments.

Professor: Oh, OK. I didn’t know you were on the team,

but I’ve had athletes in class before. It’s really not a big

deal. Uh, usually you give your schedule to the coach,

and he has an assistant pick up your work for you. But if

you’re worried about it, I can send you an email with the

assignments each week.

2.

Professor (male): Burglary statistics in the US show that

home break-ins account for a large percentage of the

theft that occurs in America. In fact, the numbers show

slightly over 2.1 million home robberies in the last year.

The total amount stolen was $3.5 billion, with each

individual crime resulting in an average loss of $1,600

dollars.

Now, the thing to remember is that most of these

burglaries occurred at homes with little or no protection.

And that’s the point I want to make today. If you

safeguard your home, you are statistically far less likely

to be burglarized. So what does it take to make your

home thief-proof? Actually, not a lot, to tell you the truth.

The first thing to remember is to lock all your doors and

windows. You’d be surprised how many people forget to

take even this basic precaution. The next thing to do

would be to invest in a home security system, and

display the sign for that security system in your front

yard. You see, most criminals won’t even bother trying to

rob a house with a security system. They just figure it’s

easier to go and find a home without one. Finally,

remember that many burglaries occur when people are

on vacation or away on a trip. So, if you are going to go

away for a while, the best thing you can do to protect

your home is to have a friend come by and check in on

your house each day.

Chapter 1

LISTENING Section

LIS

TE

NIN

G

Page 26: Insider: The Super Guide Scripts & Answer Key

A26 Insider: The Super Guide

Listening Practice 1 _ p.201

1. (C) – The key to understanding this is when the girl

corrects the professor (about halfway through the

conversation). The professor is worried that the student

wants his help in the admissions process, but she corrects

him, explaining that she only wants his advice on how to

prepare for the interview.

2. (A) – This is clearly stated by the professor in his second

response to the student.

3. (D) – This is clearly stated by the professor when he says

that he will make the student a reading list.

Building Your Listening Skills _ p.202

1. Excuse 2. these are

3. That is 4. supposed to

5. don’t you 6. This will

7. take part in the interview process

8. Is there any 9. There are

10. That would 11. It is

Listening Practice 2 _ p.203

[ f Script ]

Professor (male): OK, I got a little trivia question for you

all. Ready? What do Alaska, Japan, and Indonesia all

have in common?

Student A (female): Uh, well, I don’t know about Alaska,

but Indonesia and Japan both have a lot of earthquakes.

Student B (male): Yeah, that’s it. Alaska is a big

earthquake region, too. Uh, the whole northwestern

coast of North America is, actually.

Professor: That’s good. Uh, and not only are these areas

of high earthquake activity, they’re also areas with a lot

of volcanic activity. That’s actually not too surprising

because, as we’ll see, earthquakes and volcanoes often

go hand-in-hand. OK, next question. Anyone know why

these areas have lots of earthquakes and volcanoes?

Student B: It’s because of the movement of the earth’s

crust. Like, when you have earthquakes it’s because two

parts of the earth’s crust are moving against each other.

Professor: Great! You all are two for two today. That’s

right. As we mentioned in the last class, the earth’s crust

is made of plates which are always in motion, and it’s this

motion that causes earthquakes and volcanoes. Alaska,

Indonesia, and Japan all happen to lie on areas where

two of the earth’s plates are colliding. Actually, they are

just part of a line that geologists call the Pacific Ring of

Fire. The Ring of Fire extends from the western coast of

South America, up through the North American west

coast, crosses over the Pacific, and then runs down the

eastern coast of Asia. So it makes a kind of upside down

“U” shape that’s about 40,000 km in length. Now, all

along this line, you have plates that are colliding, which

accounts for the incredible amount of earthquakes and

volcanoes in this region. I do mean incredible by the way.

Roughly 90% of all the earthquakes and 75% of all

volcanoes occur somewhere along the Ring of Fire.

Student A: So was the 2004 tsunami because of the

Ring of Fire?

[ f Script ]

Student (female): Excuse me, Professor Gromand, uh,

these are your office hours, right?

Professor (male): That’s right. Uh, normally you’re

supposed to schedule a specific time, but ... it doesn’t

look like I got anyone signed up to come right now, so

why don’t you take a seat?

Student : Thanks, this’ll just take a minute. This is my last

year, and, uh, next semester I have to get my application

to the graduate program ready. Anyway, you know we

have to be interviewed by the department committee for

that ... and I know that you’re on that committee ...

Professor: (sounding a bit cautious) Uh, yeah, I am ...

but you know that as your professor I’ll have to absent

myself from your interview ... Normally, I take part in the

interview process, but since you’re my student, I can’t. I

won’t really have anything to do with the decision making

process in your case.

Student : Oh, I know that ... I wasn’t really coming to talk to

you about that part. Uh, really what I wanted was to ask for

some advice on how to prepare for the interview. Are there

books I should be reading? Research I should be doing?

Professor: Oh, OK, I see. I’ll have to say, it’s nice to see

you taking such a proactive role in this. Uh, yeah there’s

some books that might help ... uh, you know, give you

some background for the interview. Tell you what, I’ll

make up a reading list, and if you swing by later in the

week, I’ll have it ready for you.

Student : That’d be great. I really appreciate it.

Professor: No problem. But remember, this would be

supplemental reading, so don’t let it interfere with your

current studies.

Student wrking on applctn to grad. schl.

interview by admssns review committee

Prof. can’t take part in intrvw. b/c she is his stdnt.

Stdnt wants prof. to give advice 4 intrvw prep.

prof. will make reading list for stdnt.

Page 27: Insider: The Super Guide Scripts & Answer Key

LIS

TE

NIN

G

Scripts & Answer Key A27

1. (C) – Although earthquakes and volcanoes are mentioned

at the beginning of the discussion, they are simply things

that characterize the Pacific Ring of Fire. Answer B is far

too broad, and D is incorrect because Japan and

Indonesia are only mentioned as two locations along the

Pacific Ring of Fire.

2. (B) – Student B explains this in answering the professor’s

question about why Alaska and Japan have many

volcanoes and earthquakes. Answer B is just a paraphrase

of this information.

3. �� Eastern coastal Asia� Western coastal India�� Western coastal South America� Eastern coastal Africa

The answers are the 1st and 3rd boxes; the professor

states this clearly in describing the Pacific Ring of Fire.

Building Your Listening Skills _ p.204

1. Statement from the discussion: Uh, and not only are

these areas of high earthquake activity, they’re also

areas with a lot of volcanic activity. That’s actually not

too surprising because, as we’ll see, earthquakes and

volcanoes often go hand-in-hand.

The paraphrased sentence is really just a combination of

the information contained in these two sentences from the

discussion. Saying that two things are linked is equivalent

to saying those two things “go hand-in-hand.”

2. Statement from the discussion: As we mentioned in the

last class, the earth’s crust is made of plates which are

always in motion, and it’s this motion that causes

earthquakes and volcanoes.

The paraphrase summarizes the information in the original

statement. It omits the idea that the earth’s plates are in

constant motion, but still retains the basic explanation of

cause and effect. The phrase “seismic activity” refers to

volcanoes and earthquakes.

Listening Practice 3 _ p.205

[ f Script ]

Professor (female): You know, there’s a lot of interesting

buildings going up around the world these days ... and

I’m not just talking about the super skyscrapers that

currently seem to be all the rage. Uh, look at a lot of the

museums that have been built in the last decade or so ...

the Guggenheim in Bilbao, the ... the Museum of Modern

Art in New York ... these buildings have designs that

could rightly be called revolutionary. Uh, actually, they

should be called deconstructionist, and that’s what we’re

going to talk about today - an architectural movement

called deconstruction. OK, first, let’s use the two examples

that I mentioned ... the Guggenheim and the Museum of

Modern Art. What do these two buildings have in common?

Student A (male): Uh, they don’t really look like buildings.

Professor: OK, be a little more specific please.

Student A: Well, a building is usually ... you know, like

four walls and a roof. But these buildings ... their angles

are all off, and they have weird curves and things.

2.

Professor (male): You all are two for two today. That’s

right. As we mentioned in the last class, the earth’s crust

is made of plates which are always in motion, and it’s this

motion that causes earthquakes and volcanoes. Alaska,

Indonesia, and Japan all happen to lie on areas where

two of the earth’s plates are colliding. Actually, they are

just part of a line that geologists call the Pacific Ring of

Fire.

[ f Script ]

1.

Professor (male): OK, I got a little trivia question for you

all. Ready? What do Alaska, Japan, and Indonesia all

have in common?

Student A (female): Uh, well I don’t know about Alaska,

but Indonesia and Japan both have a lot of earthquakes.

Student B (male): Yeah, that’s it. Alaska is a big

earthquake region, too. Uh, the whole northwestern

coast of North America is, actually.

Professor: That’s good. Uh, and not only are these areas

of high earthquake activity, they’re also areas with a lot

of volcanic activity. That’s actually not too surprising

because, as we’ll see, earthquakes and volcanoes often

go hand-in-hand. OK, next question. Anyone know why

these areas have lots of earthquakes and volcanoes?

Professor: Uh, actually, no. That one was due to another

tectonic plate that’s not actually part of the Ring of Fire.

But many of the worst volcanoes and earthquakes over

the last several thousand years have been due to plate

movement in this region.

Alska, Jpn, Indnsa, many earthqkes & volcns

movmnt in earth’s crustÙ earthqks. + volcns

RRiinngg ooff FFiirree

shape from W. coast S. Am., to E coast Asia =

40,000Km

has many tectonic plates colliding

90% of earthqks + 75% of volcns

U

Page 28: Insider: The Super Guide Scripts & Answer Key

A28 Insider: The Super Guide

1. (A) – Although the concepts in answers B, C, and D are all

mentioned in the discussion, they are only discussed in

relation to the deconstructionist movement.

2. (A) – The professor says that deconstructionist architects

wish to break away from traditional architecture. Answer A

is basically a paraphrase of this idea.

3. (C) – The professor explains that deconstructionist

architecture and the cubist and expressionist movements

in painting were all attempts to deconstruct traditions in

their fields.

Building Your Listening Skills _ p.206

1. (B) – The main purpose of this section of the discussion is

to introduce the concept of deconstructionist architecture.

2. (C) – This section is primarily concerned with defining

deconstructionist architecture by explaining its most

important characteristics.

3. This section is not planned. This section, which explains

why it took so long for deconstruction to develop in

architecture, is in response to a student question.

[ f Script ]

1.

Professor (female): You know, there’s a lot of interesting

buildings going up around the world these days … and

I’m not just talking about the super skyscrapers that

currently seem to be all the rage. Uh, look at a lot of the

museums that have been built in the last decade or so …

the Guggenheim in Bilbao, the ... the Museum of Modern

Art in New York … these buildings have designs that

could rightly be called revolutionary. Uh, actually, they

should be called deconstructionist, and that’s what we’re

going to talk about today – an architectural movement

called deconstruction. OK, first, let’s use the two

examples that I mentioned … the Guggenheim and the

Museum of Modern Art. What do these two buildings

have in common?

2.

Professor (female): Good, you’ve both brought up two

important points about the deconstructionist movement.

First, deconstructionist buildings aren’t supposed to look

like normal buildings. The architects in this movement ...

uh, well, they’re trying to “deconstruct” architectural

traditions. They’re trying to do away with the conventions

of the past to make room for something completely new.

That’s what deconstruction essentially means … kind of

totally separating from or smashing the traditions of the

past. Uh, and that brings me to the second point.

Deconstructionist architecture often does resemble

modern art because it has been heavily influenced by

artistic movements, uh, particularly cubism and

expressionism.

3.

Professor (female): Good question, and the answer is

basically technology. Up until a decade or so ago, this

kind of design work in architecture just wouldn’t have

been possible. It takes really advanced engineering

techniques, computer simulation and design analysis, not

to mention cutting edge building materials, to make a

building like the Guggenheim structurally sound. You just

couldn’t have built these structures at the turn of the

century, even if you had thought up the design.

Student B (male): Yeah, they look more like some kind

of modern art sculpture than a normal building.

Professor: Good, you’ve both brought up two important

points about the deconstructionist movement. First,

deconstructionist buildings aren’t supposed to look like

normal buildings. The architects in this movement ... uh,

well, they’re trying to “deconstruct” architectural

traditions. They’re trying to do away with the conventions

of the past to make room for something completely new.

That’s what deconstruction essentially means ... kind of

totally separating from or smashing the traditions of the

past. Uh, and that brings me to the second point.

Deconstructionist architecture often does resemble

modern art because it has been heavily influenced by

artistic movements, uh, particularly cubism and

expressionism. You know, actually, those two artistic

movements were also deconstructionist movements.

They attempted to deconstruct artistic traditions in the

same way that architects are doing now.

Student B: But cubism and expressionism are both from

the beginning of the 20th century. Why’s it taken

architecture so long to catch up?

Professor: Good question, and the answer is basically

technology. Up until a decade or so ago, this kind of

design work in architecture just wouldn’t have been

possible. It takes really advanced engineering

techniques, computer simulation and design analysis, not

to mention cutting edge building materials, to make a

building like the Guggenheim structurally sound. You just

couldn’t have built these structures at the turn of the

century, even if you had thought up the design.

Guggenheim, Musm. of Mod. Art= deconstructionist blds.

Deconstruction

● don’t look like bldgs, look like mod. sculptures

● strange angles and curves

purpose:

● Break with archict. traditions, create new style

● like cubism and expressionism in art

Not possible before b/c no comptrs. or advncd. materials

Page 29: Insider: The Super Guide Scripts & Answer Key

LIS

TE

NIN

G

Scripts & Answer Key A29

| Vocabulary Review | _ p.207

1-1. configured 1-2. safeguard

1-3. precautions 1-4. hoarding

1-5. manipulation

2-1. been all the rage

. Although the band used to be very popular, they are

not anymore.

2-2. was off

. As soon as Jake entered the house he knew

something was wrong

2-3. goes hand-in-hand

. If you ask me, hard work is essential for success

| iBT Practice | _ p.207~209

QQuueessttiioonn �� –– RReeppllaayy

f PPrrooffeessssoorr: Uh� from an� uh … aesthetic point of view italso breaks up the performance into manageablesegments for the audience� A full ballet all in one gowould probably be a little much for the average person�

QQuueessttiioonn �� –– RReeppllaayy

f PPrrooffeessssoorr ((ffeemmaallee)): Ballet is an art form that combinesand synthesizes elements from three other arts:literature� music� and dance� It …

SSttuuddeenntt AA ((mmaallee)): (interrupting) Uh� excuse me�professor� I get the music and the dance part� butwhere does literature come into play in ballet?

f SSttuuddeenntt AA ((mmaallee)): … but where does literature comeinto play in ballet?

acts, and each act is separated by an intermission. Uh,

the intermission serves a number of purposes. Number

one, it provides the time needed to switch the sets,

which, as I said, are often quite elaborate. Uh, from an,

uh ... aesthetic point of view it also breaks up the

performance into manageable segments for the

audience. A full ballet all in one go would probably be a

little much for the average person.

In terms of choreography, there are a couple distinctive

features of narrative ballet. The first is called a

divertissement. This is a short departure from the telling

of the story in order to showcase the technical skills of a

particular dancer. Uh, for those of you familiar with

opera, it would be relatively similar to an aria, that’s a

solo, by the way, for the lead female voice. Uh, the other

characteristic element of narrative ballet in terms of

choreography would be the pas de deux. This is quite

simply a duet ... a dual dance by the leading man and

the leading lady of the ballet. Since nearly every

narrative ballet revolves around a love story there are

naturally a lot of these.

OK, to wrap it up, let’s talk about stories in some of the

most famous narrative ballets. Swan Lake would be at

the top of that list. In this ballet, a young prince falls in

love with a princess. Unfortunately, the princess is turned

into a swan every morning as the result of the spell of an

evil wizard. In various versions, the power of the young

couple’s love either breaks the spell, or they die together.

The point I want to make about the story though is that,

as I said earlier, it is taken from pre-existing myths. The

swan maiden is a prominent figure in both Greek and

Persian mythology. Another famous narrative ballet,

Beauty and the Beast, has its roots in a fairy tale that all

of you should recognize.

[ f Script ]

Professor (female): Ballet is an art form that combines

and synthesizes elements from three other arts: literature,

music, and dance. It ...

Student A (male): (interrupting) Uh, excuse me professor,

I get the music and the dance part, but where does

literature come into play in ballet?

Professor: A good question. Can anyone answer it?

Student B (female): Well, most ballets tell a story, don’t

they?

Student A: So they’re like musicals?

Professor: Uh, not exactly. OK, perhaps I should clarify a

bit before we move on. In a musical, the dancers sing,

and often even act, as well as dance. In ballet, the story

is told entirely through the choreography and the music

... there’s no singing, no lyrics, and definitely no acting.

Student A: Sounds like it’d be pretty hard to follow the

story to me.

Professor: Well, actually, most people already know the

story before they see a ballet. You see, many of the plots

of ballets are taken from popular myths or from well-

known fairy tales ... as I’m sure you’ll see in a bit when I

start naming a couple famous ballets.

OK, I should point out, by the way, that what we’re really

talking about today is called narrative ballet. There are

forms of ballet that don’t tell a cohesive story, but the

large majority performed are narrative ballets. So, let’s

talk about some of the major characteristics of narrative

ballets. Uh, number one, they’re typically pretty high

budget performances. They have large, elaborate sets,

full costumes, and often require a large number of

dancers. Other forms of ballet, in contrast, are far more

economical to produce. Another defining characteristic of

narrative ballet is that it will have at least two separate

Page 30: Insider: The Super Guide Scripts & Answer Key

A30 Insider: The Super Guide

1. Main Idea Question – (A)

Answers B, C, and D are just individual features of

narrative ballet mentioned in the discussion.

2. Supporting Detail Question – (B)

This is clearly stated in the discussion.

3. Organization Question – (C)

The professor discusses two examples of techniques used

in narrative ballet (the divertissement and the pas de deux)

and explains why they are used.

4. Organization/Rhetorical Purpose Question – (B)

The professor’s discussion of Swan Lake primarily involves

a summary of the plot and an explanation of where that

plot comes from.

5. Fuction Meaning Question – (B)

The key is to consider this statement with the student’s

earlier statement that he gets (understands) how music

and dance influence ballet. The contrast implied by the

student’s use of the word “but” should give you an

important clue concerning the answer.

6. Content-Linking Question – (B)

You can infer this from the fact that professor says that

breaking up the ballet into acts puts it into sections that

are manageable for most people.

Listening for Total Comprehension _ p.210

1. American companies have had problems in attracting

qualified applicants for engineering and management

positions.

2. Globalization has led to this problem because applicants

have more options concerning where they work.

3. The phrase “its best and brightest” refers to the nation’s

best and brightest people.

[ f Script ]

Professor: In recent years, many US corporations have

experienced significant difficulties in attracting qualified

personnel, especially in the fields of engineering and

management. As the world becomes more globalized,

individuals with such skills have employment options in

any number of nations, and many are choosing to work

and live outside of the US. This problem is commonly

known as “brain drain,” a phenomenon in which a nation

loses its best and brightest.

As a result, many top companies are looking for

innovative new ways to attract talent. Aside from the

traditional methods of offering handsome salaries and

bonus packages, many companies are now trying to offer

improvements in the lifestyle of their most talented

employees. This includes greater flexibility in work

schedule, longer vacations, and enhanced retirement

benefits. It is only through such efforts that companies

can now hope to compete in the global marketplace.

In recent years, many US corporations have experienced

significant difficulties in attracting qualified

personnel, especially in the fields of engineering and

management. As the world becomes more globalized,

individuals with such skills have employment options

in any number of nations, and many are choosing to

work and live outside of the US. This problem is

commonly known as “brain drain,” a phenomenon in

which a nation loses its best and brightest.

As a result, many top companies are looking for

innovative new ways to attract talent. Aside from the

traditional methods of offering handsome salaries and

bonus packages, many companies are now trying to

offer improvements in the lifestyle of their most talented

employees. This includes greater flexibility in work

schedule, longer vacations, and enhanced retirement

benefits. It is only through such efforts that companies

can now hope to compete in the global marketplace.

Page 31: Insider: The Super Guide Scripts & Answer Key

LIS

TE

NIN

G

Scripts & Answer Key A31

Supporting Detail Questions

Basic Drills _ p.214

1. Turbulence is an important keyword from the lecture.

According to the lecture, turbulence is the shaking of an

aircraft when the aircraft travels through rough air.

Important points about turbulence include the following

facts: it can be detected by radar but not totally avoided,

and it can cause injuries to passengers.

2. Disaster Relief:

Search + rescue genrlly main focus

Look for people in rubble

largest # of deaths from diseases caused by no

sanitation

sewers don’t work, and no garbage pickup

focus should be on disease prevention + sanitation

bottled water very cheap

3. (B) – This a paraphrase of the following statement from the

lecture: Uh, basically, gold companies restrict theavailability of gold to keep the price high.

Listening Practice 1 _ p.215

[ f Script ]

Student (female): Hi, excuse me. Facilities department,

right?

Man: Uh huh, what can I do for you?

Student: Well, my RA told me that we had to come down

here and put in a work order if we need something fixed.

Man: Yeah, that’s right. I’ll get you a work order form in

just a second. First, what seems to be the problem?

Student: It’s the air conditioner in my dorm room. It turns

on, but it doesn’t seem to do much good. I mean, I run it

at full blast, and my room is still hot.

Man: Hmmm ... yeah, that sounds like the coolant in your

unit needs to be recharged. We got guys going around to

all the dorms this week doing that. Normally, we try to

charge them before the students get here at the

beginning of the semester, but with all the construction

that went on over the summer, we just didn’t get to it.

years. In fact, gold prices are the highest they have been

in decades. Let’s talk about some of the reasons for that.

Uh, as with any commodity, what ultimately determines

the price of gold is the law of supply and demand. If the

demand is high and the supply is low, then prices will

rise. The catch is that while the demand for gold

fluctuates, the supply isn’t really so variable. The total

amount of new gold mined every year is just a tiny

fraction of the gold available on the world market. Most of

the available gold has already been mined, and is being

hoarded. Uh, basically, gold companies restrict the

availability of gold to keep the price high.

[ f Script ]

1.

Professor (male): One of the key factors aircraft

designers have to consider is how their aircraft will deal

with turbulence. At certain altitudes, where the air is

frequently disturbed, or during bad weather, turbulence

can cause severe buffeting of an aircraft. Although radar

can help an aircraft to identify and dodge areas of

turbulence, rough patches of air cannot be completely

avoided. In addition to the characteristic shaking that

most of us associate with it, turbulence can also cause

sudden fluctuations in altitude. While these rarely pose

any danger to the aircraft itself, they can throw

passengers about quite easily. It is therefore essential

that aircraft are designed to encourage passengers to

remain in their seats with their seatbelts fastened.

2.

Professor (female): OK, after any natural disaster, uh,

whether it’s a hurricane, an earthquake, a flood ... uh,

after any of these, people tend to focus on the search

and rescue portion of the relief effort. And, certainly,

search and rescue is important … if people are still alive

and trapped under rubble or whatever, then, of course,

we want to find them. But sometimes, I feel like the

heavy focus we place on search and rescue takes

attention from what is really the most vital aspect of

disaster relief – disease prevention.

Uh, maybe you don’t know it, but the spread of easily

preventable diseases is usually the number one killer in a

natural disaster. More people die of illnesses like cholera

and dysentery than from injuries sustained in the disaster

itself. The reason is that natural disasters often create

ideal conditions for the spread of such diseases. Uh,

there’s usually a lack of clean drinking water, and basic

sanitation measures like working sewers or garbage

pickup are often unavailable. So then, a major part of the

relief effort obviously needs to be focused on disease

prevention. That means providing clean drinking water,

ensuring a reliable supply of medications to treat

diseases, uh, and basically trying to provide people with

at least semi-sanitary conditions in which to live.

Sometimes something as simple as a bottle of clean

water, which might cost an aid agency maybe 50 cents,

can mean the difference between life and death.

3.

Professor (male): As you may or may not know, the price

of gold has been steadily increasing over the last several

Chapter 2

Page 32: Insider: The Super Guide Scripts & Answer Key

A32 Insider: The Super Guide

1. (B) – The woman clearly states that she has come to

submit a work order, and later in the conversation, she

specifies what the work order is for.

2. (C) – The man clearly states that because of the building

projects on campus during the summer, the workers were

too busy to service the air conditioners.

3. (D) – The man clearly states that the workers are

scheduled to come to the student’s dormitory next

Wednesday.

Building Your Listening Skills _ p.216

1. This is the 2. put in a

3. get you 4. seem to do

5. went on

Listening Practice 2 _ p.217

occurs. In the wild, hybridization is extremely rare. Uh, it

only happens when you have two closely related species

that have overlapping territories. Uh, so, for example, if

you have wild donkeys and wild horses living in the same

area, there is a chance that they will interbreed. But even

in those situations, chances are less than 1 in 1,000 that

any interbreeding will occur.

Student: So then why are there so many mules?

Professor: Well, that gets us to the second manner in

which hybridization can occur. It is frequently induced by

humans for our own purposes. This is especially true of

plant hybrids, uh, which are a bit easier to create than

animal hybrids. But sticking with the example of donkeys,

the reason there are so many is that we purposely breed

them on farms. Now why would we do that? The answer

is that mules are often better as work animals than either

donkeys or horses. They, uh … they kind of combine the

best qualities of the horse and the donkey. This is what is

known as hybrid vigor. Often, when you create hybrids,

you create offspring that are more robust than either

individual parent.

Now, there are drawbacks. Uh, first, as many of you

probably know, mules are sterile … they can’t produce

their own offspring. That’s because of the mismatch in

their chromosomes. You see, a horse has 64

chromosomes, while a donkey has 62, producing a mule

with 63. But when mules mate, the chromosomes don’t

match up correctly, resulting in their sterility. Uh, another

drawback is that hybrids are not always better than their

parents. Uh, for example, hybrid butterflies often lack the

protective coloration of either parent, and therefore make

easier prey for predators.

[ f Script ]

Professor (male): Hybridization is the process through

which two different species interbreed to create offspring

with a mix of the genetic qualities of both parents. Can

anyone give me an example of either a hybrid animal or

a hybrid plant?

Student (female): Mules are hybrids … they’re a mix of a

horse and a donkey.

Professor: OK, actually mules will be a good example of

many of the aspects of hybridization that we’ll talk about

today. First, let’s talk about how and when hybridization

Sorry about that. What dorm do you live in?

Student: Dorsett Hall.

Man: OK ... just let me look ... yeah, they’re scheduled to

do your dorm next Wednesday.

Student: So I got to wait until then? You do know it’s like

90 degrees outside, right?

Man: Yeah, I know. Listen, I’m really sorry. If I could

speed it up, I would, but we’re working as fast as we can.

Student: OK, I guess I’ll just have to roast until

Wednesday then.

Stdnt has to fill out work order

● needs air con. fixed

a. air con doesn’t make room cooler

Man thinks air con needs coolant to be rechrgd.

● not done yet b/c too much constrctn in summer - too

busy

● workers will come next Weds.

Hybridization = two species interbreed

exmp: horse + donkey = mule

In wild happens when similar species have very close

territories

● very rare, 1 in 1,000

Humans force animals to interbreed b/c hybrids stronger

exmpl: mule has best qualities of horse & donkey

Drawbacks

● Offspring often sterile

● May lack natural protectns of parents

Page 33: Insider: The Super Guide Scripts & Answer Key

LIS

TE

NIN

G

Scripts & Answer Key A33

1. (C) – You should be able to use the notes to help you

determine the main topic of the discussion. The first part

clearly describes the ways that hybridization can occur

(causes), and the second part discusses the advantages

and disadvantages (results).

2. � The presence of large animal populations�� Territories that merge into one another�� The presence of two genetically similar species� Genetically unique parents that can create hybrids

This is clearly stated in the discussion when the professor

says that hybridization occurs between closely related

species with overlapping territories.

3. (B) – This is clearly stated in the discussion.

Building Your Listening Skills _ p.218

1. Statement from the discussion: In the wild, hybridization

is extremely rare. Uh, it only happens when you have two

closely related species that have overlapping territories.

Uh, so, for example, if you have wild donkeys and wild

horses living in the same area, there is a chance that

they will interbreed. But even in those situations, chances

are less than 1 in 1,000 that any interbreeding will occur.

The paraphrased sentence is basically just a summary of

this section of the discussion. The clause “even when all

conditions are met” is a general reference to the specific

requirements mentioned in the discussion.

2. Statement from the discussion: Uh, for example, hybrid

butterflies often lack the protective coloration of either

parent, and therefore make easier prey for predators.

The paraphrase is essentially a generalization drawn from

the information in this specific example. The phrase

“adaptations that protect their parents” refers to the

protective coloration of butterflies mentioned in the

discussion.

Listening Practice 3 _ p.219

[ f Script ]

Professor (female): Chinese porcelain is perhaps one of

the most well-known and widely acclaimed forms of

pottery. There is a wide range of Chinese porcelain,

dating to various Chinese dynastic periods. Each period

has its own unique features. Yet, the materials used, and

the basic methods employed in the manufacture of

Chinese porcelain, largely remained the same throughout

all of these periods.

Let’s begin with the materials used in Chinese porcelain.

Most Chinese porcelain is a mixture of two materials:

porcelain stone and Chinese clay. The defining

characteristic of both materials is that they have

microscopic plates, and when worked correctly, these

plates align to form an incredibly smooth surface. In most

cases, a mixture of porcelain stone and Chinese clay

was used to form the body of a piece. Uh, the ratio of

these two materials varied depending on both the period

and the region in which the piece was produced.

Anyway, once the potter had molded the clay, it would be

left to dry. Then a glaze made of various minerals would

be added for color. Again, the piece would be left to dry,

and finally, it would be fired in a kiln. During the firing

process, the glaze and the clay would fuse together.

Now, the Chinese distinguished between what they

called high-fired and low-fired porcelain. High-fired

porcelain, found in the south, was fired in kilns with

temperatures higher than 1,300°C and typically had a

greater portion of Chinese clay in its mixture. Low-fired

porcelain, found in the north, was generally fired at

around 1,250°C and had a greater portion of porcelain

stone in its mixture.

The Chinese used two types of kilns to fire their

porcelain. The first was a dragon kiln. Dragon kilns were

typically used in southern China. They are extremely

long structures ... uh, usually around 100 feet in length.

Because of their huge size, they were built into the sides

of hills. In the north, large egg-shaped kilns were more

common. The egg-shaped kilns allowed for a faster firing

process than dragon kilns, but temperatures were more

variable at different locations within the kiln.

[ f Script ]

1.

Professor (male): OK, actually mules will be a good

example of many of the aspects of hybridization that we’ll

talk about today. First, let’s talk about how and when

hybridization occurs. In the wild, hybridization is

extremely rare. Uh, it only happens when you have two

closely related species that have overlapping territories.

Uh, so, for example, if you have wild donkeys and wild

horses living in the same area, there is a chance that

they will interbreed. But even in those situations, chances

are less than 1 in 1,000 that any interbreeding will occur.

2.

Professor (male): Now, there are drawbacks. Uh, first, as

many of you probably know, mules are sterile ... they

can’t produce their own offspring. That’s because of the

mismatch in their chromosomes. You see, a horse has

64 chromosomes, while a donkey has 62, producing a

mule with 63. But when mules mate, the chromosomes

don’t match up correctly, resulting in their sterility. Uh,

another drawback is that hybrids are not always better

than their parents. Uh, for example, hybrid butterflies

often lack the protective coloration of either parent, and

therefore make easier prey for predators.

Page 34: Insider: The Super Guide Scripts & Answer Key

A34 Insider: The Super Guide

1. (C) – Although B is discussed in the lecture, it is not the

main topic, and A and D are never discussed in the

lecture.

2. (B) – This is clearly stated in the lecture.

3. � Glazed in bright, vivid colors�� Fired at lower temperatures�� Use less Chinese clay� Use porcelain stone exclusively

The color of the porcelain of different regions is never

discussed, and the lecture explicitly states that all porcelain

used a mixture of porcelain stone and Chinese clay.

Building Your Listening Skills _ p.220

1. (B) – The rest of the lecture is concerned with describing

how Chinese porcelain is produced.

2. (A) – This section describes the process of shaping,

glazing, and firing porcelain.

3. This section is primarily concerned with describing the

differences in the materials and firing methods used in

making porcelain in the northern and southern parts of

China.

| Vocabulary Review | _ p.221

1-1. microscopic 1-2. rubble

1-3. sustain 1-4. robust

1-5. overlapped 1-6. fluctuation

2-1. catch

. The offer sounded too good to be true, so Hank

knew there must be some conditions he did not know

about.

2-2. at full blast

. My father always turns up the heat very high, even

on relatively warm days.

2-3. stick with it

. Marcy was sure that she would never be able to

learn Spanish, but Jason advised her to keep trying.

3.

Professor (female): Now, the Chinese distinguished

between what they called high-fired and low-fired

porcelain. High-fired porcelain, found in the south, was

fired in kilns with temperatures higher than 1,300°C and

typically had a greater portion of Chinese clay in its

mixture. Low-fired porcelain, found in the north, was

generally fired at around 1,250°C and had a greater

portion of porcelain stone in its mixture.

The Chinese used two types of kilns to fire their

porcelain. The first was a dragon kiln. Dragon kilns were

typically used in southern China. They are extremely

long structures … uh, usually around 100 feet in length.

Because of their huge size, they were built into the sides

of hills. In the north, large egg-shaped kilns were more

common. The egg-shaped kilns allowed for a faster firing

process than dragon kilns, but temperatures were more

variable at different locations within the kiln.

[ f Script ]

1.

Professor (female): Chinese porcelain is perhaps one of

the most well-known and widely acclaimed forms of

pottery. There is a wide range of Chinese porcelain,

dating to various Chinese dynastic periods. Each period

has its own unique features. Yet, the materials used, and

the basic methods employed in the manufacture of

Chinese porcelain, largely remained the same

throughout all of these periods.

2.

Professor (female): Uh, the ratio of these two materials

varied depending on both the period and the region in

which the piece was produced. Anyway, once the potter

had molded the clay, it would be left to dry. Then a glaze

made of various minerals would be added for color.

Again, the piece would be left to dry, and finally, it would

be fired in a kiln. During the firing process, the glaze and

the clay would fuse together.

Porcelain = famous form of Chinese pottery

Materials = Chinese clay and porcln stone

● both have tiny plates which make very smooth

pprroocceessss: 1) make body & let dry

2) brush on glaze & let dry

3) fire in kiln at temp of 1250 - 3000 C

KKiillnnss = ovens used to fire porcln

● dragon kilns

a. built into hill b/c of huge size

b. very high temp, used more Chin. clay, found in

south

● Egg-shaped shaped kilns

a. used more porcln stone, fired at lower temp, found

in north

b. faster but heat uneven

0

Page 35: Insider: The Super Guide Scripts & Answer Key

LIS

TE

NIN

G

Scripts & Answer Key A35

| iBT Practice | _ p.222~223

1. Main Idea Question – (B)

The student is not doing well in his chemistry class and is

considering dropping the course. He talks to the advisor to

learn what the procedure for dropping the course is.

2. Supporting Detail Question – (A)

The student says he has taken more courses than he

should have. He now realizes this is a mistake. He knows,

in order to do well in chemistry, he will have to study hard,

but now he doesn’t have the time.

3. Supporting Detail Question – (D)

Dropping a course must be done earlier than withdrawal. It

requires no signature from the professor and will not be

registered on the student’s permanent record. Withdrawal

will be accompanied by some kind of grade.

4. Content-Linking Question(Prediction) – (D)

It seems most likely that the student will drop the course.

To do this, the student must fill out a form. The advisor

reminds him to do this, and the student says that he’s on

his way. In other words, he’s going directly from the

advisor’s office to get the form.

5. Function Meaning Question – (C)

The advisor knows the student doesn’t want to withdraw

from the course with a bad grade and have it go on his

permanent record. So, she emphasizes this by stating it in

the form of a tag question.

QQuueessttiioonn �� –– RReeppllaayy

f AAccaaddeemmiicc AAddvviissoorr: Well� yeah� But if you wait untilthen� you’ll get a grade for the course� It’s no longerconsidered dropping� but withdrawing from a course� Ifyou’re passing and withdraw you’ll get a “W�” but if youhappen to be failing� and from what you’ve said this is areal possibility� your professor will probably give you an“F” or at best a “U�” This will also go on your permanentrecord� I doubt if you want to take that chance� doyou?

f AAccaaddeemmiicc AAddvviissoorr: This will also go on your permanentrecord� I doubt if you want to take that chance� doyou?

[ f Script ]

Academic Advisor (female): Hi, come on in. What can I

do for you?

Student (male): (hesitating) Well, I’m thinking of dropping

one of my courses, and I was wondering if you could tell

me exactly what I have to do.

Advisor: Sure, but why don’t you tell me what the

problem is first?

Student: It’s my chemistry class. I knew I probably

shouldn’t have taken it this term because of all the other

courses I’m taking, but I thought that I could get it out of

the way. But I really don’t have much idea about what’s

happening, and I don’t have the time to work on it as

much as I’d like. I thought I might as well drop it and take

it when I don’t have so many other classes.

Advisor: Have you had any tests yet?

Student: Just a few quizzes, but midterms are coming up

and I can’t risk it. I mean, I did badly on these quizzes,

and I can’t see myself improving much by midterms.

Advisor: In that case, you’d better hurry and drop the

course. I think you’ve only got a couple of days to do it.

Student: (surprised) A couple of days? But I thought I

had until four weeks before the end of the term.

Advisor: Well, yeah. But if you wait until then, you’ll get a

grade for the course. It’s no longer considered dropping,

but withdrawing from a course. If you’re passing and

withdraw you’ll get a “W,” but if you happen to be failing,

and from what you’ve said this is a real possibility, your

professor will probably give you an “F” or at best a “U.”

This will also go on your permanent record. I doubt if you

want to take that chance, do you?

Student: No way. So, okay, I want to drop the course in

the next two days. What do I have to do? Get some kind

of a form and have the professor sign it or what?

Advisor: No, you just need to fill out the form and turn it

in. There’s no need to get the professor’s signature

unless you miss the deadline.

Student: Great. Well, I’m sure glad I stopped in here. I

could have ended up in real trouble otherwise.

Advisor: Well, just make sure you get that form filled out

as soon as possible.

Student: (leaving) Thanks. I’m on my way now.

Page 36: Insider: The Super Guide Scripts & Answer Key

Listening for Total Comprehension _ p.2241. Critics say that inefficiency and a lack of innovation are

inherent parts of government space programs because

of the size of their bureaucracies.

2. Space tourism first started when the super-rich were able

to pay for a seat on a rocket in a government funded

space program.

[ f Script ]

Professor (female): For much of its history, space travel

and exploration has been the exclusive domain of a

select few of the most advanced nations in the world.

Space programs have traditionally been huge,

government-run operations with staggering budgets and

vast bureaucracies. Many critics have charged that their

size made inefficiency and a lack of innovation an

inherent part of government-run space agencies. Yet,

due to the complexity of space travel and the tight

restrictions on space related technologies, there seemed

to be little alternative.

In recent years, however, the first signs of corporate

space programs have begun to emerge. Space tourism,

in which the ultra wealthy could pay for a seat on a

governmentally funded rocket, opened the doors for

other private, for-profit ventures. There are now several

companies seeking to build their own manned rockets

with the express purpose of marketing trips into space as

a sort of exotic vacation for those with the money to pay

for such adventures. Whether these corporations will be

any more efficient or innovative in their operations than

governments have been remains to be seen, but it is

certain that space travel has entered a new era.

A36 Insider: The Super Guide

For much of its history, space travel and exploration has

been the exclusive domain of a select few of the most

advanced nations in the world. Space programs have

traditionally been huge, government-run operations

with staggering budgets and vast bureaucracies. Many

critics have charged that their size made inefficiency

and a lack of innovation an inherent part of

government-run space agencies. Yet, due to the

complexity of space travel and the tight restrictions on

space related technologies, there seemed to be little

alternative.

In recent years, however, the first signs of corporate

space programs have begun to emerge. Space tourism,

in which the ultra wealthy could pay for a seat on a

governmentally funded rocket, opened the doors for

other private, for-profit ventures. There are now several

companies seeking to build their own manned rockets

with the express purpose of marketing trips into space as

a sort of exotic vacation for those with the money to pay

for such adventures. Whether these corporations will be

any more efficient or innovative in their operations

than governments have been remains to be seen, but it

is certain that space travel has entered a new era.

Page 37: Insider: The Super Guide Scripts & Answer Key

Scripts & Answer Key A37

LIS

TE

NIN

G

Content-Linking Questions

Basic Drills _ p.228

1-1. (C) – The implication is that the team lost because their

pitcher was injured.

1-2. (A) – The implication is that Jake’s car problems are the

reason for his lateness.

1-3. (A) – There are actually two implications in this dialog.

The first is that Harry does not arrive at work by 9 pm.

The second implication is that since he is the boss’s son,

he does not have to follow the rules.

2. Fact 1: Jim has gained weight recently

Fact 2: Jim knows this, and is upset about it

Fact 3: Jim has recently started to go to the gym

Fact 4: Jim only bought fruit and vegetables at the

supermarket.

Fact 5: Jim is worried because it will be summer soon,

and he does not want to go to the beach looking as

he does.

Conclusion: The clear implication of all this is that Jim

is trying to lose weight.

3. (B) – There are a couple clues that help make this

inference. First, we know that the woman is upset because

the president has not kept his promises from the last

election. Second, the woman says that she won’t make

the mistake of voting for the president again clearly

implying that she will vote against him.

Listening Practice 1 _ p.229

[ f Script ]

Student (female): Hi, are you Professor Wilkins?

Professor (male): In the flesh. What can I do for you?

Student: Uh, I’m doing my schedule for next semester,

and I wanted to get a little more information about your

History 225 course, you know, African-Americans in the

post-Civil War era.

Professor: Oh, sure. What can I tell you about it?

Student: Basically, I wanted to know what the basic

focus would be. Will it be cultural life? Political?

Economic?

Professor: Well, we’ll be touching on all those areas, but

the primary focus is on cultural and family life ... uh, the

other stuff we really only look at in terms of how it

influenced cultural and family life.

Student: OK, that sounds like just what I was looking for.

Know how many seats are still open in the class?

Professor: I haven’t checked today, but I’m sure you can

still get in. But I should tell you there’s a lot of reading

and your grade is going to be based primarily on three

exams ... there aren’t really any papers or anything to

pad you if you don’t do well on those.

[ f Script ]

1-1.

Man: The team’s star pitcher went out of the game with a

shoulder injury after the second inning. The team ended

up losing the game by five runs.

1-2.

Woman: Jake was late to work for the third time this

week today. He’s been having some problems with his

car.

1-3.

Woman: Aren’t all company employees supposed to

start work at 9?

Man: Well, Harry’s the boss’ son. What do you expect?

2.

Man: Hey, Carry. I just saw Jim at the supermarket. Man,

he looks terrible. It looks like he’s put on quite a few

pounds since the last time I saw him.

Woman: Yeah, I know. But look, don’t say anything to

him about it ... he’s upset about it enough as it is. But on

the bright side, he started going to the gym last week. He

says he plans to go and do an hour of jogging after work

each day.

Man: Oh, I guess that explains why his shopping cart

was filled with nothing but fruit and vegetables when I

saw him, huh?

Woman: (laughing) Yeah, that’s part of it, too, I guess.

He’s all worried because summer’s just around the

corner, and he doesn’t want to go to the beach looking

like that.

3.

Woman: Man! I can’t believe this! He did it to us again!

Man: Huh? What’s going on?

Woman: Oh, it’s the President again. He’s raising taxes.

That’s like the third tax hike since he’s been in office. He

is such a snake in the grass. You remember during the

last campaign ... he was all like “I will not burden the

American people with further taxes.” You remember

that?

Man: (laughing) I do, but I guess it slipped his mind.

Woman: It’s not funny. That was basically the only

reason I voted for him. Well, we’ll see if I make that

mistake again.

Chapter 3

Page 38: Insider: The Super Guide Scripts & Answer Key

A38 Insider: The Super Guide

1. (B) – The student clearly states that she needs some

information about the professor’s History 225 course

before she registers for it.

2. (C) – The professor implies this by mentioning the amount

of reading required by the course and the fact that there

are no essays to help bring up the student’s grade if she

does not do well on the exams.

3. (D) – The professor says that he will check the student’s

record tonight and give her an answer tomorrow, clearly

implying that she should return to speak with him again

tomorrow.

Building Your Listening Skills _ p.230

1. are you 2. about your

3. Will it be 4. Will it be

5. life 6. Will it be

7. life 8. Do you

9. to pad you

Listening Practice 2 _ p.231

[ f Script ]

Professor (female): Today, we’re going to talk about the

history of a film genre that I’m pretty sure most of you are

familiar with. Who’s heard of a B-movie before? Can

anyone explain that term for us?

Student A (male): It’s like a cheap, poorly made movie.

Student B (female): (in a correcting tone) It not just any

movie. B-movies are low budget movies in the horror or

science fiction genres ... and the fact that they’re low

budget is part of their appeal.

Professor: Well, that would be the connotation that “B-

movie” carries today. Today, the term applies to low-

budget horror and sci-fi films. As to whether their low

production quality is part of their appeal or not ... well,

that just depends on who you ask. What may surprise

you is that this modern connotation is not at all the

meaning that the term B-movie carried back when it first

came into usage in the 1930s and 40s.

So, in the 30s and 40s the Hollywood system was a bit

different. Today, you have actors who are basically

freelancers. They work for a number of studios and

production companies. But in the 30s and 40s, actors

and actresses signed multi-year contracts with a single

studio. Studios kept what they called, uh, “stables” of

actors and actresses. Uh, generally there were two of

these “stables” ... the A-actors and the B-actors. The A-

actors were featured in the blockbuster films of the

studio. The B-actors ... well, think of them as kind of like

a minor league team in baseball. They starred in lesser

movies in the hope that they would get noticed, and

move up into the A-stable.

At the time, most movies were shown as double features.

Uh, you would have a popular, high production quality

movie, and then a lower production quality movie, starring

the B actors, after that. Anyway, those second movies in

the double features came to be known as B-movies.

The other point I wanted to make is that at the time, they

weren’t all horror films and sci-fi. There were a lot of

cowboy films, uh, gangster flicks ... uh, basically anything

that would kind of get the crowd’s hearts racing. So at

the time when the term was first coined, B-movies were

simply lower quality movies with second rate actors and

low production budgets. They, uh, they weren’t an entire

genre unto themselves as they are today.

QQuueessttiioonn �� –– RReeppllaayy

f PPrrooffeessssoorr: But I should tell you there’s a lot of readingand your grade is going to be based primarily on threeexams … there aren’t really any papers or anything topad you if you don’t do well on those�

Student: Oh, that’s OK. I was planning on taking the

class as an audit class anyway. I’m not really worried

about the credits. I’m just interested in the subject.

Professor: I see. You know that you have to get my

approval to audit the course, right?

Student: Really? No, I didn’t. Is that going to be a

problem?

Professor: It shouldn’t be. I’ll go over your student

records tonight and give you an answer in the morning,

OK?

Stdnt needs info re: course b/c thinking of registering

● specifically needs info on focus of class

Prof answer: focus will be cultural life

● course reqmtns: lots of reading, 3 exams, no essays

or papers to help

Stdnt no worried b/c wants to audit course but needs

permission from prof.

will get answer after prof. checks stdnts record

Page 39: Insider: The Super Guide Scripts & Answer Key

LIS

TE

NIN

G

Scripts & Answer Key A39

1. (A) – This is implied when the professor states that the

appeal of a B-movie’s low production value depends on

who you ask.

2. (B) – This is implied when the professor says that,

originally, B-movies could be any kind of film that would

get the audiences’ hearts racing.

Building Your Listening Skills _ p.232

1. Statement in the discussion: As to whether their low

production quality is part of their appeal or not … well,

that just depends on who you ask.

The phrase “that just depends on who you ask” is

commonly used in English to indicate that something is not

always true, or is a matter of opinion.

2. Statement in the discussion: The B actors … well, think

of them as kind of like a minor league team in baseball.

They starred in lesser movies in the hope that they would

get noticed, and move up into the A stable.

The comparison of B actors to members of a minor

baseball team is intended to show that they were not top

quality actors.

Listening Practice 3 _ p.233

[ f Script ]

Professor (female): OK, I want all of you to take close

notes as we talk about this next section, because it is an

essential concept in engineering. We are going to talk

about the factor of safety that is designed into a product.

The factor of safety is the amount by which a product will

exceed its design specifications before failing. Uh ... it’s

like the “over engineering” that is built into a product. Uh,

let’s take an example real quick. Let’s say that you have

a crane, and the maximum lifting load of that crane is

10,000 kg ... and let’s say that the cable on that crane is

designed to hold 40,000kg. Then, the cable has a safety

factor of four. It’s designed to hold four times more

weight than the crane is designed to lift. That means that

the cable is pretty unlikely to fail.

Student A (male): Isn’t that kind of expensive? I mean, I

would assume that the better cable is going to cost more,

but you’re paying for a cable whose full capabilities

you’re never going to use.

Professor: True, but look at it this way. What would

happen if you used a cable with a safety factor of 1?

That would mean it could only hold what the crane was

designed to lift ... and let’s say you’re lifting a maximum

load one day. Suddenly there’s a strong wind, and that

exerts more pressure on the cable than it is designed for.

Well then the cable breaks, 10,000 kg come crashing

down ... maybe on top of someone. You get the idea? A

lot of times it pays to be safe.

Student B (female): So is everything designed with a

safety factor of four?

Professor: The acceptable safety factor is primarily

determined by the consequences of an engineering

failure. If failure could lead to loss of life, safety factors

are usually pretty high, maybe four or above. In cases

[ f Script ]

1.

Professor (female): Well, that would be the connotation

that “B-movie” carries today. Today the term applies to

low-budget horror and sci-fi films. As to whether their low

production quality is part of their appeal or not … well,

that just depends on who you ask. What may surprise

you is that this modern connotation is not at all the

meaning that the term B-movie carried back when it first

came into usage in the 1930s and 40s.

2.

Professor (female): Studios kept what they called, uh,

“stables” of actors and actresses. Uh, generally there

were two of these “stables” … the A-actors and the B-

actors. The A-actors were featured in the blockbuster

films of the studio. The B-actors … well, think of them as

kind of like a minor league team in baseball. They starred

in lesser movies in the hope that they would get noticed,

and move up into the A-stable.

QQuueessttiioonn �� –– RReeppllaayy

f PPrrooffeessssoorr: Well that would be the connotation that “B�movie” carries today� Today the term applies to low�budget horror and sci�fi films� As to whether their lowproduction quality is part of their appeal or not … wellthat just depends on who you ask�

BB--mmoovviieess

● today’s meaning: horror & sci-fi films w/low prodctn.

value

HHiissttoorryy ooff tteerrmm:

● studios in 30s and 40s used two kinds of actors A

actors, and B actors

1. A actors starred in headliner films

2. B actors starred in lower quality films which were

shown after films w/A actors

Movies w/B actors were known as B movies

Not just horror films, also cowboy films, gangster films

Characterized by lots of action to get audience excited

Page 40: Insider: The Super Guide Scripts & Answer Key

A40 Insider: The Super Guide

1. (B) – This can be inferred by the student’s question about

why a high safety factor is necessary and the professor’s

answer that it often pays to be safe.

2. (C) – This is clearly stated in the discussion.

3. (A) – At the end of this section of the discussion, the

professor mentions that there are some exceptions to the

rule of when high safety factors are used, implying this will

be the next topic of discussion.

Building Your Listening Skills _ p.234

1. (A) – This section of the discussion is primarily concernedwith explaining the concept of a factor of safety.

2. (B) – The student has asked the teacher why you would

pay for capabilities that you will never use. The professor

justifies the use of high safety factors by explaining the

possible negative effects of not doing so.

3. This section of the discussion explains how engineers

determine what factor of safety to use. The professor

explains this in response to a student’s question.

| Vocabulary Review | _ p.235

1-1. exert 1-2. burden

1-3. genre 1-4. specifications

1-5. connotation

2-1. brought up

. Jessica would have never known we went to the

movies without her if Mike hadn’t mentioned it during

dinner.

2-2. is just around the corner

. Although Christmas is very soon, I haven’t done a bit

of shopping.

2-3. is a snake in the grass

. While Diane’s mother likes her new boyfriend, her

father doesn’t trust him.

Professor (female): True, but look at it this way. What

would happen if you used a cable with a safety factor of

1? That would mean it could only hold what the crane

was designed to lift ... and let’s say you’re lifting a

maximum load one day. Suddenly there’s a strong wind,

and that exerts more pressure on the cable than it is

designed for. Well then the cable breaks, 10,000 kg

come crashing down ... maybe on top of someone. You

get the idea? A lot of times it pays to be safe.

3.

Student (female): So is everything designed with a

safety factor of 4?

Professor (female): The acceptable safety factor is

primarily determined by the consequences of an

engineering failure. If failure could lead to loss of life,

safety factors are usually pretty high, maybe four or

above. In cases where a failure would not result in

serious damage or loss of life, lower safety factors are

often acceptable. There are, however, a few exceptions

to this rule.

[ f Script ]

1.

Professor (female): OK, I want all of you to take close

notes as we talk about this next section, because it is an

essential concept in engineering. We are going to talk

about the factor of safety that is designed into a product.

The factor of safety is the amount by which a product will

exceed its design specifications before failing. Uh … it’s

like the “over engineering” that is built into a product. Uh,

let’s take an example real quick. Let’s say that you have

a crane, and the maximum lifting load of that crane is

10,000 kg … and let’s say that the cable on that crane is

designed to hold 40,000kg. Then, the cable has a safety

factor of four. It’s designed to hold four times more

weight than the crane is designed to lift. That means that

the cable is pretty unlikely to fail.

2.

Student (male): Isn’t that kind of expensive? I mean, I

would assume that the better cable is going to cost more,

but you’re paying for a cable whose full capabilities

you’re never going to use.

where a failure would not result in serious damage or

loss of life, lower safety factors are often acceptable.

There are, however, a few exceptions to this rule.

Imprtnt engineering concpt = factor of safety

● Def. = amount something will exceed its design

specs.

exmp: if crane lifes 10,000kg, and cable holds

40,000kg, then factor is 4

Why use high factor of safety?

● answ: helps avoid unexpected accidents (ex. crane

cable breaks, hurts or kills people)

● high factor of safety used when failure may cause

death

● lower factor of safety used when failure will not

cause death or much damage

Page 41: Insider: The Super Guide Scripts & Answer Key

LIS

TE

NIN

G

Scripts & Answer Key A41

| iBT Practice | _ p.236~237

1. Main Idea Question – (D)

B is mentioned, but it is only an example used in the

discussion, not the main idea. C incorrectly relates ideas

from the discussion, and A is never mentioned in the

discussion.

2. Supporting Detail Question� The analysis of anatomic changes in human ancestors�� The analysis of the distribution of culture-specific

artifacts�� The analysis of specific genetic mutations in the gene

pool�� The analysis of linguistic evolution� The analysis of ancient historical records

QQuueessttiioonn �� –– RReeppllaayy

f PPrrooffeessssoorr: But over the centuries� uh� because theywere kind of isolated� their language grew apart fromthe Korean that is spoken on the mainland� So now theKorean spoken on Jeju is a bit different from mainlandKorean� They use a lot of words that aren’t used on themainland� Now� if the Koreans on Jeju would have beenisolated for long enough� say another thousand yearsor so� their Korean would have evolved into an entirelynew language�

our mother. So, for example, I could track my Y

chromosome all the way back through many generations,

from my father, to his father, to his father, etc. You can

do the same with your mitochondrial DNA and your

mother’s ancestry.

OK, now, occasionally, the Y chromosome or the

mitochondrial DNA will mutate, and uh, when it does …

it’s a fairly big event because that mutation will be

passed down through the generations after that … so

mutations kind of represent places in the family tree

where groups separated. Anyway, by tracking these

mutations we can follow the migration of genetically

distinct groups across the world. Let me think of an easy

example … Oh, OK. There is a specific Y chromosome

mutation called the M 20 mutation that occurred in India

about 30,000 years ago. Now, anyone in the world who

has this specific mutation … uh, well, we can be sure

that at some point in the last 30,000 years, their male

ancestors migrated from India. There’s another mutation

that arose in Siberia around 20,000 years ago called the

M 45 mutation. Now, if someone had both the M 20 and

the M 45 mutation, we could start to track the movement

of their ancestors … because their ancestors would have

had to move out of India and into Siberia so that they

could pick up the second mutation. See how that works?

So by tracking the sequences of these mutations in

people, we can come up with a kind of broad map of

human migration over thousands of years.

[ f Script ]

Professor (female): OK, today, we’re going to continue

talking about the migration patterns of early humans.

Now, we’ve discussed this concept in some detail

already, so let’s just review what we’ve said so far. What

are some of the ways that we can trace the migration

patterns of early humans?

Student A (male): Well, most early human cultures had

their own distinct artifacts. They had their own styles of

weapons, tools, pottery … uh, that kind of stuff. So you

can trace the spread of a certain tool style, and that

shows you the migration of a culture. Uh, like the

migration of the early Native Americans is shown through

the spread of a particular kind of spear point.

Professor: Good, looks like you all were paying attention

after all. What else?

Student B (female): Uh, I know you said something

about tracking languages, but I didn’t really get it.

Professor: Yeah, that one’s a bit complex. Basically, you

track the way that languages change, and how new

languages form. If you have two groups that speak the

same language, but are geographically isolated, over

time, their languages will grow apart. Uh, here’s a good

example. There’s a Korean island called Jeju that is

pretty far from the mainland. Now, when that island was

first settled, the people who lived there spoke just like the

other Koreans of their time. But over the centuries, uh,

because they were kind of isolated, their language grew

apart from the Korean that is spoken on the mainland.

So, now the Korean spoken on Jeju is a bit different from

mainland Korean. They use a lot of words that aren’t

used on the mainland. Now, if the Koreans on Jeju would

have been isolated for long enough, say another

thousand years or so, their Korean would have evolved

into an entirely new language. Anyway, by tracking those

kinds of language changes, we can track the migration of

early populations.

OK, so that’s a quick review of what we’ve gone over so

far. Now, what I want to do is talk about a relatively new,

and in, uh, in my opinion, quite exciting method of

tracking human migration. In the past ten years or so, it

has become possible to use DNA to track the migration

of early humans.

Student B: Sorry, Professor Schmidt? How’s that

possible? I thought everyone’s DNA was unique.

Professor: Uh, it is … but parts of our DNA are inherited

directly from our parents … uh, basically without any real

change. For example, we get our Y chromosome directly

from our father, and our mitochondrial DNA directly from

Page 42: Insider: The Super Guide Scripts & Answer Key

A42 Insider: The Super Guide

3. Supporting Detail Question – (C)

The professor clearly states that these genetic markers are

useful because we inherit them directly from our parents. A

is incorrect because the professor also states that they do

undergo mutations on rare occasions, and that these

mutations are important in tracking human migration.

4. Organization Question – (C)

The professor’s discussions of tracking artifacts and the

changes in languages are intended for review only. The

only new method covered in the discussion is the tracking

of DNA.

5. Stance/Attitude Question – (B)

This is clearly implied when the professor states that, in her

opinion, the use of DNA is “quite exciting.”

6. Content-Linking Question – (C)

This is suggested by the time scale that the professor

gives regarding how long it would have taken the language

on Jeju Island to evolve into a language that was entirely

different from Korean.

Listening for Total Comprehension _ p.238

1. They are significant because blue crabs are an important

species for the local economy.

2. They are harmful because they increase algae blooms,

which, in turn, kill other aquatic species.

[ f Script ]

Professor (male): Agricultural activities can be quite

harmful to the environment if not carefully controlled.

While much attention is paid to obviously destructive

agricultural practices, such as slash-and-burn farming,

even seemingly innocuous activities can have far

reaching and unforeseen effects.

Take, for instance, chicken farming in the Chesapeake

Bay area of the United States. The Chesapeake is one of

the nation’s greatest aquatic resources, teeming with

fish, clams, and crabs. Yet, in recent years, populations

of fish and shellfish in the Chesapeake, especially those

of the blue crab, one of the region’s most economically

important species, have fallen drastically. The primary

cause of these population declines has been algae

blooms. These algae blooms are in turn caused by water

run-off from chicken farms situated too closely to the

shores of the Chesapeake. The run-off from these farms

carries high concentrations of the nitrates and sulfates on

which the algae thrive, causing the huge blooms that kill

fish across the region. Thus, even a seemingly harmless

agricultural activity can have devastating effects on the

local environment if not properly controlled.

Agricultural activities can be quite harmful to the

environment if not carefully controlled. While much

attention is paid to obviously destructive agricultural

practices, such as slash-and-burn farming, even

seemingly innocuous activities can have far reaching

and unforeseen effects.

Take, for instance, chicken farming in the Chesapeake

Bay area of the United States. The Chesapeake is one of

the nation’s greatest aquatic resources, teeming with

fish, clams, and crabs. Yet, in recent years, populations

of fish and shellfish in the Chesapeake, especially that

of the blue crab, one of the region’s most economically

important species, have fallen drastically. The primary

cause of these population declines has been algae

blooms. These algae blooms are in turn caused by water

run-off from chicken farms situated too closely to the

shores of the Chesapeake. The run-off from these farms

carries high concentrations of the nitrates and sulfates

on which the algae thrive, causing the huge blooms that

kill fish across the region. Thus, even a seemingly

harmless agricultural activity can have devastating

effects on the local environment if not properly controlled.

Page 43: Insider: The Super Guide Scripts & Answer Key

LIS

TE

NIN

G

Scripts & Answer Key A43

Organization / RhetoricalConnection Questions

Basic Drills _ p.242

1. ● broken home is where normal family relationships

don’t exist

● father in prison (example)

● parents beat child (example)

● difficulty forming relationships (result)

● child learns not to trust anyone (explanation)

● child may be afraid to allow others to get emotionally

close to them (explanation)

2. Stdnt: probs w/ group. Work is too sloppy.

Ex. don’t check results or proofread.

Stdn wants prof. to talk to group.

Prof. doesn’t think this is good idea.

Stdnt. doesn’t understand why

Prof: When sdnt was doing teaching child was

unpopular b/c never solved his own problems. Prof

thinks the situation is similar

The example given by the professor is the student’s

experience during her student teaching. The student had a

child in her class who was unpopular because he always

told on the other children. The professor’s point is that the

student’s situation is a similar one, and therefore she

should try to handle the problem on her own.

Listening Practice 1 _ p.243

[ f Script ]

Clerk (male): Hi, can I help you?

Student (female): Yeah, I’m here about a problem with

my tuition payment. I got a letter in the mail the other day

that said I had an outstanding balance, and that I

wouldn’t be able to register for classes. But I sent in a

check in the middle of the summer.

Clerk: OK, let’s just take a look to see what the computer

says ... uh ... student ID number?

Student: 555-34-1298.

Clerk: Hmmm. Alright, here we go, Susan Johnson. OK,

I show that you paid $1,500 on July 16th, leaving you

with an outstanding balance of $2,000.

Student: The $2,000 is my student loan. I just pay the

$1,500. But why hasn’t my student loan portion been paid?

Student: He was a tattletale. He was always telling on

the other kids and getting them in trouble.

Professor: Right, basically he was unpopular because he

relied on an authority figure to solve his problems for

him. So, yeah, I could talk to your other group members,

and make them work a bit more carefully. But in the long

run, it would probably just cause more problems for you.

It’s probably better if you try to work this out on your own.

[ f Script ]

1.

Professor (female): Today, we are going to talk about the

effects that growing up in a broken home can have on a

young child’s mental state. Uh ... first let’s define what

exactly we mean by a broken home. Basically, we’re

talking about a family in which the family relationships

which we consider “normal” and “healthy” either don’t exist

or don’t function properly. Uh, maybe the father has been

arrested, and is serving a prison sentence, so there’s no

positive male role model in the house. Or maybe the

parents beat the child. Those are just two examples, but I

think you get the idea.

So anyway, what effects does this have on a growing

child? In general, these children have severe difficulties in

entering into relationships where they are required to trust

somebody. Experience has taught them that they cannot

count on their parents for proper emotional support, and

let’s face it, if you can’t trust your parents, who can you

trust? So what happens is that because these kids don’t

feel like they can trust anyone, they just avoid getting to

close to anyone. They may have acquaintances, but they

will probably avoid becoming “best friends” with anyone

because they don’t want to take the risk of placing their

trust in that person.

2.

Student (female): Excuse me, Professor Thomas? Can I

speak with you?

Professor (male): Sure, Lisa. What can I do for you?

Student: I’m having trouble with the rest of my group.

We’re supposed to be doing this project together, but

their work is so sloppy. I mean they don’t check their

results ... they don’t proofread their papers ... and every

time I talk to them about it, they just get all mad and

offended. Could you talk to them?

Professor: Well, if it’s absolutely necessary, I can ... but

it’s probably not the best solution.

Student: Why not? They have to listen to you. They don’t

have to listen to anything I say.

Professor: Well, that may be true, but look at it this way.

When you were doing your student teaching last

semester you told me there was one kid who was really

unpopular, right? Why was that?

Chapter 4

Page 44: Insider: The Super Guide Scripts & Answer Key

A44 Insider: The Super Guide

1. (B) – The student goes to the registrar’s office because

she received a letter saying her tuition had not been paid.

However, she has already sent in a check for her tuition,

so she does not understand why she got the letter.

2. (A) – The fact that the man says this just after the woman

asks why her student loan has not been paid indicates that

his response is an answer to her question. Although B and

C seem like potentially logical answers, they both ask you

to assume too much.

3. (B) – This woman says this right after the man has told her

that she should have received a letter about her student loan.

The clear implication is that she has not received the letter.

Building Your Listening Skills _ p.244

1. Let’s just 2. What is your

3. that is all 4. long is it

5. Could you 6. could have

7. would not have

Listening Practice 2 _ p.245

QQuueessttiioonn �� –– RReeppllaayy

f PPrrooffeessssoorr: It portrays modern war as a brutal�dehumanizing experience� as shown when WilfredOwens� one of the greatest of the World War I poets�wrote “What passing�bells for these who die as cattle?Only the monstrous anger of the guns�” Obviously thisis a far cry from earlier periods when men such asEmerson claimed that “War educates the senses� callsinto action the will� perfects the physical constitution�brings men into such swift and close collision in criticalmoments that man measures man�”

[ f Script ]

Professor (female): While literature about the experience

of war is certainly not unique to World War I, the First

World War does stand out as a unique period in the

genre of war literature, both for its sheer volume and for

the unique perspective it offered on the horrors of war.

Previous eras had tended to glorify the experience of war

… uh, stressing concepts such as the brotherhood of

soldiers. Even the American Civil War, which was

shocking for its brutality and carnage, produced its fair

share of highly romanticized accounts of the battlefield

experience. The literature of World War I, however, is

almost universally disillusioned, cynical, and fatalistic. It

portrays modern war as a brutal, dehumanizing

experience, as shown when Wilfred Owens, one of the

greatest of the World War I poets, wrote “What passing-

bells for those who die as cattle? Only the monstrous

anger of the guns.” Obviously this is a far cry from earlier

periods when men such as Emerson claimed that “War

educates the senses, calls into action the will, perfects the

physical constitution, brings men into such swift and close

collision in critical moments that man measures man.”

So, World War I definitely shed a new, more pessimistic

light on the war experience, but it was also remarkable

for the sheer number of soldiers who felt moved to relate

their experiences in the trenches through either verse or

prose. This, uh … may have been at least partially due to

the very nature of warfare during World War I. That era

marked the height of trench warfare. Millions of soldiers

found themselves stuck in the trenches for weeks or

months at a time, with little to do but wait for and

contemplate the horror of the next artillery barrage or the

order to attack. Under such circumstances, it can hardly

seem surprising that so many soldiers sought a diversion

through writing. Perhaps more importantly, however, the

young soldiers of World War I felt betrayed and

abandoned by their leaders and their societies. Many

had a great need to express their feelings of outrage and

betrayal, and literature was a natural outlet.

Clerk: Uh, that’s what I’m checking now. (slight pause) It

looks like you were late in turning in your application last

semester. That’s not a big deal, so don’t worry. It just

means that your loan’s going to take a bit longer to

process, that’s all. You should have gotten a letter about

that.

Student: I changed addresses, and haven’t had much

luck getting my mail forwarded. So how long is it going to

take to process my loan?

Clerk: Uh, it might be a couple of weeks.

Student: Registration is next week.

Clerk: Yeah, I know. Could you pay the balance, and then,

when your loan arrives it would just be paid directly to you?

Student: (annoyed) If I could have paid the full tuition, I

wouldn’t have taken out a loan.

Clerk: Look, I’m sorry. There’s not much I can do. You

can’t register without full payment.

Student: Great. I guess I’ll have to go ask my parents to

help me out. I’ll be back.

ssttnnddttss.. pprroobb: got letter re: outstanding tuition bill but

already sent in check

ccllrrkk: stndt. paid $1,500 but still owes $2,000

ssttnnddtt: $2,000 is paid by student loan

ccllrrkk: not paid b/c stdnt turned in appl. late, will take

several weeks

ssttnnddtt: cant wait b/c registration is next week

ccllrrkk: stdnt has to pay tuition in full before registration

ssttnnddtt: will ask parents for money

Page 45: Insider: The Super Guide Scripts & Answer Key

LIS

TE

NIN

G

Scripts & Answer Key A45

1. (A) – There are several clues to this question. The first is

that this quote comes at a stage in the lecture when the

professor is describing the pessimism of WWI literature. In

addition, it directly follows a quote from a famous WWI

poet. Finally, the professor describes Emerson’s views as

a “far cry” from those of Owens, setting up a contrasting

relationship.

2. (C) – The professor’s mention of the soldiers’ feelings of

betrayal comes at a point when he is explaining the various

reasons for why WWI soldiers wrote so much about their

experiences.

Building Your Listening Skills _ p.246

1. Statement from the lecture: While literature about the

experience of war is certainly not unique to World War I,

the First World War does stand out as a unique period in

the genre of war literature, both for its sheer volume and

for the unique perspective it offered on the horrors of war.

This is a fairly direct paraphrase of the original statement in

the lecture. A “unique perspective” describes a perspective

that is “drastically different” and the phrase “sheer volume”

in the original statement means that these soldiers wrote a

great deal.

2. Statement from the lecture: Millions of soldiers found

themselves stuck in the trenches for weeks or months at

a time, with little to do but wait for and contemplate the

horror of the next artillery barrage or the order to attack.

Under such circumstances, it can hardly seem surprising

that so many soldiers sought a diversion through writing.

The paraphrased statement basically summarizes the

information contained in this section of the lecture. The

“need to kill time” refers to the idea that soldiers had to

wait in the trenches for many weeks with nothing to do.

Listening Practice 3 _ p.247

[ f Script ]

Professor (female): So far, we have been talking about

the challenges faced by societies that have no single

culture that is clearly dominant. In doing so, we have

largely focused on societies where the various cultures

have failed to coexist peacefully. Now, I want to look at

some success stories, uh, places where two distinct

cultural groups have been able to coexist, and see if we

can prise out the secrets to their success. Can anyone

think of an example for me?

Student A: Canada. The French-speaking Canadians

and the English-speaking Canadians are pretty distinct

cultural groups.

Student B: Or South Africa. They have English-speaking

whites and Afrikaans-speaking whites.

Professor: Uh that’s true. But let’s use Canada as an

example. The South African example is a bit more

complex. So Canada does indeed have two distinct

cultures: an English-speaking culture and a French-

speaking culture.

Student B: What about the Inuit? They have a distinct

culture.

Professor: Well, the Inuit, like most other Native

American groups, have been marginalized and

suppressed. So, if we’re going to talk about Canada as a

successful balancing of cultures, we’re going to have to

overlook the Inuit for right now. Besides, the Inuit make

up a very tiny proportion of the Canadian population. The

two dominant cultures are clearly the English-speaking

and the French-speaking cultures. Approximately 7

[ f Script ]

1.

Professor (female): While literature about the experience

of war is certainly not unique to World War I, the First

World War does stand out as a unique period in the

genre of war literature, both for its sheer volume and for

the unique perspective it offered on the horrors of war.

Previous eras had tended to glorify the experience of war

... uh, stressing concepts such as the brotherhood of

soldiers.

2.

Professor (female): This, uh ... may have been at least

partially due to the very nature of warfare during World

War I. That era marked the height of trench warfare.

Millions of soldiers found themselves stuck in the

trenches for weeks or months at a time, with little to do

but wait for and contemplate the horror of the next

artillery barrage or the order to attack. Under such

circumstances, it can hardly seem surprising that so

many soldiers sought a diversion through writing.

WWI lit. unique b/c unique viewpoint on war + large

amount written

● earlier war lit. romanticized war experience

CChhaarraacctteerriissttiiccss ooff WWWWII lliitt:

● cynical, disillusioned view of war

● showed war as brutal, dehumanizing

a. exmpl: quote by Owen (very negative) contrasted

with quote by Emerson (positive)

MMaannyy ssoollddiieerrss wwrroottee.. WWhhyy??

● Nature of warfare in WWI. Soldiers had little to do

but wait in trenches for attack

● Soldiers wanted to express their feelings of betrayal

Page 46: Insider: The Super Guide Scripts & Answer Key

A46 Insider: The Super Guide

1. (B) – The professor’s response to the student’s comment

gives an important clue to this question, since the

professor basically justifies her reasons for excluding the

Inuit from her discussion.

2. (C) – The professor says that language differences are not

the only differences between French and English speaking

Canadians. She then mentions the political views of

Canadians as an example of another difference between

these two groups.

Building Your Listening Skills _ p.248

1. (C) – Although the professor does briefly mention what

has been discussed previously, she does not do this in any

detail. Therefore the main purpose of this section is not to

review information. The professor finishes this section by

stating what she wishes to discuss in the following

sections of the discussion.

[ f Script ]

1.

Professor (female): So far, we have been talking about

the challenges faced by societies that have no single

culture that is clearly dominant. In doing so, we have

largely focused on societies where the various cultures

have failed to coexist peacefully. Now, I want to look at

some success stories, uh, places where two distinct

cultural groups have been able to coexist, and see if we

can prise out the secrets to their success.

2.

Professor (female): Well, the Inuit, like most other Native

American groups, have been marginalized and

suppressed. So, if we’re going to talk about Canada as a

successful balancing of cultures, we’re going to have to

overlook the Inuit for right now. Besides, the Inuit make

up a very tiny proportion of the Canadian population. The

two dominant cultures are clearly the English-speaking

and the French-speaking cultures. Approximately 7

million French speakers live in Canada, 6 million of those

in the province of Quebec. The rest of the nation is

almost entirely made up of English speakers. Now,

language is not the only difference between English-

speaking and French-speaking Canadians. They, uh,

they generally have quite different value systems,

political views, and views on the direction the nation

should take.

3.

Student (male): So, if they’re so different, how is it a

success story?

Professor (female): Because even with these

considerable cultural differences, Canada has managed

to remain a single united nation with relatively little

conflict or resentment between the two cultures. Now,

they have done that partly because the cultures are

located in basically distinct geographic areas, with the

majority of French-speaking Canadians in Quebec.

Canada has also given a relatively high amount of

autonomy to its provinces, so the people of Quebec don’t

feel oppressed or put upon by the larger English-

speaking culture. Canada has also gone to considerable

lengths to make sure that each culture is recognized as

an integral and valued part of the Canadian national

identity.

million French speakers live in Canada, 6 million of those

in the province of Quebec. The rest of the nation is

almost entirely made up of English speakers. Now,

language is not the only difference between English-

speaking and French-speaking Canadians. They, uh,

they generally have quite different value systems,

political views, and views on the direction the nation

should take.

Student B: So, if they’re so different, how is it a success

story?

Professor: Because even with these considerable

cultural differences, Canada has managed to remain a

single united nation with relatively little conflict or

resentment between the two cultures. Now, they have

done that partly because the cultures are located in

basically distinct geographic areas, with the majority of

French-speaking Canadians in Quebec. Canada has

also given a relatively high amount of autonomy to its

provinces, so the people of Quebec don’t feel oppressed

or put upon by the larger English-speaking culture.

Canada has also gone to considerable lengths to make

sure that each culture is recognized as an integral and

valued part of the Canadian national identity.

will look at soct. w/ divided societies that are successful

exmpls: Canada + South Africa

Canada:

● 2 major cltrs.: French-speaking + English-speaking

● most French speakers in Quebec

● each cltr. has distinct values, and political views

LLiittttllee ccoonnffllcctt.. bb//cc:

● each culture lives in different area

● Canada gives lots of autonomy to provinces

● Canada stresses the importance of each culture

Page 47: Insider: The Super Guide Scripts & Answer Key

LIS

TE

NIN

G

Scripts & Answer Key A47

2. (B) – While the professor does respond to the student’s

question by stating that the Inuit have been marginalized in

Canadian society, she is not justifying this marginalization.

She is only explaining why she is not including them in her

discussion (because it would complicate her example). The

majority of this section of the discussion is dedicated to

describing the two largest parts of Canadian society,

English-speaking Canadians and French-speaking

Canadians.

3. This section of the discussion is probably the most

important part. At the beginning of the discussion, the

professor said that she wanted to look at some success

stories to try to find out why they were successful. That is

what she does in this section of the discussion. She

explains why Canadian society has been able to deal

with the divisions between English speakers and French

speakers.

| Vocabulary Review | _ p.249

1-1. fatalistic 1-2. marginalized

1-3. outstanding 1-4. acquaintance

2-1. shed new light on

. After the crash, investigators searched the crash site

hoping to find something that would help them

understand the cause of the crash.

2-2. work out

. After several hours of negotiations, Kelly was able to

agree upon a new work arrangement with her boss.

2-3. in the long run

. His investments seem to make sense now, will they

be profitable over a long period of time?

2-4. is a far cry from

. Listening to a CD of an opera at home is not nearly

as good as seeing one in person.

| iBT Practice | _ p.250~251

One of the reasons that the Dead Sea scrolls are so

important, aside from the obvious reason of their

antiquity, is that they come from a unique period in the

history of the Hebrews. In the second century B.C. the

Hebrews retook the territory around Jerusalem from the

Syrians, and reconstituted the kingdom of Jerusalem.

This is commonly known as the Second Temple Period

because they rebuilt the Temple of Solomon, the holiest

site in the Hebrew religion, at this time. So, politically, it

was an interesting time because it is one of the few times

in their early history when the Hebrews achieved an

independent state. From a cultural and religious

standpoint, it was an interesting time because there were

a number of different Hebrew sects at this time, each

with significantly different beliefs. One of these sects …

uh, actually this sect was just a small minority … a fringe

group really, was the Essenes.

The Essenes were an ascetic community that lived apart

from the rest of the Hebrew community. Their main

community, Khirbet Qumran, was located in the same

area as the caves where the Dead Sea scrolls were

found. Now, while the Dead Sea scrolls almost certainly

belonged to the Essenes, there is considerable debate

about whether the Essenes simply collected the scrolls

from other Hebrew sects, you know kind of like an early

Jewish library, or, uh, whether they wrote them all

themselves. What seems most likely, and remember I

said that there is still a good deal of debate going on, is

that the scrolls containing the actual books of the Bible

were collected from various sects, while the scrolls

dealing with community life seem to have mostly been

written by the Essenes themselves. Uh, I say this

because the Essenes had pretty unique beliefs among

the Hebrews … and uh, many of the Dead Sea scrolls

espouse beliefs that they would not have agreed with.

So, assuming that this view is correct, and many of the

scrolls were not written by the Essenes themselves, how

did they come into possession of them? Again, there are

a number of theories about this. Uh, one theory is that as

an ascetic, uh … sort of monkish, community, the

Essenes were simply heavily involved in biblical

scholarship and study … and, this is certainly a plausible

theory. But a theory that I find to be more interesting, and

a bit more compelling as well, is that the Essenes were

given the scrolls for safe keeping. You see, by the time of

the birth of Christ, the Hebrew kingdom had been

absorbed by the Roman Empire. Now, that wasn’t a

peaceful absorption. There were periodic revolts against

Roman power … and after one of these revolts in the

year 66 A.D. there was a particularly harsh crackdown

against the Hebrews by the Romans. So, perhaps

fearing the destruction of their religious texts by the

[ f Script ]

Professor (female): One of the most significant finds in

the history of modern archaeology … uh, especially for

those archaeologists interested in the history of the

Hebrews, was the discovery of the Dead Sea scrolls.

These scrolls contain some of the earliest known copies

of the Old Testament of the Bible, which those of the

Jewish faith call the Torah, and regard as the

authoritative word of God. Uh, most of the scrolls date

from somewhere between 200 B.C. and 100 B.C. Now,

those of you familiar with Jewish history obviously know

that the Old Testament certainly predates this, but again,

these are the oldest surviving copies found.

Page 48: Insider: The Super Guide Scripts & Answer Key

A48 Insider: The Super Guide

1. Main Idea Question – (B)

While the Essenes are an important part of the lecture,

they are not the main topic. Therefore A and D are

incorrect. The effects of the Dead Sea Scrolls on biblical

study are not discussed in detail.

2. Supporting Detail Question�� They are the earliest known biblical writings.� They authenticate many biblical stories.�� They date from a key point in Jewish history.� They caused Judaism to split into different sects.

The professor states that the Dead Sea Scrolls are the

earliest surviving biblical writings. The professor also states

that many of the Dead Sea Scrolls date from a period

when the Hebrews regained their independence and

rebuilt their temple.

3. Organization/Rhetorical Connection Question – (C)

The professor says that it is unlikely that the Essenes wrote

all of the Dead Sea Scrolls because many of the scrolls

contain religious beliefs that the Essenes would not have

agreed with.

4. Stance/Attitude Question – (B)

The professor states that she finds this theory to be more

interesting and more compelling. Furthermore, she gives

quite a bit of evidence to support this theory.

5. Supporting Detail Question – (C)

The professor mentions that the Essenes were geogra-

phically isolated while explaining the theory that the other

sects gave their sacred texts to the Essenes, suggesting

that their isolation was part of the reason for this.

6. Function Purpose Question – (C)

The professor has mentioned earlier in the lecture that there

are multiple theories on the origins of the Dead Sea Scrolls.

She simply wants to remind the students of this before

discussing one particular theory. The top position seems to

be frequently challenged rather than seldom challenged.

Listening for Total Comprehension _ p.252

1. The two important characteristics of European nobles

were that they were warriors, and therefore lived violent

lives, and that they were extremely religious and were

terrified of the possibility of going to hell.

2. These characteristics gave the Church power over the

nobles because they relied on the Church to forgive

them for committing acts of violence that were required

of them by their society.

[ f Script ]

Professor (male): After the collapse of the Roman

Empire, Western Europe was faced with a near complete

absence of the rule of law. Land was divided up between

rival warlords, and there was little or no centralized

authority. Considering the chaotic nature of the times, it

might seem natural that the ruling class of Europe was,

in fact, a warrior class. The nobility of Europe were highly

trained and hardened warriors, and for them, violence

was a way of life.

Yet, there was a central contradiction that lay at the heart

of this warrior society. While the European aristocracy

was engaged in near constant warfare, it was also an

extremely religious group. Like all Christians of the time,

these nobles lived in perpetual terror that their sins might

condemn them to hell in the afterlife, yet the very nature

of their society compelled them to commit acts of

violence as a matter of course. It was this constant need

to be forgiven for their sins that gave the Church such

power over the nobles of Europe.

QQuueessttiioonn �� –– RReeppllaayy

f PPrrooffeessssoorr: What seems most likely� and remember Isaid that there is still a good deal of debate going on� isthat the scrolls containing the actual books of the Biblewere collected from various sects� while the scrollsdealing with community life seem to have mostly beenwritten by the Essenes themselves�

f PPrrooffeessssoorr: … and remember I said that there is still agood deal of debate going on …

Romans, the various Hebrew sects delivered their sacred

texts to the Essenes, who were somewhat geographically

isolated, for safe keeping. The fact that the scrolls were

found in caves around Khirbet Qumran, and not within

the settlement itself, seems to suggest pretty powerfully

that the Essenes felt they needed to be hidden.

After the collapse of the Roman Empire, Western Europe

was faced with a near complete absence of the rule of law.

Land was divided up between rival warlords, and there

was little or no centralized authority. Considering the

chaotic nature of the times, it might seem natural that

the ruling class of Europe was, in fact, a warrior class.

The nobility of Europe were highly trained and hardened

warriors, and for them, violence was a way of life.

Yet, there was a central contradiction that lay at the

heart of this warrior society. While the European

aristocracy was engaged in near constant warfare, it

was also an extremely religious group. Like all

Christians of the time, these nobles lived in perpetual

terror that their sins might condemn them to hell in the

afterlife. Yet the very nature of their society compelled

them to commit acts of violence as a matter of course. It

was this constant need to be forgiven for their sins that

gave the Church such power over the nobles of Europe.

Page 49: Insider: The Super Guide Scripts & Answer Key

LIS

TE

NIN

G

Scripts & Answer Key A49

Organization Questions

Basic Drills _ p.256

1-1. (B) – The last statement by the professor shows that a

cause and effect pattern will be used.

1-2. (A) – The professor states that many things must happen

in a particular order, implying that he will describe this

sequence.

1-3. (B) – The professor states that the millions of different

kinds of insects can be categorized by a few

characteristics, implying that she will now discuss those

categories.

2. (B)

Bacteria - simple single cell orgs.

Most # on Earth

No complex internal structures

Usually parasitic (harm host) or symbiotic (benefit

host)

Body has millions of bacteria - some essential

EX. bacteria in stomach help digest food

Bacteria in soil recycle nutrients

Some bacteria create toxins

Toxins cause many diseases

The first part of the lecture defines the term bacteria, and

the second and third parts of the lecture categorize

bacteria into those that are helpful and those that are

harmful. The professor indicates that he will use this

categorizational structure when he states that bacteria can

either be symbiotic or parasitic.

Listening Practice 1 _ p.257

[ f Script ]

Student (male): Professor Dalton, could I take up a few

minutes of your time?

Professor (female): Sure, I have some before my next

section. What’s up, John?

Student: Uh, I wanted to know if there would be any way

that I could switch lab partners. Right now I’m paired up

with Roger Kisbeck ... and that’s just not working out at

all.

Professor: Well, I’m sorry to hear that. What exactly is

the problem?

Student: He’s a nice guy and all ... and I know that he

usually does well in his classes. I mean our lab results

are always right and everything ... but he takes so long to

do the labs. I know you told us to double check our steps

when we do an experiment, but this guy wants to go

back and check every step three or four times. It drives

me crazy.

Professor: Hmmm. OK, John, I don’t want you to take

this personally, but I put you with Roger for a reason.

Student: I’m sorry?

[ f Script ]

1-1. Professor (female): Thirty years ago, there was widespread

opposition to the use of nuclear power pretty much the

world over. However, as we’ll see, a number of events

have spurred a dramatic reevaluation of nuclear power.

1-2. Professor (male): Most of us assume that lift-off is the

most dangerous part of any space mission. But actually,

the potential for failure is far greater during re-entry.

There are a great many things that must occur, most of

them in a very precise time frame, in order for a

spacecraft to survive re-entry.

1-3. Professor (female): Insects are by far the most numerous

organisms on earth. There are literally millions of different

insect species, and insects can be found in essentially

every environmental niche on earth. Despite the great

numbers and variety of insects, however, those millions of

species can be placed in a surprisingly small number of

categories based on a few simple characteristics.

2. Professor (male): Bacteria are the simplest single cell

organisms found on earth. They are also the most

numerous. What differentiates bacteria from other types

of unicellular life is their lack of a cellular nucleus or other

complex internal structures. They reproduce through cell

division, and most live either symbiotic or parasitic lives.

Uh … basically that means that most bacteria can’t

survive without a host. If they form a mutually beneficial

relationship with their host, then they are symbiotic in

nature, while those that harm their hosts are parasitic.

Let’s look at a couple examples.

At any given moment, your body is host to millions upon

millions of bacteria. Uh, they’re everywhere ... your

stomach and intestines, your skin. Some of those

bacteria are essential to the function of the body. The

ones in your intestines, for example, help you digest

food. In other environments, bacteria can be equally as

useful. Bacteria in the soil, for example, play a key role in

breaking down dead organic matter and recycling

nutrients back into the soil.

Many bacteria, however, can be quite harmful. The

reason for this is that they create toxic byproducts that

are harmful to their hosts. Actually, this is the cause of

many diseases, from relatively benign illnesses like strep

throat to more deadly ones like typhoid.

Chapter 5

Page 50: Insider: The Super Guide Scripts & Answer Key

A50 Insider: The Super Guide

1. (B) – The student says that his lab partner is driving him

crazy because he takes too long to do the experiments.

The student then asks the professor to change his lab

partner.

2. (A) – The professor explains that while the student has a

good understanding of theory, his methodology is poor.

Roger, on the other hand, is just the opposite. The

professor explains that this is why she paired the two

students together.

Building Your Listening Skills _ p.258

1. time 2. Right now

3. want you 4. checking them

Listening Practice 2 _ p.259

1. (B) – The professor clearly discusses two different types of

call and response. While another blues technique (the turn

around) is briefly mentioned at the end of the discusssion,

there is no comparison made between call and response

and the turn around.

2. (D) – The professor mentions a different technique (the

turn around) at the end of the discussion, but does not

explain it in any detail. It is most likely that he will explain

this technique in the next section of the discussion.

works. The singer will sing a short section of the verse,

uh, just a phrase usually. Maybe something like (singing)

My baby done gone away.

Professor: (good naturedly) Yeah, I know. That’s why I

teach music instead of make it. But anyway, you’ll get a

short vocal phrase like that, followed by a short

instrumental phrase, uh, usually played on a guitar, but

not always, that seems to complement it. Call and

response is used to build and relieve tension within the

structure of a blues song.

Call and response can be further separated into two sub-

techniques. In leader/chorus response, the guitar line will

attempt to copy the melody of the singer, so that the guitar

line sounds like what the singer has sung. In question /

answer response, the singer will sing a melody that seems

unfinished, and the guitar line will finish that melody.

Student B (male): So how does that fit in with the 12-bar

blues pattern you told us about?

Professor: Well, each call and response typically takes a

total of two bars ... uh, one for the call and one for the

response.

Student B: Then there are 6 call and response sets?

Professor: Not usually, no. For one thing, a particular

call and response set may be repeated. For another

thing, the last two bars of a 12-bar blues pattern are

taken up by the turn around, which is the next technique

we need to talk about.

[ f Script ]

Professor (male): Today, we are going to talk about

some of the fundamental techniques in blues. The first

one we’ll talk about is call and response. Does anyone

know what this is?

Student A (female): It’s like when you play a guitar line

that goes along with the vocals.

Professor: Uh, not exactly. It would be more exact to say

that it’s an instrumental line that follows the vocals, or,

actually, that responds to the vocals. Here’s how it

Professor: You were in my Chem. 101 course last

semester, and I noticed that your experiments were ...

well, kind of sloppy. You seem to have a good

understanding of the concepts, but when you apply them

in the lab ... you skip procedures, you just accept your

results at face value without really checking them ...

that’s not good scientific method. I put you with Roger

because he’s just the opposite. His grasp of theory is not

so strong, but his methodology is excellent. I was hoping

that your good habits would kind of rub off on each other.

Student: OK, I guess ‘ll try to be a bit more patient with

him.

BBlluuee TTeecchhnn: call and response

● def. singer sings short phrase then gtr. responds w/

complimentary line

● uses: create + relieve tension/ alter mood of song

TTyyppeess:

Leader/chorus call and response:

● gtr. line follows singer

Question/Answer call and response:

● singer’s melody seems incomplete, gtr. line finishes

melody

C&R set sometimes repeated more than once - so not 6 sets

Last 2 bars for turn around (different technique)

stdt. wants change lab partners b/c partner is too slow -

checks 3-4 times

PPrrooff.. rreessppoonnssee:

● paired stdt b/c stdt does sloppy lab work

● stdt has good theory but poor method

● Roger has good method but bad theory

● profs. wants stndy to keep working w/ Roger - habits

rub off on each other

Page 51: Insider: The Super Guide Scripts & Answer Key

LIS

TE

NIN

G

Scripts & Answer Key A51

Building Your Listening Skills _ p.260

1. Statement from the discussion: But anyway, you’ll get a

short vocal phrase like that, followed by a short

instrumental phrase, uh, usually played on a guitar, but

not always, that seems to compliment it. Call and

response is used to build and relieve tension within the

structure of a blues song.

The paraphrased statement is a summary of the

information contained in this section of the discussion.

Building and relieving tension in a song would be one way

of altering the mood of the song.

2. Statement from the discussion: Not usually, no. For one

thing, a particular call and response set may be

repeated. For another thing, the last two bars of a 12-Bar

blues pattern are taken up the turn around…

In order to understand this paraphrase, you have to

consider the professor’s statements in conjunction with

the question asked by the student just before this.

Listening Practice 3 _ p.261

1. (C) – The professor’s discussion of mammoths centers

around the sequence of events that led to the evolution of

small-sized mammoths, which are a species that exemplify

island dwarfing.

2. (C) – The professor first describes a theory which focuses

on the effects of environmental pressures (the lack of

resources) and then discusses a theory which focuses on

the isolation of the gene pool.

location over a long period of time evolves to become

smaller in size than average members of the same

species living in other areas. Here … what I’ll do is give

you an example of this, and then we’ll talk about the

various theories of why it happens.

Up until around 15,000 years ago, there existed an

ancestor of the modern elephant called the mammoth.

Your typical mammoth stood around 13 feet in height at

the shoulder, so it wasn’t exactly what you would call a

dwarf. But at some point a group of mammoths somehow

got onto a small group of islands off the coast of

California and became isolated from the larger genetic

pool of mammoths. Over several thousand years, they

evolved in such a way that their average height at the

shoulders was little more than 6 feet. This is pretty much

a classic case of insular dwarfism.

There are a couple theories on why island dwarfing

occurs … uh, and they focus on very different factors.

One theory revolves around the physical environment.

Island dwarfing doesn’t always happen on an island, but it

does always happen in geographically isolated areas with

limited territory and resources. One theory suggests that

in an environment where resources are limited, larger

animals are naturally selected out of the gene pool

because they require more food to survive. Therefore, in

this situation, smaller animals are better adapted to

survival in a small area. The other dominant theory

concerns the genetic environment. Some researchers

think that reduction in size is a natural genetic response

to genetic isolation ... uh, that if you have the same

genetic group interbreeding over a long enough period of

time they will naturally grow smaller.

[ f Script ]

Professor (male): All this week, we’ve been talking about

the processes through which environment can affect the

evolutionary path of a species. Today, we’re going to

discuss a well-documented, but poorly understood,

interaction between environment and evolution.

Insular dwarfism, or island dwarfing, is a phenomenon in

which a genetic group living in a geographically isolated

[ f Script ]

1.

Professor (male): Yeah, I know. That’s why I teach music

instead of make it. But anyway, you’ll get a short vocal

phrase like that, followed by a short instrumental phrase,

uh, usually played on a guitar, but not always, that seems

to complement it. Call and response is used to build and

relieve tension within the structure of a blues song.

2.

Professor (male): Well, each call and response typically

takes a total of two bars … uh, one for the call and one

for the response.

Student (male): Then there are 6 call and response

sets?

Professor: Not usually, no. For one thing, a particular

call and response set may be repeated. For another

thing, the last two bars of a 12-bar blues pattern are

taken up by the turn around, which is the next technique

we need to talk about.

IInnssuullaarr DDwwaarrffiissmm:

● def. animals become smaller when isolated for long

time

● exmpl: nrml. Mammoth = 13 ft. but some Mam. lived

on small island so genetically & geographically

isolated, result = pygmy Mam. only 6 ft

TThheeoorriieess ffoorr IInnss.. DDwwff..:

● resrcs. limited so larger anim. die b/c need more food

● genetic evirnmt. - genetic isolation = smaller size an

natural reponse

Page 52: Insider: The Super Guide Scripts & Answer Key

A52 Insider: The Super Guide

Building Your Listening Skills _ p.262

1. (A) – This section of the lecture is primarily concerned with

providing a definition of island dwarfing.

2. (A) – In the previous section of the lecture, the professor

has given a basic definition of island dwarfing. A specific

example is now given to help clarify that definition.

3. This section of the lecture describes the two dominant

theories of why island dwarfing occurs.

| Vocabulary Review | _ p.263

1-1. spur 1-2. niche

1-3. complement 1-4. grasp

1-5. differentiate 1-6. methodology

2-1. rub off on

. George is always careful never to smoke around his

son because he does not want his son to start smoking.

2-2. accept it at face value

. If there is no way to check the truthfulness of her

story, you will just have to believe her.

| iBT Practice | _ p.264~265

[ f Script ]

Professor (male): Today, we’re going to talk about a

process called soil liquefaction. This is where a seemingly

solid soil will undergo a conversion to a liquid, uh, or at

least semi-liquid state. If you’re familiar with quicksand,

you are already familiar with the process of soil

liquefaction. In order for soil liquefaction to occur you need

two things. Uh, first you need a certain type of soil; a loose

unconsolidated sediment. Unconsolidated sediment is a

type of soil that is made of small grains rather than of large

rocks. Again, sand is a good example of unconsolidated

sediment. The other thing you need is water. (slight pause)

OK, as you can see in picture 1, in unconsolidated

sediment, there are also little pockets of air between the

grains. Now, as water soaks into the sediment, it will

begin to fill up all these little air pockets. Once all the air

pockets have been filled with water, the sediment is said

to be saturated; it’s absorbed all the water it can and still

hold together. If more water is forced into the sediment, it

starts to push the grains apart, so that they start floating

freely in the water. At this point, soil liquefaction has

occurred, and the soil will behave as a liquid rather than

as a solid. Add enough water to any unconsolidated

sediment and this will occur.

Now, this presents significant dangers to people, and I’m

not just talking about stepping into a pit of quicksand

while you’re on a hike in the woods. When an

underground water source takes on too much water,

liquefaction can occur. And if that water source happens

to be below a building or a city street … well, the damage

can be pretty severe. The foundation of the building will

begin to sink into the liquefied soil … the building may

even completely collapse. The risk is particularly severe

if liquefaction occurs on a hillside, as the soil will pour

downhill, destroying anything in its path.

Of course, in modern times we can monitor the saturation

levels of the soil, and if need be, drain water out of the

soil before liquefaction occurs. But liquefaction is still a

[ f Script ]

1.

Professor (male): All this week, we’ve been talking about

the processes through which environment can affect the

evolutionary path of a species. Today, we’re going to

discuss a well-documented, but poorly understood,

interaction between environment and evolution.

Insular dwarfism, or island dwarfing, is a phenomenon in

which a genetic group living in a geographically isolated

location over a long period of time evolves to become

smaller in size than average members of the same

species living in other areas.

2.

Professor (male): Up until around 15,000 years ago,

there existed an ancestor of the modern elephant called

the mammoth. Your typical mammoth stood around 13

feet in height at the shoulder, so it wasn’t exactly what

you would call a dwarf. But at some point a group of

mammoths somehow got onto a small group of islands

off the coast of California and became isolated from the

larger genetic pool of mammoths. Over several thousand

years, they evolved in such a way that their average

height at the shoulders was little more than 6 feet. This is

pretty much a classic case of insular dwarfism.

3.

Professor (male): There are a couple theories on why

island dwarfing occurs … uh, and they focus on very

different factors. One theory revolves around the physical

environment. Island dwarfing doesn’t always happen on

an island, but it does always happen in geographically

isolated areas with limited territory and resources. One

theory suggests that in an environment where resources

are limited, larger animals are naturally selected out of

the gene pool because they require more food to survive.

Therefore, in this situation, smaller animals are better

adapted to survival in a small area. The other dominant

theory concerns the genetic environment. Some

researchers think that reduction in size is a natural

genetic response to genetic isolation ... uh, that if you

have the same genetic group interbreeding over a long

enough period of time they will naturally grow smaller.

Page 53: Insider: The Super Guide Scripts & Answer Key

LIS

TE

NIN

G

Scripts & Answer Key A53

1. Main Idea Question – (A)

This is a paraphrase of the idea of soil liquefaction.

2. Identifying Relationship Question

3. Supporting Detail Question – (B)

The professor states that soil liquefaction is especially

dangerous when it occurs on a hillside (an inclined slope).

4. Organization Question

5. Supporting Detail Question – (C)

The professor clearly states that soil liquefaction during an

earthquake is more destructive because the destruction is

spread over a much broader area.

6. Organization/Rhetorical Purpose Question – (C)

The implication is that students would not stack things in

this manner because they know that whatever they

stacked would fall over.

Listening for Total Comprehension _ p.266

[ f Script ]

Professor (male): Although the theft of masterpiece

paintings is a popular subject in films and detective

novels, it is, in practice, not very widespread. Aside from

the obstacle posed by the impressive security measures

at most world-class museums, the theft of a masterpiece

simply isn’t very profitable for the thief. Once stolen,

because of their fame, these paintings usually prove to be

virtually impossible to sell on the black market, and thus,

there is little chance for the thief to profit from his crime.

For instance, The Scream, by Edvard Munch, was stolen

in 2004. Yet, despite having it in their possession for

more than a year, the thieves were unable to sell it, and it

was eventually returned to the museum once the thieves

were caught. When the Mona Lisa was stolen, the

situation was similar. After keeping the painting hidden for

two years, the thief was immediately caught as soon as

he attempted to sell it.QQuueessttiioonn �� –– RReeppllaayy

f PPrrooffeessssoorr: You see how the grains are stacked on top ofeach other? If you were stacking cans in your cupboard�would you stack them this way? Of course not�

f PPrrooffeessssoorr: If you were stacking cans in your cupboard�would you stack them this way? Of course not�

significant danger during earthquakes, even if the soil is

only partially saturated. Uh, to understand why, let’s go

back and look at picture 1 again.

You see how the grains are stacked on top of each other?

If you were stacking cans in your cupboard, would you

stack them this way? Of course not. Now, in unconsolidated

soil, the friction between individual grains of sand will

allow them to stack like this … and even to support a

considerable amount of weight. But the shaking motion of

an earthquake will cause them to shift into a position

more like what you see here in picture 2. (slight pause)

You can see that the grains are a lot more densely

packed, right? I mean there’s a lot less open space

between them. And remember, what’s between those

spaces? Water. So, as lower layers of the soil settle into

this more densely packed configuration, the water in

those layers gets pushed up into the next layer, and the

next, and the next, until the top layers of the soil do

become super saturated and soil liquefaction occurs. This

is a considerable danger in quite a few earthquake prone

areas. The results of earthquake liquefaction are

essentially the same as the results of regular soil

liquefaction, uh, collapsed buildings, destroyed roads,

mudslides … but the destruction is usually spread over a

much broader area. In 1964, the Japanese city of Niigata

experienced an earthquake in which widespread

liquefaction occurred. Entire sections of the city were

totally destroyed, and the loss of life was considerable. In

1989, the San Francisco area had an earthquake. Again

there was widespread liquefaction. The cost? 62 lives

lost, almost 4,000 injured, and $6 billion in damages.

Although the theft of masterpiece paintings is a popular

subject in films and detective novels, it is, in practice,

not very widespread. Aside from the obstacle posed by

the impressive security measures at most world-class

museums, the theft of a masterpiece simply isn’t very

profitable for the thief. Once stolen, because of their

fame, these paintings usually prove to be virtually

impossible to sell on the black market, and thus there is

little chance for the thief to profit from his crime. For

instance, The Scream, by Edvard Munch, was stolen in

2004. Yet, despite having it in their possession for more

than a year, the thieves were unable to sell it, and it was

eventually returned to the museum once the thieves were

caught. When the Mona Lisa was stolen, the situation

was similar. After keeping the painting hidden for two

years, the thief was immediately caught as soon as he

attempted to sell it.

1. An earthquake disrupts the arrangement of the soil.2. Soil particles settle into a new formation.3. Water is forced up as spaces between particles close.4. Upper levels of soil become saturated.

Yes NoThe occurrence of an earthquake �The presence of loose, fine-grained

�soilThe exertion of downward forces

�on the soilThe super saturation of the soil �

Page 54: Insider: The Super Guide Scripts & Answer Key

A54 Insider: The Super Guide

Identifying RelationshipQuestions

Basic Drills _ p.270

[ f Script ]

1-1.

Man: Hey, Stacy. Good to see you. What are you up to?

Woman: Oh, you know, the same old thing, just here to

finish signing up for my classes. I had to find an open

section in Biology 203.

Man: Really? No kidding, I just signed up for Professor

Johnson’s section of that class two days ago.

Woman: You got in Johnson’s section? You are so lucky.

I got stuck with Flanders.

Man: Oh no, you got to be joking. I heard he’s like a million

times tougher than the other professors. Looks like you’re

going to have your nose to the grindstone this semester.

1-2.

Professor (male): As we stated before, the slave colony

of Saint-Dominque is unique in the history of the slave

holding West Indian colonies. While slave populations on

other colonial islands also staged slave revolts, the slave

revolt on Saint-Dominque was the only one which was

successful. If we analyze that revolt, you’ll see that a lot

of factors, uh, not just in terms of conditions on the island,

but also in terms of the geo-political situation, made the

revolt and its result almost a forgone conclusion.

1-3.

Professor (male): We all know that at the beginning of

the Cold War, the US and the Soviet Union were

engaged in a space race, but what I want to really talk to

you about today is the role of German military programs

during World War II in the history of space exploration. I

know it sounds like I’m pulling your leg, but you’d be

amazed at exactly how much early American space

technology was based almost entirely on earlier designs

from the German ballistic missile program.

2.

Professor (female): When designing a new car, one

choice manufacturers are faced with is whether to go

with a diesel or a gasoline engine. As we’ll see, this has

significant impacts on the production costs, fuel

economy, reliability, and performance of the car.

OK, first, diesel engines burn a heavier fuel, diesel. What

this means is that it’s harder to ignite the fuel, the result

being that cars with diesel engines are typically harder to

start, uh, especially in cold weather. This is a considerable

drawback in marketing diesel engines in areas with cool

Chapter 6

1. It isn’t profitable to steal masterpiece paintings because

they are too famous to sell on the black market.

2. The Scream, which was stolen in 2004, and the Mona

Lisa are the two examples discussed. In both cases, the

thieves held the paintings for a long time, but were

unable to sell them, and were eventually caught.

Page 55: Insider: The Super Guide Scripts & Answer Key

LIS

TE

NIN

G

Scripts & Answer Key A55

1-1. Yes – The two students will probably compare the two

professors. The woman says that the man is lucky to be

in Professor Johnson’s class. The man, on the other

hand, expresses sympathy for the woman when he hears

that she is in Professor Flanders’ section. The students

will probably discuss the differences between these two

professors and why Professor Johnson is better than

Professor Flanders.

1-2. No – While the professor briefly compares the slave revolt

on Saint-Dominque to other slave revolts in terms of

success, the professor’s later statements make it clear

that the rest of the lecture will focus on what caused the

slave revolt on Saint-Dominque. Therefore, the structure

of the lecture will most likely be focused around cause

and effect relationships and not on comparisons.

1-3. Yes – The professor states that much of the early

American space program was based on German military

technology, and says that the students will be surprised

to learn to what extent this is true. The professor will most

likely compare the technologies of the German missile

program and the American space program to prove his

point.

2.

The professor introduces the idea that diesel engines are

found in luxury automobiles and gasoline engines are

found in economy class cars with the phrase “As a side

note.” This indicates that this is not an important point in

the lecture. The point made about it being difficult to find a

gas station with diesel fuel is simply an additional thought

that is tagged onto the large idea of fuel economy.

Listening Practice 1 _ p.271

[ f Script ]

Student (female): Excuse me? Hello? Could somebody

help me?

(Slight pause)

Employee (male): Hi, sorry we’re a little short staffed

today, and I was helping another student get set up in

one of the labs.

Student: Oh, it’s no problem. I just need to sign up to use

one of the language labs.

Employee: OK, you just need a room with a tape player

or you need a recording room?

Student: One of the recording rooms. I have to do a

project on regional accents for my linguistics class, so I

have to record a couple of my friends’ voices so that I

have data to analyze.

Employee: I see. Have you ever used one of the

recording rooms before?

Student: I’m not really a linguistics major. I just got to

take this class as a requirement.

Employee: Uh huh. Well, the first thing you’ll need to do

is sign up for an orientation session on how to use the

equipment.

Student: (amused) Dude, it’s a tape recorder.

Employee: (sounding a little annoyed) Actually, it’s

professional quality recording equipment. It’s slightly

more complex than a tape recorder and a lot more

expensive. I know that the ones in the foreign language

department are just a tape recorder, but not here.

Student: (sighing) OK, when are these orientations and

how long are they?

Employee: They only take about an hour. Uh, usually we

have one today, but since I’m the only one in today ... Uh,

the next one is Thursday 4 pm. Would you like me to put

you down?

Student: Yeah, go ahead and put me down, I guess.

climates. Another disadvantage of diesel engines is that

they are more expensive to produce … uh, because they

have to be built of heavier, more durable metals. Now,

the flip side of that is that they last longer. A gasoline

engine that is made out of mostly aluminum or low grade

steel will last maybe for 200,000 miles, but diesel

engines run for much longer if properly cared for. Uh, as

a side note, gasoline engines are almost always used in

economy class cars for this reason … uh, because

nobody really plans to keep them for a long time. You

won’t begin to see diesel engines until you get into luxury

sedans.

OK, some final points. Diesel fuel burns slower than

gasoline. This means that diesel engines burn less fuel

and get better fuel economy than gasoline engines. This

is a significant plus for diesel engines in these days of

high fuel costs, although it’s a lot harder to find a gas

station with diesel fuel than one with gasoline. In terms of

performance, it’s really a tradeoff. Diesel engines create

more torque, giving them more pulling power, while

gasoline engines have more horsepower, meaning they

can accelerate faster.

Diesel GasolineEngines Engines

Longer lasting �Used in more expensive carsHigh production costs �Lower fuel economy �More difficult to find fuel

Page 56: Insider: The Super Guide Scripts & Answer Key

1. (B) – The student states that she is not a Linguistics major

after the worker’s question about whether or not she has

used the language lab before. This implies her statement is

intended to respond to his question. Since she is not a

Linguistics major, it is likely that she has not used the

Linguistics lab before.

2. (A) – The man explains the need for the training session by

stressing the expense and sophistication of the equipment

in the language lab. Part of the way he does this is by

comparing the Linguistics lab to labs in other departments,

which only use simple tape recorders.

Building Your Listening Skills _ p.272

1. Do 2. do

3. have to do 4. got to

5. is sign up

Listening Practice 2 _ p.273

1. (D) – The professor says that one key difference between

earlier glassblowing and glassblowing after the seminar by

Littleton and Labino is that they saw glassblowing as an

art, while before it had been more of a practical craft.

2. (A) – The professor says that Littleton and Labino taught

involvement in every part of the glassblowing process as

opposed to the factory style approach of earlier times.

gum. Anyway, the bubble could be formed into various

shapes, left to cool, and once it had cooled, you would

have a solid glass object. Now that is basically the same

process that is used today. There have been

improvements in the technology, but the essential

techniques remain the same.

So anyway, throughout much of history, glassblowing was

an entirely utilitarian art. Uh, the objects produced were

glasses, bottles … stuff for everyday use. Basically,

glassblowing wasn’t so much an art as it was a craft.

That, however, changed in 1962. In that year, two men,

Harvey Littleton and Dominick Labino, held a workshop

on glassblowing in Toledo, Ohio. Littleton and Labino

changed two things about glassblowing. First, and most

importantly, they approached glassblowing as a true art.

The forms they created were abstract … uh, basically

glass sculptures. Second, Littleton and Labino stressed

individualism in glassblowing. They taught involvement in

every step in the glassblowing process, from melting the

sand and preparing the glass, to shaping and polishing it.

Before, each step had been performed by separate

individuals in an assembly line process.

Littleton and Labino’s methods and teachings became the

foundation of what is now known as the studio glass

movement. The popularity of artistic studio glass spread

quickly. Today there are over a thousand professional

glassblowing studios in the United States alone. You can

go into almost any modern art museum and find studio

glass works. This is all a testament to the influence of

Littelton and Labino.

[ f Script ]

Professor (male): While the art of glassblowing has been

around for several millennia, the studio glass movement

is relatively new. Uh, I guess before I get into the history

of the movement, I should talk a little about the art of

glassblowing itself. The art of glassblowing seems to

have first emerged in the first century A.D. in Syria. Sand

was heated to around 1400°C, at which point it melted,

and formed glass. The molten glass was then gathered

onto a hollow tube. The glassblower would then blow air

through the tube, forming a bubble in the molten glass,

uh, kind of like if you blow a bubble while you’re chewing

QQuueessttiioonn �� –– RReeppllaayy

f EEmmppllooyyeeee:: I see� Have you ever used one of therecording rooms before?

SSttuuddeenntt:: I’m not really a Linguistics major� I just got totake this class as a requirement�

A56 Insider: The Super Guide

stdnt. needs recording room b/c project in ling. class -

record accents

worker: stdnt 1st has to take training session b/c stdnt

hasn’t used recording room before

stdnt thinks this is stupid/not necessary

Worker reaction: annoyed with stdnt, equipment

expensive and complicated-more than other lang. labs.

Sessions:

Time: 4pm

next one is thurs b/c only one worker in lab today

stdnt will sign up for lab

GGllaassssbblloowwiinngg: origns: 1st century A.D. Syria

TTeecchhnnqq:

1) melt sand 2) gather melted sand on hollow tube

3) blow in tube and make bubble

4) shape bubble 5) let glass cool

HHiissttoorryy: Tradtn. use: Very practical, make things for

everyday use

LLiittttlleettoonn ++ LLaabbiinnoo: Changed approach to glassblowing –

When: seminar in Toledo 1962

Changes: glassblowing as art – glass sculptures, one

person does all work, not factory approach

Page 57: Insider: The Super Guide Scripts & Answer Key

Scripts & Answer Key A57

LIS

TE

NIN

G

Building Your Listening Skills _ p.274

1. Statement in the lecture: The art of glassblowing seems to

have first emerged in the first century A.D. in Syria.

This is a pretty direct paraphrase of the original statement.

The only real changes are the move from the mention a

specific nation (Syria) to a general region (the Middle East),

and the change in the way that time is discussed.

2. Statement in the lecture: You can go into almost any

modern art museum and find studio glass works. This is

all a testament to the influence of Littleton and Labino.

Saying that something “is a testament” to something is

roughly equivalent to saying that it shows or proves

something.

Listening Practice 3 _ p.275

Professor: Yes, but it’s not as simple as that. There are

several thousand tigers left in the world, and they are

considered to be “moderately endangered.” But let’s say

that there were only several thousand blue crabs left in

the world. Well, then the blue crab would be “critically

endangered” meaning it would be literally on the brink of

immediate extinction. So, how do we sort this out? How

do we know that several thousand tigers is a relatively

acceptable number, but that same the number of blue

crabs would be totally unacceptable? By calculating the

minimum viable population for a species.

The minimum viable population of a species is the

smallest number of animals that will give a species a

90% statistical chance of not going extinct in the next

100 years.

Student: So, how do you figure that out?

Professor: Well, it’s a pretty complex process. First, you

need to create a computer model that will analyze all the

known variables that could affect the species’ chances of

survival, uh, climate changes, spread of disease,

predation by other species, fluctuation in food supply,

that kind of thing. Anyway, after you gather all these

variables, and there’s usually quite a few of them, you

can create a computer model that will calculate the

chances of survival for the species.

So, let’s say that I want to know the minimum viable

population for tigers. I create a computer model with all

my variables, and then I run it with a hypothetical number

of tigers, uh, let’s say 2,000 total. OK, and let’s say that

when I run my model with a starting population of 2,000

tigers, they survive as a species in 99 out of 100

computer simulations. That would be a 99% chance of

survival, well within the acceptable range. Now, let’s say

I reduce my hypothetical tiger population to 300. I run the

same computer model, and this time, they only survive

50 times out of a hundred. A 50% chance of species

extinction is well below the acceptable level, so 300

tigers would be well below the minimum viable

population. Anyway, you just keep running the sim.

(simulation) with different tiger populations until you get a

number approaching 90%. Then you’ve found the

minimum viable population.

[ f Script ]

1.Professor (female): Today, I’m going to start with a

question. Who can tell me what an endangered species

is?

Student (male): It’s a species that is in danger of

extinction.

Professor: And how do we decide if a species is in

danger of extinction?

Student: Uh, I guess by how many animals are left in the

world.

[ f Script ]

1.

Professor (male): While the art of glassblowing has been

around for several millennia, the studio glass movement

is relatively new. Uh, I guess before I get into the history

of the movement, I should talk a little about the art of

glassblowing itself. The art of glassblowing seems to

have first emerged in the first century A.D. in Syria. Sand

was heated to around 1400°C, at which point it melted,

and formed glass. The molten glass was then gathered

onto a hollow tube. The glassblower would then blow air

through the tube, forming a bubble in the molten glass, uh,

kind of like if you blow a bubble while you’re chewing gum.

2.

Professor (male): Littleton and Labino’s methods and

teachings became the foundation of what is now known

as the studio glass movement. The popularity of artistic

studio glass spread quickly. Today there are over a

thousand professional glassblowing studios in the United

States alone. You can go into almost any modern art

museum and find studio glass works. This is all a

testament to the influence of Littelton and Labino.

Page 58: Insider: The Super Guide Scripts & Answer Key

A58 Insider: The Super Guide

1. (B) – The professor uses these two species to

demonstrate why total population isn’t a good indicator of

the danger of extinction for a species. According to the

professor, what is considered an acceptable number of

tigers is very different than what is an acceptable number

of crabs.

2. (C) – The professor uses a made up example of

calculating the minimum viable population of the tiger in

order to demonstrate how computer models are used in

the process.

Building Your Listening Skills _ p.276

1. (A) – This section of the discussion primarily describes the

problem of how to determine how endangered a species

is. The rest of the discussion describes the solution to that

problem (calculating the minimum viable population).

2. (B) – The professor describes the process of determining

the minimum viable population in response to the

student’s question.

3. This section of the discussion gives a detailed

explanation of how computer modeling is used to

determine the minimum viable population of a species by

using a hypothetical example to illustrate the process.

2.

Student (male): So how do you figure that out?

Professor (female): Well, it’s a pretty complex process.

First, you need to create a computer model that will

analyze all the known variables that could affect the

species’ chances of survival, uh, climate changes,

spread of disease, predation by other species, fluctuation

in food supply, that kind of thing. Anyway, after you

gather all these variables, and there’s usually quite a few

of them, you can create a computer model that will

calculate the chances of survival for the species.

3.

Professor (female): So let’s say that I want to know the

minimum viable population for tigers. I create a computer

model with all my variables, and then I run it with a

hypothetical number of tigers, uh, let’s say 2,000 total.

OK, and let’s say that when I run my model with a

starting population of 2,000 tigers, they survive as a

species in 99 out of 100 computer simulations. That

would be a 99% chance of survival, well within the

acceptable range. Now, let’s say I reduce my

hypothetical tiger population to 300. I run the same

computer model, and this time, they only survive 50

times out of a hundred. A 50% chance of species

extinction is well below the acceptable level, so 300

tigers would be well below the minimum viable

population. Anyway, you just keep running the sim.

(simulation) with different tiger populations until you get a

number approaching 90%. Then you’ve found the

minimum viable population.

[ f Script ]

1.Professor (female): Today, I’m going to start with a

question. Who can tell me what an endangered species

is?

Student (male): It’s a species that is in danger of

extinction.

Professor: And how do we decide if a species is in

danger of extinction?

Student: Uh, I guess by how many animals are left in the

world.

Professor: Yes, but it’s not as simple as that. There are

several thousand tigers left in the world, and they are

considered to be “moderately endangered.” But let’s say

that there were only several thousand blue crabs left in

the world. Well, then the blue crab would be “critically

endangered” meaning it would be literally on the brink of

immediate extinction. So how do we sort this out? How

do we know that several thousand tigers is a relatively

acceptable number, but that same the number of blue

crabs would be totally unacceptable?

WWhheenn iiss aanniimmaall eennddggrrdd??

Total #s not good enough b/c acceptable #s different

from animal to animal

● exmpl: several thousand tigers OK, several thousand

crabs = extinction very soon

● Minimum viable population used to detrmn if anim.

is endgrd.

● def. minimum number w/ 90% chance survival for

100 years

CCaallccuullaatteedd MMiinn.. VViiaabbllee PPoopp..:

1. make computer model with all variables that affect

population

2. run computer model w/ different starting pops.

3. stop when species survives 90 out of 100 computer

sims.

Page 59: Insider: The Super Guide Scripts & Answer Key

LIS

TE

NIN

G

Scripts & Answer Key A59

| Vocabulary Review | _ p.277

1-1. utilitarian 1-2. viable

1-3. durable 1-4. staged

2-1. is on the brink

. The baseball player is very close to breaking the

homerun record, and is expected to do so any day now.

2-2. have his nose to the grindstone

. Although Jason is successful, he always seems to

be working really hard and never seems to enjoy his

success.

2-3. the flip side of it

. While his illness gives him a few more days to study

for the exam, it also means that he will have a lot of

make-up work when he gets back.

2-4. have been pulling my leg

. When Bob told me that he won the lottery, I knew he

must have only been joking.

| iBT Practice | _ p.278~279

Student B (male): I read it, too, and I kind of had the

same feeling. Yeah, there are some good parts, but was

it really necessary to put in so much information on

whaling? Maybe I just missed something.

Professor: Yeah, a lot of the story, actually the whole

story, can be considered an allegory. Do you know what

that means?

Student A: Isn’t that the same as a metaphor? I mean,

there’s like a double meaning or something, right?

Professor: Well, there is a similarity. Allegories tend to be

longer and more involved while metaphors are shorter. I

mean a whole story or novel can be an allegory.

Student B: Yeah, but ... I still don’t actually understand

what an allegory is.

Professor: Yeah, I guess I did kind of skim over that.

Okay so, on the surface you have one story. Let’s say

you’re writing about a tree and its branches, leaves,

something like that. But at the same time maybe, under

the surface, you’re writing about, let’s say, society.

Student A: But how do you know the writer’s just not

writing about a tree?

Professor: Well, he’ll usually give you some hints, some

references that show what he’s doing. I mean, almost all

fables are allegories that end with some moral. The

moral shows the reader that the fable has actually been

referring to something else all along.

Student B: So you’re saying that Melville was writing

about whaling on the surface, but was trying to say

something more, I don’t know, important under the

surface. But I don’t understand why he needed to hide

these bigger ideas under the surface. Why not just say

them directly?

Professor: Well, you’re raising an interesting point, and

actually it’s a point that separates literature from

philosophy, and sometimes the line between them blurs.

Philosophy tends to present an organized system of

thoughts whereas literature may make similar points

without having to give any sort of organized structure to

support them. But Melville also had a more practical

reason for writing one story on the surface and another

below the surface.

You see, Dana had stimulated not only Melville’s, but the

whole nation’s interest in maritime adventure. Melville’s

first books were very similar to Dana’s and were quite

popular. But when he tried to write books that were more,

well, thoughtful, I guess you could say they were not very

well received. So, he was stuck. He could write popular

books with little philosophical depth, which would make

his publishers happy, or he could write the more

thoughtful books that he wanted to, and please few

[ f Script ]

Professor (male): If a Harvard student hadn’t gotten the

measles in 1834, we might never have had one of our

most famous novels. I guess I’d better explain. When

Richard Dana, the Harvard student I’m talking about, got

the measles, it affected his eyesight. Well, in those days

it was considered routine for Harvard students to go off

and visit Europe. But Dana had other ideas. He

somehow believed that going out on a sea voyage would

help strengthen his eyes. In any event, he was never the

traditional Harvard student and really never had the

money to go on a tour of Europe anyway.

So, he signed on board a ship to work as a common

sailor. And he sailed down around Cape Horn to

California on to Hawaii and then back again. The voyage

took two years, and during that time, he experienced

many adventures all the while writing them down in his

journal. You see, his goal was to come back and write a

book about the bad treatment of sailors. And so he

published the book Two Years before the Mast in 1840. It

became an instant sensation and is now considered an

American classic. But this is not the book that I want to

talk about today. I want to talk about someone who read

the book and was influenced by it.

Today, I want to talk about Herman Melville and his novel

Moby Dick, considered by many not only one of the

greatest American novels, but one of the greatest novels

in the English language. Have any of you read it?

Student A (female): Well, I tried, but I kind of got tired of

all the descriptions of whaling and all.

Page 60: Insider: The Super Guide Scripts & Answer Key

A60 Insider: The Super Guide

1. Main Idea Question – (C)

Certainly allegory is discussed and some examples of its

use given, but these examples are quite limited and

generally given only to clarify the definition of allegory.

Melville and Dana are only briefly compared. And though

the professor states that Moby Dick is a great novel, he

says little to support this assertion.

2. Organization/Rhetorical Purpose Question – (A)

All fables are allegories in that the surface story is often just

there to support a deeper meaning which is generally

expressed as a moral at the end of the fable. The

professor mentions them to give an example of a well

known type of allegorical writing.

3. Organization Question

4. Supporting Detail Question – (B)

If Melville had wanted to make his novels more popular, he

wouldn’t have used any allegory at all. In fact, Melville

wanted to write more philosophical novels and tried to use

allegory in order to please both himself and his publisher.

The publisher would have been happier, however, if he

had used no allegory at all, since this would have made the

book more popular.

5. Function Purpose Question – (D)

A student still doesn’t understand the meaning of the term

“allegory,” and the professor realizes that he really didn’t

explain it very well. His statement can be seen as an

apology for not doing a very good job in explaining the

term and moving on too quickly to a new topic. He is not

apologizing for speaking too quickly, just for moving along

too quickly.

6. Function Meaning Question – (A)

There is a separation between literature and philosophy.

Sometimes, however, this distinction is lost and there may

be some overlap (“the line blurs”), so it may be hard to

separate the two. Novels like Moby Dick may be

considered as literature even though at the same time they

may be expressing strong philosophical ideas.

Listening for Total Comprehension _ p.280

[ f Script ]

Professor (female): Many people think of bone as one

solid material, but that’s not really true. Bones are a

complex composite of various materials, both organic

and inorganic. Bones are made of two basic types of

bone material: compact bone and cancellous, or spongy,

bone. Compact bone is just that, compact. It is very

dense and very strong, and, as a result, compact bone

material forms the outer layers of a bone, where we

require the greatest level of protection. Because of its

density, it forms the majority of total bone weight, even

though it does not form the majority of bone material by

volume. Cancellous bone is less dense and has a

sponge-like structure, and can be found in the middle of

bones. In some bones, called long bones, there is a

hollow tube-like structure in the center of the bone called

the medullar cavity. The medullar cavity is filled with

bone marrow. Bone marrow is made of stem cells, and is

responsible for the production of new blood cells, as well

as the re-growth of outer bone tissue.

QQuueessttiioonn �� –– RReeppllaayy

f PPrrooffeessssoorr: Well� you’re raising an interesting point� andactually it’s a point that separates literature fromphilosophy� and sometimes the line between themblurs� Philosophy tends to present an organized systemof thoughts whereas literature may make similar pointswithout having to give any sort of organized structureto support them� But Melville also had a more practicalreason for writing one story on the surface and anotherbelow the surface�

f PPrrooffeessssoorr: Well� you’re raising an interesting point� andactually it’s a point that separates literature fromphilosophy� and sometimes the line between them blurs�

QQuueessttiioonn �� –– RReeppllaayy

f SSttuuddeenntt BB:: Yeah� but ��� I still don’t actually understandwhat an allegory is�

PPrrooffeessssoorr:: Yeah� I guess I did kind of skim over that�

f PPrrooffeessssoorr:: Yeah� I guess I did kind of skim over that�

people but himself. In my opinion, Moby Dick was a way to

try to do both in the same book. For the average reader,

it gave them whaling stories. For the more thoughtful

reader, there were allegory, metaphor, and symbolism.

Student A: Well, did it work?

Professor: Unfortunately, no. When the book came out,

it was a complete failure.

1. Herman Melville reads Dana.2. Herman Melville goes to sea.3. Herman Melville writes popular novels.4. Moby Dick is published.

Page 61: Insider: The Super Guide Scripts & Answer Key

LIS

TE

NIN

G

Scripts & Answer Key A61

1. Compact bone material is dense, strong, and is found in

the outer layers of bone because of the protection it

provides.

2. Cancellous bone is soft, spongy, and less dense than

compact bone material. It is found in the interior of

bones.

Stance / Attitude Questions

Basic Drills _ p.284

[ f Script ]

1-1.

Man: Hey, Jessica. I saw an announcement for this

really interesting art seminar this Saturday, so I signed

us both up. That cool with you?

Woman: (hesitantly) Sure.

1-2.

Man: Hey, are you ready to go to the picnic yet?

Woman: Yeah. Oh, I heard that it might rain later this

afternoon, so maybe we should bring an umbrella.

Man: (sarcastically) Good idea. I hadn’t thought of that.

1-3.

Man: So did you go to Tom’s party last night?

Woman: Yeah, I did. Oh! By the way, I met your friend

John there.

Man: (laughing) Oh, yeah? So what’d you think?

Woman: (in an upbeat voice) He’s really different, isn’t

he?

2-1.

Man: Hey, Jessica. Did you hear about the new university

policy about alcohol being off limits at the football games?

(One second pause)

Man: (in a disagreeing voice) Well, I think it’s the dumbest

thing I’ve ever heard.

2-2.

Woman: I’m really excited about this research trip. How

about you, Bob?

(One second pause)

Woman: (in a pleased, agreeing tone) No joke! This is

really going to help beef up my resume.

3.

Professor (male): Today, we’re going to talk about Social

Darwinism. Basically, this was a theory that applied

Darwin’s theory of evolution to human societies.

According to this view, there’s a constant struggle for

survival between different civilizations, and civilizations

that are conquered or subjugated suffer this fate

because they are inherently inferior. As a result, as

history progresses, civilization progressively improves

because lesser civilizations are eliminated by stronger

ones.

Now, given its total lack of supporting historical evidence,

this theory would be laughable if it wasn’t for the

Chapter 7Many people think of bone as one solid material, but

that’s not really true. Bones are a complex composite of

various materials, both organic and inorganic. Bones

are made of two basic types of bone material: compact

bone and cancellous, or spongy, bone. Compact bone is

just that, compact. It is very dense and very strong,

and, as a result, compact bone material forms the outer

layers of a bone, where we require the greatest level of

protection. Because of its density, it forms the majority

of total bone weight, even though it does not form the

majority of bone material by volume. Cancellous bone is

less dense and has a sponge-like structure, and can be

found in the middle of bones. In some bones, called long

bones, there is a hollow tube-like structure in the center

of the bone called the medullar cavity. The medullar

cavity is filled with bone marrow. Bone marrow is made

of stem cells, and is responsible for the production of new

blood cells, as well as the re-growth of outer bone tissue.

Page 62: Insider: The Super Guide Scripts & Answer Key

A62 Insider: The Super Guide

1-1. (C) – The woman’s voice does not sound enthusiastic,

but neither does she sound confused. In addition, the

context of the situation does not give any clear indication

that the woman should be confused. The man made the

decision without her knowledge. Therefore the

information must come as a surprise to the woman.

Under the circumstances, it would be logical for her to be

somewhat hesitant in her answer.

1-2. (A) – The man’s tone of voice is clearly unfriendly. In

addition, the woman has stated something fairly obvious

(that they should bring an umbrella since rain has been

forecast) as if it were a very original idea. Under these

circumstances a sarcastic response could be expected

from the man.

1-3. The woman says this in a positive manner. Her voice

retains a high pitch and sounds very cheerful.

Considering this, what she most likely means is that

John is very unique.

2-1. (B) – The way that the man stresses the word “I” in his

response suggests that his opinion is different from the

woman’s. Therefore, the woman must have agreed with

the policy.

2-2. (A) – The woman’s tone indicates that she is agreeing

with the man. In addition, the expression “no joke” is

often used in casual English to agree with something

someone has said.

3. The professor has an extremely negative opinion of the

theory. He says that it would be laughable (meaning it

should not be taken seriously) if not for the horrible uses

it was put to. The professor also stresses that the theory

lacks any supporting historical evidence.

Listening Practice 1 _ p.285

1. (B) – The professor sounds very hesitant after the student

asks him about letting her miss class. He says that once

he does that for one student he has to do it for everyone,

suggesting he does not want to make any exceptions to

his policy.

2. (C) – The student says that she is planning to go to

graduate school, and that the professor’s course will help

her prepare.

Building Your Listening Skills _ p.286

1. Do you 2. what did you

3. If 4. How about if you

meets. I heard that you’re pretty strict on the attendance

policy, so I wanted to come and see if you would let me

slide before I signed up for the course. I got a friend in

the class, so I can get all the notes, and I promise I’ll

never be late with any of the work.

Professor: Hmmm … I don’t know … you do that for one

student, pretty soon you have to do that for everyone. Is

this a required course for you?

Student: Not really. But I’m planning on going to grad

school in a year, and this course sounds like it would

really help me prepare.

Professor: What about taking the course a different

semester? It’s offered both spring and fall terms.

Student: This is the only semester I have any room in

my schedule. Next year, it’s going to be all required

classes.

Professor: OK, how about this? How about you sign up

to take it as an audit course, you know for no credit? If

you audit the course, you won’t get a grade, and then the

attendance policy won’t really come into play. It’s not a

required course for you, and it sounds like what you

really want is the experience … so the grade part

shouldn’t be a big deal.

Student: (sounding unsure) Well, I would prefer to take it

for credit, but … if this is the only way, I guess it’ll have to

do.

[ f Script ]

Student (female): Excuse me, Professor Weston? My

name’s Kate Dressler. I’m thinking of taking your Psych.

304 course next semester and I wanted to talk to you

before I did. Have a minute?

Professor (male): Uh, sure, come on in. (slight pause)

So, what did you want to know?

Student: OK, here’s the thing. I really want to take your

course, but I’m doing an internship next semester, and

on Thursdays I have to work at the same time your class

sickening uses that it was put to at the turn of the 20th

century. This theory was used to justify almost every type

of injustice from European domination of colonial holdings

to movements to sterilize the poor and unemployed to

wholesale attempts at genocide.

Girl wants to take prof. course. But has intershp - Thurs

class time

Can she miss class? Knows prof. is strict re: attendance

Prof. not sure – have to do for everyone

Not required course, but will help stdnt prep for grad schl.

stdnt has no time next sem.

Prof. suggstn: take as audit, then no grade = no prob.

for attend

Page 63: Insider: The Super Guide Scripts & Answer Key

LIS

TE

NIN

G

Scripts & Answer Key A63

Listening Practice 2 _ p.287

1. (B) – The professor says that the division of the liver is

based on external appearance, and has nothing to do with

its internal functions. He also tells the students not to pay

too much attention to this distinction. Combined, these

two statements suggest the professor thinks this

distinction is misleading.

2.

Building Your Listening Skills _ p.288

1. Statement from the lecture: The liver is kind of a jack of

all trades.

A “jack of all trades” usually refers to a person who has

many talents. In this context, the phrase is used to

describe the fact that the liver serves many functions.

2. Statement from the lecture: The liver also acts as kind of

a storage area for excess carbohydrates. The liver will

remove some of the carbohydrates from the incoming

blood, store them, and release them later when the body

needs them.

The paraphrased statement is essentially a summary of

these two statements from the lecture.

[ f Script ]

1.Professor (male): The liver is, uh, kind of a jack of all

trades. It performs a number of vital functions within the

body. Number one, it aids in digestion of food by

producing bile, an enzyme fluid that the stomach needs

to dissolve fats. The liver also plays an important role in

processing food.

2.Professor (male): In the liver, carbohydrates are

metabolized into more useful forms and sent to the rest

of the body. The liver also acts as kind of a storage area

for excess carbohydrates. The liver will remove some of

the carbohydrates from the incoming blood, store them,

and release them later when the body needs them.

Finally, the liver removes toxins from the bloodstream,

and converts them to forms that can be safely excreted

from the body. The liver performs several other minor

functions but these are the big three.

[ f Script ]

Professor (male): At 1.4kg, the liver is the largest internal

organ in the human body. The liver rests on the right side

of the body just below the diaphragm, and to the right of

the stomach. It is covered by the visceral peritoneum, a

sheaf-like membrane that protects the liver from friction

caused by rubbing against other organs. Traditionally,

the liver has been considered to be divided into four

lobes, but, uh, I don’t want you to pay too much attention

to this distinction because it’s based primarily on external

appearance and has little to do with the internal functions

performed in these four areas. The liver has the unique

ability to regenerate … uh, no other internal organ has

this ability. It is thought to be the result of a type of stem

cell called a hepatocyte. Anyway, you can remove almost

75% of the liver, and that remaining 25% will regenerate

into a full, functional liver again.

The liver is, uh, kind of a jack of all trades. It performs a

number of vital functions within the body. Number one, it

aids in digestion of food by producing bile, an enzyme

fluid that the stomach needs to dissolve fats. The liver

also plays an important role in processing food. After

food has been fully digested in the large and small

intestines, and the nutrients have entered the

bloodstream, that nutrient-laden blood travels to the liver

where it is further processed. In the liver, carbohydrates

are metabolized into more useful forms and sent to the

rest of the body. The liver also acts as kind of a storage

area for excess carbohydrates. The liver will remove

some of the carbohydrates from the incoming blood,

store them, and release them later when the body needs

them. Finally, the liver removes toxins from the

bloodstream, and converts them to forms that can be

safely excreted from the body. The liver performs several

other minor functions but these are the big three.

Liver largest organ – loctn: below diaphragm, right of

stomach

Covered by visc. pertinm. – protects liver

Liver regenerates – unique ability

Essential functions:

Aids in digestion – makes bile (dissolve fat)

Metabolizes carbohdrts. in blood & send to body

Store area for extra carbohdrts. – release when needed

Yes NoRegulating carbohydrate levels �Regeneration of body tissue �Removal of poisons from the blood �Production of digestive fluids �Production of new blood cells �

Page 64: Insider: The Super Guide Scripts & Answer Key

A64 Insider: The Super Guide

Listening Practice 3 _ p.289

1. (A) – The phrase “you’ve done your homework” is often

used in conversational English to indicate that a person is

knowledgeable about something. This suggests that the

professor thinks the student is correct.

2. (B) – The professor uses the example of a train coming

towards someone to illustrate the Doppler Effect.

Building Your Listening Skills _ p.290

[ f Script ]

1.Professor (female): Today, we’re going to talk about a

particular effect of waves, and how that effect is used in

astronomy. First, remember that we have already said

that all energy in the universe, whether it is the form of

sound, light, or X rays, exists in wave form. Waves, as

we have said before, have both a wavelength and a

frequency, which determine how we perceive that

energy. Uh … can anyone give me an example of that?

Student (male): Well, like with sound waves, uh, longer

wavelengths are lower tones, and shorter wavelengths

are higher tones. Or in light, shorter wavelengths

correspond to colors closer to blue, and longer ones

account for colors like red.

2.Professor (female): So, the effect I want to talk about is

called the Doppler Effect. The Doppler Effect is a

QQuueessttiioonn �� –– RReeppllaayy

f PPrrooffeessssoorr:: Uh … can anyone give me an example of that?

SSttuuddeenntt:: Well� like with sound waves� uh� longerwavelengths are lower tones� and shorter wavelengthsare higher tones� Or in light� shorter wavelengthscorrespond to colors closer to blue and longer onesaccount for colors like red�

PPrrooffeessssoorr:: You’ve done your homework� great�

[ f Script ]

Professor (female): Today, we’re going to talk about a

particular effect of waves, and how that effect is used in

astronomy. First, remember that we have already said

that all energy in the universe, whether it is in the form of

sound, light, or X rays, exists in wave form. Waves, as

we have said before, have both a wavelength and a

frequency, which determine how we perceive that

energy. Uh … can anyone give me an example of that?

Student (male): Well, like with sound waves, uh, longer

wavelengths are lower tones, and shorter wavelengths

are higher tones. Or in light, shorter wavelengths

correspond to colors closer to blue, and longer ones

account for colors like red.

Professor: You’ve done your homework, great. So, the

effect I want to talk about is called the Doppler Effect.

The Doppler Effect is a phenomenon in which the

apparent wavelength of a wave relative to a stationary

observer changes when the source of the wave is in

motion. Uh, let’s look at an example real quick. You’re

standing next to a railroad track, and you hear the sound

of a horn from a train that is coming towards you. Now,

as that train approaches, the tone of the horn seems to

get higher and higher. Once the train reaches you and

starts to move away, the tone of the horn seems to get

lower. Now, the sound wave isn’t really changing, but

your perception of the wave is, because its source is in

motion. So that’s the basic idea of the Doppler Effect.

When a wave source is moving towards you, its

wavelength seems to get shorter. When the object is

moving away, the wavelength seems to get longer.

Student (male): So, what does this have to do with

astronomy?

Professor: Well, remember that I said all energy in the

universe exists as a waveform, including light and X-rays

… uh, the two energy forms that most of our

astronomical observations are based on. So, sometimes,

we observe an object, let’s say a star, and the light

coming from that star seems redder than it should be.

This, by the way, is called redshift. Well, because of the

Doppler Effect, we know that star is moving away from

us, and by measuring the amount of the redshift we can

even determine the speed at which it is moving away

from us.

All energy = wave

wavelength & freq. – perceptn. of energy

ex. sound - long wavelength = low tone, short = high

light - short wavelength = blue light , long = red

Doppler efct. – wave source moving = change in perctn.

of energy

Ex. train coming sound gets higher, leaving sound

gets lower

Use in astronomy:

Some stars too red (called redshift) = star is moving

away from us

Can measure redshift to find speed of star

Page 65: Insider: The Super Guide Scripts & Answer Key

LIS

TE

NIN

G

Scripts & Answer Key A65

1. (B) – A is incorrect because the professor never actually

mentions the Doppler Effect in this section. The students

need to understand the behavior of waves in order to

understand the professor’s explanation of the Doppler

Effect, so the professor reviews this information first.

2. (A) – The professor uses the example of the train to help

her explain the Doppler Effect, which is, in effect, the link

between wavelength and the perception of energy.

3. This section of the discussion describes the how the

Doppler Effect can be used in astronomy. At the

beginning of the lecture, the professor says that she will

talk about an important effect of waves (the Doppler

Effect) and how it is used in astronomy. Since she has

just finished explaining what the Doppler Effect is, it

makes sense that she will now discuss how it is used.

Furthermore, the student prompts this part of the

discussion by asking the professor what the Doppler

Effect has to do with astronomy.

| Vocabulary Review | _ p.291

1-1. inherently 1-2. metabolize

1-3. regenerate 1-4. excrete

1-5. subjugate

2-1. are off limits

. My doctor says that I have to lose some weight, so I

guess I can’t have any fatty foods or sweets for a while.

2-2. beef up

. In an attempt to improve its international reputation,

the nation has begun to take a more active role in

mediating foreign conflicts.

2-3. let her slide

. The police officer caught Alice speeding, but he did

not give her a speeding ticket.

| iBT Practice | _ p.292~293

[ f Script ]

Student (male): Hi, Professor Bennett, are you busy

now?

Professor (female): Tom! Hi, no, come on in. I always

got time for a good student.

Student: (a little embarrassed) Thanks. Uh, the other

day I saw up on the department bulletin board that you

were looking for research assistants this summer, and I

wanted to check up on that.

Professor: Oh, OK. Well, let me give you a quick run

down of what I need, and then I’ll answer any questions

you have. I’ll probably have a couple for you, too.

Student: OK.

Professor: What I’m going to be doing this summer is

running a study analyzing the relationship between

income, education, and voting patterns. What I need are

people to help me collect the data.

Student: (sounding disappointed) Oh, so you’re really

only looking for surveyors?

Professor: Well, surveying is part of it. I’ll probably try to

get about 10 students or so to help me with that part. But

I’ll also need a couple students, maybe two or three, who

can help me crunch the numbers. Uh, do you have any

background in statistical math?

Student: I’ve taken Statistics 101 and 201 so far.

Professor: That should be enough. Who were your

professors? Do you mind if I talk with them?

Student: No, that’s fine. I did OK in both classes. I had

Williams for 101 and Hall for 201.

Professor: OK, the next thing would be the scheduling.

I’m going to need people every evening for about a week

phenomenon in which the apparent wavelength of a

wave relative to a stationary observer changes when the

source of the wave is in motion. Uh, let’s look at an

example real quick. You’re standing next to a railroad

track, and you hear the sound of a horn from a train that

is coming towards you. Now, as that train approaches,

the tone of the horn seems to get higher and higher.

Once the train reaches you and starts to move away, the

tone of the horn seems to get lower. Now, the sound

wave isn’t really changing, but your perception of the

wave is, because its source is in motion. So that’s the

basic idea of the Doppler Effect. When a wave source is

moving towards you, its wavelength seems to get

shorter. When the object is moving away, the wavelength

seems to get longer.

3.Student (male): So what does this have to do with

astronomy?

Professor (female): Well, remember that I said all energy

in the universe exists as a waveform, including light and

X-rays … uh, the two energy forms that most of our

astronomical observations are based on. So, sometimes,

we observe an object, let’s say a star, and the light

coming from that star seems redder than it should be.

This, by the way, is called redshift. Well, because of the

Doppler Effect, we know that star is moving away from

us, and by measuring the amount of the redshift we can

even determine the speed at which it is moving away

from us.

Page 66: Insider: The Super Guide Scripts & Answer Key

A66 Insider: The Super Guide

1. Main Idea Question – (C)

The student says that he saw a sign on the bulletin board

saying that the professor is looking for students to help her

with a research project this summer, and that he wishes to

help her.

2. Identifying Relationship Question

3. Stance/Attitude Question – (B)

When the professor describes the first part of the research

project, the student sounds rather disappointed, and asks

her if it is only surveying work. In addition, once the

professor explains that there are other things she needs

help with, the student seems to be more interested and

enthusiastic. This suggests that he wishes to do

something more important than surveying.

4. Supporting Detail Question – (A)

Although the professor never directly states this, it is

clearly implied. She first asks the student what math

courses he has taken, and then says that the courses he

has taken should be enough. She then asks if she can

speak to his professors. The clear implication is that she

wishes to find out how the student did in these courses to

determine if he is good enough in math to help her. This is

supported by the fact that the student tells the professor

that he got good grades in both courses in response to

her request.

5. Function Purpose Question – (C)

The question immediately follows the professor’s

explanation of the work schedule for the student

assistants. This clearly implies that she wants to know if

the student can help her during these times.

Listening for Total Comprehension _ p.294

[ f Script ]

Professor (male): It is one of history’s great ironies that

one of the nations most heavily involved in the trans-

Atlantic slave trade was also the nation most

instrumental in its abolition. By the early 18th century,

with Spanish naval power in sharp decline, England had

taken a leading role in the importation of African slaves

for sale not only in its own colonies, but also in those of

France and Spain as well. Thus, England was the

biggest player in the slave trade at a time when that

trade was reaching its height. Yet, in 1808, England

outlawed its own slave trade, and then, over the next 30

years, took an active role in pressuring other European

nations to end theirs as well.

While historians have long debated England’s motives in

ending its slave trade, none have debated the impact of

the act itself. While black market slave trading still

occurred, the flow of African slaves to the New World

significantly decreased. This, in turn, eventually brought

an end to slavery in most of the Caribbean colonies, and

it was only in the United States that slavery continued to

thrive.

QQuueessttiioonn �� –– RReeppllaayy

f PPrrooffeessssoorr:: OK� the next thing would be the scheduling�I’m going to need people every evening for about aweek to conduct the surveys and I’ll need the numbercrunchers two afternoons a week for pretty much thewhole summer� Is that doable for you? Are youworking this summer?

f PPrrooffeessssoorr:: Are you working this summer?

to conduct the surveys, and I’ll need the number

crunchers two afternoons a week for pretty much the

whole summer. Is that doable for you? Are you working

this summer?

Student: I got a summer job, but it’s just working for my

uncle’s construction company. I can pretty much take off

whenever I want. That won’t be a problem.

Professor: Great. Uh, now you know that this is unpaid

work, right? I’d like for it be paid, but my research grant

just isn’t enough to cover that.

Student: Yeah, I know there’s no money. Again, I’m not

really worried about that. I just thought it would look good

on my application to grad schools.

Professor: Well, that it will. I just wanted to make sure

you understood the situation. OK, so far everything looks

like this will work out well. Just let me talk to your

statistics professors, and then I’ll give you an answer.

OK?

Student: Sure, no problem. Thanks again for your time.

Yes NoThe work will be unpaid. �All researchers will need to assist

�with the analysis of dataMost research positions will only

�last a week.Only political science majors will

�be accepted.The experience will be valuable

�to students in the future.

Page 67: Insider: The Super Guide Scripts & Answer Key

LIS

TE

NIN

G

Scripts & Answer Key A67

1. England was the leading nation in importing and selling

slaves to New World colonies.

2. England first outlawed its own slave trade, and then

began to pressure other nations to do the same.

Function Questions

Basic Drills _ p.298

1-1. The woman has just invited the man to go to a baseball

game but he does not have any money. Telling the

woman that he does not get paid until next week is a

way of turning down her invitation and explaining why

he is turning it down.

1-2. The man is finished eating. By telling the waitress that

he is done with this (meaning the plate), he is telling her

that she should take the plate away.

1-3. The man has just told the woman that he intends to

make a stop on the way to take her to the airport, but

she has very little time before her flight. By reminding

him of her flight time, she is objecting to his plan.

2. The woman’s response is not meant to be taken literally.

The implied meaning is that she does not know where

the building is. Several context clues should help you

understand this. First, the woman’s voice is rather

tentative and unsure. Second, the man’s response

(Thanks anyway) indicates that she has not been able to

help him.

[ f Script ]

1-1.Woman: Hi, Mike. How are you?

Man: Good, you know … nothing new really. What’s up?

Woman: Oh, I just called to tell you a bunch of us are

going to the baseball game on Friday and I wanted to

know if you wanted to come.

Man: Ah, I don’t get paid until next week.

1-2.Man: Excuse me, Miss? I’m done with this.

1-3.Man: Oh, I just remembered, I have to swing by the post

office on the way to the airport and drop off these letters.

Woman: (sounding a little annoyed) George, my flight

leaves at 3, remember?

2.Man: Hi, excuse me. I was looking for the engineering

hall. It’s supposed to be over on this side of campus

somewhere. Could you tell me where it is?

Woman: (sounding unsure) Oh … that’s a good question.

Man: OK. Well, thanks anyway.

Chapter 8It is one of history’s great ironies that one of the nations

most heavily involved in the trans-Atlantic slave trade

was also the nation most instrumental in its abolition.

By the early 18th century, with Spanish naval power

in sharp decline, England had taken a leading role in

the importation of African slaves for sale not only in

its own colonies, but also in those of France and Spain

as well. Thus, England was the biggest player in the

slave trade at a time when that trade was reaching its

height. Yet, in 1808, England outlawed its own slave

trade, and then, over the next 30 years, took an active

role in pressuring other European nations to end theirs

as well.

While historians have long debated England’s motives

in ending its slave trade, none have debated the impact

of the act itself. While black market slave trading still

occurred, the flow of African slaves to the New World

significantly decreased. This, in turn, eventually

brought an end to slavery in most of the Caribbean

colonies, and it was only in the United States that

slavery continued to thrive.

Page 68: Insider: The Super Guide Scripts & Answer Key

A68 Insider: The Super Guide

Listening Practice 1 _ p.299

1. (D) – The additional statement “That’s the university

policy” has two purposes. First, it reconfirms that there is

no way the student can get his money back. But it also

serves to distance the director from her refusal. It implies

the director has no choice in the matter, and is simply

following university policy.

2. (C) – Her example is intended to show the student how

much he could lose if he waits to get a new card. Her

question is intended to reinforce this idea.

Building Your Listening Skills _ p.300

1. couldn’t even 2. what you all

3. Do you 4. get your

Listening Practice 2 _ p.301

[ f Script ]

Professor (male): It’s not going to surprise anyone ...

when I say that most flowers rely on bees to be

pollinated. And we also know why bees are interested in

the flowers in the first place. They get some nectar for

their troubles. So, my question is, would a bee still visit a

flower ... if it had no nectar?

Student A (female): Probably not. I mean, what would be

in it?

QQuueessttiioonn �� –– RReeppllaayy

f DDiirreeccttoorr:: Hey� we’ve had cases where people waitedtoo long before reporting their lost card ��� and endedup losing their entire savings account and owing moneyfor purchases made on it besides� You wouldn’t want toend up like them� would you?

f DDiirreeccttoorr:: You wouldn’t want to end up like them�would you?

QQuueessttiioonn �� –– RReeppllaayy

f DDiirreeccttoorr:: I’m afraid not� Sorry� That’s the universitypolicy�

[ f Script ]

Director of Security (female): Hello, what can I do for

you today?

Student (male): (hesitating) Well ... I have a problem

concerning my ID card. I went to the dining hall to eat

today, and when I reached in my pocket to get my card, it

was gone. I mean, I always carry it with me. Anyway, I

went back to my dorm and at first I couldn’t even get in

my building. I had to call the house manager to let me in

and, well ... to make a long story short, it looks like my

card’s been lost or stolen. The house manager said I

should come over here and see what you all can do.

Director: Hmmm ... I see. Well ... the main problem

would be ... if your card was stolen. It’s possible whoever

stole it could use it to charge something at the campus

store, take books from the library, or use it to gain

entrance to your dorm. The first thing we’ll have to do ...

is deactivate it. Understand what I mean?

Student: (not understanding the problem) Not really.

Director: Make it invalid, so it can’t be misused.

Student: Hold on a second. What if someone finds it ... and

turns it in? Then I’d have a card that wouldn’t work anyway.

Director: I’m sorry, but the university can’t take that

chance. We simply have to deactivate it, and then we

can issue you a new card.

Student: (surprised) Really? How long will that take?

Director: No time at all. We can get your new ID within

an hour ... depending on how many people are waiting.

But it’ll cost you $25.

Student: $25! That’s something I can’t really afford right

now.

Director: No problem, we can take it out of your student

deposit. You’ll just have to fill out some forms. OK?

Student: (accepting her proposal) But what if I find it

later or someone turns it in? Can I get this money back?

Director: (firmly) I’m afraid not. Sorry. That’s the

university policy. Look, if your card has really been

stolen, you could be losing a lot more than $25. Sure,

you could have us wait before we deactivate it, but the

longer we wait ... the more you risk losing money. Hey,

we’ve had cases where people waited too long before

reporting their lost card ... and ended up losing their

entire savings account and owing money for purchases

made on it besides. You wouldn’t want to end up like

them, would you?

Student: No, I suppose not. Okay, so where do I go to

have a card made?

Stdnt: Lost ID card, couldn’t get in dorm, house

mangr. told to come secrty. off.

Dir: if card stolen, other person can use so have to

deactivate

Stdnt: doesn’t want to, but Dir: insists

Cost = $25 but stndt doesn’t have – not prob. Can take

from stndt deposit

Stdnt can’t get $ back. Stdnt should replace soon to

avoid risk

Page 69: Insider: The Super Guide Scripts & Answer Key

LIS

TE

NIN

G

Scripts & Answer Key A69

1. (B) – The student’s response contains an implied concept.

She asks “what would be in it,” but what she really means

is “what would be in it for the bee.” In other words, what

would a bee get by visiting a flower with no nectar?

2. (A) – The professor knows that if the student thinks about

what they have previously said about the memory of

bees, he will realize that his statement is very unlikely.

Building Your Listening Skills _ p.302

[ f Script ]

1.Professor (male): It’s not going to surprise anyone ...

when I say that most flowers rely on bees to be

pollinated. And we also know why bees are interested in

the flowers in the first place. They get some nectar for

their troubles. So, my question is, would a bee still visit a

flower ... if it had no nectar?

Student (female): Probably not. I mean, what would be in

it?

QQuueessttiioonn �� –– RReeppllaayy

f SSttuuddeenntt BB:: So are you saying that bees are incapable oflearning which flowers have nectar ��� and whichflowers don’t?

PPrrooffeessssoorr:: You remember in the last class we talkedabout the memory of bees?

SSttuuddeenntt BB:: Yeah� you said bees have a very goodmemory … uh� can remember the precise directions toa nectar source that is miles away� So … I guess theyshould be able to figure this out� right?

f PPrrooffeessssoorr:: You remember in the last class we talkedabout the memory of bees?

QQuueessttiioonn �� –– RReeppllaayy

f PPrrooffeessssoorr:: They get some nectar for their troubles� So�my question is� would a bee still visit a flower ��� if ithad no nectar?

SSttuuddeenntt AA:: Probably not� I mean� what would be in it?

f SSttuuddeenntt AA:: Probably not� I mean� what would be in it?

Professor: Yeah, that seems to be the sensible answer,

but there are flowers that depend on bees that don’t give,

you know, any nectar in return. Darwin himself was

puzzled by this and ... he always thought that there must

be nectar in these flowers, even if he couldn’t find it.

Well, in this case, he was wrong ... because these

flowers had no nectar. And most of these flowers are

from the orchid family.

Student B (male): So, are you saying that bees are

incapable of learning which flowers have nectar ... and

which flowers don’t?

Professor: You remember in the last class we talked

about the memory of bees?

Student B: Yeah, you said bees have a very good

memory … uh, can remember the precise directions to a

nectar source that is miles away. So … I guess they

should be able to figure this out, right?

Professor: Exactly, it’s not an issue of the bees not being

able to learn which flowers don’t produce nectar.

Student A: So, in that case there are two puzzles. The

first is how orchids can attract a bee without giving nectar

and ... the second is why the bee can’t learn this

association.

Professor: Well, actually the big problem is, you know,

why flowers would evolve that didn’t produce nectar in

the first place. Most flowers do. In other words, is there

any advantage to the flower in not giving nectar? Let’s

look at how orchids attract bees. One orchid looks like,

um, a female bee. When the male bee comes down to

mate with it, it carries off some of the orchid’s pollen.

Another orchid produces, you know, an aroma that is

similar to that produced by female bees. And when the

male bee goes to investigate, it also carries off some

pollen. Now orchids often produce several blossoms on

the same plant. One plant, the green-winged orchid,

produces many blossoms and no nectar. So when the

bee visits the plant and finds no nectar, he does not visit

this same plant again.

Now, the flowers on the orchid could self-pollinate. Many

plants have this capability. But there’s a problem. Self-

pollination tends to lead to weak, sickly seeds. It’s not the

best alternative. If the bee found nectar in one blossom,

he’d probably visit all the blossoms on the same plant

and ... we’d end up with self-pollination. See what I

mean? So, when the bee gets frustrated and flies off,

there is no self-pollination. Instead the bee flies off in

search of other flowers and ends up cross pollinating the

orchids, leading to better genetic diversity.

Most flower need bees for pollintn. so produce nectar,

orchids need bees but no nectar

Bees have excellent mem. How do orchids attract bees?

Why can’t bees learn?

Orchids smell like female bees, so attract males.

Orchids use bees b/c cross pollintn better than self-

pollintn.= more genetic divrsty.

Page 70: Insider: The Super Guide Scripts & Answer Key

A70 Insider: The Super Guide

1. Statement from the discussion: Yeah, that seems to be

the sensible answer, but there are flowers that depend

on bees that don’t give, you know, any nectar in return.

The professor says this in response to the student’s

statement that bees probably wouldn’t visit flowers with no

nectar. If the student’s answer seems sensible, then the

fact that some flowers rely on bees but don’t produce

nectar must seem illogical.

2. Statement from the discussion: If the bee found nectar in

one blossom, he’d probably visit all the blossoms on the

same plant and ... we’d end up with self-pollination.

This is a pretty direct paraphrase. Only minor vocabulary

changes have been made.

Listening Practice 3 _ p.303

QQuueessttiioonn �� –– RReeppllaayy

f PPrrooffeessssoorr:: Passive solar technology relies on directsunlight to create heat� This is by far the most commonuse of solar power� I mean you’ve all opened thecurtains on a cold winter day to let the sunlight in andlet it warm up the house before� right?

f PPrrooffeessssoorr:: I mean you’ve all opened the curtains on acold winter day to let the sunlight in and let it warm upthe house before� right?

time they started to get interested, the oil prices fell back

a little and … well, you know people generally don’t

change unless they have to. As a country, our dedication

to developing solar energy has not been very strong.

Yet, some development has continued. With interest in

the environment growing, it seems like the cleanest way

to produce energy for our homes. And more and more

people are working solar principles into housing design.

I’d like to divide solar energy use into two classes that I’ll

call passive and active. Now, passive solar uses the

direct sunlight to accomplish some kind of heating, while

active solar converts sunlight into electricity and uses it in

a variety of ways.

Passive solar technology relies on direct sunlight to

create heat. This is by far the most common use of solar

power. I mean you’ve all opened the curtains on a cold

winter day to let the sunlight in and let it warm up the

house before, right? Well, you may not have know it, but

you were actually using passive solar heating. Now, you

can imagine that if you had large, south facing windows,

you could capture even more of the sun’s heat. And

that’s the basic principle ... behind housing designs that

use passive solar heat. So, of course, they want to

capture as much of the heat from the sun that they can.

Not only are large windows important, but you also need

the sunlight to fall on material that will readily absorb the

heat.

Now, the more active solar uses, the ones that produce

electricity by using photovoltaic cells, are also far more

expensive. Don’t expect this kind of solar power to pay

for itself for a long, long time. That probably explains why

this type of solar energy hasn’t become very widespread.

Also, what happens when the sun goes down? Well, you

don’t produce any energy. You have to use batteries to

store energy for you to use at night. Unfortunately, these

batteries are not cheap, and they lose a lot of the energy

during the storage process.

[ f Script ]

Professor (male): Probably when most of you hear of ...

solar power, you probably think it’s a relatively recent

development. But way back in the 1890s, solar power

was booming. In some places in California, more than

30% of the homes had solar water heaters, and it looked

like the US was well on its way to a solar future.

But ... then something happened to change all this. By

1920, huge gas ... and oil deposits were found

throughout the western US. This meant that there was

going to be cheap fuel for everyone and interest in solar

power decreased.

But then in 1973, during the first oil shock, as it’s

sometimes called, people realized ... how dependent the

U.S. had become on oil. Suddenly people began once

more to look at the possibility of ... solar power. But every

Professor: Yeah, that seems to be the sensible answer,

but there are flowers that depend on bees that don’t give,

you know, any nectar in return. Darwin himself was

puzzled by this and ... he always thought that there must

be nectar in these flowers, even if he couldn’t find it.

2.Professor (male): Now, the flowers on the orchid could

self-pollinate. Many plants have this capability. But

there’s a problem. Self-pollination tends to lead to weak,

sickly seeds. It’s not the best alternative. If the bee found

nectar in one blossom, he’d probably visit all the

blossoms on the same plant and ... we’d end up with

self-pollination. See what I mean? So when the bee gets

frustrated and flies off, there is no self-pollination. Instead

the bee flies off in search of other flowers and ends up

cross pollinating the orchids, leading to better genetic

diversity.

Page 71: Insider: The Super Guide Scripts & Answer Key

LIS

TE

NIN

G

Scripts & Answer Key A71

1. (A) – The professor states that the oil shock of the 1970s

briefly generated renewed interest in solar power, but that

oil prices dropped enough for people to lose interest again.

2. (A) – Just before this question, the professor states

passive solar is the most common usage of solar energy.

His assumption that all students have done this before is

meant to reinforce his point.

Building Your Listening Skills _ p.304

1. (A) – The rest of the lecture discusses the two separate

types of solar power. This section of the lecture simply

notifies the students of this.

2. (C) – This section of the lecture is primarily concerned with

defining the term passive solar, which the professor does

through examples.

3. The purpose of this section is primarily to describe the

drawbacks that have prevented active solar power from

becoming more widespread.

| Vocabulary Review | _ p.305

1-1. density 1-2. photovoltaic

1-3. sensible 1-4. passive

1-5. aroma 1-6. ambient

2-1. swing by

. The clinic’s main hours are from 10am to 4pm, but it

remains open late into the night so that people can

come when it is most convenient for them.

2-2. be booming

. The stock market may be growing rapidly now, but I

doubt it will last.

| iBT Practice | _ p.306~307

[ f Script ]

Professor (male): Did you know that space has a

temperature? Yeah, I know it’s weird to think about. I

mean, we think of space as this vast, cold emptiness, but

it’s true … space really does have a temperature …

about 2.7 degrees Kelvin to be exact. Uh, for those of

you who don’t know, Kelvin is the temperature scale

frequently used by physicists. It’s based on the

movement of atoms. Uh, zero degrees Kelvin, or

absolute zero, is the total absence of heat, in Celsius

that’s minus 275 degrees. So, yeah, space is pretty darn

cold, but it does have a measurable temperature. Today,

we’re going to talk about where that temperature comes

from, and what it tells us about the universe.

That temperature is part of cosmic microwave

background radiation, sometimes known by its initials,

CMB. CMB is an ambient energy source that emits over

the entire energy spectrum, from heat all the way up to

microwaves, where it reaches its strongest point. It

comes from the Big Bang … uh, actually you can think of

CMB as kind of the echo of the Big Bang. Hmmm …

maybe it’s better if I start at the beginning. Right after the

Big Bang, matter was ejected at super high

temperatures, uh, billions or maybe even trillions of

degrees Kelvin. At these temperatures atoms can’t form.

The universe was just a kind of hot soup of electrons and

[ f Script ]1.Professor (male): I’d like to divide solar energy use into

two classes that I’ll call passive and active. Now, passive

solar uses the direct sunlight to accomplish some kind of

heating, while active solar converts sunlight into

electricity and uses it in a variety of ways.

2.Professor (male): Passive solar technology relies on direct

sunlight to create heat. This is by far the most common use

of solar power. I mean you’ve all opened the curtains on a

cold winter day to let the sunlight in and let it warm up the

house before, right? Well you may not have know it, but

you were actually using passive solar heating. Now, you

can imagine that if you had large, south facing windows,

you could capture even more of the sun’s heat. And that’s

the basic principle ... behind housing designs that use

passive solar heat. So, of course, they want to capture as

much of the heat from the sun that they can. Not only are

large windows important, but you also need the sunlight to

fall on ma.terial that will readily absorb the heat.

3.Professor (male): Now, the more active solar uses, the

ones that produce electricity by using photovoltaic cells,

are also far more expensive. Don’t expect this kind of

solar power to pay for itself for a long, long time. That

probably explains why this type of solar energy hasn’t

become very widespread. Also, what happens when the

sun goes down? Well, you don’t produce any energy. You

have to use batteries to store energy for you to use at

night. Unfortunately, these batteries are not cheap, and

they lose a lot of the energy during the storage process.

Solar power not new – 1890s – California 30% solar

1920s found much gas, so no interest in solar

1970s oil shock restarted interest, but not a lot

US not dedicated to solar, but some use in house design

Passive solar – sunlight creates heat

Ex. large S. windows to warm house

Use large windows & matrl. absorbs heat

Active solar – sunlight create electrcty. – phtotovoltaic cell

Very expensive, not widespread

Nighttime no electricy – storage bats. expensive

Page 72: Insider: The Super Guide Scripts & Answer Key

A72 Insider: The Super Guide

1. Main Idea Question – (B)

The main topic of the lecture is the discovery of CMB and

what it tells us about the universe.

2. Identifying Relationship Question

3. Supporting Detail Question – (C)

The professor states that although CMB was predicted by

the Big Bang theory, there was not much interest in it until

the 1960s, when scientists at Bell Laboratories were

searching for the source of satellite interference.

4. Organization Question��� The origins of CMB�� The discovery of CMB� The effects of CMB on life on Earth�� The scientific implications of CMB� The arguments in opposition to CMB

5. Supporting Detail Question – (B)

Since CMB was predicted by the Big Bang theory, its

discovery strongly suggests that the theory is correct.

6. Stance/Attitude Question – (A)

The professor realizes that he has not provided the

students with enough information about what exactly CMB

is, so he gives them more background information by

explaining how it was created.

QQuueessttiioonn �� –– RReeppllaayy

f PPrrooffeessssoorr:: CMB is an ambient energy source that emitsover the entire energy spectrum� from heat all the wayup to microwaves� where it reaches its strongest point�It comes from the Big Bang … uh� actually you canthink of CMB as kind of the echo of the Big Bang�Hmmm … maybe it’s better if I start at the beginning�

f PPrrooffeessssoorr:: … maybe it’s better if I start at the beginning�

protons. After about 400,000 years, the temperature was

down to around 3,000° Kelvin, and at this point atoms

began to form. Further cooling came with further

expansion, until the temperature of space reached its

current level of 2.7 degrees Kelvin. So CMB is literally

the heat left over from the Big Bang.

So how do we know this? Well, one of the predictions of

the Big Bang theory, first proposed in the 1920s and

further developed in the late 40s, was that there should

be some form of background radiation left over from the

Big Bang. For quite a while, not many people were very

interested in looking for this background radiation

because the Big Bang theory wasn’t taken very seriously

by a lot of people. It wasn’t until the space age that

interest in background radiation grew again. But it wasn’t

because people were interested in the Big Bang. It was

… well, you see there was some form of background

radiation that was interfering with satellite

communications, and scientists were interested in finding

its source. Was it coming from the sun? From a nearby

galaxy? Nobody really knew. So a group of scientists

working for Bell Laboratories began surveying the sky to

find its source. Basically, they looked for the direction

from which the radiation was the strongest. Logically,

that should be the direction of the source. But, uh, to

their amazement … and everyone else’s too, the

radiation was of uniform intensity in every direction. It

existed everywhere in space. The only explanation for

this was that this was the background radiation predicted

by the Big Bang.

OK, on to what CMB tells us about the universe. Well,

first, and most importantly, it’s the best evidence we have

to date that the Big Bang theory is correct. Now, that

doesn’t mean that the Big Bang theory is an undeniable

fact … maybe future discoveries will disprove it. But the

presence of CMB, and the lack of any other way to

explain it, makes the Big Bang theory the best

explanation for the origin of the universe that we have to

date. CMB also gives us clues as to how the first

galaxies formed. That’s because, uh … because CMB

isn’t exactly equal in every region of space. There are

parts that are slightly warmer or slightly colder. The

difference isn’t much, only a fraction of a degree, but it’s

significant. It tells us that there were, uh … lumps in the

Big Bang, uh, areas of greater density. And it’s these

areas of greater density that we think formed the first

galaxies.

True FalseCMB is of nearly equal strength in

�any direction of the sky.CMB levels fluctuate widely. �CMB is decreasing in temperature

�as the universe grows.The evidence for CMB remains

�largely speculative.

Page 73: Insider: The Super Guide Scripts & Answer Key

LIS

TE

NIN

G

Scripts & Answer Key A73

Listening for Total Comprehension _ p.3081. They attempt to claim refugee status because US and

Australian law prohibit deporting refugees if there is a

risk they will be killed in their home country.

2. The second problem is that boat people often seek to

cross into the US and Australia in very unsafe boats, and

the governments of the two countries don’t want people

to risk their lives in this way.

[ f Script ]

Professor (female): In modern times, both the United

States and Australia have had to deal with the issue of

so called “boat people.” Simply put, boat people are

citizens of another nation, generally an impoverished

one, who seek a better life in the United States or

Australia. The catch is that, lacking visas, boat people

seek to enter these nations illegally through their poorly

patrolled shorelines.

Boat people present two problems for the governments

of the United States and Australia. The first is that boat

people often seek to claim refugee status once they have

reached land. According to US and Australian law,

refugees cannot be forcibly returned to their country if

that will place their lives at risk. Thus, there must be long

and often expensive legal proceedings to determine the

risk to boat people before they are deported. The second

problem is that the overseas crossing itself is extremely

dangerous. Coming from impoverished countries, the

boats used to reach the US and Australia are often old

and dangerous. Therefore, the governments of the US

and Australia wish to discourage boat people from risking

their lives in a dangerous and illegal crossing.

In modern times, both the United States and Australia

have had to the deal with the issue of so called “boat

people.” Simply put, boat people are citizens of another

nation, generally an impoverished one, who seek a

better life in the United States or Australia. The catch is

that, lacking visas, boat people seek to enter these

nations illegally through their poorly patrolled

shorelines.

Boat people present two problems for the governments of

the United States and Australia. The first is that boat

people often seek to claim refugee status once they have

reached land. According to US and Australian law,

refugees cannot be forcibly returned to their country if

that will place their lives at risk. Thus, there must be

long and often expensive legal proceedings to determine

the risk to boat people before they are deported. The

second problem is that the overseas crossing itself is

extremely dangerous. Coming from impoverished

countries, the boats used to reach the US and Australia

are often old and dangerous. Therefore, the

governments of the US and Australia wish to

discourage boat people from risking their lives in a

dangerous and illegal crossing.

Page 74: Insider: The Super Guide Scripts & Answer Key

P A R T ABrainstorming and Note Taking

Exercise 3 _ p.323

1. milty. forced to spending gov.cutbacks

2. Many survival strat. early humans = mod. apes

3. Freshman grades expectns univ. # of tutors

4. 1940-50 pub. desire for represntl. and - abstract art

5. Tall trees advantg small trees b/c more sun

6. Oil explortn. proj. use 30% compy’s ann. income + are

largest expendtr.

7. Ape monekys b/c apes no tail, large size, more intel.

8. Atom clocks used when exact time crucial b/c more

accurate conv. clocks

Exercise 4 _ p.324

1. Availbty drinking H2O pop. growth + polutn

2. Univ. planning revise stdn sthl. policy several

incidents.

3. World Bank overseas econ. dev. in 3rd world natns.

4. Adol. behvr. hormone levls.

5. Life of star linked w/ size; large size = short life

6. Stdn need time for proj. b/c/ sick 2 wks.

7. Emply not follow safety procdrs explosn

8. Hot + cold air collide severe T-storm + tonadoes

9. US war of Indp. indp. movmnts other natns.

10. Hybrid cars emisns. b/c better fuel econ.

11. Good mngrs. prodtvy. w/ incentives + rewds

12. Sand dunes consistent wind; = wave formation

Exercise 5 _ p.325~326

1.

2.

3.

4.

[ f Script ]

1. The availability of drinkable water is decreasing due

to population growth and pollution.

2. The university is planning to revise its policy on

student athletics in response to several embarrassing

incidents.

3. The World Bank overseas economic development

projects in many 3rd world nations.

4. Adolescent behavior is often the result of the

increased levels of hormones in their bodies.

5. The lifespan of a star is linked to its size; the larger

the star, the shorter its lifespan.

6. The student needs more time to work on her report

because she was sick for two weeks.

7. The employee’s failure to follow company safety

procedures led to the deadly explosion.

8. When masses of hot and cold air collide, severe

thunderstorms often form and produce tornadoes.

9. The American War of Independence sparked similar

independence movements in other nations.

10. Hybrid cars help reduce car emissions due to their

greater fuel economy.

11. Good managers increase the productivity of their

employees through the use of incentives and

rewards.

12. Sand dunes are formed by a consistent wind, much

the same as how waves are formed.

Chapter 1

SPEAKING Section

A74 Insider: The Super Guide

Next yr, only incoming stdn go orientn active. b/c

upperclassmen more responsibilities Soph, jr. + sr

stdns can choose how many acts. attnd, but

recommend attend many as poss.

Univ. winter brk to 10 days chng to semstr sys.

Short brk. make up for late start

All stdn on campus must report to dorm when class

begins or fined for late.

Pidgin lang. = fusion of 2 langs.

Pidgin 2 groups w/ different lang have reg

contact e.g. trade

Pidgin helps understanding b/w 2 groups.

Common cause = colonization

simplified version of colonizers lang.

Monopoly = one group controls markt or serv.

no competition

B/c no comp. provider sets price + amount, and

doesn’t care re: quality

Will not lose bus. b/c customer has no options

Page 75: Insider: The Super Guide Scripts & Answer Key

Exercise 6 _ p.327

1.

2.

3.

[ f Script ]

Student A (male): Jill, I need to use the computer.

You’ve been on it long enough.

Student B (female): (upset and rambling) Well, I can’t

figure out how to break it to her.

Student A: Break what to whom?

Student B: My mother … she’s going to receive a letter

on Monday telling her that I’m failing all of

my classes.

Student A: Oh. That’s bad, very bad. Well, I don’t think

you should be writing her an email. Outline

how you are going to explain yourself and

then give her a call.

Student B: Oh gosh. I don’t know if I can bear telling

her over the phone. She’ll yell my ear off.

Student A: Hmm … Well, why not go home this

weekend, take her out to dinner and break it

to her lightly when she’s in a good mood?

Student B: Tell her to her face? I’m not sure if I can

muster up the courage.

Student A: Well … any way you go about it, she’s going

to be upset.

Student B: Yeah. But there has to be some way that

would do the least damage to our

relationship. Well, go ahead and use the

computer. I’m going to think about this some

more in my room.

process. A month at least. It could even be

the end of the semester before you know

anything.

Student : That doesn’t help me much right now.

Employee: Well, at the end of the semester you’ll have

an opportunity to change roommates. So,

for the time being, could you stay with your

parents and commute?

Student : It’s an awfully long way to drive. But filing a

complaint doesn’t sound like such a great

idea either, and I’d still have to put up with

my roommate. I’ll give it some thought and

get back to you.

[ f Script ]

Student (male): Is this the Student Affairs Office?

Employee (female): Sure is. How can I help you?

Student : I need to change dorm rooms. My roommate

is driving me crazy.

Employee: Well, if you don’t feel like your RA is able to

handle the problem, you could file a formal

complaint here in the office. We may be able

to move you to a different room, but we can’t

promise anything due to the shortage of

dorms on campus.

Student : Well, I have to do something. Exams are

coming up and I really have to get some sleep.

Employee: Just out of curiosity, how far away do your

parents live?

Student : It’s about a 45-minute drive. Why?

Employee: Well, I shouldn’t be saying this, but

complaints can take an awfully long time to

[ f Script ]

Man : Hey, Tina. Could I ask you for a favor?

Woman: Sure, Matt. What’s up?

Man : Well, I was wondering … Do you think it would

be possible for me to catch a ride with you to

school in the mornings?

Woman: Uh … sure, I guess that’s cool. But why don’t

you want to drive yourself anymore?

Man : It’s not that I don’t want to. It’s that I can’t. That

new campus parking policy says that people

who have two or more parking tickets on

campus can no longer drive to school.

Woman: Wow. That seems kind of harsh.

Man : Tell me about it. I mean I know it’s my fault

that I got the parking tickets, but the school is

totally overreacting. Besides, it’s partially their

fault if you ask me.

Woman: How so?

Man : Well if they had enough parking spaces

around the academic buildings, students

wouldn’t have to park illegally.

Scripts & Answer Key A75

SP

EA

KIN

G

Man needs to ride w/ woman

Man can’t drive to schl. b/c has too many prkng

tcks. + can’t drive on campus

Man thinks schl. policy too harsh ... schl overreact

Man also thinks part schls. fault b/c not enough

parkng near acad. blg. stdn park illegally

Stnd wants chng dorm room b/c roommate

Emp. suggests file complaint but can’t promise new

room b/c shortage on campus

Also suggest stnd stay w/ parents b/c complaints

take long time

Stdn thinks too far to drive but complain also not a

good idea

Page 76: Insider: The Super Guide Scripts & Answer Key

Exercise 7 _ p.328

1.

2.

[ f Script ]

Professor (female): Now, many organisms have lived

and died on this planet. Have you ever wondered what

happens to their remains? Well, it does one of two

things: some of it will continue to decay into inorganic

material, while some of it becomes mineralized, or turned

into stone, by natural forces over a long period of time.

The results of both processes have advanced our

technology and scientific understanding.

Let’s discuss mineralization first. Sometimes when

organisms decay, various minerals will partially or

completely replace the organic particles. Basically what

happens is that the creature’s original shape or structure

is preserved, and we end up with a fossil. Scientists have

used fossils to keep a record of the origins and history of

life, noting the physical changes of plants and animals

throughout history. By comparing the changes a species

undergoes, we have been able to understand the

specifics of evolutionary development. Also, because

similar fossils have been found in different continents, we

know that the continents were once connected. Thus,

fossils help us understand how the earth has changed.

The other process organic material undergoes is further

decomposition. If the organism does this, it will eventually

break down into materials rich in hydrocarbons. We call

them fossil fuels, and they include oil, coal, and methane,

or natural gas. See, hydrocarbons are extremely

combustible and release vast amounts of energy when

burned. Because they are as easy to obtain as fossils,

we use them as our main source of energy. They provide

electricity for our technology in agriculture, industry, and

transportation, just to name a few areas. They also

power the technology we use for research, making more

scientific discoveries possible.

[ f Script ]

Professor (male): To continue our discussion of project

management, let’s take a hypothetical example. Let’s say

you’re a project manager for a cell phone company, and it

is your job to develop a cell phone that uses less power.

Now, you have a total of thirty engineers working for you,

and you have to decide how to use them. One option

would be to have all of them work in one team to design a

phone that meets the project requirements. But what

happens if they are all working on a design and it ends up

not working? Then at best your project is going to

experience major delays and at worst it may fail entirely.

That’s why many smart managers would separate their

engineers into several independent teams, with each

team working on its own separate design. Now working

this way does have some disadvantages. Uh, obviously

developing several designs is going to be a lot more

expensive than developing one. In addition, you’re going

to have more work as a manager because you will have

to oversee each separate team and their design, so

there’s more for you to keep track of than if you had all

your engineers working on the same design. But the big

advantage is that it is highly unlikely that each separate

design will be a complete failure. So by increasing the

number of design teams, you help to guarantee the

overall success of the project.

A76 Insider: The Super Guide

Hypothetical sit: proj mngr has to dev new phone and

has 30 engnrs – how to use them?

Option 1: all work together to make 1 phone design

Prob: If design doesn’t work, proj fails

Smart mngrs make teams of engrs. Each team

works on own design.

Disavatnage: Slower, more expensive, more work as

mngr b/c track all teams

Advantage: More chance for success b/c not all

designs fail

When organisms die 2 options: either mineralize or

decay – both important to science

Mineralization: Minrls replace parts of body,

keep shape of animal = fossil

Fossils help science b/c show chngs in animls +

plants, help undrstnd evoltn. Similar fossils in diff

places show continents connected

Decomposition: animals decay into fossil fuels i.e.

oil, nat. gas

decomp. Provides main source of enrgy.

Stdn has to tell mom failing classes

– mom will get letter on Mon.

Man thinks she should call mom, not email

Also suggests she go home for weekend + tell mom

Page 77: Insider: The Super Guide Scripts & Answer Key

The Basics of Sentence Structure

Exercise 1 _ p.330

1. My younger brother plays basketball for the school.2. The man in my class will most likely go home for the

holidays.

3. Cheetahs, able to run at speeds of 60 mph, are the

fastest animals in all the world.

4. Walking back to his car, Mark found $40 lying on thesidewalk.

5. The hardworking detective miraculously recovered the

money stolen form the bank.

Exercise 2 _ p.331

1. Jack disagreed with the dangerous plan.

2. All reports are due tomorrow at the latest. 3. Reacting to an instinctive urge, the birds flew south for

the winter.

4. Jake awoke from his nightmare frightened and disoriented.

5. All new employees attending the training session are

expected to arrive on time.

6. Korean car makers, having made significantimprovements to their vehicles, are currently enjoying

increased sales in the US.

Exercise 3 _ p.331

1. Having resolved their earlier differences, Jack and

Lisa now work well together.

2. Driven by a strong demand on the foreign market,the fledgling company’s profits increased dramaticallyin the second quarter.

3. Deborah, hoping to finish her coursework before theend of the semester, is working on her master’s thesis

day and night. 4. Tired and exhausted, Martin finally crossed the finish

line nearly 40 minutes after most of the other runners.

5. Using a mixture of stealth and teamwork, the pride of

lions hunts antelope to feed the hungry cubs.

Exercise 4 _ p.333

1. You need to finish the project by Tuesday, or you might

get fired.

2. We have an exam this week, so our teacher won’t give

us one next week.

3. Jack plays football and works after school.

4. Helen won’t donate money to charity, but she will work

as a volunteer.

Exercise 5 _ p.333

1. After receiving several generous donations, the

school decided to significantly increase funding to the

soccer team, and it will also build a new art center.

2. Jessica told Nick his favorite dog had just died, and hebegan to cry uncontrollably.

3. Asteroids do pose a danger to the Earth, but comets,

being of much larger size, pose a much greater one.

4. Germany began to rebuild its army during the 1930s, so

Britain and France, fearing another German attack,

increased their own military spending in response.

Exercise 6 _ p.335

1. Bryan left the party after he saw Kelly there with another

guy.

2. Although Taiwan used to be one of the top exporters to

the US, its exports have been surpassed by those of

Japan and Korea.

3. Even though Greg loves history, he changed his major

to engineering because it is too hard to get a job with a

history degree.

Exercise 7 _ p.335

1. President Nixon was forced to resign in disgrace afterhe was caught illegally spying on his political opponents.

2. Mt. Everest is widely considered the most dangerous

mountain on earth because so many climbers have died

on it. 3. While great white sharks rarely attack humans

intentionally, sometimes they mistake surfers for seals

because from beneath the water, a person on a

surfboard looks a lot like a seal.

Exercise 8 _ p.337

1. Hawks are excellent hunters, but eagles are better onesbecause they have better eyesight than hawks.

2. After Mitch and Jessica got into an argument at the

party, she left crying, so he followed her to apologize.

3. When the weather turns cooler in the fall, trees go into a

dormant state and animals begin to gather extra food

because they are preparing for the winter.

Exercise 9 _ p.337

1. When the carmaker, suffering from decreased sales,

was forced to cut its workforce, many people were left

without jobs, so they organized a rally to protest the loss

of their jobs.

2. Although France has acquired a great amount of art

Chapter 2

Scripts & Answer Key A77

SP

EA

KIN

G

Page 78: Insider: The Super Guide Scripts & Answer Key

from other cultures, it acquired much of this illegallyduring the colonial period and many nations are now

demanding a return of their art.

Exercise 10 _ p.338

I once took a trip to South America when I was in high

school. I had never been out of the country before. I was

unsure of what to expect, but I was very excited. As the day

of the trip approached, I grew more and more excited, and I

could hardly wait. Finally, the day of the trip arrived. I went to

the airport, but I nearly missed my flight because I had

forgotten my passport. Fortunately, I managed to get my

passport from my house and return to the airport in time.

Although the trip had nearly been ruined, the rest of it went

perfectly.

Exercise 11 _ p.339

The woman’s problem is that her computer is broken, andshe needs it to finish her report for her biology class. The

man suggests that she either go to the computer lab to

finish her report or that she borrow a laptop fromsomeone. I think the woman should ask to borrow a laptop

from someone because if she borrows one she can work at

her own pace. She can also work in her own room, and thisis better for her because her room is probably quieter than

the computer lab. Therefore, I think this is the best choice for

her.

Exercise 12 _ p.340

In the eighth grade, I went to Paris with my family. Of course,

we saw a lot of amazing sites, but my favorite part of the trip

was the food. Although I had never tried French food before,

I fell in love with it. There were so many amazing dishes to

try. In the morning there were croissants and quiche, and

there were all kinds of bistros to have lunch in. As much as I

loved these, dinner was even better and I looked forward to

it every day. By the end of the trip, I had probably gained ten

pounds, but I didn’t care. I have my entire life to exercise

and lose weight, but food like that only comes once in a

lifetime.

Exercise 13 _ p.340

In the eighth grade, I went to Paris with my family for twoweeks. Of course, we saw a lot of amazing sites such asthe Eiffel Tower and Versailles, but my favorite part of the

trip was the food. Although I had never tried French food

before, I fell in love with it almost instantly. There were so

many amazing dishes to try. In the morning there were

croissants and quiche, and there were all kinds of cute littlebistros to have lunch in. As much as I loved these, dinner

was even better and I looked forward to it every day morethan anything else. By the end of the trip, I had probably

gained ten pounds from all the food, but I didn’t care. I have

my entire life to exercise and lose weight, but food like that

only comes once in a lifetime.

A78 Insider: The Super Guide

Page 79: Insider: The Super Guide Scripts & Answer Key

Varying Word Choice andGrammar Usage

Exercise 1 _ p.343

1. Scientists agree that the environmental problems we

face are quite severe.

2. Many struggling artists have to consider changing their

profession so they can lead a more comfortable lifestyle.

3. The university is handily located in a part of the city with

many shops and restaurants with moderately priced

items.

4. Getting the job at Mt. Sinai Hospital was the fulfillmentof a lifelong dream to be a distinguished surgeon at one

of the nation’s finest hospitals.

5. The professor doubts whether the student’s rude

comment deserves a response.

Exercise 2 _ p.345

1. In my opinion, the increase in security at airports is a

good thing. It helps keep air passengers safe from

potential disasters, and they should appreciate being

kept safe. Many passengers complain about increased

security because of the inconvenience that it poses for

them, but I believe that a few minutes of inconvenience

are worth it to make the skies safer for passengers. In

the end, increased security may very well be an

inconvenience to most passengers, but it is necessary to

keep them safe.

2. I believe that it is necessary for the government to

establish a minimum wage. Without one, many

employers would take advantage of their employees by

offering them very low wages. Earning lower wages

would force them to compensate for this by working

longer hours to earn enough money to live. When the

employee is forced to do this, his or her labor is being

exploited by the employer because they know that the

employee must somehow earn at least the essential

money that he or she needs to survive.

Exercise 3 _ p.347

1. In conducting research for their final exam papers, it is

important for students to remember to only use reputable

sources.

2. The meager selection of fruits, vegetables, and diary

items provided by the school dining service for

vegetarian students really upsets the woman.

3. Whatever it takes to pacify the angry group of student

protesters, the university is willing to do.

4. His teacher’s preoccupation with assigning difficult

projects rather than teaching interesting material has

upset the student.

5. Taking pity on those they hurt is something that people

with certain types of personality disorders are incapable

of doing, as the professor explains.

6. Orienting themselves with the campus and the

surrounding community is something newly transferred

students need to do before scheduling classes.

Exercise 4 _ p.348

1. In my opinion, the popular vote should be the basis for all

elections. This is, in my view, the most accurate way to

hold elections. The respect it shows for the will of the

people makes the popular vote the best system. This

respect, in my opinion, makes the popular vote the only

sensible way to conduct elections.

2. All citizens should be provided with free medical care by

the government, in my opinion. This is the government’s

responsibility because of the expense of health care,

which is too much for most people. Most people don’t

even go to the doctor when they are sick because of the

cost. If free health care were provided by the

government, however, this would not be the case.

Chapter 3

Scripts & Answer Key A79

SP

EA

KIN

G

Page 80: Insider: The Super Guide Scripts & Answer Key

Organizing Your Response

Exercise 1 _ p.351

Exercise 3 _ p.353

New York City and London have a number of important

similarities. First, both New York and London are the most

famous cities in their nations. Additionally, just as London

currently has a growing population of immigrants, New York

City was the entry point for most European immigrants to the

US during the 19th century. Finally, London is an important

cultural center, and New York City is also.

Exercise 4 _ p.353

A number of important distinctions can be made between

London and New York City. One such distinction is that

while London is more than 1,400 years old, New York City is

only about 400 years old. Another important difference is

that although Britain is both the governmental and financial

capital of Britain, New York is only the financial capital of the

US, not the governmental capital. Finally, London is located

in the southern part of Britain, as opposed to New York,

which is located in the northern part of the US.

Exercise 5 _ p.355

According to the reading, the university is going to build 3

more computer labs over the summer. The reading says that

the new labs will be located in the engineering department,

but that they will be available to all students. It also says in

the reading that students can use the computer lab for up to

an hour without an appointment, but that they must make a

prior appointment if they want to use a computer for longer

than that.

Exercise 6 _ p.355

The woman’s problem is that she can’t get higher than a C

on her papers. She says that she proofreads them, but that

there are always still errors on her papers. The man says

that she might have to admit that she’s not a good

proofreader and get someone to proofread her papers for

her. He points out that the writing lab in the English

department does this for free.

Exercise 7 _ p.356

[ f Script ]

Professor (male): Normally, we think of cats as having

excellent senses. So, some of you might find it surprising

to learn that most cat species are basically color blind.

Lions are a good example of this. Lions have almost no

ability to distinguish colors. So why would a hunter have

such bad color vision?

As most of you probably know, most cats are nocturnal

hunters. So what is most important for them is to see well

in low light conditions. On a moonlit night, a lion can see

just as well as you or I can in the middle of the day; they

just can’t see in color. Basically, over millions of years,

cats have evolved to sacrifice their color vision in favor of

better night vision.

[ f Script ]

Man: Hey Lisa, how’s it going?

Woman: Not so good. I got another C on a paper. This

always happens. I work really hard on a paper,

but then I get a low grade because I have a

bunch of typing errors in the paper.

Man: Don’t you proofread your papers before you turn

them in?

Woman: Of course. I probably proofread them like 3 or 4

times, but I never seem to catch all my

mistakes.

Man: Maybe you just have to acknowledge that you’re

not a good proofreader, and get someone else

to proofread your papers before you turn them

in.

Woman: Who could I get to do that?

Man: Well, the writing lab in the English department

will proofread your papers for free. Why don’t

you start taking your papers over there before

you turn them in?

Chapter 4

A80 Insider: The Super Guide

If I could visit any area in the world, I would visit Lebanon.

One major reason for this is that I have a friend who is

from Lebanon, and from what he has told me, it sounds

like a really interesting place. He is always telling me

about how great the food is, and how nice the people are.

Additionally, I have read a lot about how Lebanese

society is a mixture of different cultural influences, and I

find this to be really interesting. Another major reasonis that Lebanon is supposed to be a nation of amazing

beauty. It is located right on the Mediterranean and is

supposed to have great beaches. Furthermore, some

parts of Lebanon are quite mountainous, and I love

mountain landscapes. A final point is that the Middle

East in general has always been an area I have wanted

to visit, and Lebanon seems like a good place to start.

Page 81: Insider: The Super Guide Scripts & Answer Key

In the lecture the professor points out that most cats are

color blind. He says that since most cats hunt at night, it is

most important for them to be able to see in low light. He

goes on to say that cats have evolved over millions of years

to have good night vision at the expense of their ability to

see colors. According to the passage, animals that can see

well at night have more rod cells than cone cells in their

eyes. Rod cells are better at detecting low amounts of light,

but are not good at detecting colors. This explains why cats

have good night vision but poor color vision.

Adding Elaboration andSupport

Exercise 1 _ p.359

1. You have to have determination if you want to be a

doctor.

2. I think doing the same thing in your job every day is

tedious.

3. I would like to go to Hawaii because it is a beautiful place.

4. I think the most important thing is for teacher to be

compassionate.

5. It is dangerous to smoke while you are at the gas station.

Exercise 2 _ p.359

1. Air pollution can damage the lungs of people.

2. Students get both physical and social benefits from

playing sports.

3. It is important to eat food that is low in fat and cholesterol.

4. Going to college helps a person find a good job in the

future.

5. Too much stress can keep a person from sleeping properly.

Exercise 3 _ p.360

1. I think one of the most important things to find in a friend

is a sense of humor. If a person has a good sense of

humor, especially if they can laugh at themselves, we

can have fun teasing and joking with each other in a

friendly way. Also, people with a sense of humor usually

spread their energy to other people, For example, my

friend John is a very funny guy. When he is around,

everyone is more active. For these reasons, I think a good

sense of humor is an important thing to find in a friend.

2. I prefer to study by myself rather than in a group. In my

experience, studying in a group is not an efficient way to

study. Most people have many differences in the way

they learn, and it makes it difficult for them to study

together. When I study on my own, however, it is more

effective and efficient. There are no other people, so I

can just concentrate on studying. Also, when I study

alone, I study at home, so the environment is quiet and

there are no distractions, which makes it easier to

concentrate on my studies.

Exercise 4 _ p.362

1. Claim of fact 2. Fact

3. Claim of fact 4. Fact

5. Claim of fact

Chapter 5

Scripts & Answer Key A81

SP

EA

KIN

G

Page 82: Insider: The Super Guide Scripts & Answer Key

Exercise 5 _ p.362

1. Internet crimes such as fraud have risen to unacceptable

levels in recent years.

2. It discourages them from going outside and getting the

exercise they need to grow into healthy adults.

3. There is much less air pollution and the traffic is better.

4. Nearly every high paying job in the modern world

requires a high degree of education.

5. He created countless useful inventions during his

lifetime.

6. Cars have made society more mobile, for example.

7. Every year, there are hundreds of murders and thousands

of robberies.

Exercise 6 _ p.363

1. I think it is important for a city to spend money on public

facilities like museums and parks. For one thing, these

kinds of facilities help to increase tourism. People aremore likely to visit a city with beautiful parks andmuseums. Take Paris, for example. People travelfrom all over the world to see the museums of Paris.Additionally, parks and museums improve the quality of

life for people living in the city. This is especially true of

children living in the city. Children living in a city withgood parks have a place to play and enjoy theoutdoors. In addition, children have more learningopportunities if they live in a city with goodmuseums. Finally, parks and museums increase the

pride that citizens feel for their city, because they canfeel like they live in a city that appreciates beautyand culture.

2. In my opinion, it is a bad idea for people to take online

courses instead of attending regular college classes.

There are a number of reasons for my opinion. First, the

education one receives through online courses is inferior

to the education provided in a traditional college setting.

When taking an online course, you don’t get as muchcontact with the professor. You can’t ask theprofessor questions directly, and this limits yourlearning opportunities. Another thing to keep in mind is

that in an online class, you have no real contact with the

other students, which puts you at a disadvantage. Muchof what you learn in college comes from havingdiscussions with other students. Furthermore, thepeople you meet in college could be importantbusiness contacts in the future, and you can’t makethose kinds of contacts taking online classes.

Exercise 7 _ p.364

1. One animal that I think makes a very good family pet is a

cat. First of all, cats are small, so they don’t make much

of a mess. Cats aren’t big enough to knock thingsover, as dogs sometimes do, and they don’t leave asmuch fur on the furniture either. In addition, since cats

require less care than dogs or other kinds of pets, they

are convenient pets for busy people. Cats do not needto be taken outside every day like dogs do. Thismeans that cats can be left alone in the house for aday or two if you need to take a trip. While some

people may say that cats are not as affectionate as

dogs, I don’t think this is true. Cats love their owners just

as much as dogs do. My cat, for example, runs up tomeet me at the door every day when I get home.

Finally, cats generally live longer than dogs, so you can

enjoy their company for a longer period of time.

2. Generally, I think that private schools are better than

public schools. One of the reasons I believe this is that

private schools are better funded than public schools.

While public schools must rely on taxes for theirfunding, private schools are free to charge whateverthey wish for tuition, and as a result often have muchlarger budgets. Since they are better funded, they can

hire more highly qualified and experienced teachers,

whereas public schools may not be able to afford to hire

such teachers. Furthermore, private schools can choose

which students they accept and which students they

don’t, which means there are fewer disciplinary problems

in a private school. Public schools, on the other hand,must accept any and all students, even if a studenthas a history of bad behavior. Finally, private schools

are able to give more personalized attention in theclassroom to individual students, so students receive a

better education. Private schools generally havesmaller class sizes, so a teacher can spend more timewith each student and help them more effectivelywhen they have a problem.

A82 Insider: The Super Guide

Page 83: Insider: The Super Guide Scripts & Answer Key

Improving Pronunciationand Flow

Exercise 2 _ p.367

1. Ten dangerous fighters sought the title.

2. Just because Kenny can cheat doesn’t mean he

should.

3. The thing that is most needed is better safety

procedures.

4. Four of the clocks got damaged by accident.

5. Does Thomas care if she sees his book?

Exercise 4 _ p.369

1. Ned says he called Jessica to emphasize the point.

2. The shed was built after the farm house.

3. Be sure to check the tires before your trip.

4. The pilot lost his job due to cutbacks.

5. The accident occurred as she slowed to a stop.

Exercise 5 _ p.371

1. (B)

2. (A)

3. (B)

4. (C)

5. (A)

6. (A)

Exercise 6 _ p.371

1. The preacher talked with his congregation after the

service.

2. Golden showers of sunlight fell on the open field.

3. Don’t go caving without an electric torch.

Exercise 7 _ p.373

1. No pause

2. Whatever his motivations were / Jason had no right to

speak to Jessica like that.

3. Most birds / and even some land mammals / migrate for

the winter / but others stay where they are all year long.

4. Prior to the company’s announcement of its third quarter

earnings / expectations had been high among many

analysts.

5. Every parent knows that a child / of any age / should not

be left unsupervised / but many parents do so anyway.

Exercise 8 _ p.373

The woman disagrees with the policy / because she thinks

that more physical education classes are unnecessary. —First of all / she is afraid that the new physical education

requirements will replace other required classes. / She is

most afraid that important academic classes / like literature /will be cut / and she doesn’t believe that adding the new

physical education classes is important enough to do that. —Also she thinks that her health is her own business / and not

that of the university. / She states that she can easily get

enough exercise in her spare time / without having to take

the new classes. — Finally, / she complains that it will be

difficult to fit two additional classes into her schedule.

Chapter 6

Scripts & Answer Key A83

SP

EA

KIN

G

Page 84: Insider: The Super Guide Scripts & Answer Key

P A R T BPersonal Preference Task

Preparing Your Response

Exercise 1 _ p.380

Exercise 2 _ p.381

1. My first job interview was a valuable experience.

2. The interview did not go well, and I did not get the job.

I learned to deal with failure and how to speak under

pressure.

3. One learning experience that I thought was particularly

valuable was when I went on my first job interview, which

did not go well at all. This was a valuable experience

because I learned to deal with failure and how to speak

under pressure.

Exercise 3 _ p.382

1. I did poorly on my first interview.

2. I learned how to deal with failure from this experience.

At first I was very upset that I didn’t get the job and I was

very discouraged. But later I realized that everyone fails

sometimes, and that the important thing was to learn

from one’s failures.

3. I learned to how to speak under pressure. This was a

valuable lesson because as an adult I often have to

speak under pressure. Learning how to do this has

helped me in other job interviews, and it has helped me

in my job also because I sometimes have to do

presentations for the company.

4. This was a particularly valuable learning experience for

me because it taught me to deal with failure. When I first

learned I hadn’t gotten the job, I was very upset. Later,

however, I realized that everyone fails sometimes, and

that the most important thing is to learn from those

failures. Furthermore, this experience also helped me by

teaching me how to speak under pressure. Leaning how

to do this has helped me in other job interviews and in my

job, where I sometimes have to do presentations for the

company.

Exercise 4 _ p.383

1. Doing poorly on my first interview was an important

learning experience.

2. This was an important experience because it taught me

to learn from my failures and how to speak in pressure-

filled situations.

3. In conclusion, doing poorly on my first interview was an

important learning experience because it taught me to

learn from my failures and how to speak in pressure-filled

situations.

Guided Practice 1

Exercise 1 _ p.384

Exercise 2 _ p.384

1. I believe the Red Cross helps mankind a great deal.

2. The Red Cross provides medicine and medical care in

areas where it is unavailable, such as in poor countries

and in war zones.

3. An organization which I think benefits humanity is the

Red Cross. The Red Cross works to provide medical

care and medicine to places where it would otherwise be

unavailable.

Exercise 3 _ p.385

1. The Red Cross

2. The Red Cross provides cheap medicine to poor nations.

For example, the Red Cross works to provide basic

immunizations to poor children throughout Africa and

Asia. This helps mankind because it helps to treat and

control diseases.

3. The Red Cross also works to provide medical care for

people trapped in war zones. In many conflict areas,

medical care would not exist if the Red Cross did not

provide it. By providing such medical care, the Red

Cross saves many lives every year.

Chapter 1

A84 Insider: The Super Guide

Operates all over

the world

Helps provide

medicine in poor

countries

Provides care

for people in war

zones

RReedd CCrroossss

Did not do well

in interview

Learned to deal

with failure

Learned to speak

under pressure

FFiirrsstt JJoobbIInntteerrvviieeww

Page 85: Insider: The Super Guide Scripts & Answer Key

4. This organization helps mankind by providing

inexpensive medicine to poor countries. For example, in

many parts of Africa and Asia, the Red Cross provides

immunizations for poor children. This is important

because the Red Cross helps fight disease by doing so.

Furthermore, it also provides medical care in war zones,

where there would be none if not for the Red Cross. By

providing such medical care, the Red Cross saves many

lives every year.

Exercise 4 _ p.385

1. I believe the Red Cross benefits mankind greatly.

2. The Red Cross provides medicine and medical care in

areas where it would not exist without them.

3. In conclusion, I think the Red Cross is an organization

that is beneficial to humanity because it provides

medicine and medical care in areas where it would

otherwise not be available.

Guided Practice 2

Exercise 1 _ p.386

Exercise 2 _ p.386

1. I dislike washing clothes the most.

2. I have to wash my clothes. Then I have to hang them up

and wait for them to dry. Finally, I have to fold all my

clothes and put them away.

3. The chore I dislike the most is washing my clothes. Since

I don’t have an electric dryer, it takes a really long time

because I have to wait for my clothes to dry.

Exercise 3 _ p.387

1. I don’t like to do my laundry.

2. I don’t like doing my laundry because I can’t really avoid

doing it. I can put off doing other chores if I really want

to, but not washing my clothes isn’t really a choice. The

fact that I have to do it no matter what really bothers me.

3. I hate washing my clothes, because once I’ve washed

them, I have to fold them all and put them away. If I only

had to wash my clothes, it wouldn’t be so bad, but folding

them and putting them all away takes forever.

4. One of the things I dislike about doing laundry is that I

can’t really avoid doing it. I can put off doing other chores

if I really want to, but not washing my clothes isn’t really

a choice. The fact that I have to do it no matter what

really bothers me. I also hate the fact that once I’ve

washed all my clothes, I have to fold them all and put

them away. If I only had to wash my clothes, it wouldn’t

be so bad, but folding them and putting them all away

takes forever.

Exercise 4 _ p.387

1. Doing laundry is my least favorite chore.

2. I don’t like washing my clothes because I can’t delay it

even if I want to, and it takes a really long time because I

have to put away all my clothes after I am done.

3. In short, doing laundry is my least favorite chore. I don’t

like washing my clothes because I can’t delay it even if I

want to, and it takes a really long time because I have to

put away all my clothes after I am done.

Additional Practice _ p.388~389

1. [ Sample Response ]

My favorite movies are definitely science fiction movies

because they combine the best aspects of other movie

genres. Science fiction movies usually have a lot of

action, so you get all the adventure that you would get

watching an action movie. However, there is also a lot of

humor written into the scripts of many science fiction

films. A great example of this would be the film Men in

Black, which was both fast paced and hilarious. Another

point is that science fiction films almost always have

great special effects. This means that even on occasions

when the plot might be a little slow, science fiction films

are still visually pleasing. In short, I love science fiction

films because they offer not only the best of other

genres, but stunning special effects as well.

2. [ Sample Response ]

One event that I really believe has changed the word in

recent years is the rise of China as an economic power.

One significant effect of this has been that it has sparked

new competition for natural resources. As China’s

economy grows, it needs more and more oil, coal, and

steel. This has increased the power of nations rich in

natural resources because they now have another large

customer. China’s economic growth has also opened

new markets for many companies. Traditionally, the US

and Europe have been the major markets for most

products. Now, however, the importance of China’s own

Scripts & Answer Key A85

SP

EA

KIN

GI have no electric

dryer

Can’t really

avoid doing it

Have to fold

all my clothes

afterwards

WWaasshhiinnggCCllootthheess

Page 86: Insider: The Super Guide Scripts & Answer Key

Paired Choice Task

Preparing Your Response

Exercise 1 _ p.392

Exercise 2 _ p.393

1. I prefer to eat smaller meals.

2. I heard that eating smaller meals was healthier. Eating

several small meals keeps me from getting hungry

between meals.

3. In my opinion, it is preferable to eat four or five small

meals each day rather than three large meals. I feel that

this is healthier for you and it keeps you from getting

hungry between meals.

Exercise 3 _ p.394

1. It is healthier to eat several small meals a day.

2. I read in the paper that people who eat four or five small

meals a day are typically in better shape and live longer

than people who eat three large meals a day.

3. I don’t get hungry between meals.

4. If I eat three large meals a day, I get pretty hungry

between meals and I end up eating snack foods that

really aren’t good for me. If I eat four or five smaller

meals throughout the day, however, I don’t get as hungry

between meals, and I can avoid unhealthy snack foods.

Exercise 4 _ p.395

1. I prefer to eat small meals throughout the day.

2. It is healthier, and I don’t get hungry between meals.

3. In conclusion, I prefer to eat small meals throughout the

day because it is healthier and I don’t get hungry

between meals.

Chapter 2

A86 Insider: The Super Guide

domestic market is growing. As a result, companies must

consider not only the needs of the European and US

markets, but the needs of the Chinese market as well. In

conclusion, I feel China’s rise has changed the world

because it has increased competition for natural

resources and shifted some of the focus away from US

and European markets.

3. [ Sample Response ]

In my opinion, an ideal neighbor would be a person who

is polite and considerate, but basically minds his or her

own business. Obviously, when I meet my neighbors on

the street or in the hallway of our apartment building, I

would like them to be polite. In such situations, the

perfect neighbor would say hello and perhaps ask how I

was doing. That, however, would the limit to our

conversation. I’m a pretty private person, so I wouldn’t

want to have detailed conversations about my life with

my neighbor. Therefore, a perfect neighbor would also

mind his or her business. Basically, my idea of a perfect

neighbor is someone who greets me when they see me,

but not much else.

4. [ Sample Response ]

It is my belief that the most important issue faced by

mankind in 20 years will be environmental damage.

I think that our environmental problems will most likely

only get worse in the future. First of all, many of these

problems are long term. For example, I read that even if

we made significant reductions in air pollution today,

temperatures around the globe would still continue to

rise. So in some sense, future environmental problems

are unavoidable because they are the result of the

pollution we have already created. Furthermore, I don’t

really see any governments taking serious steps to

reduce pollution. There doesn’t seem to be any real

funding to find cleaner fuel sources or clean up the

environment, so I don’t expect the situation to get any

better. In short, I feel the most pressing issue humanity

will face in the future will be environmental damage

because such damage is a long term problem and

because I don’t see any real efforts to try and fix the

problem.

BBaassiicc cchhooiiccee::

Many small meals

PPrrooss ooff yyoouurr cchhooiiccee::

• Healthier

• Avoid unhealthy snack

foods

CCoonnss ooff ootthheerr cchhooiiccee::

• Get too hungry between

meals

Page 87: Insider: The Super Guide Scripts & Answer Key

Guided Practice 1

Exercise 1 _ p.396

Exercise 2 _ p.396

1. I prefer big vacations.

2. Big vacations are more memorable and I can go to

places I’ve never been to before.

3. Personally, I like to take a bigger, more expensive

vacation every few years rather than go on a cheap

vacation every year. I prefer big vacations because I feel

they are more memorable and because I can go to

places I’ve never been before.

Exercise 3 _ p.397

1. Big vacations are more memorable.

2. If you only go on vacation every few years, you have

more money to spend while on vacation. This allows you

to do more exciting and more memorable things. For

example, I could afford to go skiing every year at a local

ski resort. If I saved up for a few years, however, I could

afford to take a ski trip to the Alps, which would be far

more memorable.

3. I can go to places I have never been before.

4. If I went on vacation every year, I probably wouldn’t have

the money to go to new places every time. If I only go on

vacation once very few years, however, I can afford to go

somewhere I’ve never been to. Since seeing new things

is one of my favorite parts of going on vacation, this is a

better system for me.

Exercise 4 _ p.397

1. I prefer big vacations.

2. On a big vacation, I can go to new places and do more

exciting things, which is more memorable.

3. In short, I would rather take a big vacation every couple

of years because on a big vacation, I can go to new

places and do more exciting things.

Guided Practice 2

Exercise 1 _ p.398

Exercise 2 _ p.398

1. I think it is better to give employees extra time off.

2. Most people have enough money, but not enough time to

enjoy it. Additionally, if people are given more time off

work, they will do their jobs better.

3. In my opinion, rewarding employees by giving them time

off is a better company policy than giving cash bonuses.

I feel this way because most people don’t have enough

time to enjoy the money they do have, and because I

think more time off helps people to do their jobs better.

Exercise 3 _ p.399

Most people complain that they don’t have time to do the

things they want to do. So even if a company gave its

employees more money, they probably wouldn’t have time to

enjoy the extra cash. If a company gives employees extra

time off, however, they will finally have the time to go out

and do the things they normally don’t have time to do.

Another reason I think this is a better policy is because if

employees have more time off, they will be better rested and

less stressed when they return to work. This will help them

do their jobs better.

Exercise 4 _ p.399

To sum up, I think companies should reward their

employees with time off work because the extra time is

something their employees can really use and enjoy, and

because they will return to work rested and ready to do a

good job.

Scripts & Answer Key A87

SP

EA

KIN

G

BBaassiicc cchhooiiccee::

Big vacation

PPrrooss ooff yyoouurr cchhooiiccee::

• Big vacations are more

memorable

• Anticipation is fun

CCoonnss ooff ootthheerr cchhooiiccee::

• Don’t want to go to places

I have already been

BBaassiicc cchhooiiccee::

Extra days off

PPrrooss ooff yyoouurr cchhooiiccee::

• Time is more important to

workers than money

• Extra time off helps

workers do their jobs better

CCoonnss ooff ootthheerr cchhooiiccee::

• Have money, but no time

to enjoy it

Page 88: Insider: The Super Guide Scripts & Answer Key

Additional Practice _ p.400~401

1. [ Sample Response ]

I’d rather spend my free time outside than inside for a

number of reasons. First, I have an office job, so I

already spend most of my day inside. My free time is my

only time to really get outside and enjoy nature, so I try to

spend as much of my free time outside as possible. For

example, I love to go to the park on the weekends, or

even just to take a short walk at night. Another reason

that I prefer to be outdoors is that I really enjoy sports. I

particularly love playing soccer and baseball, and

obviously you can’t play these sports inside. So basically,

I’d rather spend my free time outdoors because it allows

me to get out and enjoy nature and because my favorite

sports are outside sports.

2. [ Sample Response ]

For me personally, I think it’s better to put my money in

the bank rather than invest it in the stock market,

because I don’t know that much about stocks and I am

not yet financially stable. First, investing in the stock

market is risky, especially if you don’t know what you are

doing, which I don’t. If I invested my money in the stock

market, it seems like there is a high chance that I would

end up losing it, so I would rather not risk it. In addition,

since I am still young and not yet financially stable, I

need quick access to the money I have saved in times of

emergency. If you buy stocks, it’s not always so easy to

get your money back if you need it quickly. Therefore,

this doesn’t seem like a good option for me. In brief, I feel

it’s a better idea for me to put my money in the bank

because I don’t know enough about the stock market

and because I occasionally need quick access to my

money.

3. [ Sample Response ]

In my opinion, it is more just for a nation to punish a drug

smuggler with life imprisonment rather than the death

penalty. First, I feel the death penalty is wrong in all

cases. It’s always wrong to take a life. I don’t even think

that the death penalty should be used to punish murders,

so I definitely don’t think it should be used in cases of

drug smuggling. In addition, I don’t think the death

penalty prevents people from smuggling drugs. There is

so much money to be made in drug smuggling that there

will always be someone willing to try it, even if they could

be executed for it. So basically, I think it’s better to give

drug smugglers life in prison because I am opposed to

the death penalty in all situations and because I don’t

think it would help anyway.

4. [ Sample Response ]

In my view, parents should let their children make their

own decisions rather than be very strict with them. One

reason I feel this way is that I believe that we learn from

our mistakes. If a child is free to make his or her own

decisions in life, then of course, they will occasionally

make bad decisions. But they will also learn from their

mistakes, which will help them make better decisions in

the future. Children whose parents never let them make

their own decisions never get these kinds of learning

experiences. Another point to make is that most children

hate to be controlled by their parents. The more parents

try to control their child, the more that child will try to

rebel against his or her parents. Basically, it’s better to let

kids make their own decisions so they can learn from

their mistakes and because they are going to resist their

parents’ control anyway.

A88 Insider: The Super Guide

Page 89: Insider: The Super Guide Scripts & Answer Key

Fit and Explain Task

Overview _ p.405

( Format of the Conversation

Preparing Your Response

Exercise 1 _ p.406 Exercise 2 _ p.407

The woman does not agree with the new policy concerning

the change to the final examination schedule announced by

the Academic Affairs office. The woman thinks that

administering final exams after winter break will force

students to spend their breaks studying for tests when they

would rather be having fun with family and relaxing.

Student A: Even if I study two weeks before break, I’ll

still have to study again to refresh my

memory, because winter break is three

weeks long.

Student B: What if I have to study during my family’s

winter vacation? This is –

Student A: (interrupts) – a terrible idea! Students

should be given a choice between two

options – either have winter break mark the

official end of the semester or opt to delay

semester’s end by taking final exams and

handing in papers at the start of the new

semester.

Student B: Great idea. So, either way, the semester

would end at the start of winter break,

whether officially or unofficially.

[ f Script ]

Student A (female): I’m really peeved about the new

policy about exam week.

Student B (male): What new policy?

Student A: You didn’t hear the announcement? Finals

are after winter break.

Student B: Oh. Well, I guess we’ll have more time to

study.

Student A: Yeah, but I really liked the idea of leaving

for winter break knowing that I had

completed all my exams and the semester

was finished. The holidays are for fun and

relaxation with family. Now I’ll have to

stress about classes and study like crazy.

Student B: But you could always study before you

leave for winter break.

[ f Script ]

Woman: Nathan, did you see this editorial?

Man : The one about Professor Dresner’s dismissal?

Yeah, I saw it, and I have to agree; it is

outrageous.

Woman: Come on, Mike. He’s brought the university a

lot of bad press. I mean, he kind of deserves it.

Man : You really think the university has the right to

fire him for speaking his mind? What about his

right to free speech? He should have the right

to say what he thinks. It doesn’t matter if you,

me, or the university agrees with him or not.

Woman: So you think he should be able to say

whatever he wants?

Man : Well, maybe not whatever he wants ... but

certainly he should have the right to voice his

opinions about the government. I mean, this is

a university. It’s supposed to be a place where

opinions and ideas flow freely. How’s that

supposed to happen when the university fires

people for saying what they think? Say what

you want about Dresner, but I think the

university is way out of line here.

Chapter 3

Scripts & Answer Key A89

SP

EA

KIN

G

MMaaiinn SSppeeaakkeerr’’ss OOppiinniioonn:: w. disagrees; upset new

policy will make holidays less fun

SSppeeaakkeerr’’ss RReeaassoonniinngg 11:: winter brk time for fun &

relaxation + stud. shouldn’t have to worry a/b tests

& papers

SSppeeaakkeerr’’ss RReeaassoonniinngg 22:: since winter brk so long stud.

can’t just study b4 brk.; they would forget info;

studying sometime during 3 wk. vac. period nec.

AAlltteerrnnaattee ssoolluuttiioonn((ss)):: give stud. choice b/w 2 opts. …

DDeettaaiillss::

- start of brk marks off. end of semester

- stud. can delay off. end of sem. by taking exams @

start of new semester after brk.

Page 90: Insider: The Super Guide Scripts & Answer Key

Exercise 3 _ p.408

According to the woman, the new final exam schedule would

force students to alter their holiday plans even if they studied

prior to winter break because since winter break is three

weeks long, students will have to study sometime before the

end of break so as not to forget information. The student is

quite annoyed by the fact that there seems to be no way for

even the most diligent and responsible student to escape

spending winter break studying and worrying about

schoolwork. In addition, she points out that she would rather

complete all her exams prior to winter break and leave for the

holidays with the knowledge that she’d successfully

completed the semester.

Exercise 4 _ p.409

1. The woman thinks the new policy is unfair because it will

make winter break less fun and relaxing

2. The woman thinks that the new policy will have a

negative impact on students’ holidays because even if

they studied before break, they would still have to alter

their vacation plans to study again before the end of the

three week break.

3. In conclusion, the woman disagrees with the new change

to the final exam schedule because students will have

less time to relax, have fun, and properly enjoy their

breaks. The student is quite unhappy with the new policy

because less time will be spent participating in traditional

holiday activities. She would prefer if students either got

to end the semester officially at the start of winter break

or extend the semester unofficially by opting to take

exams at the start of the new semester.

Guided Practice 1

Exercise 1 _ p.410

Exercise 2 _ p.410

The man does not like that the student council created a

new policy on campus elections without proposing the policy

to the student body first. He thinks that members of the

student council should not make unilateral decisions on

matters that concern the entire student body.

Exercise 3 _ p.411

The man is dissatisfied with the student council decision to

enact an important policy without first consulting the

students whom it represents. In his opinion, the decision was

not unreasonable because he believes that freshmen are

indeed too inexperienced to run for president and seniors

will be too preoccupied with numerous graduation activities

and requirements to be successful presidents. However, he

still thinks that the decision of who is eligible to run for

school president should be made by the student body.

Student B: So you don’t think it’s unfair to restrictcertain students from running?

Student A : Well … uh … I think their reasoning for thedecision is perfectly logical. Freshmen andseniors would really make awful presidents.But student council is supposed torepresent the student body, not itself. Theyshould have allowed us to at least chime in… better yet … they should have proposedthe policy and put it up for a general vote.

Student B: Well, I guess it’s too late now. I’ve alwaysthought student council members are a bitself-absorbed. This just proves it.

[ f Script ]

Student A (male): Hey, Jackie. Have you heard aboutthat obnoxious new student councilannouncement?

Student B (female): Oh. The one about elections?Yeah. I think it’s kind of –

Student A: (Interrupts) Obnoxious is what it is. It’s notthat I hate the policy. I’m just annoyed thatthey made the decision unilaterally. Theyhad a closed-off debate about something that concerns the entire student body and then decided to simply announce the resultsof their so-called ‘exhaustive’ debates.

A90 Insider: The Super Guide

MMaaiinn ssppeeaakkeerr’’ss ooppiinniioonn:: m. thinks policy reasonable

b/ disagrees w/ council tactics

RReeaassoonniinngg 11:: thinks council membs. should have

consulted stud. bdy 1st

DDeettaaiillss:: council membs. wrk 4 students & shouldn’t be

allowed to make unilateral dec.

RReeaassoonniinngg 22:: thinks frshmn. & seniors not good. pres.

Candidates b/ dec. should be left up to stud. bdy

DDeettaaiillss:: frshmn. 2 inexp. & seniors 2 busy

AAlltteerrnnaattee ssoolluuttiioonn((ss)) ((iiff pprroovviiddeedd)):: stud. council

should propose policy 2 students

DDeettaaiillss:: stud. bdy should be allowed to vote on whether

proposal should become off. sch. policy

Page 91: Insider: The Super Guide Scripts & Answer Key

Exercise 4 _ p.4111. The man disagrees with the student council’s decision to

announce a policy it created without permission from the

student body.

2. The man argues that the student council does not have

the right to enact important policies without first

presenting them to the student body because they

represent the student body, not themselves.

3. In short, although the man is dissatisfied with the student

council’s decision to announce a decision it made

unilaterally, he doesn’t think the policy itself is wrong.

However, he would have preferred if the student council

had proposed the policy first and then put it up for a vote,

leaving the final decision on whether there should be

restrictions on who can run for the office of president to

the student body.

Guided Practice 2

Exercise 1 _ p.412

Exercise 2 _ p.412

The woman thinks that the Admissions Office policy requiring

applicants for the recruiter position to have at least a 3.5

GPA is unfair. She feels this way because has been looking

forward to applying for the position, but she doesn’t have a

high enough GPA.

Exercise 3 _ p.413

The woman thinks that the Admissions Office should

consider other factors like her participation in extracurricular

activities and her difficult course load when deciding who

can apply for the position. Furthermore, she thinks she would

be a better recruiter than most other applicants because she

has ties with many people in her community and the

surrounding communities. These school officials and

community members are looking forward to developing a

relationship with the university.

Exercise 4 _ p.413

To sum up, the woman is disappointed that the Admissions

Office has created such a strict policy that doesn’t take into

account factors beyond GPA. Since the student is sure that

people who just do it for a little extra cash.

Student B: You bet. I come from a really small town

and have excellent relationships with a lot of

school officials there and the surrounding

towns. Everybody is really excited that I will

be coming back to help build a relationship

between the communities in that area and

this university.

Student A: You should complain to the Admissions

Office. Maybe you can get them to change

their policy.

Student B: Yeah. Or at least leave room for exceptions.

I am definitely going to write an official

complaint.

[ f Script ]

Student A (male): Hey, Kate … have you read this flyer?

Student B (female): No.

Student A: Well, it looks like you won’t be able to apply

for that summer recruiter position with the

Admissions Office.

Student B: Why not?!

Student A: Because you have to have at least a 3.5

GPA to apply. Isn’t your GPA like a 3.0?

Student B: Well, 3.2 to be exact. This is really unfair.

I’m an athlete, on student council and a

physics major. I have way more

extracurricular commitments and a much

harder course load than most students with

really high GPAs.

Student A: And you’d be a much better recruiter than

Announcement from the Admissions Office

Effective next month, only students with at least a 3.5

GPA can apply to be academic recruiters for the

university. Working as a recruiter is a fun but also very

serious responsibility. The Admissions Office wants to

guarantee prospective students are exposed to the top

academic achievers of our university. Likewise, the

university would like those in excellent standing

academically to be able to share their experiences and

recipes for success to the young achievers they will be

recruiting. All other positions in the Admissions Office

are open to all students. This restriction applies only to

the position of academic recruiter.

Scripts & Answer Key A91

SP

EA

KIN

G

- W. thinks policy unfair … She wants 2 apply for post.

b/ GPA not high

- Thinks other factors should be considered …

e.g. extracurrs. , diff. of courses

- Believes she would be more committed than others b/c

- ties 2 community

- p/p in her town want r/n w/ Univ.

- Admiss. Off. should revise new policy or mk rm 4

exceptions

Page 92: Insider: The Super Guide Scripts & Answer Key

she would make a better academic recruiter than most other

students with higher GPAs, she is going to complain to the

Admissions office and get them to either change the policy

or make exceptions.

Additional Practice _ p.414~417

1.

[ Sample Response ]

The woman disagrees with the policy that limits the use of the

campus computer labs. She thinks it is unfair that she has to

be inconvenienced because of the behavior of a few other

students. First of all, the policy would pose an inconvenience

because the woman does not own a computer, and the

computer lab is the only place she can do homework. While

she could ask to use someone else’s computer, she feels

uneasy about imposing on others. She also complains about

the fact that she will have to get a new student ID, since they

are required to use the computer labs now. The policy will

also result in the woman using the computers less often

because the labs will be closing earlier and she works until

late at night. In the end, the policy is especially unfair for the

student because she is dependent on using the computer

lab. She doesn’t understand why her life has to be

inconvenienced for offenses committed by other students.

2.

[ f Script ]

Man : Hey, Lori, did you read the letter in this week’s

newspaper?

Woman: About the activity fees? Yeah, I thought the

guy made a pretty good point.

Man : Really?

Letter to the Editor

First, I would like to thank the editor for bringing to our

attention the new activity fees that students will be

required to pay at registration next year. I myself feel

that the extra hundred dollars is unjustified. It’s nice that

the university wants to build a new campus center, but I

would have appreciated, at the very least, some

advance notice about the new fees, and at most, a say

in whether or not we as a student body even wanted a

new campus center to begin with, especially if we’re

going to be paying the bill for the next few years.

[ f Script ]

Man : Hey, Mandy. Did you read that announcement

in today’s paper?

Woman: About the computer labs? Yeah, I’m not very

happy about it. I don’t have my own computer,

so the computer lab is the only place I can go

to work on papers.

Man : You have an ID, right?

Woman: Yes, but it’s old. I’ll have to get a new one

before I can use the computer lab again, and

that’s going to cost me. Not to mention those

new hours.

Man : You should still have plenty of time during the

day.

Woman: I have classes all day. I have a couple of hours

in between classes, but that’s not enough time

to get any real work done. I usually go in the

evening and stay late. And now I can’t.

Man : Can’t you ask someone in your dorm if you

can use their computer?

Woman: I couldn’t impose. And I shouldn’t have to.

I mean, I haven’t done anything wrong. It’s not

fair that I have to suffer because of a few

misbehaving freshmen.

Announcement from Student Technology Services

Recently it was discovered that a small group of

students has been abusing use of the Internet in

campus computer labs. Due to these abuses, Student

Technology Services will be changing its policy

concerning lab use. Beginning in January, all students

will be required to sign in using a valid student ID in

order to use the campus computer labs. Anyone without

a student ID will be turned away. As an additional

measure, the closing time will be changed from 9:00PM

to 7:00PM.

A92 Insider: The Super Guide

– woman disagrees

– thinks policy unfair b/c she doesn’t own comp. & can

only do hmwk in comp. lab

– hesitant to ask others to use their comp.

– ALSO … stud. ID not up2date … problm. b/c

– labs require IDS

– will be forced to purchase new ID

– FINALLY … b/c of classes, wrk … etc. can only use

lab @ nt. … prob. b/c

– will be unable to use labs often

Page 93: Insider: The Super Guide Scripts & Answer Key

[ Sample Response ]

The woman agrees with the letter to the editor concerning

the new mandatory student activities fee. She thinks that the

new activities fee is a bad idea. The first reason she gives

for her opinion is that the new fee is too expensive.

Secondly, she points out that the university failed to notify

the student body of the extra fee, which she finds very

disagreeable. In addition, she does not think that she will

use the new campus center, and doesn’t understand why

she should have to pay so much money for something she

won’t use. In her opinion, a better solution might have been

to charge a smaller fee to renovate the old campus center,

instead of building another, unnecessarily larger new

campus center. Finally, while the student is willing to pay the

fee, she finds it unfair and would have preferred if students

were consulted before the decision to build an expensive

new campus center was made.

3.

[ f Script ]

Woman: Hey, Nick. How’s it going?

Man : Alright, I guess. Have you registered for

classes yet?

Woman: Not yet. I just met with my advisor today. I

assume you heard the announcement?

Man : Yeah. I don’t understand why I should have to

wait to see my advisor when I already know

what classes I’m taking.

Woman: Well, I think it’s a good thing. I mean, I meet

with my advisor every semester anyway, so

it’s no big deal.

Man : Really?

Woman: I make it a point to always talk with my

advisor. I know a lot of people who have really

been helped by their advisors. Required

classes change every few years, you know.

Keeping in touch with your advisor is a good

way to stay on track.

Man : I guess you make a good point.

Woman: You wouldn’t want to end up wasting time on

classes you don’t need. Just a few minutes

with your advisor could save you a lot of time

in the long run.

Announcement from the Office of the Registrar

Beginning with the upcoming fall registration period, all

students, regardless of year, will be required to see an

advisor before they are allowed to formally register.

Upperclassmen working on a concentration will need to

speak with a faculty member from their respective

departments. Underclassmen not yet working toward a

major should contact the office of the registrar to be

assigned an academic advisor. Students registering in

person at the office will have to present a signed form

from an advisor in order to register.

Woman: He has a right to be a little angry. I know I am.

I have to come up with an extra hundred

dollars now every semester because of this

campus center, and the school didn’t even

bother to tell me.

Man : So you don’t want a new campus center?

Woman: I don’t think I’d even use it. I mean, I wouldn’t

have minded paying a small fee to renovate

the campus center, because I know it might

benefit other people, but a totally new one

that’s twice the size? I just don’t see the need.

And the cost is outrageous!

Man : Well, enrollment has gone up a lot in the last

few years. Maybe they’re trying to keep the

facilities up to date.

Woman: It would’ve been nice to have a say in the

matter, that’s all.

Scripts & Answer Key A93

SP

EA

KIN

G

- woman agrees … activities fee bad idea b/c

- fee too expensive

- Univ. didn’t notify stud.

- She will not use new facilities … doesn’t want to

pay fee 4 it

- BETTER SOLUTION

- chrg smaller fee to renovate old centr … lrgr centr

not nec,

- FINALLY … Univ should have let stud. have say in

bldng plans

– woman agrees … in favor of all stud. seeking

advising b/c

– she sees advisor often & finds it helpful

– other stud. should do same b/c …

– class requirements chng often

– advisor keeps stud. up2date

– ALSO – mtg w/ advisor – time saver; avoid taking

classes you don’t need b/c of gd. advice

Page 94: Insider: The Super Guide Scripts & Answer Key

[ Sample Response ]

The woman agrees with the announcement requiring all

students to meet with their academic advisor before

registration. She is in favor of the new policy because she

sees her advisor regularly and thinks it would be good for all

students to keep close contact with their advisors. For one

thing, she notes that class requirements change every few

years, and an advisor can help his or her students stay aware

of important changes. Secondly, she says that meeting with

an advisor every semester can save a lot of time because

being unaware of certain policy changes can result in taking

unnecessary classes. In sum, the student considers the new

policy a positive change because whether or not students

realize it, seeing their advisors will help them make important

academic decisions and avoid making mistakes that could

negatively affect their academic future.

4.

[ Sample Response ]

The woman disagrees with the policy making physical

education classes a graduation requirement. She thinks

requiring students to take two physical education classes is

unnecessary. First of all, she fears that new physical

education requirements will interfere with other required

classes. She is most afraid that important academic classes

like literature will be cut from student’s schedules. She

doesn’t believe that requiring students to take new physical

education classes is worth the cost of cutting what she

thinks are more important classes. Furthermore, the student

strongly thinks that her health is her own business that can

be addressed in her spare time. In conclusion, the student

would rather focus on taking important academic courses

instead of potentially compromising her academic schedule

for required classes that she perceives to be a waste of time.

[ f Script ]

Man : So Sara, did you hear about the new

graduation requirements?

Woman: Yeah, I did. Two credits of physical education.

It’s such a waste of time, you know?

Man : What do you mean?

Woman: Well, think about it. They’ll have to make room

for those two new classes in the curriculum, so

that means two other requirements will have to

be cut — serious requirements like literature or

art.

Man : I didn’t think about that. But physical education

classes are easy! Maybe it’s a blessing after all.

Woman: More like a curse. I can learn to keep myself fit

on my own time. I pay my tuition to take

serious academic classes, not aerobics

classes or tennis lessons!

Man : Well, you’ve got a point there.

Woman: I just don’t see the point. My major concentra-

tion doesn’t require any physical education

classes, so I never planned for any. But now I

have to find a way to fit two new classes into

my schedule.

Announcement from the Office of the Registrar

In the interest of our student body’s health and wellness,

all incoming freshmen for the next academic year will be

required to take at least two physical education classes

in order to graduate. In addition, other underclassmen

still working to complete their general requirements may

also be subject to the new policy, and should meet with

their advisors to discuss the matter further. Please refer

to the new student handbook for complete details.

A94 Insider: The Super Guide

– woman. disagrees … more physical ed. Classes not

nec. +…

– new phys. Ed. Classes might replace other

required classes

– important acad. Classes … e.g. Lit. will be cut

– new phys. Ed. Classes not worth cutting adac. Classes

– ALSO – health is indiv. Biz not Uni biz … can get

nuff exercise in spare tm

– Finally … fitting 2 + classes in schedule too diff.

Page 95: Insider: The Super Guide Scripts & Answer Key

General / Specific Task

Overview _ p.419

( Format of the Lecture

Preparing Your Response

Exercise 1 _ p.420

Exercise 2 _ p.421

The professor’s discussion of how the body regulates its

own temperature relates to the concept of homeostasis in

that the process of temperature regulation is a clear example

of the self-regulating process of homeostasis. In the

passage, the concept of homeostasis is defined as any self-

regulating process by which biological systems achieve and

maintain equilibrium.

Exercise 3 _ p.422

1. The body automatically reacts to external changes in

order to keep a constant body temperature. This is an

example of homeostasis.

2. External changes trigger automatic reactions and

adjustments within the body to maintain a constant

temperature. The hypothalamus detects changes in body

temperature and responds to them. An example of this is

the way that the body reduces blood flow to the arms

and legs to conserve heat around more important body

parts, or increases blood flow to help dissipate body

minute. On an average day, the air temperature may

vary by as much as 30 degrees between night and day,

yet our bodies must maintain exactly the same

temperature. So how is that temperature maintained?

Well, it turns out that the body has a built in system for

keeping itself at that ideal temperature.

This system is controlled by the hypothalamus, a portion

of our brain. If the hypothalamus detects a dip in body

temperature, it will begin to reduce blood flow to our legs

and arms. This has the effect of conserving heat around

the more vital parts of the body such as the chest and

abdomen. On the other hand, if an increase in body

temperature is detected, the hypothalamus directs the

body to release heat and cool itself. Sweat glands begin

to perspire. The circulatory system kicks in and allows a

greater amount of blood to circulate closer to the surface

of the skin, which also releases heat. And, in no time at

all, the body has successfully stabilized its own

temperature.

[ f Script ]

Professor (female): OK, today we are going to talk about

temperature regulation within the human body. Most of

you already know that the optimal human body

temperature is 98.6°F. If our bodies stray even just a few

degrees from this optimal temperature, it can cause

serious damage or even death. Just think about that for a

[ f Script ]

Professor (male): You may not realize it, but school

administrations have a great deal of power over a

student’s future. School administrations often attempt to

identify which students are most likely to attend college

and then tailor each student’s education toward those

expectations. Uh, for example, there are special schools

that are designed to provide students with specific job

training, called, vocational schools. In such a school, a

student may attend classes on how to fix cars instead of

traditional history classes. Generally, students who

attend vocational schools are students from lower

income families who are unlikely to be able to afford

college. This can be beneficial to these students

because it provides them with the skills needed to get a

decent job without a degree, but since the students

receive less traditional academic instruction, attending a

vocational school greatly reduces the chances of ever

being accepted into college.

Even in a traditional high school, the administration has a

great deal of influence over a student’s future. Students

with the best grades and best test scores are often

placed in “advanced placement” or college preparation

courses. As these classes are generally small, only the

best students are admitted to them. Since most top

universities greatly favor students who have been in such

classes, students in regular classes find their choices of

college reduced.

Narrator (male): The professor discusses the ways in

which a school administration can influence a student’s

future. Explain how his discussion relates to the concept

of tracking.

Chapter 4

Scripts & Answer Key A95

SP

EA

KIN

G

MMaaiinn IIddeeaa:: human body’s self-reg. of temp.

KKeeyy PPooiinnttss:: hypothalamus detect. chngs in body temp.

If too cold, blood flow to hold heat

If too hot, blood flow

CCoonnnneeccttiioonnss ttoo tthhee rreeaaddiinngg:: body temp. regulation an

ex. of homeostasis

Page 96: Insider: The Super Guide Scripts & Answer Key

heat.

3. In the lecture, the professor explains how the body

reacts to external changes in order to keep a constant

body temperature. According to the professor, the

hypothalamus detects changes in body temperature and

responds to them. An example of this is the way that the

body reduces blood flow to the arms and legs to

conserve heat around more important body parts, or

increases blood flow to help dissipate body heat. These

are examples of homeostasis because they are

automatic and they keep the body in equilibrium.

Exercise 4 _ p.423

1. The body’s regulation of its temperature is an example of

homeostasis.

2. The hypothalamus automatically detects changes in

body temperature and increases or reduces blood flow to

restore normal body temperature.

3. In conclusion, the body’s regulation of its temperature is

an example of homeostasis. The hypothalamus

automatically detects changes in body temperature and

increases or reduces blood flow to restore normal body

temperature.

Guided Practice 1

Exercise 1 _ p.424

Exercise 2 _ p.424

The professor’s discussion of Mozart’s unusual musical

ability relates to the concept of nature vs. nurture in that

Mozart’s talent seems to have been the product of both his

education and his natural abilities.

Exercise 3 _ p.425

1. It is an example of the concept that shows how blurred

the line between nature and nurture is in regard to how

humans come to possess certain traits, specifically in this

case, musical abilities.

2. The professor says that Mozart’s father was a great

musician, and so Mozart grew up with an excellent

education in music. The professor also says that

Mozart’s progress in music was unusually fast, and that

other musicians with great teachers did not become

nearly as talented as Mozart.

3. The concept of nature vs. nurture is exemplified in the

discussion of Mozart’s unusual musical talent, and the

question of how he came to possess it. According to the

professor, one possibility is that Mozart’s talent was the

product of his father’s teaching. The professor points out

that Mozart’s father was a world famous musician

himself, and so Mozart had one of the best teachers

possible. This view of Mozart’s talent exemplifies the

nurture side of the debate. The professor also points out

that Mozart might have inherited a high level of natural

talent from his father. The professor notes that while

other musicians also had great teachers, none of them

were as talented as Mozart. This view exemplifies the

nature side of the debate.

view is also supported by the fact that he stood out so

much from other great musical composers of his time, all

of whom had excellent teachers as well. Clearly there

was something inherently special about Mozart that

made him so gifted.

[ f Script ]

Professor (female): For centuries, philosophers and

scientists have been debating the nature and origins of

genius. Does genius only arise in carefully cultivated

circumstances, like a rare and delicate flower? Or is genius

an inborn trait? It seems that genius is most often a product

of both. To show you what I mean, I’ll use the example of

one of the greatest musical geniuses in history.

We all know of Mozart, the great composer and child

prodigy who was composing music by the age of five and

operas at 14. These are incredible achievements for a

child, ah, almost unbelievable. Some might claim that

Mozart was a product of his environment. His father was a

famed musician in his own right, and perhaps the greatest

musical teacher in Europe. He conducted all of Mozart’s

early training and remained an important influence

throughout Mozart’s life.

Yet even with such an excellent teacher, the pace of

Mozart’s progress was remarkable. I mean, he was

composing world class music at the age of five, an age

when most of us can barely tie our own shoes. Clearly that

points to an incredible level of inherent ability. This

A96 Insider: The Super Guide

MMaaiinn ssppeeaakkeerr’’ss ooppiinniioonn:: Genius is the product of both

env. + natural ability

KKeeyy PPooiinnttss:: Mozart’s father great musician therefore

Mozart had excellent teacher (ex. of environment)

Mozart’s progress much faster than other musicians;

other musicians had great teachers but not as talented

as Mozart (ex. natural ability)

CCoonnnneeccttiioonnss ttoo tthhee rreeaaddiinngg:: Example of Mozart shows

example of nature vs. nurture debate.

Page 97: Insider: The Super Guide Scripts & Answer Key

Exercise 4 _ p.425

1. The professor’s discussion of Mozart’s unusual musical

ability relates to the concept of nature vs. nurture in that

it poses an example of the complicated nature of the

debate, that is, whether Mozart’s musical genius at such

a young age was inborn or whether it was a product of

his environment.

2. Mozart’s father was a renowned musician and music

teacher, so it may be possible that even as a five-year-

old, his training might have fostered the musical genius

in Mozart, making his talents a product of his

environment, or nurturing. On the other hand, some of

his father’s physical traits that may have been especially

conducive to learning music could have been passed on

to him genetically, making his talents inborn, or part of

his “nature.”

3. In conclusion, the complications in the debate of nature

vs. nurture, and the question of whether acquired skills

can be passed on genetically, are all present in the case

of Mozart and his unusual childhood genius. According

to the professor, Mozart and other geniuses probably

acquire their talent through a combination of both nature

and nurture.

Guided Practice 2

Exercise 1 _ p.426

Exercise 2 _ p.426

The professor’s discussion of women in the military suffering

from eating disorders relates to the concept of anorexia

nervosa because many of the conditions and causes of

anorexia as stated in the reading passage are present in the

lives of female soldiers.

Exercise 3 _ p.427

1. Many of the pressures women in the military are exposed

to are the same ones that the reading claims lead to

eating disorders.

2. The professor says that women in the military are often

not fully accepted and are harassed by male members of

the military. The professor also says that the women who

choose to join the military often hold themselves to

unreasonable standards.

3. According to the professor, many of the pressures

women in the military are exposed to are the same ones

that the reading claims lead to eating disorders. The

At the top of the list of those disorders are eating phobias

and a fixation on body image. Many women in the

military are obsessed with having a perfect body. They

may see attaining physical perfection as a way to gain

acceptance among male soldiers, and as a result, adopt

unhealthy and even dangerous exercise and diet

regimens. The problem, in fact, is so widespread that

one researcher estimates that eight percent of women on

active duty have a distorted body image and eating

phobia. This is more than double the percentage of

women in the civilian population who are diagnosed with

eating phobias each year. So, uh, it seems the effects

the military has on the mental health of women,

specifically, may have to be studied more closely.

[ f Script ]

Professor (male): The military can be a pretty toughenvironment for women. Female soldiers are, uh ... well,they are often not fully accepted by their malecounterparts. They may undergo consistent ridicule orharassment, which can be highly damaging to their ownsense of self-worth. Add to this the fact that manywomen who choose to enter the military are highly drivenand set rigorous, and sometimes unreasonable,standards for themselves, and what you get is a situationthat is ripe for a number of disorders.

Anorexia Nervosa

Anorexia nervosa is a medical term that describes a

type of eating disorder characterized by a distorted body

image and a manic obsession with weight control. It can

and often does result in a fatally low body weight. It is a

condition that affects mostly young men and women in

industrialized Western nations. While there is no single

cause of anorexia, researchers find that those with traits

like perfectionism or psychological disorders like

obsessive compulsive disorder are more likely to develop

the illness.

Scripts & Answer Key A97

SP

EA

KIN

G

MMaaiinn iiddeeaa:: women in military have rate of eating

disorders

KKeeyy PPooiinnttss:: competition with men obsessing over

body; high physical/ psychological stress

aggravates disorder; perfectionist tendencies

common

CCoonnnneeccttiioonnss ttoo tthhee rreeaaddiinngg:: perfectionist tendencies +

psychological stress as aggravating eating

disorders and being present in female soldiers; only

female soldiers from Western nations (specifically

US) included in study; anorexia sufferers have

self-esteem, female soldiers made to feel inferior to

males

Page 98: Insider: The Super Guide Scripts & Answer Key

professor also says that the women who choose to join

the military often hold themselves to unreasonable

standards. In the lecture, the professor says that women

in the military often attempt to perfect their bodies as a

way to gain acceptance among male military members

and to meet their own unreasonably high standards.

Exercise 4 _ p.427

1. The lecture is basically about how women in the military

face additional pressures which lead to a higher rate of

eating disorders in the military.

2. Women in the military often try to reach unrealistic levels

of body perfection in an attempt to gain acceptance

among male soldiers and to meet their own high

standards.

3. In conclusion, the lecture explains how the high rates of

eating disorders among women in the military are a

result of the pressures women in the military face,

specifically the wish to gain acceptance among male

soldiers and meet their own high standards.

Additional Practice _ p.428~431

1.

[ Sample Response ]

The professor’s discussion of the technological advances of

the Cold War relates to the concept of Social Darwinism in

that the competition between the US and the USSR led to

advances in society and also the downfall of a society.

According to the professor, the competition between the US

and the USSR led to many useful advances, such as the

development of supercomputers. But the professor also says

that led to the downfall of the USSR because the USSR

could not keep up with US in research spending. This

example illustrates the two major claims of Social

Darwinism: that competition between societies leads to

social and technological advancements, and that only the

most fit societies survive.

supercomputer, built by the US in 1976 to facilitate secret

military and espionage operations against the USSR.

Since that time, of course, supercomputers have become

essential to the scientific community and have allowed

any number of important scientific discoveries.

Now while the Cold War was responsible for some of the

most important technological advancements of the 20th

century, it also eventually led to the downfall of the

Soviet Union. The USSR could not keep pace with the

US in terms of spending on research and development of

new technologies. Eventually, the USSR was driven to

collapse by its struggle to keep pace with the US. So

while the Cold War resulted in the development of many

useful technologies, it also brought about the downfall of

a nation.

[ f Script ]

Professor (female): The end of World War II brought

about a bitter rivalry between the United States and the

Soviet Union. Fearing the spread of Communism into

western European nations, the US engaged in a

protracted Cold War with the USSR. While there was no

direct fighting, this race for political, military, and

ideological supremacy defined the post World War II era.

Both the US and the USSR invested incredible amounts

of resources into developing new technologies that could

be used against the other side. One excellent example of

this would be the supercomputer. Weighing in at some

30 tons, the CRAY-1 was the world’s first

Social Darwinism

Social Darwinism is the theory that the driving force in

the evolution of societies is competition among groups,

races, and even nations for survival and dominance.

Proponents of this theory believe that different societies

are locked in a struggle for existence, and that only

those societies fit for their environments (be it a political

environment or an economic one) will “survive” the

struggle. Supporters of this theory also believe that it is

this struggle that drives the advancement of human

civilization.

A98 Insider: The Super Guide

MMaaiinn IIddeeaa:: the Cold War defined post WWII era

KKeeyy PPooiinnttss:: US + USSR competing for

scientific/technological superiority; new

technologies; ex. super-comp. built by US to use

against USSR

Competition downfall of USSR b/c USSR

could not match US spending on military.

CCoonnnneeccttiioonnss ttoo tthhee rreeaaddiinngg:: Cold war is ex. of Soc.

Darwinism. Competition advances in US and

downfall of USSR

Page 99: Insider: The Super Guide Scripts & Answer Key

2. [ Sample Response ]

In the lecture, the professor explains how whites in the 19th

century tried to develop a scientific justification of racism.

According to the professor, a new justification of slavery was

needed because most slaves had converted to Christianity,

so the white slave owners needed to develop other theories

for why Africans should be enslaved. Basically, they claimed

that the facial features of the slaves proved they were

inferior to whites. The professor points out that there was no

real scientific basis for this claim, but that people believed it

because it provided a way to justify slavery. In short, people

believed in a pseudoscience because it confirmed the beliefs

of the community, just as it states in the reading.

3.

[ f Script ]

Professor (female): Scientists have long known thatnuclear fusion could provide us with an almost endlesssource of energy. Unfortunately, constructing a workingfusion reactor has proven extremely difficult.

Reproducing sustainable fusion in the laboratory hasbeen difficult for two reasons. First, fusion reactors haveyet to produce any significant amount of energy — thatis, no net energy, or energy beyond that which is used tocreate a nuclear reaction. Scientists have been able tocreate fusion reactions on Earth, but so far it hasrequired more energy to create the conditions requiredfor a fusion reaction to occur than is actually produced bythe reaction. Thus, while we can create a fusion reaction,we have yet to find a way to use fusion to create energyon Earth.

But this is only half the problem. Once a fusion reactionhas been achieved, scientists must find a way to sustainit. This means dealing with temperatures so high thatthey would destroy any material they came in contactwith. So how do you sustain a process that is going todestroy the machine that creates it? This is the otherproblem scientists must overcome before viable fusionreactors can be built on Earth.

Nuclear Fusion in Stars

Nuclear fusion is the process by which the nuclei of

different atoms fuse together to form a single nucleus. In

order to fuse, nuclei must be subjected to enormous

amounts of heat and pressure. In the natural world,

such conditions are only found in the cores of stars.

Temperatures inside a star’s core can reach tens of

millions of degrees, and the immense gravity of the star

exerts a great amount of pressure on nuclei. When

nuclei fuse, they release energy, primarily in the form of

heat, thus inducing other nuclei to fuse as well.

[ f Script ]

Professor (male): Today, we are going to discuss the

way that science was used, well, misused actually, in the

19th century to create justifications for the continued

enslavement of Africans. Now early in the history of

American slavery, the practice had been justified by

claiming that African slaves were not Christians and it

was therefore accepted to enslave them. By the

beginning of the 18th century, however, most slaves had

converted to Christianity and a new justification was

needed.

What arose were a number of scientific theories which

attempted to prove that Africans were actually a

separate, inferior species to white Europeans.

Researchers and scientists wishing to prove the inferiority

of Africans seized upon the physical differences,

especially the differences in facial structure, between

Africans and Europeans. They claimed that the facial

structure of Africans proved they were more primitive and

less advanced than whites, and there was a whole

system of measuring facial features to help support this.

Now not a single bit of this actually had any basis in

scientific fact, but the study of these facial differences

gained widespread popularity because it provided whites

with a convenient justification of slavery.

Pseudoscience

Pseudoscience is any variety of thought, theory, or

concept that claims to be scientifically founded but

actually has no solid grounding in science or has not

been acknowledged as true or plausible by the scientific

community. Fields of study often characterized as

pseudoscience usually lack theories or claims that can

be tested or verified utilizing the scientific method.

Despite their lack of scientific evidence, pseudosciences

may gain widespread acceptance among the general

populace if they espouse ideas that help to confirm the

beliefs or values of the community.

Scripts & Answer Key A99

SP

EA

KIN

G

MMaaiinn IIddeeaa:: study of facial featrs. created by whites to

justify slavery

KKeeyy PPooiinnttss:: traditional just. of slav. not useful b/c

most slaves convert to Christianity

In 19th cent. whites try to prove Afrc. inferior w/

science – ex. facial features

No scientific basis, but convenient just. of slav., so

accepted as fact.

CCoonnnneeccttiioonnss ttoo tthhee rreeaaddiinngg:: Study of facial features

is example of pseudoscience

Page 100: Insider: The Super Guide Scripts & Answer Key

[ Sample Response ]

In the lecture the professor explains why it is so difficult to

use fusion to create energy. This relates to the concept of

fusion in stars because most of the difficulties described by

the professor relate to replicating the conditions inside a

star. The reading states that in a star, nuclear fusion is

driven by very high levels of heat and pressure. The

professor notes that creating the heat and pressure for

fusion to occur on Earth actually takes more energy than is

created in the fusion reaction. Furthermore, the professor

points out that since fusion reactions release huge amounts

of heat and energy, containing a fusion reaction would be

very difficult on Earth. In short, using fusion to create energy

on Earth is very difficult because scientists first have to create

and then contain the conditions that exist inside a star.

4.

[ Sample Response ]

In the lecture, the professor explains why the US placed

Japanese Americans in internment camps but not German

or Italian Americans. Basically, it was the result of

xenophobia. The professor points out that there was a long

history of racist attitudes towards Asian Americans in the

United States. This coincides with the claim in the reading

that xenophobia is based on existing prejudices. In addition,

the professor says that the shock and the anger created by

the surprise attack on Pearl Harbor fed into those racist

attitudes and eventually led to the internment of Japanese

Americans. This directly relates to the claim made in the

reading that xenophobia is usually triggered by a traumatic

experience. In short, the internment of Japanese Americans

after the attack on Pearl Harbor was basically a xenophobic

reaction.

First, the vast majority of Japanese Americans were

living in California and other western states at the time.

Now across America, and in this region in particular,

there had been a longstanding history of racist attitudes

towards Asians. Most white Americans did not view

Asians as their equals and did not fully accept them into

American society. German and Italian Americans, on

other hand, had a common cultural background with

other white Americans and were more or less readily

accepted into American society.

The other factor leading to the detention of Japanese

Americans was the attack on Pearl Harbor. After the

surprise attack by the Japanese Navy, most Americans

were shocked, horrified, and above all, extremely angry.

This anger fed into the racist attitudes concerning Asians

that already existed in American society and eventually

led to the placement of Japanese in internment camps.

[ f Script ]

Professor (female): As many of you know, during World

War II, over 100,000 Japanese Americans were rounded

up and put in detention camps on suspicion that they

might be working for the enemy. Now German and Italian

Americans were not placed in detention camps, yet the

governments of their homelands were also at war with

the US. So why, then, were Japanese Americans singled

out as potential enemies of the state?

Xenophobia

Xenophobia is an excessive and irrational fear of

individuals who are different from oneself. Xenophobia

often centers on a fear and aversion to a specific social

group. Generally, xenophobia is rooted in a preexisting

cultural prejudice but is often exacerbated or brought to

the surface by a traumatic experience involving a

member of a certain group. Xenophobia can result in

discriminatory or violent behavior towards a group of

individuals even in the absence of provocation or

justifiable cause.

A100 Insider: The Super Guide

MMaaiinn IIddeeaa:: Internment of Jap. Am. racial pred. +

Pearl Harbor

KKeeyy PPooiinnttss:: long hist. of racist att. towards Asian Am.

esp. in Calif. + western states… white Am. did not

see Asians as equals

After Pearl Harbor Am. shocked + angry... anger

fed racist att. internment camps

CCoonnnneeccttiioonnss ttoo tthhee rreeaaddiinngg:: causes of internment of

Jap. Am. = causes of xenophobia

MMaaiinn IIddeeaa:: the difficulties of producing fusion energy

KKeeyy PPooiinnttss:: fusion reactors yet to produce net energy;

hard to replicate high pressure and temp in lab; hard

to generate enough heat to cause more reactions; still

experimental

CCoonnnneeccttiioonnss ttoo tthhee rreeaaddiinngg:: reproducing fusion

reactions involves high temps and pressure of stars;

sustained fusion in stars due to gravity confining

nuclei until fused, while confinement on earth still

impossible

Page 101: Insider: The Super Guide Scripts & Answer Key

Problem / Solution Task

Overview _ p.433

( Format of the Conversation

Preparing Your Response

Exercise 1 _ p.434

Exercise 2 _ p.435

The man’s problem is that his mom wants him to transfer to

another school with better resources, but he doesn’t want to

be away from his girlfriend. The woman suggests that he

either transfer schools and drive to see his girlfriend or try

explaining the situation to his mom. In my opinion, the best

solution is to just explain the situation to his mom.

Exercise 3 _ p.436

I feel this is a better solution because it’s his life and he

needs to make his own decisions about his future. If he

really thinks that being near his girlfriend is more important

than going to a school with better funding, that is his choice.

He should just explain this to his mom. In addition, as the

man points out in the conversation, the university his mom

wants him to transfer to is far away, and he would not get to

see his girlfriend very often. Long distance relationships are

hard to maintain, so I think he should stay at the same

school if he wants to keep dating his girlfriend.

Man : Yeah. Well, my girlfriend Amy … she says she

won’t transfer with me because she loves the

English department here.

Woman: Hmm … OK. Why not just transfer and

commute to see each other?

Man : See, I got into a school that’s kind of far. It’s at

least a three hour drive from here. It would be

difficult for us to see each other regularly.

Woman: Oh. OK. Maybe you should explain all of this

to your mother. Maybe she’ll understand that

transferring to a new school could hurt your

relationship with your girlfriend.

Man : Hmm … I hadn’t thought of that. My mom is

already pretty excited about the school. I’m not

sure if I want to upset her by telling her I don’t

want to transfer.

Woman: Looks like you have a pretty tough decision to

make.

[ f Script ]

Man : Hey, Nia … I could really use your advice.

Woman: What’s up?

Man : My mother thinks I should transfer to another

university.

Woman: Really, why?

Man : Well, the biology department here just lost a

lot of funding. I’d have a lot more resources if I

transferred to a university with a well-funded

program.

Woman: OK. So I’m guessing you don’t really want to

leave.

[ f Script ]

Man : Hey, Julia. I think I’m going to have to skip out

on going to the movies tonight. Sorry.

Woman: Oh … OK. What’s up?

Man : Well, I got this huge project in my sociology

class, and I’m not even close to finished. It’s

due on Friday … I don’t know how I’m ever

going to finish it.

Woman: Well, today’s only Wednesday. You got tonight

and all of tomorrow to work on it. I’m sure

you’ll get it done.

Man : That’s the thing. Thursday is my heaviest class

day. I’m in class until 9 o’clock tomorrow night.

I just don’t see how I’m going to find the time.

Woman: Well, if this project is that important, just skip

your classes tomorrow, and spend the whole

day working on the project. I do that

sometimes when I have a big exam to study

for.

Man : Yeah, but some of my other professors might

not appreciate me skipping their classes like

that.

Woman: I think they’d understand, but if you’re really

worried about it, you can just drink tons of

coffee, and stay up tonight and tomorrow night

to finish the project. It won’t be fun, but I’m

sure you can do it.

Chapter 5

Scripts & Answer Key A101

SP

EA

KIN

G

SSttuuddeenntt’’ss PPrroobblleemm:: mom wants him to transf. to other

univ. b/c biology dept. lost funding. Prob is that

girlfrnd doesn’t want to transfr.

SSoolluuttiioonn 11:: transfer to other univ. + drive to see his gf

CCoonnss ttoo ssoolluuttiioonn:: new univ. more than 3 hrs. away

can’t see gf. often

SSoolluuttiioonn 22:: explain situation to mom

Page 102: Insider: The Super Guide Scripts & Answer Key

Exercise 4 _ p.437

1. I think he should explain the situation to his mom.

2. Deciding which school he goes to is his choice, and it will

be too hard to continue the relationship if he transfers.

3. In conclusion, deciding which school he goes to is his

choice, and it will be too hard to continue the relationship

if he transfers. Therefore, I feel the best solution is to talk

to his mom and explain why he doesn’t want to transfer.

Guided Practice 1

Exercise 1 _ p.438

Exercise 2 _ p.438

The man’s problem is that his computer crashed and he has

a paper due in a week. The woman offers two possible

solutions: that he take his computer to the tech support

office or that he write the paper over again. Personally, I

think he should he should take the computer to the tech

support office.

Exercise 3 _ p.439

I feel taking the computer to the tech support office is the

best solution because otherwise he is just wasting a lot of

work. If he has already written the paper, he should at least

try to get the files back before he starts all over again. In the

conversation, the woman points out that if he explains the

situation, the tech support office can probably rush the job

for him. Therefore, I don’t think his concern that he won’t get

his computer back in time is a big deal.

Exercise 4 _ p.439

1. I think he should at least try to get his computer fixed.

2. Starting his paper over again will mean he has wasted a

lot of work, and the woman said that the tech support

office can probably rush the job if he explains the

situation.

3. In conclusion, starting his paper over again will mean he

has wasted a lot of work, and the woman said that the

tech support office can probably rush the job if he

explains the situation. Therefore, I think it would be better

for the man to at least try to get his computer fixed.

[ f Script ]

Woman: Hey, Brian. How’s it going?

Man : Not so great. I was working on my term paper

last night at home and my computer crashed.

Woman: I’m sorry to hear that. Did you have everything

backed up on a disk?

Man : Unfortunately, no. So I lost everything. And the

paper is due in a week!

Woman: You know, the university offers free technical

support. You can just take your computer up

to their office and see if they can recover the

files for you.

Man : I’ve heard only bad news about the tech

support department. They’re always really

busy, so the wait time can be a week or more.

By that time the paper will be due.

Woman: Maybe if you explained your situation, they

could put a rush on it for you. Or … you could

start over on the paper. I know it’s not ideal,

but you still have a week, right?

Man : I suppose it’s possible. Most of my research

was from books, and I kept hard copies of

electronic resources, so the raw materials are

there. That still leaves a lot of work, though.

Woman: Well, whatever you decide to do, good luck.

A102 Insider: The Super Guide

SSttuuddeenntt’’ss PPrroobblleemm:: his comptr crashed while working

on a term paper

DDeettaaiill:: didn’t back up files; paper due in a week

SSoolluuttiioonn 11:: take the comp. to the tech. suppt office

DDeettaaiill:: tech suppt might take too long, and may not be

able to recover his files

SSoolluuttiioonn 22:: write the paper again from what he has left

DDeettaaiill:: still has resrch matls and still has a week before

the paper is due

Page 103: Insider: The Super Guide Scripts & Answer Key

Guided Practice 2

Exercise 1 _ p.440

Exercise 2 _ p.440

The woman’s problem is that she needs to get her car fixed,

but the appointment is at the same time as her make-up

exam. The man suggests that she either reschedule her

appointment with the mechanic or ask the professor to

postpone the exam. I think it would be better for her to get

her car fixed at another time.

Exercise 3 _ p.441

Rescheduling the appointment with the mechanic is a better

option because her exam is obviously more important. As

the woman points out in the conversation, her professor has

already said this is her only chance to take the make-up

exam. If she misses the exam, she might fail the class. In

addition, her brother will understand why she couldn’t help

him move. He can probably find someone else to help him

anyway.

Exercise 4 _ p.441

In short, I feel passing her exam is the woman’s most

important responsibility, and her brother can probably get

someone else to help him anyway. Therefore, I believe the

woman should reschedule her appointment and take her

exam.

Additional Practice _ p.442~445

1.

[ f Script ]

Man : Hi, Haley. How’s it going?

Woman: Not so great. I just scheduled my thesis

defense a few days ago, you know, only to

find out now that my boyfriend’s senior art

exhibition is opening on the same day. I can’t

disappoint him by not going, but I certainly

can’t miss my thesis defense.

Man : Well, the exhibition will last a week. You might

miss the opening, but you might make it up to

him by having him take you on a special tour

of the exhibit at a later date.

Woman: I suppose I could do that. He’d still be pretty

disappointed with me, though. I don’t want to

ruin his big day.

Man : Of course, another option would be to try and

move your thesis defense to another time of

day, so that you could make it to at least part

of the opening.

Woman: I don’t know how flexible they are in the

department; they get swamped with theses

during spring semester. And anyway, I still

might miss the opening. There are also written

and oral exams that go along with it, and I

have no idea how long those will take.

Man : Think about it a while. I’m sure you’ll decide on

something that suits the both of you.

[ f Script ]

Man : Hey, Lisa … why do you look so confused?

Woman: I need to take my car to the mechanic, but I

have a make-up exam in two hours.

Man : Is that a problem?

Woman: Well, the mechanic just said that it will take at

least three hours for him to fix my car, which

means that I won’t get back in time to take that

test.

Man : Hmm … Why don’t you just postpone your

appointment with the mechanic?

Woman: Today is Friday. The mechanic is closed on

weekends. If I don’t get that car fixed today,

then I won’t be able to drive into the city this

weekend to help my brother move into his

new apartment.

Man : Oh. Well, can’t you ask your teacher to

postpone the test?

Woman: I wish. I had to cancel once before, and she

said this is my last shot to make up this exam.

If I don’t show up in her office in two hours,

she’ll fail me on the exam.

Man : That doesn’t sound good.

Woman: Yeah. Well, I need to make a decision soon,

because the clock is ticking.

Scripts & Answer Key A103

SP

EA

KIN

G

SSttuuddeenntt’’ss PPrroobblleemm:: she needs to take her car to a mech

but has an exam in 2 hrs

DDeettaaiill:: 3 hrs. to fix car, will miss exam

SSoolluuttiioonn 11:: postpone her appt w/ mech

RReelleevvaanntt DDeettaaiill((ss)):: mech closed all weekend; needs the

car to help bro. move

SSoolluuttiioonn 22:: ask the prof to postpone the exam

RReelleevvaanntt DDeettaaiill((ss)):: the prof said today only day for

exam

Page 104: Insider: The Super Guide Scripts & Answer Key

[ Sample Response ]

The woman’s problem is that she has scheduled her thesis

defense and exams on the same day as the opening of her

boyfriend’s art exhibition. In my opinion, the best solution is

to miss the opening and visit the exhibition with her

boyfriend at a later time. Her boyfriend might be

disappointed, but he should understand that she has to take

her exams and defend her thesis. Besides, as the woman

points out, she doesn’t even know how long the exams will

take, so even if she tries to take them a little earlier, she still

might miss the exhibition. Therefore, I think she should just

tell her boyfriend she can’t come. At least that way he won’t

be expecting her to be there.

2.

[ Sample Response ]

The man’s problem is that he is afraid that he might fail one of

his classes and that he will lose his scholarship if his GPA is

too low. The professor suggests that he either study really

hard to try and bring his grade up or that he drop the course

and take it again next semester. I think the man’s best option

is to drop the class. First, as the professor points out, he

would basically have to get all A’s for the rest of the class, and

even then he might not pass. If his grade is that bad, he needs

to just drop the course. Furthermore, while the man is worried

that his schedule next semester is too full to take the class

again, if he loses his scholarship he won’t even be in school

next semester. In conclusion, I think the man should just drop

the class because it seems like his grade is too low to bring

up, and keeping his scholarship is the most important thing.

3.

[ f Script ]

Woman: Hey, Josh. What’s up?

Man : I just got a call from the restaurant where I

work. Apparently, they don’t need me

anymore. That’s how they said it, anyway.

Woman: You got fired? Oh, I’m so sorry.

Man : I don’t know what I’m gonna do. The only way

I could get my parents to agree to pay my

tuition is if I got a job and paid for housing and

living expenses. The deadline for housing fees

is coming up in less than a month. I don’t

know what my parents will do if they find out I

can’t pay it.

Woman: Have you checked with the student

employment office? They might be able to find

you some work pretty quickly. They have a lot

of contacts in the community. Some of it’s on

campus, too.

Student : I have a pretty full schedule next semester,

and taking more classes than I can handle

might bring all of my grades down and still

lower my average.

Professor: Well, I’m leaving it up to you.

[ f Script ]

Student (male): Excuse me, Dr. Snyder. Do you have a

minute?

Professor (female): Of course. What’s up?

Student : Well, I know I’m not doing very well right

now, and I’m wondering if I’ll even be able to

pass your class.

Professor: Let me look up your grade on my computer.

[pauses] Hmm. No, this doesn’t look good.

Student : I really need to pass this class. If I don’t

maintain my grade point average I could

lose my scholarship. Is there anything I can

do to bring my grade up to passing?

Professor: Well, you can work very hard, make all A’s

for the rest of the semester, and maybe

make a C in the class, although I can’t swear

to it. Of course, as far as your grade point

average is concerned, you may still have a

problem, since all the time and effort you’ll

be putting into this class may take time away

from other classes, and bring down your

grades overall.

Student : Any other options?

Professor: Well, it’s still mid-semester, and you can

drop out of this class without a penalty if you

want, and try it again next semester.

A104 Insider: The Super Guide

SSttuuddeenntt’’ss PPrroobblleemm:: might not pass class

DDeettaaiill:: he could lose his scholarship if GPA too low

SSoolluuttiioonn 11:: work extra hard to bring his grade up to

passing

DDeettaaiill:: will take time away from other classes

SSoolluuttiioonn 22:: drop out of the class and repeat it next

semester

DDeettaaiill:: student has full sched. next sem.

SSttuuddeenntt’’ss PPrroobblleemm:: sched. her thesis def. + exams

same day boyfrnd’s art exhbtn.

DDeettaaiill:: she does not want to miss either of them

SSoolluuttiioonn 11:: miss 1st day of exhbtn + go w/ boyfrnd

diff. day.

DDeettaaiill:: bf. will be disappointed

SSoolluuttiioonn 22:: try to sched. thesis def. exams earlier in the

day

DDeettaaiill:: doesn’t know how long thesis def. will take

Page 105: Insider: The Super Guide Scripts & Answer Key

[ Sample Response ]

The student’s problem is that he has just been fired from his

job and payment for campus housing is due soon.

Additionally, the student’s parents may not pay his tuition for

the coming year if he cannot afford to pay for housing. In my

opinion, the best solution to the problem is for the student to

take out a student loan to cover the cost of housing. I think

this is the best solution because a small loan will help him

pay for housing and give him a little more time to find a job.

Also, this solution is best because going to the student

employment office might not help. As the woman points out,

there is no guarantee they will be able to help him find a job.

4.

[ Sample Response ]

The student’s problem is that he has an important paper due

on the same day that he is to attend his sister’s wedding. In

addition, he has already promised his family that he will

arrive early to help them get ready for the event. In my

opinion, the best solution to the student’s problem is to turn

in his paper before he leaves. I think this is the best solution

because it will not result in any deductions from his grade.

Likewise, his grade in the class will not drop as it would if he

turned in the paper late. In conclusion, I think that handing in

the term paper before leaving for the wedding is the best

solution. It may mean more work for the student, but it is the

solution that is least detrimental to the student’s grades.

working extra hours to save money for the

trip. Honestly, I don’t know how much more

work I can realistically have done before I

leave ...

Professor: I understand. I guess I can make an

exception. I’ll let you hand in the paper after

you return. But I still don’t think it’s fair to the

other students who are turning theirs in on

the 12th, so … I’ll have to deduct 30% off the

top.

Student : Thirty percent?! But … ahh … that’s a big

deduction. That’s really going to bring down

my class grade. I don’t know … I’m not sure

if my grade point average can take that kind

of a hit. It’s already been slipping this

semester, and I have scholarships to think

about.

Professor: Well … you still have the option of handing

the paper in early.

[ f Script ]

Student (male): Professor Smith … do you have a

minute to talk?

Professor (female): Sure, come on in.

Student : Okay … you said that the last paper for the

class is due on May 12th, but I will be out of

town for my sister’s wedding on that day. I

sort of promised my family that I’d be there

early to help get everything ready.

Professor: Well, congratulate your sister on my behalf.

But … I’m afraid your paper is still due on

May 12th. You can hand in the paper early, if

you’d like.

Student : Oh, but I have several other papers to hand

in before I leave for the wedding. And I’m

Man : That’s not a bad idea. I need to look at other

options though, in case they can’t place me

anywhere.

Woman: There are always student loans. You could

take out a small loan to pay for housing. You’d

have to be able to pay it off, of course. But it

would give you a little more flexibility in finding

a new job.

Man : Thanks. I’ll think it over.

Scripts & Answer Key A105

SP

EA

KIN

G

SSttuuddeenntt’’ss PPrroobblleemm:: lost job + has to pay for campus

housing soon

RReelleevvaanntt DDeettaaiill:: parents only pay his tuition if he

pays housing and living expenses

SSoolluuttiioonn 11:: go to student empl off.

RReelleevvaanntt DDeettaaiillss:: off. has lots of contacts but no

guarantee of job.

SSoolluuttiioonn 22:: take out a student loan

RReelleevvaanntt DDeettaaiillss:: would give him time to find a job;

but will have to be able to pay it off later SSttuuddeenntt’’ss PPrroobblleemm:: paper due on the same day as sis.

wedding

RReelleevvaanntt DDeettaaiillss:: promised sis. be there early to help get

ready

SSoolluuttiioonn 11:: hand in the paper early

RReelleevvaanntt DDeettaaiillss:: already has many assignments to

do before leaving + is working extra to save

money for travel

SSoolluuttiioonn 22:: hand in the paper late

RReelleevvaanntt DDeettaaiillss:: the professor will deduct 30% from

the paper grade if late

Page 106: Insider: The Super Guide Scripts & Answer Key

Summarization Task

Overview _ p.447

( Format of the Lecture

Preparing Your Response

Exercise 1 _ p.448Exercise 2 _ p.449

2. Example 1: Improved transportation

Example 2: Improved urban infrastructure

3. The lecture is mainly about how the Industrial Revolution

led to changes and improvements in England. The

lecture shows that changes in transportation and urban

infrastructure are two kinds of improvements caused by

the Industrial Revolution.

the city in search of work. These two changes alone

necessitated and hastened two major developments of

the period: improved transportation for goods and

laborers; and improvements to England’s urban

infrastructure.

The Industrial Revolution placed new demands on

England’s transportation system. Raw materials had to

be transported to factories, and finished goods had to be

shipped from factories to various cities where they could

be sold. A large web of turnpikes, or toll roads, began to

emanate from the capital city of London, built by private

companies often under contract with large manufacturers

in need of better inland transportation. In addition, the

railway system was a direct result of England’s need for

more efficient transportation methods during the

Industrial Revolution.

The Industrial Revolution also brought about a surge in

the urban population. As more and more people moved

into the overcrowded cities, conditions grew worse and

worse. For decades, the working classes lived in squalid

conditions, with no underground sewage, no clean water,

and cramped housing. Slowly, however, the

infrastructure of England’s cities was rebuilt and

improved. Improved sewage and plumbing systems were

installed, and large apartment blocks were built to

accommodate the rising urban population.

[ f Script ]

Professor (male): The Industrial Revolution brought

about a number of very sudden changes to the country’s

basic infrastructure. England’s many inland rivers proved

inadequate for transportation as the Revolution wore on,

creating a need for alternative methods. In addition,

urban industrial centers saw an unprecedented rise in

population, with laborers migrating from the country to

[ f Script ]

Professor (male): Reducing crime in a neighborhood can

be a difficult task. Sometimes simply hiring more police

officers is not effective in preventing crime. In these

cases, there are a number of things members of the

community can do to help prevent crime in their

neighborhood.

One particularly effective method of reducing crime in a

neighborhood is to set up a neighborhood watch

program. A watch program is basically a group of

residents who patrol the neighborhood at night. If they

see anyone committing a crime or anyone who looks

suspicious, they call the police. The programs are

effective for a number of reasons. First, if criminals know

that a community has a neighborhood watch program,

they will generally avoid that community, because they

know they are much more likely to get caught in such a

neighborhood. Second, since the members of the watch

group live in the neighborhood, they are more familiar

with it than police are. This makes them more effective at

patrolling the neighborhood.

Another type of program that can help reduce crime is a

neighborhood activities center. You see, a surprisingly

large amount of crime is committed by teenagers. In

many such cases, teenagers are simply bored and have

nothing to do, so they may begin to drink alcohol or do

drugs and end up getting in trouble as a result. By

opening an activities center, the community provides

teenagers with things to do in their free time. This keeps

them out of trouble and helps reduce crime in the

neighborhood.

Chapter 6

A106 Insider: The Super Guide

MMaaiinn CCoonncceepptt:: Industrial Rev. many changes in

England’s infrastructure

EExxaammppllee 11:: improved transportation

DDeettaaiillss:: raw matrls. needed at factories. + finished

goods needed in cities

Turnpikes and railroads built to move matrls. +

goods

EExxaammppllee 22:: improved city infrastructure

DDeettaaiillss:: during Indus. Rev. more urban pop.

Living conditions were terrible. New sewage sys. +

large apt. blocks built to improve conditions.

Page 107: Insider: The Super Guide Scripts & Answer Key

Exercise 3 _ p.450

1. The professor says that better transportation systems

were needed to move raw materials to the factories and

finished goods to the cities.

2. Turnpikes, roads, and a railway system were built to

provide better transportation.

3. The professor says that the urban population grew

during the Industrial Revolution, and that living conditions

for the working classes were terrible. As a result,

improvements to urban infrastructure had to be made.

4. Sewage systems were built, and large apartment blocks

were built to house the growing population.

Exercise 4 _ p.451

1. Changes brought about by the Industrial revolution

necessitated changes in England’s infrastructure.

2. Improved transportation: Better transportation systems

were needed to carry goods to and from the factories.

Improved city infrastructure: The large increase in urban

population made improvements to the city infrastructure

necessary.

3. In conclusion, changes brought about by the Industrial

revolution necessitated changes in England’s

infrastructure. Better transportation systems were

needed to carry goods to and from the factories, and the

large increase in urban population made improvements

to the city infrastructure necessary

Guided Practice 1

Exercise 1 _ p.452 Exercise 2 _ p.452

2. Example 1: Town hall meetings

Example 2: Initiatives

3. In the lecture, the professor explains that although the

US is basically a representative democracy, there are a

few examples of direct democracy in the US. The

professor discusses town hall meetings and initiatives as

examples of this.

Exercise 3 _ p.453

1. The first example the professor gives of direct

democracy in the US is a town hall meeting, in which all

the adult members of a small community meet to decide

on the town’s policies.

2. During the meeting, individual members of the

community get a chance to express their opinion about

proposed policies. Then, at the end of the meeting, all the

members vote on whether or not to approve the policy.

3. The second example of direct democracy is the creation

of state laws through the use of initiatives.

the increase or not. Town hall meetings generally only

occur in very small communities. In larger communities,

gathering all the members of the community together at

one time is simply unrealistic.

In some states, most notably California, some laws are

created by initiatives. First, a citizen proposes a new law,

but in order to have his or her law considered, that citizen

must collect a certain amount of signatures from people

who approve of the proposed law. If the required number

of signatures is collected, then the proposed law is put

on the ballot during the next set of state elections. If

more than 50% of the public votes in favor of the

proposed law, then it is approved and becomes law.

[ f Script ]

Professor (female): In general, the United States is a

representative democracy, which means that the public

does not directly play a part in creating and approving

laws. Instead, the public elects officials like congressmen

and senators who create and approve laws. There are,

however, a few examples of direct democracy in the

United Sates. In direct democracy, the public plays an

active role in the creation and approval of laws.

One example of direct democracy in the US is the town

hall meeting. In a town hall meeting, the adult members

of a community will meet to decide upon the official

policies of the local government. For example, a

proposed increase in taxes might be proposed and

decided upon in such a meeting. During the meeting,

individual citizens will be given an opportunity to express

their opinion of the proposed tax increase, and at the end

of the meeting there will be a vote on whether to approve

Scripts & Answer Key A107

SP

EA

KIN

G

MMaaiinn CCoonncceepptt:: There are some examples of direct dem.

In US

EExxaammppllee 11:: Town hall meeting

DDeettaaiillss:: members of community meet to discuss town

policy

Indiv. get chance to express opinion; at end of

meeting members vote

Only practical in small comm.

EExxaammppllee 22:: Initiatives

DDeettaaiillss:: Citzen proposes law, must get enough people to

sign

If enough sign, then pub. votes on law in next elect.,

more than 50% vote yes then law passes

Page 108: Insider: The Super Guide Scripts & Answer Key

4. In an initiative, an individual citizen proposes a new law. If the

citizen can get enough signatures of people who approve

of the law, the public votes on the law during the next

election. The professor says that more than 50% of the

voters have to vote in favor of the law in order for it to pass.

Exercise 4 _ p.453

1. There are some examples of direct democracy in the US.

2. Town hall meetings: Town hall meetings are examples of

direct democracy because each person gets to voice his

or her opinion and vote on policy changes.

Initiatives: Initiatives are examples of direct democracy

because a law can be created by an individual citizen

and then approved in a public vote.

3. In conclusion, town hall meetings and initiatives are

examples of direct democracy in the US. In both town

hall meetings and initiatives, the public plays a direct role

in the creation and approval of public policy and laws.

Guided Practice 2

Exercise 1 _ p.454 Exercise 2 _ p.454

In the lecture, the professor discusses bioelectricity.

According to the professor, bioelectricity is normally used

involuntarily, but its use can be voluntary in some animals.

Exercise 3 _ p.455

The professor says that most uses of bioelectricity are

involuntary, and uses the example of the nervous system to

show this. According to the professor, the nervous system

allows the body and the brain to communicate through the

use of electric signals. For example, the nervous system

sends electric signals from the brain to the heart, telling the

heart when to beat. The professor also shows that electrical

signals are used to transmit information, such as feelings of

pain, from the body to the brain. Sending these kinds of

signals is involuntary in nature. In contrast to humans, the

professor says that some animals can use bioelectricity on a

voluntary basis. Electric eels can voluntarily release an

electric shock to kill their prey, and electric catfish use

electricity to help them find their prey and avoid enemies.

Exercise 4 _ p.455

In conclusion, involuntary uses of bioelectricity are those

which we have no control over, such as the transmission of

electric signals through the nervous system. Voluntary uses

of bioelectricity, on the other hand, are those which can be

controlled, and are only found in some animals.

grown man. Eels use their bioelectricity to kill prey and to

defend themselves, but other aquatic creatures, such as

the electric catfish of Africa, use this energy for detection,

whether foraging for food or looking out for potential

predators.

[ f Script ]

Professor (female): When you think about electricity,

what sorts of images come to mind? A lightning bolt?

Electronic devices? What about inside your body? You

may not realize it, but there is a substantial amount of

what’s called bioelectricity coursing through your body

right now. In general, we might define bioelectricity as

any electric current that flows within a living organism.

Now, much of this is involuntary, but sometimes

bioelectricity can be used voluntarily.

As I said, most examples of bioelectricity are involuntary.

Take your nervous system, for example. The nervous

system uses pulses of electricity to send signals from

your brain to the rest of your body. For example,

bioelectrical signals are sent to the heart, telling it when

to beat. Electrical signals are also used by the nervous

system to transmit information from the body to the brain.

If you put your hand on a hot stove, you’ll obviously feel

pain. That is because electrical signals will travel through

the nervous system to tell your brain that your hand is

being burnt. In both of these cases, the body’s use of

bioelectricity is involuntary.

There are, however, voluntary uses of bioelectricity. This

is not something humans are capable of, but it is most

commonly seen in animals such as the electric eel. The

electric eel has specialized organs that can release

substantial amounts of bioelectricity, enough to stun a

A108 Insider: The Super Guide

Bioelectricity: any form of elec. current inside the body,

can be voluntary or involuntary

Invol bioelec:

Ex) nervous system uses elec to send signals from

brain to body.

Elec signals control heartbeat + send pain signals to

brain

Vol. biolelec not in humans but other animals

Ex) elec eel can release biolelec to stun or kill other

animals

Elec. catfish uses biolelec to detect prey + enemies

Page 109: Insider: The Super Guide Scripts & Answer Key

Additional Practice _ p.456~459

1. [ Sample Response ]

In the lecture, the professor explains how, despite the 1st

Amendment’s guarantee of freedom of the press, there is

still some censorship in the US. The professor points out

that the FCC has the power to fine a broadcasting company

or take its license. The professor also says that the FCC

uses these powers to punish a broadcasting company if it

broadcasts offensive material. But since the FCC is simply

enforcing its opinion of what is offensive, this is a form of

censorship. The professor also points out that major

corporations are able to practice censorship because they

are the ones who buy ads from the broadcasting company.

Since broadcasting companies rely on the money from these

ads, they can’t express opinions that would anger major

corporations, because those corporations could retaliate by

refusing to buy more ads. In short, the American media

aren’t really as free as they seem because both the FCC

and major corporations are able to practice some forms of

censorship.

2.

[ f Script ]

Professor (male): When the Earth first formed nearly 5

billion years ago, it was a hot volcanic planet pretty much

completely covered in molten lava. Uh, not exactly the

kind of place you would expect life to form. Over time, a

number of significant events began to slowly transform

the Earth into a place where life could develop.

One such type of event was the repeated impact of

comets. Now, some of you may be asking yourself, “How

could a comet hitting the Earth help create life?” To

answer that, think about what comets are made of.

Comets are basically big hunks of ice and dust. As

comets collided with the Earth billions of years ago, they

deposited significant amounts of ice. That ice of course

melted to form water, which we all know is a basic

requirement for any form of life. The dust carried in

comets was also helpful because it carried other

chemicals that are necessary for the formation of life.

Another event that we believe was crucial for the

development of life was the formation of the moon. The

moon played a number of roles in the development of

life. First, the moon’s gravity creates tides on Earth.

These tides stirred up chemicals in the early oceans

which we believe eventually combined to form the first

life forms. The tides also helped to wash the nutrients

these new life forms needed to survive into the sea.

[ f Script ]

Professor (female): Most of us have been taught that the

1st Amendment guarantees freedom of the press, and

that the American media is free of censorship. Today,

however, I’m going to show you a couple examples of

censorship in America, and I think you’ll see that the

press isn’t as free as many of us believe.

The FCC, or the Federal Communications Commission,

is a federal agency that in some ways actively censors

the media. You see, the FCC issues broadcasting

licenses to media companies, and it also has the power

to revoke those licenses if it decides the company is

broadcasting inappropriate content. The FCC can also

issue fines to companies who broadcast materials the

FCC deems to be offensive, and it often issues such

fines. Since not every single American sees the same

things as being offensive, the FCC is basically imposing

its opinion of what is offensive and what is not on the

American public, and this is of course a form of

censorship.

It may surprise you to learn that major corporations are

also responsible for censorship of the media.

Broadcasting companies make their money from ads,

right? Without money from ads, the broadcasting

companies would quickly go out of business. And who

pays for these ads? Major corporations. So when a

corporation doesn’t agree with the views expressed by a

broadcasting company, it can retaliate by refusing to buy

more ads. In effect, this forces the broadcasting

company to only express opinions the corporations will

approve of.

Scripts & Answer Key A109

SP

EA

KIN

G

1st amend. guarantees freedom of press but there is

censorship in US

FCC: fed. Agency censors media

FCC can punish broadcast co. if broadcast offensive

material, can issue fine or take license.

Not all Americans think same thing off.

FCC imposes its opinion = censorship

Major corps. Censor media

Major corps buy ads and so have control over

broadcasting co.

Can refuse to buy ads if don’t like opinions of

broadcasting co.

Page 110: Insider: The Super Guide Scripts & Answer Key

[ Sample Response ]

In the lecture, the professor explains that the early Earth was

not a place where life could develop, and that a number of

events had to happen to make Earth more suitable for life.

According to the professor, the earth was repeatedly hit by

comets in its early history. Since comets are made mostly of

ice and dust, these impacts brought water, which is essential

for all life, to the earth. The professor also points out that the

dust in the comets contained chemicals that are important to

life. Another event that was important for the development of

life was the formation of the moon. The moon’s gravity

created tides, which mixed up the chemicals in the ocean

and allowed them to combine to create the first life forms. So

basically, comet impacts brought the materials needed to

create life, and the formation of the moon created the tides

which allowed those chemicals to combine and form life.

3.

[ Sample Response ]

In the lecture, the professor explains that cancer is a huge

problem and that early detection of cancer is very important.

The professor points out, however, that the nature of

different kinds of cancer influences how screening is done.

To illustrate this, she uses two examples: cervical cancer

and prostate cancer. Cervical cancer is caused by a virus

called HPV. The professor says that since any sexually

active woman can get the virus, there is no way to tell which

women are at greatest risk. As a result, all adult women are

encouraged to get regular screening for cervical cancer.

Prostate cancer, on the other hand, is caused by factors that

can be identified, namely genetics and obesity. Therefore,

men with a history of prostate cancer in their family or

overweight men are at greater risk, and doctors focus their

screening efforts on men in these groups.

With other cancers, however, it is possible to identify

groups who are at high risk of developing that particular

form of cancer. Prostate cancer, for example, is a cancer

that is common in men. It can be caused by a number of

factors, the most important of which are genetics and

obesity. Men who are severely overweight or have a

history of prostate cancer in their family are at

significantly greater risk of developing prostate cancer

themselves. Consequently, doctors focus their screening

efforts on men who fall into these groups. Men who are

not overweight and do not have a family history of

prostate cancer are not as strongly encouraged to get

screened.

[ f Script ]

Professor (female): One of the most significant threats tohuman health is cancer. Every year, millions of dollars arespent researching new cures for cancer, and billions arespent treating those with cancer. Early detection of cancerplays a significant role in a cancer patient’s chances forsurvival, so doctors often stress cancer screening forseemingly healthy individuals. The natures of differenttypes of cancer, however, determine how that screening isconducted.

Cervical cancer is the abnormal growth of cells that makeup the cervix, or a section of the female reproductivesystem. The most frequent cause of cervical cancer isinfection by HPV, which is a sexually transmitted virus thatcomes in many strains. It is estimated that certain strainsof HPV cause nearly 100 percent of cervical cancers. Thismeans that any sexually active woman is potentially at riskof developing cervical cancer. It is basically impossible toidentify which women are at greater risk. Therefore,doctors encourage all adult women to be screened forcervical cancer on a regular basis.

A110 Insider: The Super Guide

Cancer a huge problem – much money spent on research

and treatment.

Early detection important docs. screen patients.

Different types of cancer = different screening

Cervical cancer occurs in women

Almost all cases caused by HPV virus

not poss. to identify at risk group all adult women

screened

prostate cancer occurs in men

caused by genetics and obesity men w/ history of

pros. cancer in fam. or overweight at higher risk.

Docs. focus screening on at-risk groups

Early earth not good place for life but several events

changed environment

Comet impacts:

Comets made of ice impacts brought water to earth

Dust in comets had chem. important to life

Formation of moon:

Moon’s gravity creates tides

Tides stir chemicals, chem. combine to form life

Tides also wash nutrients into water so help survival

Page 111: Insider: The Super Guide Scripts & Answer Key

4.

[ Sample Response ]

The lecture explains how plate tectonics can be used to

explain why earthquakes and volcanoes occur. According to

the professor, the earth’s crust is made of plates that move

very slowly. In some instances, those plates try to slide

against each other but are held in place by friction.

Eventually, the pressure will be greater than the force of

friction, and the plates will jerk forward. In doing so, they will

release the energy that causes an earthquake. In other

cases, two plates collide and one plate is forced underneath

the other one, where it begins to melt. Pressure forces the

melted rock up through cracks in the upper plate, and when

it reaches the surface, it forms a volcano.

[ f Script ]

Professor (male): OK, today we are going to talk about

the theory of plate tectonics and how that theory can be

used to explain some natural disasters. According to this

theory, the earth’s crust is divided into large plates, all of

which float upon the earth’s liquid mantle. Now, since

these plates float, they also move, although very slowly,

and it is this movement that helps to explain a number of

natural disasters, most notably earthquakes and

volcanoes.

In some places where two plates meet, the two plates

move in such a way that they slide against each other. At

first, the friction between the two plates will keep them

from moving. But over time, the pressure between the

plates will build and build. When the pressure becomes

greater than the force of friction, the plates will move in a

sudden jerk. The resulting release of energy causes the

massive vibrations on the earth’s surface that we refer to

as earthquakes.

Volcanoes are also created by the movement of the

earth’s plates. In some places, two plates are pushed

together, and as this happens, the edge of one plate is

forced beneath the other one. As the edge of this plate is

forced into the liquid magma, it melts. As pressure builds,

this magma is forced upwards through the cracks in the

earth’s crust. When it reaches the surface, it is released

in a volcanic eruption.

Scripts & Answer Key A111

SP

EA

KIN

G

Plate tectonics: earth’s crust divided into plates that float

on liquid mantle

Plate tec. helps explain earthquakes + volcanoes

Earthqks:

Plates try to slide but friction holds them

Pressure builds, when press. more than friction, plates

jerk + release energy

Energy causes earthqks.

Volcanoes

2 plates collide + one plate pushed under into mantle

Plate melts, pressure forces magma up through cracks in

crust

When magma reaches surface = volcano

Page 112: Insider: The Super Guide Scripts & Answer Key

,

A112 Insider: The Super Guide

P A R T AThe Basics of SentenceConstruction

Phrases

Exercise 1 _ p.471

1. The leopard’s spots help it to hide from its prey.

2. When the right foot of the player touched the foul line,

the referee blew the whistle.

3. Average winter temperatures in the Arctic Circle

drop below zero.

4. The vast majority of start-up companies fail in their first

three years.

Exercise 2 _ p.471

1. Maintaining a high grade point average is essential to

any student hoping to go college.

2. Crying and upset, the witness admitted she had lied to

protect her boyfriend.

3. Ned came home to find his door forced open and his

belongings scattered on the floor.

4. Planning the trip scheduled for next week is Laura’s

responsibility.

Exercise 3 _ p.471

1. The Tigers the only team to go undefeated are the

favorites to win the championship.

2. The film about King Kong a giant ape who destroys the

city was entertaining.

3. The dancer a girl of both beauty and grace was the star

of the show.

4. The climbers decided to attempt their ascent of the

mountain during the spring traditionally the safest time of

year in terms of weather.

Clauses

Exercise 1 _ p.473

1. The city’s air pollution is the result of the high number of

cars on the road and of its outdated factories but little is

being done to solve the problem.

2. Although the modifications to the ship’s design were

thought to be minor they had a significant impact on its

performance.

3. Most wild animals are afraid of humans but they can still

be dangerous when they are threatened.

4. While Mark usually displays excellent sportsmanship he

was thrown out of the last game because of his behavior.

Exercise 2 _ p.473

1. Last semester our science teacher taught us how

batteries work.

2. That he had committed the crime was understood to be a

proven fact.

3. Although Nick is actually from New York, Miami is where

his heart is.

4. When Jessica found what Mark had hidden under her

book she screamed at the top of her lungs.

Exercise 3 _ p.473

1. Most professional sports are seasonal in nature because

athletes need time to rest and recover between seasons.

2. The waves in Hawaii are better than in most of California,

so Hawaii is the surfing capital of the US.

3. After CD players were released for mass marketing in

the late 1980s, they quickly replaced tape players.

Sentence Structures

Exercise 1 _ p.475

1. simple sentence 2. compound sentence

3. complex sentence 4. compound-complex sentence

Exercise 2 _ p.475

1. The team realized it was thirteen points back with only

minutes to go in the final quarter, and it began to lose

any hope of victory.

2. The team began to lose any hope of victory when it

realized that it was thirteen points back with only minutes

to go in the final quarter.

3. Realizing it was thirteen points back with only minutes to

go in the final quarter, the team began to lose any hope

of victory.

Chapter 1

WRITING Section

, ,

,

,

,

,

,

,

,

,

Page 113: Insider: The Super Guide Scripts & Answer Key

Scripts & Answer Key A113

WR

ITIN

G

Subject / Verb Agreement

Exercise 1 _ p.477

1. Every one of the houses in the neighborhood

: singular

2. The gold worth millions of dollars

: singular

3. Nearly all of the findings taken from the study

: plural

4. Broad consensus among the various members

: singular

Exercise 2 _ p.477

1. The biggest problem facing many new companies are

the dozens of regulations that are often unfamiliar to the

company management.

2. Most of the talk, including the portions on political reform,

financial stability, and future economic growth, are

available on tape for anyone who is interested.

3. Seemingly minor breakdowns in social etiquette, the

failure to apologize after bumping into someone for

example, is Michael’s biggest complaint about life in the

city.

4. In the suspect’s house, underneath his mattress, was

found the murder weapon and the stolen property, which

was enough to convict him of the crime.

5. Not only the investors, but also the company itself are

expected to suffer severe financial losses after the world

of financial analysts learns of its improprieties.

Pronoun Usage

Exercise 1 _ p.479

1. If you talk to people who know Michael and Kathy, theywill tell you that the couple is deeply in love.

2. Despite the dire predictions for climate change in the

near future, it is the next generation which is the onethat will experience the full effects.

3. Compiling lists of all his business expenses is a long and

laborious process, and Henry hates doing it. 4. The Venus flytrap is one of those rare types of organisms

that are able to nourish themselves using chemicals

taken from both the soil and the bodies of other

organisms.

Exercise 2 _ p.479

1. The nation’s failure to deal with increasing poverty, which

led to a huge revolt, was reported widely in the press.

2. Tensions arose between the Romans and the Hebrews,

who felt the Romans didn’t respect the Hebrew god.

Verb Tense Consistency

Exercise 1 _ p.481

1. In recent years, there have been claims from a number

of influential people who say that television is not good

for young children. However, there are a several reasons

why television is not as damaging to children as some of

these people claim. First, television can be a source of

education for children, as a large number of children’s

shows are highly informative and instructional in nature.

These programs teach children important values, such

as respect for their parents.

2. One experience that I would like to forget is my first

football game in high school. During that first game, it

seemed like I did everything wrong, even though I

practiced every day for months before the start of the

season. In the first half, I made a long pass that I thought

would result in a touchdown, but the other team

intercepted it. From there, my problems continued. I

dropped the ball several times and I was tackled twice.

When the coach took me out of the in the third quarter I

was so embarrassed because I had done everything

wrong.

Parallelism

Exercise 1 _ p.483

1. Freedom of speech and the ability to vote in elections

are the rights of every citizen.

2. The workers had already completed the necessary

adjustments, checked all components twice, and gone

home for the night.

3. The very definition of a Renaissance man, Leonardo da

Vinci was a painter, a sketch artist, and an inventor.

4. The mayor suggested several new initiatives:

constructing a new art complex with a large outdoor

theater, increasing funding to promote local artists, and

hosting an annual art festival.

Exercise 2 _ p.483

1. The company manager set out three goals for the year:

to increase sales, to decrease accidents on the factory

floor, and to increase productivity by streamlining the

construction process.

2. Public intimidation, strict regulation of the media, and

imprisonment of political enemies with little or no judicial

process are all characteristics of totalitarian

governments.

is

is

is

were

are

were

Page 114: Insider: The Super Guide Scripts & Answer Key

A114 Insider: The Super Guide

Placement of Modifying Words, Phrasesand Clauses

Exercise 1 _ p.485

1. Our mother frequently told us to wash our hands before

we ate dinner.

2. As the smartest student, Melvin was always pressured

to let other students copy his homework.

3. Mr. Bruner allowed the boys to play baseball in his yard

as an act of kindness, even though they were loud and

noisy.

4. Many scientific discoveries have been made quite byaccident while investigating a seemingly unrelated

phenomenon.

Exercise 2 _ p.485

1. Seeing that it was inadequately prepared, the college

delayed its freshmen orientation activities.

2. The politician incorrectly thought that, by a series of false

promises, he would be able to gain the support of voters

in the next election.

3. The applicant made sure on his resume to note where he

had previously worked.

Sentence Fragments and Run-onSentences

Exercise 1 _ p.487

1. It is my belief that people rely on their cars too much.

Most people use their cars even for the short trips where

they could just walk instead. A good example is when

they are going to a store that is just around the corner or

things like that. In such situations it is not necessary to

drive, but most people drive anyway. Even people who

are unwilling to walk even these short distances could

drive less if they would just plan ahead and try to run all

of their daily errands at one time instead of making

numerous short trips.

2. Air quality in many parts of the world has grown

drastically worse in the last three decades because of a

greater reliance on coal to generate power. Coal, which

is full of impurities that make their way into the air, is

utilized by many developing countries for power

generation due to the simplicity of the technology. Coal-

fired plants are easier to build than plants that run on

natural gas or oil because it is easier to safely store coal

than to store either natural gas or oil. Furthermore, coal-

fired plants are obviously simpler and safer than nuclear

power plants. However, they also cause far more air

pollution than other types of power plants.

| Review | _ p.488

1. Not a single one of the rules, regulations, or restrictions

my school imposes on its students makes a bit of sense

to me.

2. Due to numerous failures to enforce company policy and

his misuse of the company credit card he was fired last

week.

3. Like the Egyptian Empire and the Greek city-states

before it, the Roman Empire depended heavily on the

use of slave labor.

4. The city council, for fear of angering voters,

abandoned its plan to increase taxes and resigned itself

to another year of belt tightening.

5. If John would just stop complaining about it and do his

chores, we could go to the zoo this afternoon.

6. During the course of the semester, the medical students

learned how to identify the symptoms of an illness, to

assess the severity of the illness, and to decide what

drugs to prescribe.

7. All indications, including anecdotal evidence taken from

numerous patients, suggest the treatment is both

effective and safe.

Page 115: Insider: The Super Guide Scripts & Answer Key

Scripts & Answer Key A115

WR

ITIN

G

Paragraph Structure

The Basic Elements of a Paragraph:Main Ideas

Exercise 1 _ p.491

1. A few years back, I took an extended vacation to the

island of Bali. Of all the vacations I have been on inmy life my trip to Bali was the most memorable. One

of the things that struck me the most was the way that

much of the traditional culture of the island remained

intact, especially in the city of Ubud located in center of

the island. There, many families still live in the traditional-

style compounds, complete with a personal temple for

the family. The other thing that sticks in my memory

about that trip is that it cemented my relationship with

one of my closest friends. My friend and I were good

friends before the trip, but during our time on Bali, we

really bonded and became much closer friends.

2. Indirect main idea: The English Pilgrims were poorly

prepared for life in America and could not have survived

without the help of the Native Americans.

Exercise 2 _ p.492

1. A child’s earliest years are the most important to

development.

2. An effective advertising strategy is essential to the

success of a company.

3. Photons are extremely useful in astronomical

observation.

Supporting Ideas

Exercise 1 _ p.494

I do believe that advertising can tell you a great deal about a

culture. If you look at a lot of American advertisements, you’ll

see that many of them feature people who are young,

attractive, and successful, and seem to imply that if you buy

these products, you will be too. The stress that American

commercials place on these qualities clearly shows that youth,

success, and attractiveness are highly valued in American

culture. In my experience, the same is true in other countries;

their commercials highlight the qualities that are valued in

their respective cultures. Therefore, by paying attention to

the qualities stressed in advertisements, you can begin to

learn about the underlying values of a particular culture.

Exercise 2 _ p.494

1. I agree that it is important to be honest in all situations. A

person’s level of honesty is one the basic characteristics

that he or she will be judged by. Since it is important to

have the good opinion of others, it is essential to be

honest. Honesty is especially important between friends

and family members because trust is the foundation of

these relationships. It is trust that allows us to be

completely open with our family and friends, and it is this

openness that makes these relationships so special.

2. I believe that it is very important for a city to try to

preserve its old buildings and monuments. These

structures are the only connections to the city’s past.

They provide citizens with a visible reminder of their

history. In addition, a city’s old monuments and buildings

can be valuable for the tourism industry. Take

Westminster Abbey in London, for example. Every year,

thousands of tourists visit London specifically to see this

historic site. Finally, the presence of well-maintained

monuments and historic buildings adds beauty to a city.

This is important because it gives people a sense of

pride in their city.

Exercise 3 _ p.495

I think it is better for university students to have a job while

they are in school rather than to just focus on their studies. I

feel this way for a number of reasons. First, students who

work while they are in college learn to handle multiple

responsibilities because they have to worry about both their

grades and their job. As adults, they will have to do the

same thing; they will need to balance their responsibilities to

their families and their responsibilities at work. In addition,

students who work during college can gain valuable

experience that will help them after they graduate. Many

working students have jobs that relate directly to their fields

of study, and thus they learn more than they could in the

classroom alone. This experience gives them a valuable

advantage when they look for a job after graduation. Finally,

working students don’t need to worry about paying their

tuitions and are less likely to have large student loans when

they graduate. This means they will have more money for

other things, like starting a family, or buying a house.

Chapter 2

Page 116: Insider: The Super Guide Scripts & Answer Key

A116 Insider: The Super Guide

Organization Within A Paragraph

Exercise 1 _ p.497

Sentence Order: 3, 1, 4, 6, 8, 2, 5, 7

Exercise 2 _ p.497

Lions display a level of sociality that is unique among feline

species. Lion societies are known as prides. These pridesare organized into a strict hierarchy. Male lions have the

highest ranking in the pride, and among the male lions one

will be the dominant male. Female lions and cubs, incontrast, are at the bottom of the social ladder. Lion prides

also display high levels of cooperation. Male lions, forexample, work together to defend the territory of the pride. If

one male is attacked, the other males will come to its

rescue.

Paragraph Structures

Exercise 1 _ p.500

1. (B) 2. (A)

Exercise 2 _ p.501

Before I started teaching several years ago, I worked in an

insurance office, and like most people, I viewed my job as

simply a way to pay the bills. I didn’t hate working at the

insurance office; it just didn’t seem very interesting or

important to me. After a few years, I was sick of working in

the office and decided to try teaching. Being a teacher

changed the way I thought about my work. I found it really

interesting, and, since I was working with kids, I felt like I

was really making a difference in society. I was making less

money as a teacher, but that didn’t matter to me because,

for the first time, I felt like I was doing something important.

I’ve been teaching for five years now, and it has become an

important part of who I am as a person. What I learned from

this experience is that if you choose the right job, work can

be more than just something you do for money: it can be

something that adds meaning and purpose to your life.

Exercise 3 _ p.504

1. (B) 2. (A)

Exercise 4 _ p.505

I prefer to start work early and finish early, rather than go to

work late and leave late. First of all, if I am able to leave

work earlier it gives me more time to go out after work. I can

go out to see a movie, or have a drink with my friends.

Basically, it gives me more social opportunities. If I started

work later, however, I would not finish until much later at

night. Consequently, my social life would be much more

limited. I wouldn’t have time to go out after work, and I would

only be able to see my friends on the weekends.

Exercise 5 _ p.506

Everyone knows that parents serve as the primary role

models for children. Children usually develop the same traits

they see in their parents. This is why I believe the most

important quality for a parent to have is responsibility. A child

with irresponsible parents will grow up to be an irresponsible

adult. As an adult, he will be unable to teach responsibility to

his own children, because he did not learn the quality from

his own parents. This creates a cycle of irresponsibility in a

family that is very hard to stop.

Page 117: Insider: The Super Guide Scripts & Answer Key

Scripts & Answer Key A117

WR

ITIN

G

Essay Structure, Editing &Revision

Introductions

Exercise 1 _ p.510

1. The quality of a person’s boss is very important to how a

person views his or her work; people who have bad

bosses are far more likely to dislike their jobs.

Consequently, the most important qualities for a boss to

have are a clear set of expectations and a willingness to

listen to his or her employees.

2. With the help of modern medicine it has become possible

to delay death for increasingly longer periods. While the

fear of death is understandable, our increasingly long life

spans are beginning to cause problems for society.

Exercise 2 _ p.511

Although many people claim that television has negative

effects on young children, it can also be a powerful

educational tool. Television can not only teach young

children important life lessons, but it can also improve their

language skills.

Body Paragraphs

Exercise 1 _ p.513

If someone were to propose the construction of a new

shopping center in my community, I would definitely oppose

it. Although a large shopping center would certainly lead to

the creation of jobs for relatively unskilled workers, it would

also drive small stores out of business and create further

traffic problems in the community.

First of all, many members of my community are self-

employed and own their own small shops. My uncle, for

example, runs a small bakery, and has done so for the last

thirty years. Small business owners such as my uncle would

not be able to withstand the intense competition that would

be created if a large shopping center were constructed. They

would not be able to compete with the lower prices that

much larger business chains are able to offer and would

most likely be driven out of business. So, while the

construction of a new shopping center would create jobs for

some, it would also ruin the livelihoods of many others.

Furthermore, since most of the new jobs created would be

lower-paying jobs, such as cashier and sales positions, the

overall effect on the local economy would be largely

negative.

The construction of a new shopping mall would also further

add to traffic congestion, which is already a significant

problem in the community. A new shopping center would

centralize the workforce and commerce within the

community. Essentially, most community members, whether

they were going to work or simply going shopping, would be

going to the same place. Clearly, this would have a

disastrous effect on traffic.

Exercise 2 _ p.514

1. One reason it is incorrect to define success by how much

money one earns is that wealth does not always bring

happiness. In my opinion, many wealthy people are too

busy earning money to truly enjoy themselves. For

example, my uncle owns a chain of auto repair shops,

and he has made a lot of money. However, he hasn’t

taken a vacation in more than five years because he is

always working. Similarly, he loves to golf, but he rarely

goes because he spends most weekends in his office.

2. Another reason that success should not be judged solely

on the basis of wealth is that this kind of narrow,

materialistic view is bad for society. An individual’s

contributions to society should be an essential part of

any definition of success. A person who donates his or

her time by volunteering may not earn as much money,

but that person is actively working to improve his or her

community. By contrast, people who spend all of their

time trying to earn more money do not really contribute to

their community in a meaningful way.

Conclusions

Exercise 1 _ p.516

Conclusion B

Exercise 2 _ p.517

In conclusion, children are better off growing up in urban

rather than rural areas because they gain a more

multicultural perspective and get a better education.

Exercise 3 _ p.518

In conclusion, while it is natural to become attached to one’s

pet, pets are not family members. Treating them as such is

both unreasonable and unfair to them.

Chapter 3

Page 118: Insider: The Super Guide Scripts & Answer Key

A118 Insider: The Super Guide

Editing and Revision

Exercise 3 _ p.522

If I were to live in a dormitory, I would prefer the university to

choose my roommate for me rather than make the choice

myself. This seems like it would be the best way, and it

would give me a better chance to change roommates if

things did not work out.

First of all, I don’t see how choosing my roommate would

benefit me in any way. I would have to choose a stranger to

live with anyway because I would be new to the university

and wouldn’t know anyone. So it would not matter whether it

was the school who decided or if I did. Either way, I wouldn’t

know anything about my roommate. If the result would be

the same either way, why would I want to bother making the

decision?

In addition, allowing the university to select a roommate for

me could actually be a good thing. If it was the school’s

choice and it did not work out, I could argue that the

university had the obligation to find a new roommate for me,

since it made the decision. However, if I made the choice, it

would be difficult to argue that the university had the

responsibility to find me a different roommate. They could

simply say that it had been my choice and that I had to live

with the consequences.

In conclusion, it would be better if the university chose my

roommate for me because the decision would be the same,

and it would give me a better chance to change my

roommate.

P A R T BIndependent Writing Task

Pre-Writing: Guided Practice

Exercise 1 _ p.530(Sample)

Exercise 2 _ p.531(Sample)

1. When I was about 12 years old, I had a pen pal in

England to whom I wrote a letter every month. I very

much enjoyed it.

2. I enjoyed the anticipation of waiting for new letters to

come in. It definitely made me see some of the virtues in

letter writing, and I still write letters now and then.

3. I think it really is the best way to communicate over long

distances. It has its downsides, like waiting, but in the

end it’s completely worthwhile.

Exercise 3 _ p.531(Sample)

Pros of letters

• More personal than other

ways of communicating

• More formal than a simple

email

• Letter writing is a time

honored tradition

Cons of letters

• Postal service too slow,

not efficient enough

• Better to talk to people

face to face than in writing

• Requires too much work:

writing, mailing, waiting

Chapter 1

• MMaaiinn IIddeeaa:: I agree with the statement that writing

letters is the best way to communicate over long

distances

- Supporting Detail 1: First, I enjoy the

anticipation of waiting for a new letter to arrive

at my house

- Supporting Detail 2: Second, I think that

writing or reading a letter is a very personal

experience that can’t be matched by email or

telephone

- Supporting Detail 3: Third, letter writing is a

distinguished and time honored tradition and I

enjoy keeping it alive in some small way

Page 119: Insider: The Super Guide Scripts & Answer Key

Scripts & Answer Key A119

WR

ITIN

G

Writing an Introduction: Skill Building

Exercise _ p.534~535(Sample)

1. I prefer to relax when I am on vacation. I prefer this

because it helps me to forget about the stresses of

everyday life, gives me an opportunity to rest and regain

my energy, and allows me time for leisure activities such

as reading.

2. I think that teenagers should not be allowed to buy

alcohol. They are not willing to drink responsibly, they

are more likely to drink and drive, and they are at risk of

putting themselves in danger.

3. I disagree with the idea that people put too much value in

their appearance. It is important to have a good

appearance because it helps you feel good about

yourself, helps you look more professional at work, and

makes you appear more approachable to others.

4. I believe that space exploration is relatively unimportant.

I believe this for three main reasons: scientists can

gather a lot of data about space from earth; other planets

and solar systems are too far away to travel to; and

money spent by the government on space exploration

would be better spent on other things.

Writing an Introduction: Guided Practice

Exercise 1 _ p.536(Sample)

Exercise 2 _ p.536(Sample)

1. I once went to a restaurant that was very crowded, and

the only seats available were in the section that allowed

smoking. I had a negative experience because of all the

smoke burning my eyes and making me cough. I could

not enjoy my dinner.

2. It made me think that smokers should have the courtesy

to go outside and smoke. I don’t want people to be left

out, but if you cannot be courteous to others, then you

should be made to leave.

3. I think that smoking should be banned in all public places.

It is a disturbance as well as a public health issue.

Exercise 3 _ p.537(Sample)

Exercise 4 _ p.537(Sample)

I believe that smoking should be banned in all public places,

including restaurants and bars. First of all, second hand

smoke can be hazardous to other people’s health. Second, I

think that most people who do not smoke would find the

cigarette smoke to be quite irritating. Finally, there are those

people who really are allergic to smoke, as it can be a very

potent allergen.

Body Paragraphs: Skill Building

Exercise 1 _ p.539

1. (C)

2. (D)

3. (C)

4. (B)

Pros

• Those who do smoke

would not be forced to go

outside

• Smokers might learn to

be more courteous

toward non-smokers

• Those who smoke would

be able to do so

whenever they wanted

Cons

• Secondhand smoke can

be dangerous for others

• Some people are allergic

to cigarette smoke

• Those who do not smoke

would find the smoke very

irritating

• MMaaiinn IIddeeaa:: I think that smoking should be banned

in all public places.

- Supporting Detail 1: Secondhand smoke can be

dangerous for other people.

- Supporting Detail 2: Some people are quite

allergic to cigarette smoke.

- Supporting Detail 3: Those who do not smoke

would find the smoke very irritating.

Page 120: Insider: The Super Guide Scripts & Answer Key

A120 Insider: The Super Guide

Exercise 2 _ p.540

* The author starts to digress when he talks about his

Spanish classes, but it still seems possible that he might

eventually connect this example to the main idea. Just

when it is time to relate this first example back to the main

idea, however, he begins to talk about his French classes.

It is very clear from this point on that he has gone off topic

completely.

Exercise 3 _ p.540(Sample)

1. Elderly people should not drive because they pose a

danger to pedestrians. Elderly drivers are bound to have

diminished eyesight or blurred vision. When a driver has

especially bad vision, he or she becomes a hazard to

other drivers and also to pedestrians.

2. Students who are required to wear uniforms to school

face less peer pressure and teasing than they might

otherwise. Clothing is a marker of style and popularity

among teenagers, so the pressure to dress well in order

to be accepted can be intense. Dictating what a

teenaged student will wear to school every day might

decrease some of the pressure.

3. I disagree with the statement that people are essentially

good because I fail to see it in the real world. For

example, if my car were to break down on the freeway,

far more people would drive right past me than stop and

help. Therefore, I don’t see how most people could be

essentially good when most people refuse to help others

in need.

Body Paragraphs: Guided Practice

Exercise 1 _ p.541(Sample)

1. I went to college for a while in the city. The dormitories

were many blocks from campus, so I had to ride the

elevated train everyday to school. I depended on it being

on time every day, and it was.

2. A friend was forced to ride the bus to work for a while

because his car needed repairs. The bus system is very

complicated in the city, but his bus was always within five

minutes of being on time.

3. In cities like New York, many people rely completely on

public transportation to get them to work on time. If it was

not successful in doing this, I hardly think that so many

people would put so much trust in public transportation.

Exercise 2 _ p.541(Sample)

Public transportation is better than driving a car because the

public transportation system is generally more reliable in the

long run than a car is. When I attended college in a major

city, I relied on public transportation to get to class. It was

always on time and never let me down, unlike my car, which

needed repairs constantly. In major cities like New York,

millions of people rely on public transportation to get them to

work on time, and many people do not even own cars. I

doubt that they would have so much trust in public

transportation if it were not very reliable.

Conclusions: Skill Building

Exercise 1 _ p.543

1. (A)

2. (B)

3. (A)

Exercise 2 _ p.544(Sample)

1. Just as human beings have a right to health and

happiness, so do animals.

2. I believe that endangered species should be protected in

part by the government.

3. Personally, on the weekends I prefer reading or watching

a movie rather than going out to clubs and partying.

4. Popular culture icons like sports players, musicians, or

movie stars are among the most overpaid people.

I agree with the statement that all American school

children should be made to learn a second language. I

think that learning a second language gives us a new

way of seeing the world. It fosters understanding

between cultures. America is a melting pot of cultures,

and we must all be able to coexist with one another.

Language is a step toward this. When I was in high

school I took two Spanish classes. It was very difficult

trying to learn a foreign language, even one that is as

similar to English as Spanish is. But I worked very hard,

and at the end of the second Spanish class I felt that I

had learned a great deal. When I went to college, I

decided to take French classes because I so wanted to

travel to Paris. This proved to be yet another challenge,

but, with a lot of hard work, I managed to pass all of my

classes. The summer after that, I finally got a chance to

go to Paris and I had the time of my life.*

Page 121: Insider: The Super Guide Scripts & Answer Key

Scripts & Answer Key A121

WR

ITIN

G

Exercise 3 _ p.545(Sample)

2. Three ways in which people commonly travel are by

plane, by train, and sometimes by boat.

3. Meteors, climate change, and famine have all been

theorized as causing the extinction of the dinosaurs.

4. Using foreign oil is a complicated practice because oil is

a limited resource, it causes political conflict in various

world regions, and it is more expensive than domestic oil.

Conclusions: Guided Practice

Exercise 1 _ p.546(Sample)

1. In conclusion, I am of the opinion that hunting and killing

animals is not the best solution to the problem of animal

overpopulation.

2. As an animal living in close proximity to humans, deer

are an especially adaptable animal, and I think it would

be more humane to let them find their own way rather

than give hunters a license to shoot them down.

Exercise 2 _ p.547

1. In conclusion, I much prefer swimming in the ocean to

lying in the sun when I go to the beach.

2. Compared to just lying in the sun, swimming in the ocean

is more fun, it offers better protection against sunburn,

and makes for great exercise.

Editing and Revising: Skill Building

Exercise 1 _ p.549(Sample)

1. I believe that animals deserve the same right to health

and happiness as human beings. While an animal may

not be totally self-aware, it does feel some basic

feelings. A domesticated animal that is abandoned feelslonely and confused for sure. This animal is accustomed

to being fed regularly and being provided for completely,and in the wild it will not survive.

2. I agree with the statement that celebrity culture has a

negative influence on children. My young niece wants to

be just like Paris Hilton. Many children look up to

celebrities as role models. They see the fame and thewealth that goes along with being a celebrity, butwhat they do not understand is that beyond the fameand fortune celebrities are flawed human beings likeeveryone else.

Editing and Revising: Guided Practice

Exercise _ p.550

Incorrect:

I believe that pet ownership improves a person’s quality

of life. Pets give companionship[awkward phrasing] and

is[wrong verb tense] good for one’s mental and physical

health. Additionally, being responsible for the life of

another being teaches a wonderful lesson in responsibility.

First, owning a pet improves mental health. A pet canmake us feel not lonely[awkward phrasing]. Especiallyfor depressing person[sentence fragment, incorrect

adjective form]. For example there is a thing called “pet

therapy” and many psychologists use it to help the

mentally ill.[more support needed] It shows that pets can

help a person improve their[incorrect pronoun] mental

health.

Second, pet ownership could[inappropriate verb tense]

improve the[unnecessary article] physical health. Having

to walk a dog every day offers the perfect opportunity for

a little extra exercise. In addition to mental disorders,pets have been used by medical doctors for elderlypeople or those people with serious illnesses toimprove their quality of life[very awkward word order

and sentence construction]. A pet provides an additional

reason to live, and this type of pet therapy has been

shown to increase the expected lifespan of patients.

Finally, owning pet[article needed] teaches us all a good

lesson in responsibility. If a child has a pet when theyare young, then the child will be[article missing] goodparent later. Because having[article] pet is likehaving[article] baby.[not enough support in general] It’s

not easy and it takes a lot of work and care.

In conclusion, pet ownership does in fact improve one’s

quality of life. A nice, affectionate pet can make us feel

better about ourselves and the world we live in. In

special cases, it can even give us a reason to live.

• MMaaiinn IIddeeaa:: Whenever I go to the beach, I prefer to

swim and play in the ocean rather than lie in the sun

and tan.

I. FFiirrsstt SSuuppppoorrttiinngg DDeettaaiill: There is much more fun

to be had swimming in the ocean than simply

lying in the sun.

II. SSeeccoonndd SSuuppppoorrttiinngg DDeettaaiill: Also, being in and

out of the cool water helps protect you from the

sun’s potentially harmful rays.

III.TThhiirrdd SSuuppppoorrttiinngg DDeettaaiill: Finally, swimming

and playing in the ocean is an excellent source of

exercise.

Page 122: Insider: The Super Guide Scripts & Answer Key

A122 Insider: The Super Guide

(Sample)

Additional Practice _ p.552~553

(Sample)

1. I prefer to work in a quiet office setting. While it is not

quite as exciting as working outdoors, working in an

office provides a quiet atmosphere in which to work, a

closer set of coworkers and friends, and an important

sense of purpose.

There are many advantages to having a quiet place in

which to work. For one, I think that office workers are

more productive than those who work outdoors. There

are fewer distractions than there are in an outdoor

setting. A calm environment with few distractions can

lead to greater output and better quality of work.

Working in an office, one is able to make friends with his

or her coworkers more easily than working in an outdoor

setting. In an office, many people with different

personalities work very close to one another, often in

small cubicles or open offices where desks are side by

side. This closeness makes it easier to make friends with

those working around you. A group of workers that have

this kind of relationship generally share a strong team

attitude and can work well together to accomplish a goal.

Finally, it is especially important at any job to have a

sense of purpose. If one does not feel like he or she has

a special purpose in the workplace, he or she will not feel

very motivated to work. In an office, where people work

in close proximity and are on generally friendly terms, it

is easy to feel a sense of purpose at work. As part of a

team, an office worker will have specific duties assigned

only to him or her. This sense of purpose, then, comes

from being an important part of a team.

In conclusion, I think that it is better to work in an office

setting than to work outdoors. Three important

advantages to working in an office are the quiet

atmosphere, the opportunity to make close friends, and

an important sense of purpose. A close team working in

a stable environment will be more productive and

probably more satisfied as workers.

2. I disagree with the statement that teenagers should face

serious consequences for illegally downloading music

from the Internet. First of all, most people who get music

from the Internet are simply sharing files with other music

fans. Second, file sharing is not the same thing as

illegally downloading copyrighted music. Finally, I think

that people have the right to come together and share

their favorite music.

First of all, the illegal part of downloading music has to

do with buying music from a website that does not pay a

portion of the profits to the musicians themselves. Selling

someone else’s copyrighted music is definitely a crime.

However, most if not all of these websites have been

closed down by the government. Now there are ways to

buy music on the Internet that are essentially the same

as the old ways, but are perfectly legal because money is

paid to the artists. Most people now use these services.

Still, some people do not buy music on the Internet, but

get it for free through file sharing programs. This practice

is what the government claims is illegal. However, it is

not the same as earlier services that violated copyright

laws. These programs allow music lovers to come

together and trade their favorite songs with one another.

No purchases take place, no one makes any money, and

everyone participates equally in the sharing. It is just like

trading cassette tapes with a friend, and no one has ever

Correct:

I believe that pet ownership improves a person’s quality

of life. Pets can be great companions, and they can be

good for one’s mental and physical health. Additionally,

being responsible for the life of another being teaches a

good lesson in responsibility.

First, owning a pet improves mental health. A pet can

help a person not feel lonely. This is especially good for

a depressed person. For example, there is a thing called

“pet therapy” which many psychologists use to help the

mentally ill. Petting a cat or playing with a puppy can

improve someone’s mood and calm them down. In this

way pets can help a person improve his or her mental

health.

Second, pet ownership can improve physical health.

Having to walk a dog every day offers the perfect

opportunity for a little extra exercise. In addition to

psychologists, medical doctors also use pets to help

their patients. Doctors use pets to improve the mood of

very ill patients. They can also be used to improve the

quality of life of elderly patients. Having a pet provides

an additional reason to live. This type of pet therapy

prolongs the lives of certain patients.

Finally, owning a pet can teach us all a good lesson in

responsibility. A child that is taught to be responsible for

his or her pet will grow up with a good sense of

responsibility and perhaps even become a good parent

one day. One must always think of the pet’s needs in

order to care for it well, which is also very true of

children.

In conclusion, pet ownership does in fact improve one’s

quality of life. A nice, affectionate pet can make us feel

better about ourselves and our world. Sometimes, it can

even give us a reason to live.

Page 123: Insider: The Super Guide Scripts & Answer Key

Scripts & Answer Key A123

WR

ITIN

G

been arrested for doing that.

People have the right to get together on the Internet and

share their interests with one another. These people who

share files are genuine music lovers who only want to

spread the word about their new favorite bands, and I

find nothing illegal about that. In addition, file sharing

does not hurt the musicians or the record companies. It

is actually an excellent source of word-of-mouth

marketing.

In conclusion, I do not think that teens, or anyone, should

be held legally responsible for “illegally downloading”

music. This term “illegal downloading” really refers to

friends sharing music over the Internet, something that is

harmless and should not in any way be considered a

crime.

Integrated Writing Task

(Sample Integrated Writing Task _ p.557

[ f Script ]

Professor (female): When most Americans hear the

name Benedict Arnold, the first word that comes to mind

is “traitor.” Yet, while Arnold certainly was guilty of

treason, his motives were not as straightforward as many

believe. In fact, few of us really know much about Arnold

or the real reasons behind his decision to betray the

American struggle for independence.

First of all, Arnold did not turn traitor simply because he

hated fellow officers and grew to despise the American

struggle for nationhood. There is much evidence that

Arnold’s Loyalist-leaning wife Peggy Shippen played a

much more significant role in pushing him to turn his

back on the revolution than any feuds he had with

competing officers. In fact, there is evidence that

Shippen and Arnold’s co-conspirator, Major John André,

were former lovers and that Shippen initiated the

correspondences that led to Arnold agreeing to betray

the Americans. Furthermore, although Arnold harbored

resentment after being passed over for important

promotions, he always knew that his close friendship with

George Washington was a very significant advantage he

ultimately held over any competing forces in the Patriot

army.

Secondly, although Arnold lived extravagantly and was in

debt, his wife Peggy was the daughter of a wealthy

Philadelphia family. Therefore, the 20,000 pounds he

was promised can hardly be considered an incentive for

Arnold to disgrace himself, risk his life, and betray his

countrymen.

Finally, although the thought of becoming a famed British

General might have crossed Arnold’s mind a few times, it

is well documented that British officials did not trust

Benedict Arnold and he likely was well aware of this. He

was always perceived as a risk. Since he had such

significant ties to and strong former allegiance to the

Patriots, he could have potentially betrayed the British

and reignited Patriot loyalties. The British may have also

suspected that he was a double agent all along. Arnold,

therefore, could have had no illusions about the

possibility of a career in the British Army.

Chapter 2

Page 124: Insider: The Super Guide Scripts & Answer Key

A124 Insider: The Super Guide

(Example 1: Contradiction Format _ p.557

(Example 2: Problem / Solution Format _ p.558

(Example 3: Supporting Argument Format_ p.559

[ f Script ]

Professor (male): After a rocky period of alienation in the

mid-nineties, the Dutch method of drug enforcement is

beginning to catch on. The Dutch have been criticized by

most of Europe in the past for their liberal drug policies,

when, in actuality, they have the lowest rate of drug

addiction in Europe. This seems to be the case with all

countries that are loosening their grip on harsh drug

laws. It seems the more progressive the policy, the better

it works, and nowhere is this more true than in Holland.

The Dutch system involves government-sponsored drug

rehabilitation for anyone and everyone in need. They

favor rehabilitation programs over imprisonment,

believing that drug addiction is a disease and should be

treated as such. Medical assistance is given in the form

of medicinal heroin, administered either to addicts

weaning themselves off of drugs or even continuing drug

addicts. Either way, it reduces crime and serves the

public interest.

The legalization of certain drugs is another major step

forward. I mentioned medicinal heroin, but the Dutch also

have very liberal rules regarding marijuana. “Coffee

shops” can sell up to five grams of marijuana to a single

patron, the same amount that is legal to have on one’s

person in public. Legalizing and regulating the sale of

One interesting advance has been the development of

chips with power monitoring systems. You see, a chip

doesn’t actually need to run at full speed all the time. So

these chips calculate how much of their processing

power the computer needs at the moment, and they

reduce their power usage accordingly. So when the

computer doesn’t need much processing power, the chip

is using less electiricity and therefore creating less heat.

When the computer needs the full processing power of

the chip, it increases its energy usage and immediately

drops it back down once the processing task is

completed. This greatly reduces the problem of chips

overheating.

Engineers have also started to design chips with dual

processors. In reality, these chips are basically two

processors stacked together. They increase computer

performance by splitting the processing tasks between

the two processors. The nice thing is that, since they use

the same manufacturing technology as older chips,

engineers don’t have to figure out how to fit more circuits

on the chip in order to increase performance.

[ f Script ]

Professor (female): As all of you know, in CPU design,

the name of the game is speed. The faster your chip, the

easier it will be to sell. But creating faster chips has

become more and more problematic with each passing

year... uh, primarily due to heat issues and limitations in

the manufacturing process. However, a number of

computer companies are now working on some pretty

innovative solutions to these problems.

[ f Script ]

Professor (male): OK, uh, I know that many of us have

this image of the American Revolution as kind of a David

and Goliath story... uh, that the colonists were facing

almost impossible odds in fighting the mighty British. But

if you actually sit down and examine the situation, you’ll

find that this image is a deceptive one.

For example, much has been made of British naval

power and their control of the seas. To be sure, the

British Navy certainly was the dominant sea power at the

time. But the colonists had been running smuggling

operations for decades and were skilled at evading

British ships. Therefore, the British were never really able

to cut off the supply lines of the colonists. Uh, in addition,

the colonists received considerable support from the

French, the only European power capable of challenging

British dominance of the seas. French aid proved to be a

decisive factor in the colonists’ victory.

Another misconception is the image of the Colonial Army

as a small, rag-tag band of volunteers with little

experience. Many of the men who fought in the

Revolutionary War were veterans of the French and

Indian War or of other conflicts with the Native American

tribes. In reality, it was the British who lacked the

necessary training. They were used to fighting large

armies that fought on open fields using traditional tactics,

not small groups of rebels who used hit and run tactics

as the colonists did. The British were completely

unprepared for this type of warfare.

OK, finally, the colonists were technologically superior to

the British in the one area where it truly mattered... their

rifles. Colonial rifles were more accurate and had a

longer range than those used by the British Army. These

superior firearms, along with their hit and run tactics,

gave them a great advantage over the British.

Page 125: Insider: The Super Guide Scripts & Answer Key

Scripts & Answer Key A125

WR

ITIN

G

Note Taking

Exercise 1~2 _ p.564~565(Sample)

Main Idea: Music

Downloading is illegal and

damaging to the music

industry.

Supporting Point 1:

Music downloading same

as stealing CDs.

Supporting Point 2:

Music downloading

damages the recording

ind.

Supporting Point 3:

Illegal music downloading

hurts small artists

Main Idea: Music

Downloading is neither

illegal nor is it damaging to

the music industry.

Supporting Point 1: Most

music downloading is legal.

File sharing no different

from letting a friend borrow

a CD. Many sites make

users pay for music now.

Supporting Point 2: Music

downloading not destroying

music ind. Music ind.

makes profit from online

music stores. Online music

has software, so can’t

share illegally.

Supporting Point 3: Many

small artists rely on online

music; helps distribute their

music to public. Many

benefit from free music

sharing.

community is their only access to the music masses.

Major labels have created divisions for alternative artists,

many of whom gained popularity by allowing their music

to be shared freely online.

[ f Script ]

Professor (male): It is assumed that all music sharing on

the internet is illegal, detrimental to music artists and

record labels, and harmful to emerging alternative artists.

All of these are wrong assumptions.

First of all, a large portion of people who share music

files do just that, share their legally obtained music files

with the online community. The sources of almost all of

the music files shared through file-sharing programs are

CD collections of the average, dedicated music collector.

The sharing of content from legally purchased CDs is

neither illegal when you loan your CD collection to a

friend nor is it illegal when you download it onto your

computer and loan it to fellow music lovers on the

internet.

Second, record labels would have you believe that file-

sharing is destroying the music industry. Well, uh, we all

know that the music industry and millionaire recording

artists are a long way from bankruptcy. The industry

already makes a profit from online music stores. They

are also creating safeguards to make music sold in

online music stores difficult to share illegally. For

example, nearly all the music sold online is copyright

protected and the software makes it impossible to

download your music collection on more than one

computer. This corrects the mistake made by the music

industry by not initially protecting the CD format by

making it illegal to share CD content on a mass level.

Finally, many in the file-sharing community argue that far

from doing harm to artists outside the major label

system, they are helping to publicize and boost

alternative artists into fame by sharing music online. In

fact, little-known artists have benefited the most from free

music exchange because for many, the internet music

soft drugs helps delineate them from more destructive

hard drugs, hopefully making marijuana less of a

“gateway drug,” as some people call it.

Educational programs in Holland differ from those in

many other, more conservative nations. Holland’s

educational programs for pre-teens and teens deal in

frank, often disturbing facts concerning drug addiction.

These programs include open discussions with teens

who have experimented or considered experimenting

with drugs. Such a liberal approach acknowledges the

intelligence and free will of youth and carries more

credibility with youth culture.

Page 126: Insider: The Super Guide Scripts & Answer Key

A126 Insider: The Super Guide

Preparing Your Response

Exercise _ p.567(Sample)

Writing an Introduction

Exercise _ p.569(Sample)

1. Downloading music from the internet is not illegal and it

does not harm the music industry.

2. Downloading online music is not illegal.

3. Online music does not hurt the music industry.

4. Online music is not harmful to small artists.

5. In his lecture, the professor discusses the

misconceptions concerning online music. The points he

makes directly contradict the reading’s claims that

downloading online music is illegal, that it harms the

music industry, and that it is harmful to small artists.

Body Paragraphs

Exercise 1 _ p.571(Sample)

According to the professor, most music downloading is

actually legal. Since most shared music files come from

legally purchased CDs, trading music files online is actually

no different from lending a CD to a friend.

Exercise 2 _ p.571(Sample)

The professor claims that the recording industry has already

begun to use file sharing for its own purposes. The music

industry makes a profit from selling music in online stores

and it uses copyright software to make sure that music files

are not illegally copied.

Exercise 3 _ p.573(Sample)

1. According to the professor, most music downloading is

actually legal. Since most shared music files come from

legally purchased CDs, trading music files online is

actually no different from lending a CD to a friend. This

directly contradicts the reading’s claim that downloading

online music is illegal.

2. In addition, the professor claims that the recording

industry has already begun to use file sharing for its own

purposes. The music industry makes a profit from selling

music in online stores, and it uses copyright software to

make sure that music files are not illegally copied. This

disproves the point made in the reading that the music

industry is being damaged by online music.

3. Finally, the professor notes that many smaller bands

have actually benefited from online file sharing. File

sharing allows smaller bands to distribute their music to a

wider audience and encourages innovation in the music

industry. The reading’s claim that online music sharing

hurts small artists is contradicted by this point.

Writing a Conclusion

Exercise _ p.575(Sample)

In conclusion, the professor’s lecture shows that

downloading online music is not nearly as bad as many

people claim, and in many ways it is helpful. In contradiction

to the claims made in the reading, the professor points out

that it is not illegal, nor does it harm the music industry or

small artists.

MMaaiinn ccoonnttrraaddiiccttiioonn bbeettwweeeenn tthhee lleeccttuurree aanndd tthhee

rreeaaddiinngg:: Music downloading is not illegal and it does

not harm the music industry.

PPooiinntt 11 ooff tthhee lleeccttuurree: Online music sharing is like

letting a friend borrow a CD, and many music

downloading sites now charge customers for the music.

HHooww ppooiinntt 11 ooff tthhee lleeccttuurree ccoonnttrraaddiiccttss ppooiinntt 11 ooff tthhee

rreeaaddiinngg: The reading claims that music downloading

is no different from stealing a CD from a music store.

The lecture clearly contradicts this point.

PPooiinntt 22 ooff tthhee lleeccttuurree: The music industry makes a

profit from the music they sell in online music stores,

and use copyright software to make sure the music

cannot be copied illegally.

HHooww ppooiinntt 22 ooff tthhee lleeccttuurree ccoonnttrraaddiiccttss ppooiinntt 22 ooff tthhee

rreeaaddiinngg: The reading claims that music downloading

is damaging the music industry, but, according to the

lecture, the music industry is actually benefiting from

it.

PPooiinntt 33 ooff tthhee lleeccttuurree: Many small artists benefit from

music downloading because it helps them to distribute

their music to the public.

HHooww ppooiinntt 33 ooff tthhee lleeccttuurree ccoonnttrraaddiiccttss ppooiinntt 33 ooff tthhee

rreeaaddiinngg: The reading claims that small artists are hurt

most by music downloading, but the lecture clearly

contradicts this claim.

Page 127: Insider: The Super Guide Scripts & Answer Key

Scripts & Answer Key A127

WR

ITIN

G

Editing and Revising Your Essay

Exercise _ p.577(Sample)

The lecture makes three major points that refute the claims

made in the reading. First, the reading says that it is illegal

to share music on the internet. However, the professor

points out that this is untrue. According to the professor,

sharing music online is no different than allowing a friend to

borrow a CD, so it is certainly not a form of piracy.

Next, in the lecture the professor also mentions that sharing

music on the internet does not hurt the music industry. The

music industry actually makes a profit from selling music

online, and it uses copyright software to ensure that files are

not copied and distributed illegally. This contradicts the

reading, which claims that online music is destroying the

music industry.

Finally, the lecture points out that file sharing is actually

helpful to smaller artists, who are able to distribute their

music more effectively through the use of online music. This

contradicts the reading’s claim that online music is harmful

to small artists.

Guided Practice 1

Exercise 1~2 _ p.578~579(Sample)

Main Idea: Quitting

smoking can be very

difficult for a number of

reasons.

Point 1: Smokers are

physically addicted to

nicotine, and addiction is

very difficult to control.

Point 2: Quitting smoking

causes withdrawal, and

many start smoking again

to avoid withdrawal

symptoms.

Point 3: Smoking is also a

psychological addiction,

and psychological

addiction can last for

many years.

Main Idea: There are a

number of therapies and

methods available to people

who wish to quit smoking.

Point 1: Nicotine Replace-

ment Therpay (NRT)

supplies small amounts of

nicotine through gum,

patch, etc. NRT gradually

reduces level of nicotine in

body. Helps beat physical

addiction and reduces

withdrawal symptoms.

Point 2: Support groups

can help smokers

overcome psychological

addiction. In meetings they

admit their problems.

Regular attendance of

meetings helps to avoid

temptation to smoke again.

Point 3: Smokers can also

use hypnotherapy to block

the mental urge to smoke.

Hypnotic suggestion are

very powerful and effective

in preventing a return to

smoking.

Smokers who attend these meetings, like alcoholics,

admit their powerlessness over their dependence on

nicotine and use a combination of sponsorship and

regular attendance of meetings to address their

temptation to light up a cigarette. Other smokers opt to

hire hypnotherapist. Hypnotherapy techniques can help a

person block the psychological impulses that cause him

or her to smoke. The smoker is first hypnotized and then

given instructions on how to deal with the urge to smoke.

Because these instructions are given while the patient is

hypnotized, they are very powerful and usually quite

effective.

[ f Script ]

Professor (female): Solving the problem of nicotine

addiction is something millions of smokers around the

world struggle with on a continuous basis. Few people

are able to quit smoking without a struggle, and many

have to make several attempts before they are

successful, if they ever are. However, there are a

number of treatment options that can help smokers quit

the habit.

The first is called Nicotine Replacement Therapy, or

NRT. Nicotine replacement therapy or NRT provides low

doses of nicotine which greatly minimize the withdrawal

symptoms that make it impossible for most smokers to

quit. NRT comes in the form of gums, patches, oral

sprays, inhalers and lozenges. The level of nicotine

these products release is based on the level of the

smoker’s addiction, for example, light, average, or heavy

smoker. As time goes on, the smoker can reduce the

amount of nicotine he or she gets from NRT.

Beating the physical addiction of smoking, however, is

only half the battle. Smokers who wish to quit must beat

the psychological addiction as well, and this can be very

difficult. Some smokers choose to also attend Nicotine

Anonymous groups. These groups adopt the strategies

used by Alcoholics Anonymous to deal with alcoholism.

Page 128: Insider: The Super Guide Scripts & Answer Key

A128 Insider: The Super Guide

Exercise 3 _ p.580(Sample)

Exercise 4 _ p.580(Sample)

1. There are a number of therapies available to help people

stop smoking.

2. The three problems discussed in the reading are the

physical addition to nicotine, the experience of

withdrawal, and psychological addiction.

3. In the lecture the professor explains a number of

strategies to help people stop smoking. These strategies

address the problems of nicotine addiction, withdrawal

symptoms, and psychological addiction, which are

discussed in the reading.

Exercise 5 _ p.581(Sample)

1. The first strategy discussed by the professor is nicotine

replacement therapy. In this treatment, smokers are

given small doses of nicotine through the use of a gum,

patch, or some other method. The amount of nicotine

they are given is slowly reduced over time, This helps to

overcome the physical addiction to nicotine and lessens

the unpleasant symptoms of withdrawal, which according

to the reading are two of the reasons it is so difficult to

stop smoking.

2. The professor also discusses using group therapy as a

way to quit smoking. According to the professor, these

groups use methods similar to those of Alcoholics

Anonymous. During meetings, members discuss their

addiction to smoking and how it affects their lives.

According to the professor, regular attendance at these

meetings can help smokers resist the long term

psychological urge to smoke. According to the reading,

overcoming the psychological addiction is another major

hurdle to quitting smoking over the long term.

3. Finally, the professor says that some people may need

hypnotherapy to stop smoking. In this method of

treatment, the smoker is hypnotized and instructed to

resist the urge to smoke. According to the professor,

these instructions are very powerful and effective. This

method is another way in which smokers may deal with

the long term effects of psychological addiction.

Exercise 6 _ p.582(Sample)

1. Smokers who wish to quit have a number of treatment

options that can help them do so.

2. The treatment options described in the reading directly

address the reasons why people find it so difficult to stop

smoking.

3. In conclusion, the lecture outlines a number of different

treatment options available to people who want to quit

smoking. These options directly address the reasons

why people find it so difficult to quit.

Exercise 7 _ p.582(Sample)

In the lecture, the professor explains a number of strategies

to help people stop smoking. These strategies address the

problems of nicotine addiction, withdrawal symptoms, and

psychological addiction, which are discussed in the reading.

The first strategy discussed by the professor is nicotine

replacement therapy. In this treatment, smokers are given

small doses of nicotine through the use of a gum, patch, or

some other method. The amount of nicotine they are given is

slowly reduced over time. This helps to overcome the

physical addiction to nicotine and lessens the unpleasant

symptoms of withdrawal, which according to the reading are

two of the reasons it is so difficult to stop smoking.

The professor also discusses using group therapy as a way

to quit smoking. According to the professor, these groups use

methods similar to those of Alcoholics Anonymous. During

meetings, members discuss their addiction to smoking and

how it affects their lives. According to the professor, regular

attendance at these meetings can help smokers resist the

long term psychological urge to smoke. According to the

reading, overcoming the psychological addiction is another

major hurdle to quitting smoking over the long term.

Finally, the professor says that some people may need

hypnotherapy to stop smoking. In this method of treatment,

the smoker is hypnotized and instructed to resist the urge to

smoke. According to the professor, these instructions are

very powerful and effective. This method is another way in

which smokers may deal with the long term effects of

psychological addiction.

In conclusion, the lecture outlines a number of different

treatment options available to people who want to quit

smoking. These options directly address the reasons why

people find it so difficult to quit.

Lecture Point 1: Nicotine

Replacement Therapy

Lecture Point 2: Support

groups – Nicotine

Anonymous

Lecture point 3:

Hypnotherapy

Addresses problems of

physical addiction and

withdrawal symptoms

Addresses problem of

psychological addiction

Addresses problem of

psychological addiction

Page 129: Insider: The Super Guide Scripts & Answer Key

Scripts & Answer Key A129

WR

ITIN

G

Guided Practice 2 Exercise 1~2 _ p.583~584(Sample)

Exercise 3 _ p.585(Sample)

Lecture Point 1: Mars

rovers have operated for

3 years.

Lecture Point 2: Mars

rovers are advanced

robots capable of making

independent decisions.

Lecture point 3: NASA

controllers have placed

rovers in high-risk

situations.

Supports the reading’s

claim that unmanned

missions are preferable

because they can operate

for longer periods and

accomplish more.

Supports the reading’s

claim that modern robots

can perform most of the

functions of human

astronauts.

Supports the reading’s

claim that unmanned

probes can be placed in

more dangerous situations

than human astronauts.

Main Idea: From a

scientific standpoint,

unmanned space missions

are more valuable than

manned missions.

Point 1: Unmanned

missions can conduct

scientific research for

longer periods.

Point 2: Modern robots

can perform nearly all of

the same functions as a

human astronaut.

Point 3: Robots can be

placed in high risk

situations that may be

scientifically rewarding.

Main Idea: NASA’s

unmanned Mars Rover

mission has been its

greatest success.

Point 1: The two robots of

the mission have been

operating on Mars for over

three years. The length of

their stay has allowed

them to gather much info

about the planet. Both

continue to operate.

Point 2: Rovers are the

most advanced robots

ever. They can make their

own choices for navigation

and can evaluate what

areas they should explore.

Point 3: NASA has already

placed both robots in high-

risk situations when it was

felt that the potential

scientific rewards

outweighed the risks to the

robots.

[ f Script ]

Professor (male): What do you think would be the most

successful NASA mission of all time? Many of you would

probably say the Apollo missions, which put the first

human explorers on the moon. Most scientists, however,

would disagree with you. From the standpoint of scientific

discovery, the Mars Rover mission, which put two

unmanned, robotic probes onto the Martian surface in

2004, is by far NASA’s greatest success.

These two robots have been operating and conducting

experiments on the surface of the planet for over three

years. During that time, they have discovered a wealth of

new information about the planet, its history, and the

possibility of life on other worlds. Their extended

presence on Mars has allowed us to learn more about

the planet in the last three years than we did in the 30

years prior to their landing. And their work isn’t done yet.

Both of the rovers are still operable and continue to carry

out scientifically valuable experiments.

Another reason for the rovers’ success is that they are by

far the most advanced robots that NASA has ever built.

Their powerful array of cameras, sensors, and computers

allow them to analyze the landscape and choose the

best way to navigate the terrain. They are also able to

make decisions regarding which areas would be the

most valuable to explore.

Finally, these two robots are just that – robots. If they are

destroyed or damaged, it’s unfortunate, but it’s not the

end of the world. There have already been a number of

situations in which scientists at NASA have chosen to

order the rovers to engage in potentially risky

experiments. In these cases, the scientists have decided

that the potential rewards of the experiment outweighed

the risks to the robot.

Page 130: Insider: The Super Guide Scripts & Answer Key

A130 Insider: The Super Guide

Exercise 4 _ p.585(Sample)

1. The Mars rovers are the most successful mission in

NASA’s history, even more successful than the manned

Apollo missions.

2. The reading discusses the advantages of longer mission

lengths, more advanced robots that can perform more

functions, and the ability to place robots in high-risk

situations.

3. In the lecture, the professor discusses the reasons why

the Mars Rover missions are considered to be NASA’s

greatest success. The professor’s discussion directly

supports the reading’s claims that unmanned missions

are able to operate for longer time periods, capable of

performing many of the same functions as human

astronauts, and able to be used for high-risk operations.

Exercise 5 _ p.586(Sample)

1. The professor points out that the Mars rovers have been

in operation for more than three years and are still

conducting useful experiments. According to the

professor, during those three years the rovers have

made numerous important discoveries and added more

to our understanding of Mars than the previous 30 years

of study. This directly supports the reading’s claim that

unmanned missions are more valuable to science

because they are able to operate for longer periods.

2. The professor also discusses how the rovers are able to

make their own decisions regarding the course they take

and the areas they decide to explore. This is due to their

advanced sensors and computers. The professor’s

discussion of these points supports the point made in the

reading that modern robots are able to perform most of

the same functions as human astronauts.

3. Finally, the professor says that the controllers of the

rovers can place them in high-risk situations where it

would be unacceptable to use human astronauts.

According to the professor, the rovers have already been

placed in several situations where there was significant

risk that they would be damaged or destroyed because

their human controllers felt the rewards outweighed the

risks. This supports the reading’s claim that unmanned

robots can be risked in ways that human astronauts

cannot.

Exercise 6 _ p.587(Sample)

1. NASA’s greatest success has been the unmanned Mars

Rover missions.

2. It is because the rover missions involved unmanned

robots that they were so successful.

3. In conclusion, the professor’s discussion of the Mars Rover

missions supports the reading’s claim that unmanned

probes are more valuable to science than manned

missions. It is precisely because the rovers were unmanned

robots that they were able to accomplish so much.

Exercise 7 _ p.587(Sample)

In the lecture, the professor discusses the reasons why the

Mars Rover missions are considered to be NASA’s greatest

success. The professor’s discussion directly supports the

reading’s claims that unmanned missions are able to

operate for longer time periods, capable of performing many

of the same functions as human astronauts, and able to be

used for high-risk operations.

The professor points out that the Mars rovers have been in

operation for more than three years and are still conducting

useful experiments. According to the professor, during those

three years the rovers have made numerous important

discoveries and added more to our understanding of Mars

than the previous 30 years of study. This directly supports

the reading’s claim that unmanned missions are more

valuable to science because they are able to operate for

longer periods.

The professor also discusses how the rovers are able to

make their own decisions regarding the course they take

and the areas they decide to explore. This is due to their

advanced sensors and computers. The professor’s

discussion of these points supports the point made in the

reading that modern robots are able to perform most of the

same functions as human astronauts.

Finally, the professor says that the controllers of the rovers

can place them in high-risk situations where it would be

unacceptable to use human astronauts. According to the

professor, the rovers have already been placed in several

situations where there was significant risk that they would be

damaged or destroyed because their human controllers felt

the rewards outweighed the risks. This supports the

reading’s claim that unmanned robots can be risked in ways

that human astronauts cannot.

In conclusion, the professor’s discussion of the Mars Rover

missions supports the reading’s claim that unmanned probes

are more valuable to science than manned missions. It is

precisely because the rovers were unmanned robots that

they were able to accomplish so much.

Page 131: Insider: The Super Guide Scripts & Answer Key

Scripts & Answer Key A131

WR

ITIN

G

Additional Practice

1 _ p.588~590

Note Taking >>(Sample)

Planning Your Response >>(Sample)

Lecture point 1: Global

warming may be part of a

natural cycle.

Lecture point 2:

Interglacial periods are

common in the earth’s

history.

Lecture point 3: There is

no evidence that higher

levels of CO2 cause more

severe storms.

Connection to the

reading: Refutes the idea

that greenhouse gases are

responsible for increased

temperatures.

Connection to the

reading: Refutes the idea

that glacial retreat is due to

greenhouse gases.

Connection to the

reading: Refutes the idea

that greenhouse gases are

responsible for stronger

hurricanes.

Main Idea: Global

warming and extreme

weather patterns are the

direct result of greenhouse

gas emissions.

Point 1: The large

temperature increases of

the last century and the

increases predicted for the

future can only be

explained by greenhouse

gases.

Point 2: Greenhouse

gases are also

responsible for the rapid

retreat and disappearance

of glaciers.

Point 3: The increase in

high-power hurricanes is

due to high levels of CO2

in the atmosphere.

Main Idea: Global

warming and extreme

weather patterns may not

be caused by greenhouse

gases.

Point 1: The current

increase in temperature

may be part of a natural

cycle. Over the last 12,000

years, there have been

many cycles of warming

and cooling. There is

usually around 1,000 to

1,500 years between each

cycle. The last warming

cycle was about 1,000

years ago.

Point 2: Periods of glacial

melting are also a natural

cycle. The earth is

currently in an interglacial

period, and there is no

evidence that this period is

more severe than previous

periods.

Point 3: There is no direct

evidence that higher levels

of CO2 result in more

severe storms. It is only a

mathematical theory.

[ f Script ]

Professor (female): Much has been made of the

potential disaster posed by global warming, but many

scientists remain skeptical of the idea, claiming that more

evidence is needed. Indeed, many of them are right.

Some of the evidence used to support the idea of global

warming is questionable, and it’s important that we

explore these questions before creating a mass panic.

Some scientists argue that the recent spike in

temperatures is simply one of many relatively harmless

warming trends that have occurred for millennia. A study

of seabed sediments off the coast of Ireland revealed a

12,000-year-old history of nine different climate cycles,

lasting 1,000-1,500 years apiece, during which both

warming and cooling took place – with no disastrous

consequences. Additionally, history tells us of smaller

warming trends within these larger ones, such as the

unexpected increase in temperatures that occurred about

a thousand years ago, during the Middle Ages.

As for melting glaciers, well, the earth is already 10,000

years into an interglacial period – um, that is, a period of

temperate conditions that occurs within an ice age. The

previous interglacial period lasted about 28,000 years,

and some geologists say that, to the best of their

knowledge, retreating glaciers, in relation to temperature,

are no more severe now than they were at this point in

the last interglacial period.

And of course, there’s the issue of severe weather. This

is based on the idea that high concentrations of CO2 in

the atmosphere correlate with more severe weather

patterns. While this idea does represent sound

mathematics, no empirical evidence has been found to

support it. That doesn’t necessarily mean it isn’t true, but

it does bear further study.

In short, evidence tells us that climate shifts are

inevitable, and determining whether this particular

climate shift will prove to be the catastrophe that most

scientists claim simply requires more research.

Page 132: Insider: The Super Guide Scripts & Answer Key

A132 Insider: The Super Guide

(Sample Essay)

In her lecture, the professor discusses alternative

explanations to many of the phenomena that scientists

blame on greenhouse gases. Specifically, she questions

whether greenhouse gases are responsible for global

warming, the retreat of glaciers, and increasingly severe

storms.

First, the professor points out that there is evidence that the

earth undergoes fairly regular periods of cooling and

warming. According to the professor, there have been nine

such warming periods in the last 12,000 years, with the last

one occurring about 1,000 years ago. Since there are

typically around 1,000 to 1,500 years between each cycle,

this casts doubt on the reading’s claim that the current

increase in temperatures is due to greenhouse gases.

The professor also notes that glaciers experience similar

periods of expansion and retreat, although the period is

much longer. The earth is currently in an interglacial period,

so it seems logical that glaciers are retreating. Again, this

casts doubt on the claim that the melting of glaciers is the

result of greenhouse gases.

Finally, the professor addresses the claim that high levels of

CO2 are responsible for recent increases in severe

hurricanes. According to the professor, there is no evidence

that higher CO2 levels lead to stronger storms. She says the

idea is only a mathematical theory and must be studied

more before it can be proven or disproved.

In conclusion, the professor points out that there are many

other possible explanations for many of the phenomena that

we commonly blame on greenhouse gases.

2 _ p.591~593

Note Taking >>(Sample)

Main Idea: The electoral

college has a number of

problems as a method of

electing a president.

Point 1: The candidate

who wins a majority of the

popular vote in a state

wins all of the electoral

votes. Makes many voters

feel like their vote does

not matter.

Point 2: Makes the

national popular vote

unimportant. It is possible

to win the popular vote

but still lose the electoral

vote.

Main Idea: There a

number of changes that

could be made to the

electoral system to make it

fairer.

Point 1: States could use a

proportional system for

assigning electoral votes.

Each state would keep the

same number of electoral

votes, but those votes

would be distributed among

candidates according to the

percentage of the popular

vote they received, rather

than all being given to the

winning candidate.

Point 2: If states made

interstate compacts, they

could agree to assign all of

their electoral votes to the

candidate who won the

national popular vote. If

enough states joined the

compact, it would basically

bypass the electoral

college.

still have its allocated number of electoral votes, but,

instead of all the state’s electoral votes going to the party

that wins the majority within the state, it would allow

these votes to be split according to which candidate

receives what percentage of the popular vote. This

system would more accurately reflect the views of both

parties within a given state.

Another criticism is that it undermines the idea of rule by

the people, in the sense that the national popular vote

can sometimes have little bearing on the electoral vote.

One proposed remedy is interstate compacts. An

interstate compact allows for cooperation between states

over broad policy issues. As far as elections are

concerned, any number of states might agree with one

another to devote all of their electoral votes to the

candidate who wins the national popular vote. If enough

states enter into the compact, they could essentially

bypass the Electoral College altogether and allow the

election to be decided by the national popular vote.

[ f Script ]

Professor (male): So, uh, we’ve been talking about the

recent elections and how Americans elect their

presidents, um, indirectly. It may seem wrong that the

winner of the popular vote can still lose the election, but it

is, according to our Constitution, completely legitimate.

Ultimately, it’s the Electoral College that casts the final

ballots. The system has come under fire in the past few

years, and there are two interesting reforms that could

make this system more, uh, democratic in nature. The

first would simply revise the electoral system, while the

second devises a perfectly legal way to bypass it

altogether.

One complaint is that the candidate who receives the

majority of the votes within a state wins all of that state’s

electoral votes. A proposed alternative is a proportional

representation system. In this system, each state would

Page 133: Insider: The Super Guide Scripts & Answer Key

Scripts & Answer Key A133

WR

ITIN

G

Planning Your Response >>(Sample)

(Sample Essay)

The professor’s lecture is about potential changes that could

be made to fix the problems in the way that the American

president is elected. The suggestions that the professor

makes directly address the problems that are raised in the

reading.

One of the suggestions that the professor makes is that the

electoral college could be changed so that states assigned

their electoral votes to different candidates according to the

percentage of the popular vote that each candidate won,

rather than assigning all of the states votes to the candidate

who won a simple majority of the popular vote. According to

the professor, such a system would more accurately reflect

the views of members of all political parties in a state. This

would eliminate the problem of minority voters feeling like

their votes for president did not count.

Another possible solution discussed by the professor is an

interstate compact. In an interstate compact, a number of

states make an agreement to assign all of their electoral

votes to the winner of the national popular vote, rather than

the winner of the popular vote within their own state. This

would help to ensure that the winner of the national popular

vote actually became president.

In conclusion, the professor states that the problems with the

electoral college could be solved by assigning electoral

votes proportionally or by using interstate compacts.

3 _ p.594~596

[ f Script ]

Professor (male): Advertising isn’t quite the business it

once was. Advertisers are losing ground. DVR devices

allow television viewers to skip commercials entirely, and

most people are too fed up with constant advertising to

take it seriously anyway. With this in mind, rogue

advertising firms that specialize in so-called guerrilla

marketing campaigns are gaining increasing popularity.

While these stealthy, low-cost campaigns can turn quite

a profit for companies, they can also be risky and

deceptive.

Take, for instance, the following campaign carried out by

a third-party advertising company in support of a recent

cult film. The agency placed boxes containing specially

designed LED light boards depicting characters from the

animated film in various locations around eleven major

US cities. This was done without the purchase of formal

advertising space or even the consent of the targeted

cities, and when discovered, one city treated the boxes

as explosive devices, spending millions of dollars to call

in bomb squads and police forces to shut down areas of

the city. Multi-million-dollar lawsuits were filed against the

studio, and the film gained a great deal of bad press.

Other incidents have not been so overt, however, and err

more on the side of deception than spectacle. A major

electronics company, in an effort to generate buzz for a

new digital camera, recently paid a number of actors to

walk about Times Square as if they were tourists. They

stopped a passersby to innocently ask if the stranger

would take their picture for them. As the unwitting victim

snaps the photos, the two “tourists” talk up the camera’s

state-of-the-art features, as if offering the person a

friendly tip. This tactic, according to most, is skirting the

bounds of media ethics.

Lecture point 1: States

could assign electoral

votes proportionally.

Lecture point 2: States

could enter interstate

compacts.

Connection to the

reading: Would solve the

problem of voters not

feeling like their votes

matter.

Connection to the

reading: Would ensure

that the winner of the

national popular vote won

the electoral college vote

also.

Page 134: Insider: The Super Guide Scripts & Answer Key

A134 Insider: The Super Guide

Note Taking >>(Sample)

Planning Your Response >>(Sample)

(Sample Essay)

In the lecture, the professor discusses two examples of

guerrilla marketing that illustrate why it is considered

dangerous and unethical. In the first example, the professor

describes how one company put small lighted signs in large

cities to create a buzz about a new movie. However, they did

not notify city officials that they were going to put up the

signs, and a few of their signs were mistaken for bombs.

This is exactly the kind of guerrilla marketing hoax that is

described in the reading. As the reading says, this particular

hoax backfired and caused a huge public disturbance.

The other example discussed by the professor is of a camera

company that hired actors to advertise its product. The actors

walked around the city pretending to be tourists and asked

people to take their picture using the company’s camera. As

the people took pictures, the actors would talk about how

great the camera was. Since the people thought the actors

were real people, they didn’t realize that they were listening

to an advertisement. This is an example of a company using

an undercover spokesperson, which is described in the

reading. The professor agrees with the reading, stating that

this practice is deceptive and somewhat unethical.

In conclusion, the examples given by the professor illustrate

the dangerous and unethical nature of guerrilla advertising.

4 _ p.597~599

[ f Script ]

Professor (male): So, in the past few years you’ve

probably heard a lot of this term “trickle-down

economics.” What does it mean? Well, first of all, it’s a

derogatory term for what’s actually called “supply-side

economics.” The idea is that tax breaks for the wealthy

and for corporations will result in economic growth which,

over time, will “trickle down” to benefit the middle and

lower classes, who will invest their extra earnings in the

economy.

However, uh, when one particular version of supply-side

economics was introduced in the early 1980s, it led to a

long series of short recessions that stretched well into

the next decade. By the early 1990s, unemployment was

at its highest rate in years, and average yearly earnings

had dropped by more than 20%. Those left jobless faced

longer unemployment, and when they did return to work,

they were paid significantly less than before. This led to

less consumer spending, jeopardizing the important role

of consumers in this proposed “cycle” of economic

growth, and the economy suffered greatly from it.

And these major corporations that had benefited from the

tax cuts, well, they still weren’t paying their taxes,

depriving the government of billions in tax revenue that,

frankly, could’ve been quite useful around this time.

Facing a difficult recession, the safety of offshore tax

havens was more appealing than ever.

Additionally, the gap between the upper and lower

classes had slowly been widening since the early 1980s,

and with the recession that gap very quickly became a

chasm. Apparently, the tax breaks given to the rich and

industrious had failed to “trickle down” as expected – the

rich had either squandered their companies’ profits or

padded their own pockets with the extra money – and

the lower class suffered more than ever.

Lecture point 1: Guerrilla

marketing technique

caused a bomb scare.

Lecture point 2: Actors

pretended to be real

camera owners.

Connection to the

reading: Proves that

guerrilla marketing can

backfire and cause large

public disturbances.

Connection to the

reading: Proves that some

guerrilla marketing

techniques can be

deceptive and unethical.

Main Idea: Guerrilla

marketing can be effective,

but it can also be

dangerous and unethical.

Point 1: Guerrilla

marketing often involves

pranks and hoaxes that

can create serious public

disturbances if taken

seriously or misunderstood.

Point 2: Some guerrilla

marketers disguise

themselves as real people.

This is misleading and

unethical.

Main Idea: Guerrilla

marketing strategies can

be risky and deceptive.

Point 1: One company

was advertising for a

movie and put light boards

of the movie’s characters

in large cities without

telling authorities. Several

of the devices were

mistaken for bombs.

Point 2: An electronics

company paid actors to

walk around asking people

to take pictures of them.

As the people took the

pictures they would talk

about how good the

camera was. The tactic is

somewhat unethical.

Page 135: Insider: The Super Guide Scripts & Answer Key

Scripts & Answer Key A135

WR

ITIN

G

Note Taking >>(Sample)

Planning Your Response >>(Sample)

(Sample Essay)

In his lecture, the professor explains why supply-side

economics does not actually work. The points he makes

directly contradict the major claims of the reading.

First, the professor points out that when supply-side

economics was introduced in the 1980s, it led to a series of

recessions. As a result, salaries decreased and

unemployment rose. Consumer spending dropped, and the

overall economy suffered. This directly contradicts the

reading’s claim that supply-side economics benefits the

economy because it increases productivity and creates more

jobs.

The professor also points out that lowering corporate taxes

did not discourage companies from using offshore tax

shelters to avoid paying their taxes. Instead, the poor

economic conditions encouraged companies to continue

using those tax shelters, and the government lost large

amounts of tax revenue. This refutes the reading’s claim that

lowering corporate taxes encourages companies to pay a

higher percentage of their tax burden.

Finally, the professor says that, in contrast to what the

reading claims, lowering taxes for the rich did not benefit the

lower classes. Instead of using the money they saved

through lower taxes to help the poor, the rich simply kept the

extra money, and the gap between the rich and the poor

increased.

Lecture point 1: Use of

supply side economics

caused a recession and

high unemployment.

Lecture point 2:

Companies continued to

use offshore tax shelters.

Lecture point 3: Lowering

taxes for the rich did not

help the poor and

increased the income gap.

Connection to the

reading: Refutes the claim

that supply side economics

helps to create more jobs.

Connection to the

reading: Refutes the claim

that lowering corporate

taxes encourages compa-

nies to pay their taxes

rather than avoiding them.

Connection to the

reading: Refutes the claim

that supply side economics

is good for both the rich

and the poor.

Main Idea: Supply side

economics benefits both

the rich and the poor and

helps the economy.

Point 1: Lowering

company taxes increases

productivity and creates

more jobs.

Point 2: Companies are

less likely to set up

offshore tax shelters if

corporate taxes are

lowered.

Point 3: Lowering taxes

for the rich eventually

leads to lower taxes for

the poor as well.

Main Idea: Supply side

economics is bad for the

economy and only benefits

the rich.

Point 1: Supply side

economics was used in the

1980s. It caused high rates

of unemployment, reduced

consumer spending, and

damaged the economy.

Point 2: Even with lower

corporate taxes,

companies still tried to

avoid paying their full

share of taxes by using tax

shelters.

Point 3: Lowering taxes for

the rich did not help the

poor. The income gap

between the rich and the

poor continued to grow.

Page 136: Insider: The Super Guide Scripts & Answer Key

A136 Insider: The Super Guide

Part 1>> Set 1 _ p.604

1. Vocabulary Question – (D)

A key context clue in the sentence is that an “amalgam of

ideas and beliefs” can be brought together.

2. Negative Fact Question – (D)

The first sentence of paragraph 1 contains paraphrases of

all the answer choices except (D).

3. Rhetorical Purpose Question – (B)

Answer choice (C) can be eliminated because it has little to

do with the main focus on the passage. Answer choice (A)

seems like a likely choice, but the paragraph specifically

mentions the influence that the Greeks had on Western

civilization. Therefore, (B) is the best choice.

4. Vocabulary Question – (D)

Answer choices (B) and (C) can be eliminated because

they make little sense in the context of the sentence.

5. Sentence Simplification Question – (A)

Monarchies often pass on the right to rule to the next

generation. This keeps power in one family and gives the

society the feeling that the government and the ethics

behind the entire society have not changed over time. This

sense of continuity can give the society a feeling of stability.

It is not a false sense of continuity as is stated in choice D

but a real sense of stability as is stated best in choice A.

6. Fact Question – (C)

The paragraph directly states that “The amount of power

held by the monarch varies with the society.”

7. Vocabulary Question – (C)

The next sentence gives a context clue when it states that

the poor “would support anyone who would overthrow

their oppressors”.

8. Negative Fact Question – (B)

Answer choice (B) directly contradicts the passage.

Paragraph 4 states that tyrants are people who overthrow

aristocrats.

9. Fact Question – (D)

This is directly stated in the last two sentences in

paragraph 4.

10. Inference Question – (A)

Most people who became aristocrats were wealthy to

begin with. In principle, aristocracies may have been a

good idea. Even founding them on military heroism was

not necessarily wrong. The problem was that this heroism

could not be passed on to future generations and, over

time, aristocracies tended to deteriorate. At least this is

what seems to be suggested here.

11. Fact Question – (A)

Democracies of the past did not truly give power to all of

the people. In the U.S., for example, only white males

originally had the right to vote. Modern democracies,

where all adults had the right to vote, did not appear until

the 20th Century.

12. Sentence Insertion Question – This sentence should be

placed at the third square.

One would expect this sentence to follow one where the

continuation of wealth was discussed. The sentence

following should explain why such power was undeserved.

The location that does both of these is at square three.

13. Categorization Question

(A) Occurs when the head of state is overthrown (X)

(B) Perhaps the oldest form of government (Monarchy)

(C) Occasionally has only a symbolic purpose

(Monarchy)

(D) Aristotle criticized it as rule by the poor (X)

(E) Connected to military heroism (Aristocracy)

(F) Leaders were supposed to symbolize the best

qualities of the society (Aristocracy)

(G) Head of state is given the right to rule for life

(Monarchy)

(H) Often replaced by democracies (Aristocracy)

(I) Over time lost the values on which it was founded

(Aristocracy)

The two incorrect answer choices can easily be eliminated

because they refer to tyrannies and democracies. Most

other answer choices should be fairly obvious. Monarchs

rule for life, and many modern monarchies are purely

symbolic. In addition, the passage explicitly mentions that

monarchies are perhaps the oldest form of government.

The remaining answer choices clearly relate to

aristocracies.

Part 2>> Set 1 _ p.609

1. Referent Question – (A)

Read the sentence for an important context clue. The

opening phrases states that: “Not only have they mapped

out these layers,” indicating that “they” are people, thus

researchers.

2. Fact Question – (D)

Skim the paragraph for the sentence that directly states

the answer. It occurs early on: “In the past, many

speculated that the inner earth was hollow and inhabitable,

much like the surface.”

3. Fact Question – (A)

The answer is never directly stated in the paragraph.

However, it is paraphrased, so understanding the main

ideas should indicate the correct choice. The phrase

“Though they haven’t had the opportunity to directly

iBT Practice Test - Reading

Page 137: Insider: The Super Guide Scripts & Answer Key

Scripts & Answer Key A137

Pra

ctic

e T

es

t

observe the inner earth” is a major clue.

4. Vocabulary Question – (B)

The correct choice is the most logical one. The primary

focus of this paragraph is the top layer of the Earth, and

the phrase “this stratum” also references this focal point.

The other choices are irrelevant to the paragraph.

5. Fact Question – (C)

The correct answer choice actually paraphrases something

which is directly stated. The passage states that solids in

the crust “are much lighter than the materials that sink to

the lower levels.”

6. Vocabulary Question – (B)

This vocabulary word describes the area in which these

materials reside, or the mantle itself. In the next sentence,

the word “slimy” also describes the mantle and renames

the vocabulary word: “they are slowly shifting due to the

large amounts of heat that travel through the slimy layer.”

Thus, the choice is “oozy,” a synonym for “viscous” and

“slimy.”

7. Vocabulary Question – (A)

Pay attention to the context in which the vocabulary word

appears, especially the part of the sentence that describes

how the energy reverberates off the outer core “because it

cannot travel through liquid.” Because it cannot pass

through, the energy ricocheted, or bounced, off the outer

core.

8. Fact Question – (B)

Again, this is another fact that is directly stated in the

passage. Simply find the sentence that contains the

highlighted phrase. It states: “This flowing of the liquid core

produces another critical phenomenon that affects the

earth: a planetary magnetic field.”

9. Vocabulary Question – (C)

Carefully consider the context in which the word appears.

The paragraph describes how “the lowest level of the earth

is the most difficult for scientists to study.” Also, most of

the language indicates that scientists are guessing but not

certain about the details regarding the inner core. Thus,

“conjecture” means “guesswork.”

10. Sentence Simplification Question – (B)

It is important to recognize not only the ideas being

discussed in the highlighted sentence but also their

relationship to each other. Basically, two forces act on the

inner core: heat and pressure. They both have an effect:

the heat makes it hotter than the liquid outer core, but the

pressure keeps it solid. Choice (B) correctly restates this

relationship, and the other choices either distort or omit

the information.

11. Negative Fact Question – (D)

As with most negative fact questions, following the logic of

the argument is the quickest way to eliminate incorrect

choices. You may also skim the paragraph for these

choices, as the answer directly contradicts this statement:

“They theorize that, as liquid from the outer core begins to

cool and solidify, the inner core grows.”

12. Inference – (C)

This is a large inference question, and it asks you to make

connections between large ideas discussed throughout

the passage. Thus, it is never directly stated in any one

place, but it describes a pattern which should be evident

through a clear understanding of the passage and its main

ideas. Early on, the passage mentions how scientists can

use multiple techniques in observing the surface, but they

can only use sonar and earthquake waves to study the

mantle, and the outer core deflects earthquake waves.

Thus, they must largely theorize about the lowest levels.

This is because many ways of obtaining information

become useless at deeper levels.

13. Sentence Insertion Question – The sentence should be

placed at the third square.

The insertion sentence is referring to a previous subject

with the phrase “these less direct methods,” which are

used to study the mantle. This should follow a description

of the methods being used, which is the sentence which

describes how scientists observe energy waves beneath

the surface.

14. Prose Summary Question – (A), (E), (F)

The ideas presented within the introductory sentence are

the key to answering this question. Basically, these ideas

involve the composition of the inner earth and the study of

it by scientists. Choice (A) mentions “studies” and the

interchanging pattern of the layers, which is described

throughout the passage. Choice (E) describes how

researchers have discovered how energy travels

throughout the interior, a major idea within the passage,

and choice (F) mentions the method of reading energy

waves and the increasingly metallic composition of

materials at deeper levels. All mention the interior of the

earth and the study of it, while the incorrect choices only

describe minor or incorrect details.

>> Set 2 _ p.613

15. Vocabulary Question – (C)

Use details from this paragraph as contextual clues that

indicate the words meaning. For instance, “numerous

stories of lepers cast out of society, forced to wear signs

and ring bells to alert the healthy to their ‘unclean’

presence,” and “Leprosy was once believed to be the

outward manifestation of one’s sins.” These show that the

most logical meaning for “stigma” is “shame.”

16. Vocabulary Question – (A)

The word “outward” is an important clue for the meaning

of “manifestation.” Sinfulness is an abstract quality, so it

can’t be seen. However, the outward appearance, or

Page 138: Insider: The Super Guide Scripts & Answer Key

A138 Insider: The Super Guide

manifestation, of leprosy would indicate this quality.

17. Fact Question – (B)

Skim the paragraph. The answer is directly stated in this

sentence: “Lepers were gathered into ‘colonies’ and

removed from civilization to avoid transmitting to the public

what was for millennia believed to be a highly contagious

disease.”

18. Referent Question – (A)

Find the sentence that precedes this that mentions the

word “areas,” for it directly states the referent. It reads,

“Africa and Asia are also among the most heavily afflicted

areas in the world.”

19. Vocabulary Question – (B)

The vocabulary word is renamed by a synonym in a later

sentence; it states that poor living conditions “helped the

disease take root and increase to dangerous new levels.”

As stated earlier, poor living conditions contributed to the

disease’s proliferation, which is a synonym for increase.

20. Sentence Simplification Question – (A)

The correct answer paraphrases all of the key details

mentioned within the highlighted sentence. The four main

details are the poor living conditions, the poor health care,

the overcrowded populations, and the regions being within

Asia and Africa. All of these details appear in both

sentences. The other choices either omit or distort key

information.

21. Rhetorical Purpose Question – (C)

Read the paragraph closely in order to understand the

main idea, which is that leprosy is still a problem in certain

areas that have poor living conditions. The first sentence of

this paragraph provides an important contextual clue:

“Despite recent success against this disease, there are still

at least 600,000 new cases of leprosy reported each year .

. .” This indicates that improvements have been made but

that leprosy has never completely vanished and thus still

remains a problem.

22. Referent Question – (D)

It is important to understand the central topic being

discussed in this paragraph, which is transmission of the

disease. Skimming the paragraph reveals several

references to this mystery, such as: “Scientists are still

uncertain of the exact mechanisms of infection” and “the

method of infection remained unclear.” Thus, the only

mystery being discussed is how infection occurs.

23. Vocabulary Question – (B)

Look for an important contextual clue in this sentence. “As

had been postulated (though never proven) by many

scientists before” shows that whatever was postulated

was never proven, so it was just a theory.

24. Fact Question – (A)

Skim the paragraph. The answer is directly stated: “the

disease could pass through open wounds and through the

mucous membranes of the nose.”

25. Inference Question – (B)

While it is never directly stated, the correct answer choice

summarizes many of the ideas described in this

paragraph. The paragraph focuses on how extensive

treatment can cure leprosy, and, while the damage is

irreversible, it can still be treated without further hindering

the patient. Thus, victims no longer need to be excluded

from society, and they are free to live normal lives.

26. Sentence Insertion Question – The sentence should be

placed at the fourth square.

Pay attention to the insertion sentence’s transitional

phrase: “As a result of this treatment and drug therapy”.

Logically, this should come after a discussion of the

treatment and therapy that to which it refers. Thus, the

insertion sentence belongs at the end of the paragraph.

27. Prose Summary question – (A), (C), (E)

There are three key ideas in the introductory sentence that

are expressed by the correct answer choices. The key idea

is the advancements in treating leprosy, and the other two

are the areas affected by these advancements: the

physical damage and social stigma. Choices (A) and (C)

address the progress made in treating the physical effects

of the disease, while (E) mentions that leper colonies are

no longer needed, showing that the social stigma has

vanished. The other answer choices contain either minor

details or incorrect information.

Part 3>> Set 1 _ p.618

1. Fact Question – (B)

The paragraph states that it is ultimately a star’s mass (i.e.

its total amount of matter) that determines all aspects of its

life and death.

2. Vocabulary Question – (C)

Main sequence stars are stars that are able to create

enough heat and pressure to resist the compressive force

of gravity. This heat and pressure is created through

fusion. Thus, as long as a star has fuel for fusion, it will

remain in the main sequence.

3. Referent Question – (D)

The context of the paragraph should clearly indicate that

(D) is the correct answer. The pronoun there is possessive

and is connected to the term “cores.” Thus, the correct

antecedent for “their” must be something that has a core.

This eliminates answer choices (B) and (C).

4. Fact Question – (B)

The paragraph clearly states that “the compressive force

of gravity is counter-balanced by the outward pressure

and heat created by fusion.”

Page 139: Insider: The Super Guide Scripts & Answer Key

Scripts & Answer Key A139

5. Rhetorical Purpose Question – (A)

The sun is used as part of a larger comparison between

large stars and small stars. The main point of this

comparison is to illustrate how the mass of a star affects

its life span.

6. Vocabulary Question – (B)

The main focus of the paragraph provides an important

context clue for the meaning of this term. The main focus

of the paragraph is what happens when a star begins to

run out of fuel. Logically, “dwindle” must mean the same

thing as “lessen.”

7. Fact Question – (D)

Paragraph 3 clearly states that when stars run out of

hydrogen (their primary fuel) they will attempt to fuse

heavier elements (i.e. use a secondary fuel source). As

long as a star is able to fuse heavier elements, it will remain

in the main sequence and its death will be delayed.

8. Negative Fact Question – (C)

Answer choice (C) is clearly contradicted by the main idea

of the passage. The mass of a star influences every part of

a star’s life, including how long it remains in the main

sequence. Therefore, all main sequence stars cannot have

equivalent life spans.

9. Vocabulary Question – (B)

The sentence provides an important context clue in stating

that when a state of “equilibrium” is achieved, the force of

gravity is matched by the repulsive force between

electrons.

10. Referent Question – (D)

The antecedent for “they” must be something that is visible

to astronomers. That eliminates (A) as a logical answer

choice. Furthermore, both “they” and “neutron stars”

operate as subjects of different clauses within the

sentence, making (D) the most likely answer choice.

11. Inference Question – (C)

Paragraph 4 clearly states that any star with a mass that is

less than 1.5 times that of the sun will end its life as a

white dwarf. Therefore, the sun is logically destined to

become a white dwarf at the end of its life.

12. Sentence Insertion Question – The sentence should be

placed at the second square.

The insertion sentence contains several context clues that

indicate where it should be placed. First, it contains the

pro-form “these stars.” Therefore it needs to be placed

after a sentence that discusses some type of star. Second,

it states that these stars “test the very laws of physics.”

This logically matches up with the idea that neutron stars

are bizarre.

13. Categorization Question

(A) Result from the deaths of smaller stars (White

dwarfs)

(B) Emit little visible light (Neutron stars)

(C) Take billions of years to cool (White dwarfs)

(D) Collapse stopped by the repulsion of sub-atomic

particles (BOTH)

(E) Experience a more extensive collapse (Neutron

stars)

(F) Fate of every main sequence star (X)

(G) Invisible to astronomers (X)

The two incorrect answer choices should be fairly easy to

identify; the passage clearly states that not every main

sequence star shares the same fate and that both neutron

stars and white dwarfs are visible to astronomers. Both

electrons and neutrons are types of sub-atomic particles,

so the collapse of both neutron stars and white dwarfs is

stopped by the repulsive force between sub-atomic

particles. The remaining answer choices should be

obvious.

>> Set 2 _ p.622

14. Fact Question – (D)

The last sentence states that “The persistence of these

myths showed how for centuries Americans selectively

examined evidence and distorted science in order to

support their own agendas against the natives.” In other

words, the earth mounds were misunderstood.

15. Inference Question – (C)

The answer choice paraphrases important information and

examples within the paragraph, specifically how the

theories weren’t so much genuine studies as they were

projections of certain beliefs. The following sentence

indicates this: “Moreover, particular details of different

mound-builder theories reflected the specific prejudices of

the people who supported them.”

16. Vocabulary Question – (D)

Look for contextual clues within the sentence. It states that

“many Americans accepted the mound-builder theories as

facts, despite the dubious evidence that supported these

theories.” “Despite” indicates that the evidence didn’t

provide strong support for the theories, so it was doubtful.

17. Vocabulary Question – (A)

This vocabulary question requires you to understand the

key idea of the passage, which is that Americans

automatically believed that the natives couldn’t have built

the mounds. This was a common assumption, or

presupposition, of the theories.

18. Sentence Simplification Question – (B)

The key idea of the highlighted sentence is the contrast of

the natives’ supposed ignorance with the theorists’ real

ignorance, which is proven by their inability to recognize

native language. The correct answer choice contains these

ideas and correctly connects them together. The other

choices either omit or distort information.

19. Referent Question – (A)

Pra

ctic

e T

es

t

Page 140: Insider: The Super Guide Scripts & Answer Key

A140 Insider: The Super Guide

Follow the logic of this paragraph’s argument. The settlers

believed that their conquests were revenge for imagined

conquest of the mound-builders. Thus, when the Americans

“were essentially avenging these vanished peoples,” they

thought that they were avenging the mound-builders.

20. Rhetorical Purpose Question – (D)

The correct answer choice paraphrases a key idea of the

paragraph, which is how some Americans used the

theories for their own purposes, which included gaining

native territory. This is strongly hinted at when the author

writes that “these theories also served the Americans’

agenda of seizing native lands.”

21. Referent Question – (C)

Read the sentence carefully. The key word that hints to the

referent is “inscribed,” as in “All of these were inscribed

with different languages.” Naturally, only tablets would be

inscribed with a language.

22. Fact Question – (B)

The answer is directly stated, so skim for the sentence that

contains this information. It describes how the hoaxes

contained “different languages, such as Mayan, Chinese,

Hebrew, and Egyptian, and this supposedly proved that

non-native civilizations built these mounds.”

23. Fact Question – (C)

Because the keywords are highlighted, simply skim the

paragraph for the sentence that directly states this

information. It reads: “Also, early evidence includes

accounts by Spanish and French explorers who stayed

with various native tribes and learned of the construction of

many mounds.”

24. Negative Fact Question – (B)

This is a large negative fact question that addresses many

key ideas that are discussed throughout the entire

passage. Thus, it is important to keep track of the main

ideas from the passage. These include inherent racism of

the theories, their wild speculation, and their use of

unreliable evidence. Only choice (B) is a minor detail, and it

is actually contradicted by information in the last

paragraph.

25. Sentence Insertion Question – The sentence should be

placed at the second square.

The topic of the insertion sentence is an example of how

some groups use mound-builder theories to support racist

beliefs. This should naturally follow the claim that some

groups actually do have racist beliefs and thus use

mound-builder theories. This claim is made at the very end

of paragraph 4.

26. Prose Summary Question – (A), (B), (E)

Use the introductory sentence as a guide for selecting the

correct answer choices. The main focus of the introductory

sentence is the popularity and prevalence of mound-

builder theories. The correct choices all address this in

some way: (A) mentions how they have always been

opposed even when popular, (B) explains that they were

popular in spite of the lack of evidence, and (E) indicates

that popular beliefs were supported and justified by the

theories. The other choices are either minor details or

incorrect.

Page 141: Insider: The Super Guide Scripts & Answer Key

Scripts & Answer Key A141

Part 1(Questions 1-5) _ p.628

1. Main Idea Question – (C)

When the student first comes to the help desk he does not

yet know that he needs RAM for his computer. He only

knows that his Internet connection is not working properly.

2. Supporting Detail Question – (B)

The student says that he has already been to the facilities

department, but that they told him there was a problem

with his computer, not with the Internet connection.

Therefore, they told him to visit the computer help desk.

3. Supporting Detail Question – (B)

The help desk worker says that she can’t be sure, but that

the most likely problem is that the student does not have

enough memory. According to the woman, the fact that

the student’s computer is slow in general supports this.

4. Content-Linking Question(Prediction) – (B)

The student will most likely go to the computer store that

the woman has told him about to get more RAM installed

in his computer.

5. Function Purpose Question – (B)

The woman is trying to indicate that while she thinks she

knows what the problem is, she is not 100% sure. When

the student first comes to the help desk he does not yet

know that he needs RAM for his computer. He only knows

that his Internet connection is not working properly.

QQuueessttiioonn �� –– RReeppllaayy

f WWoommaann:: OK� that could be due to any number ofthings� Usually … yeah� right� OK� it looks like youdon’t have that much RAM on your computer� Itprobably runs kind of slow in general� right?

f WWoommaann:: OK� that could be due to any number of things�

Student: Is it something I can get done today? Like I

said, I got a big project to do.

Woman: There’s a computer shop right across from the

campus bookstore. They usually do pretty quick work,

and adding RAM is pretty simple. If they can’t do it today,

just buy the RAM, bring it back here, and I’ll put it in for

you.

Student: OK. I really appreciate it.

[ f Script ]

Student (male): Excuse me. Is this the computer help

desk?

Woman: Yup. What can we do for you?

Student: I’m living in Granger Hall, and I’m having

problems with my Internet.

Woman: OK, actually the Facilities department is in

charge of the Internet lines in the dorms, so you’ll need

to go see them to get the line fixed.

Student: I just came from there, they said there’s nothing

wrong with the line, and that it had to be a problem with

the settings on my computer, so I should come and talk

to you guys.

Woman: Oh. OK. Uh … we don’t have anyone who can

go out to the dorm and take a look at your computer right

now, so can I get your name and room number, and we’ll

send someone out tomorrow?

Student: Actually, I have a notebook, so I just brought it

with me. I was kind of hoping that you could help me

today. I got a big project to do tonight.

Woman: Sure, if you got it with you, let’s take a look.

(slight pause) So when you hook up your computer you

don’t get any connection at all?

Student: No, I get a connection, but the dorms are

supposed to have high speed Internet, and it takes

forever for sites to come up.

Woman: OK, that could be due to any number of things.

Usually … yeah, right. OK, it looks like you don’t have

that much RAM on your computer. It probably runs kind

of slow in general, right?

Student: Uh, yeah, kind of.

Woman: Well, that’s the first thing you could do is

upgrade your RAM.

Student: I don’t see how that affects my Internet

connection.

Woman: It doesn’t, but if your computer can’t handle all

the information coming in over the high speed

connection, then it’s going to take a while for pages to

come up. Drop some more RAM in your computer and

that shouldn’t be a problem.

Student: Will that be expensive?

Woman: Not really. Maybe 60 or 80 bucks.

iBT Practice Test - Listening

Pra

ctic

e T

es

t

Page 142: Insider: The Super Guide Scripts & Answer Key

A142 Insider: The Super Guide

(Questions 6-11) _ p.630

6. Main Idea Question – (D)

Very little is said about advertising itself. The main point of

this lecture is to define a number of ways to approach

budgeting for advertising.

QQuueessttiioonn ���� –– RReeppllaayy

f PPrrooffeessssoorr:: These kinds of things can be measured� Imean� you can always make customer surveys orsomething� So� what I’m saying is that ��� youradvertising is not so much tied to the budget� but to�um� other factors that may eventually lead to increasedsales� But again� this is advertising and there are neverany guarantees�

f PPrrooffeessssoorr:: But again� this is advertising� and there arenever any guarantees�

The third way to set an advertising budget is to have the

advertising goal-oriented. Let’s say you want to increase

sales, you know, by a certain amount. Well, you could

increase advertising until this goal is achieved. Of

course, there’s the chance that it may not be achieved,

but that’s the chance you always have to take in

advertising. But other goals may not be simply financial.

For example, you might want to increase the general

recognition of your product or ... you may want to reach a

certain number of viewers, listeners, or readers, you

know, potential clients. These kinds of things can be

measured. I mean, you can always make customer

surveys or something. So, what I’m saying is that ... your

advertising is not so much tied to the budget, but to, um,

other factors that may eventually lead to increased sales.

But again, this is advertising, and there are never any

guarantees.

Well, there is a fourth method of setting an advertising

budget, and even though it is probably the worst way, I

have to say that it’s the way used by too many

companies. This is what is referred to as the, um,

residual approach. Now, “residual” means what’s left

over. In the case of such an advertising budget

approach, a company decides on the advertising budget

after it finishes with all other expenditures. Advertising

gets whatever is left over. In other words, there is no

connection between advertising and any marketing

principles. Whether advertising succeeds or fails is left

totally up to chance. During good times it may not matter

much, but during bad times a low advertising budget

could contribute to the collapse of your company.

Okay, so these are ways a company can budget for

advertising, but it doesn’t address the results of one

study that found that 97% of all advertising is wasted.

We’ll talk about how to more effectively reach customers

in the next lecture.

[ f Script ]

Professor (male): A famous quote about, um, advertising

goes like this: “I know that half of my advertising budget

is wasted, but I’m not sure which half.” (laughing) So ...

what does that mean? Well, basically it says that it is

very difficult to measure the effect of advertising on

sales, you know. Companies know it’s important but

have a hard time knowing exactly how to set an

advertising budget ... that makes sense. The reason for

this is that advertising is not the only variable that affects

sales. Other factors also come into play. These are such

things as consumer and business confidence, levels of,

um, disposable income, availability of product, availability

of, you know, competing products, and, oddly enough,

the weather.

Many companies often decide on their advertising

budgets, um, based on the percentage of sales. But this

also varies according to the product being sold.

Pharmaceutical companies generally allot around 20% of

sales to advertising, carmakers about 1%, and other

consumer goods manufacturers average somewhere in

the neighborhood of 8 to 10%.

Now, there are four basic ways to set an advertising

budget. The first way is ... through a fixed budget or an

advertising budget that does not vary from year to year.

No matter what the sales are, the budget stays at a fixed

percentage. This can be good because the budget can

be predictable and ... um, a company may not need to

worry about overspending on advertising. I mean,

occasionally, in any field, there are sometimes

advertising wars that break out, and companies have

been destroyed by them, you know. A fixed budget

would stop a company from getting involved in such a

war. On the negative side, however, it fails to take into

account the effect ... advertising has on sales. I mean, if

advertising is strongly influencing sales, then increasing

the percentage spent on it would seem to make sense.

But this would be prevented in a fixed budget system.

Another way to set an advertising budget is, um,

competitor-based advertising. The idea here is to match

the percentage of the competitors in your industry. For

example, if you’re a pharmaceutical company ... and you

know all the other companies are spending 20% on

advertising, then you do the same. The rationale here is

that if it’s working for them, then it will probably work for

you. See what I mean? Anyway, the problem, as I see it

anyway, is that it’s kind of a lazy way to manage ... your

advertising budget. With a little research, um, you may

find that increasing or decreasing the budget may be

better for your particular products.

Page 143: Insider: The Super Guide Scripts & Answer Key

Scripts & Answer Key A143

7. Organization Question – (A)

The professor gives some specific statistics as to what

percentage of sales is allocated to the advertising budgets

of certain types of companies. He specifically mentions

pharmaceutical and automobile companies.

8. Supporting Detail Question – (B)

As the name suggests, goal-oriented budgets depend on

reaching some target or attaining a specific outcome. This

could be either financial or non-financial, such as gaining

product recognition. This would not be directly connected

to an increase in sales, but might eventually do so. For this

reason, the first choice is not really correct. It suggests that

increased sales are the only goal in such a budget.

9. Supporting Detail Question – (C)

The professor refers to such budgets as “lazy” in that they

don’t require much in the way of market or advertising

research.

10. Supporting Detail Question – (D)

The professor directly states that this is the worst method

for planning a budget. It takes into account no marketing

principles at all. If it succeeds, it does so by pure luck,

since there is really no plan behind it.

11. Function Purpose Question – (A)

When someone says “there are no guarantees,” they mean

that the outcome of something is not predictable.

Advertising is not a science. What should work may

sometimes not. This does not mean that most advertising

is ineffective or that marketing principles do not apply; it

simply highlights advertising’s unpredictability.

(Questions 12-17) _ p.633

future of any state and any town depended on how close

it was to the railroad. California began to put a lot of

pressure on the government to help get a railroad to

California, but, well, let me ask you, what could

encourage the railroad company to build a railroad to

California? I mean, it would take a lot of money and a lot

of work to do this, and what would they get out of it?

Student B (male): Yeah, I think that in cases like this the

government had to give them some money or something

to encourage them to do it.

Student A: Maybe they could share the cost or

something.

Professor: Well, I guess I should tell you that the

Congress authorized building the transcontinental

railroad in 1862. And what else was going on in 1862?

Student B: The Civil War was starting.

Professor: That’s right, and the US government was in

no position to give money away when they had a war to

fight. So what they did was loan the railroad companies

millions of dollars ...

Student A: (interrupting) But what good was that? This is

only a loan. The companies will have to pay it back. I

can’t see that as much of an incentive.

Professor: That’s true, and if all they got was a loan, I

doubt if the railway would have been built. But the

government also gave them millions of acres of western

land which they probably figured wasn’t worth much

anyway. But the railways thought that if they could sell

this land, they could use the money to repay these loans.

Any idea on how they could sell land that was in the

middle of nowhere?

Student B: Well, if you want my honest opinion I’d say

they’d have to trick people into buying it.

Student A: I don’t know. If they gave it at a good price to

people who normally couldn’t afford land. And you’ve got

to remember, it was on the railroad line.

Professor: Well, I don’t know if they tricked people, but

they did what any business does that wants to sell

something. They did a good job of salesmanship. They

advertised throughout the East and even in Europe.

They offered free transportation to the land. And they

also offered low interest, long term loans.

Student B: Did it work?

Professor: Yes and no. The main people who took the

land were farmers. They grew crops, all right, but they

had to pay the railroads to ship them to market. Often

they grew one crop, wheat, which depended on

fluctuations on the international wheat market, something

small farmers had never experienced before. Sometimes

[ f Script ]

Professor (male): So, when gold was discovered in

California in 1849, there was a great deal of interest that

developed in going west to get rich. Now, imagine you’re

a young man living in the East and you have this idea

that you can get rich by finding gold in California. Well,

your first problem is getting there. So tell me, how would

you get to California in 1849?

Student A (female): I think they had to go across the

country by wagon train, didn’t they?

Professor: Yeah, well, that was one of the ways. The

other way was to go by sea, and since there was no

Panama Canal, it meant going around all of South

America. Any way you looked at it, getting to California

was not an easy thing to do. But many did it and, in

doing so, shifted a significant number of people to

California. In fact, enough people lived in California by

1850 to grant it statehood.

Now, at this same time, the railroad had been developing

throughout the East. It was apparent to everyone that the

Pra

ctic

e T

es

t

Page 144: Insider: The Super Guide Scripts & Answer Key

A144 Insider: The Super Guide

12. Main Idea Question – (D)

The California Gold Rush was mentioned, but only

because it began to stimulate the need for a

transcontinental railroad. A complete picture of life in

railway towns was not given. We only know that they were

farmers who were trying to survive under the power of the

railroads. The expansion of the railroads may be a main

topic, but it seems too broad, as only western expansion is

discussed. In addition, it does not take into account the

politics and social changes that surrounded this

expansion. Therefore, answer D best summarizes the

entire scope of the discussion.

13. Organization Question – (A)

The professor gives an example of a town which may be

dependent on one crop such as wheat and how its

relationship with and dependence on the railway could

develop. The farmers may have depended on wheat and

the price of wheat, but it is not really related to how they

depended on the railway. The two were linked, but the

type of dependence was not compared by the professor.

No particular town was named, and no real statistics were

given.

14. Content-Linking Question – (C)

Although a student suggests the railroads may have been

dishonest, the professor doesn’t really agree. He basically

states that they made use of some clever advertising and

attractive incentives that led people to buy the land. In

other words, they were clever businessmen. They could

not force anyone to initially buy the land. That would have

been their choice.

15. Supporting Detail Question – (B)

The professor explains that the timing was bad for the

expansion of the railroad. The Civil War had just begun,

and the government needed to be more careful about how

it spent its money, since it seemed likely that much money

would be needed to fight this war.

16. Supporting Detail Question – (D)

If the town grew only wheat and the price of wheat was

high, the town would do well. It, perhaps, could be argued

that dependence on one crop was a bad farming practice,

but, in fact, it was closer to a bad business practice.

However, it was only bad in the event of low prices for that

one crop. In truth, all of this could be overridden by too

much dependence on the railroads. Since the railroads

loaned most of these farmers the money to pay for the

land, the inability to repay the loan during a bad year could

lead to the entire town being abandoned, thus forming a

ghost town.

17. Function Purpose Question – (D)

The expression “to not take something lying down” means

to refuse being pushed around. In this case, the farmers

refused to give in to the power of the railroad and even

organized to fight it. To emphasize this attitude in the

farmers, the professor uses this expression.

QQuueessttiioonn ���� –– RReeppllaayy

f PPrrooffeessssoorr:: Maybe� but they didn’t take this lying down�They eventually banded together to fight the railroadsand eventually succeeded in having laws passed thatcurbed their power�

f PPrrooffeessssoorr: Maybe� but they didn’t take this lying down�

groups of farmers lived near the same railway stop,

forming small towns. But if the price of wheat, for

example, was low, the whole town could lose money.

And it was not unusual for these farmers to go more and

more into debt to the railroads. Sometimes bankruptcy

caused whole towns to disappear and form ghost towns.

Student A: So what you’re saying is that the railroads

completely controlled the fate of towns in the West. How

did the government let this happen?

Professor: I think they just didn’t see how much power

they were giving to the railroads when they first passed

the law. Yeah, you’re right. The railroad could raise their

prices at will. They could cut off service. They could drive

a town out of existence.

Student B: But in some way it was the farmers’ fault for

not seeing how dependent they would be on the railroad.

Professor: Maybe, but they didn’t take this lying down.

They eventually banded together to fight the railroads

and eventually succeeded in having laws passed that

curbed their power.

Page 145: Insider: The Super Guide Scripts & Answer Key

Scripts & Answer Key A145

Part 2(Questions 1-5) _ p.637

1. Main Idea Question – (D)

Although the student begins with an explanation of why he

was absent last week, his real intention in visiting his

professor is to get the work that he missed last week.

2. Supporting Detail Question – (A)

The professor says that regardless of any letter that was

sent out, the student should have personally informed her

that he was going to be absent.

3. Organization Question – (C)

The easiest part is getting the notes, because there are

posted on the Internet. Giving the student the homework

from last week will also be relatively simple, and the

professor says she will do that in the next class. The most

difficult part will be getting ready for the exam next week

because the student does not have much time to get

caught up.

4. Supporting Detail Question – (C)

At first, the student is thinking about just skipping the

assignments so that he can spend more time getting ready

for the exam. But the professor suggests that he do them

after the exam. That way, he will at least get partial credit

for the assignments.

5. Function Purpose Question – (B)

The expression “to cut someone some slack” is commonly

used in conversational English to mean “to make an

allowance for someone.” In this case, the student wants

the professor to make an allowance for him concerning the

deadlines of the assignments.

QQuueessttiioonn �� –– RReeppllaayy

f SSttuuddeenntt:: OK … but doing last week’s work� this week’swork� and studying for the test seems like a lot� Isn’tthere any way you can cut me a little slack?

f SSttuuddeenntt:: Isn’t there any way you can cut me a little slack?

your work, so a couple missed assignments wouldn’t be

the end of the world. But I got a better idea. Why don’t

you just wait until after the exam, and then do those

assignments. You know that I accept late work. I just

deduct points for the lateness. That would be a lot better

than just not doing the assignments at all.

Student: Yeah, I guess that’s a good idea. Thanks.

[ f Script ]

Student (male): Hi, Professor Wallace, got a second?

Professor (female): Oh, Nick. Hi. We missed you last

week in class, were you sick?

Student: No, It was nothing like that. My anthropology

class went on a research trip to study Appalachian

communities. Uh … there was a memo sent out by the

Anthropology department.

Professor: Well, it must have missed me because this is

the first I’ve heard of it. You know in the future, you

should really notify your professors personally,

regardless of any memos being sent out by the

department.

Student: I know. I’m sorry. I was just so busy getting

ready for the trip and all …

Professor: So, I guess you’re here about makeup work,

right?

Student: Uh, yeah.

Professor: Well, I don’t have any of it prepared, because

this is the first I’ve heard about this. Uh, let’s see … for

the lectures you missed you can just get my lecture

notes off the Internet.

Student: Yeah, I already did that.

Professor: OK, good. Regarding the homework

assignments … I guess I can have them ready for you by

Thursday, and then I’ll give you a week to complete them

since you were out for a week. How’s that sound?

Student: That’s fair enough.

Professor: Now, the tricky one is the exam you have

next week. You really need to spend time going over the

notes from last week’s lectures to prepare for that. And

I’m reluctant to give you an extension on the exam

because then I have to write a whole new exam for you.

Student: OK … but doing last week’s work, this week’s

work, and studying for the test seems like a lot. Isn’t

there any way you can cut me a little slack?

Professor: I don’t know. It’s not like this was a sudden

thing. I mean you knew about this trip, right? The

responsible thing to do would have been to come and

see me before. Then you wouldn’t be in this situation.

Student: Yeah, I guess I dug my own grave. How much

is the homework from last week worth? If I don’t make it

up, is that going to really kill my grade?

Professor: Not really. You’ve been fairly consistent in

Pra

ctic

e T

es

t

Page 146: Insider: The Super Guide Scripts & Answer Key

A146 Insider: The Super Guide

(Questions 6-11) _ p.639

6. Main Idea Question – (C)

A and B are too narrow to be the main point of the lecture,

and D is never mentioned in the lecture.

7. Organization Question – (B)

The author lists several toxic substances, such as

asbestos and depleted uranium, that are found in modern

battlefields.

8. Supporting Detail Question – (C)

The professor clearly states that the illnesses of Vietnam

veterans turned out to be from exposure to a herbicide

that was used in Vietnam, and specifically from a chemical

byproduct (DDT) that was in the herbicide.

QQuueessttiioonn ���� –– RReeppllaayy

f PPrrooffeessssoorr:: A few years after the Vietnam War� veteransstarted to report a whole slew of medical problems�Uh� increased rates of cancer� kidney and liverproblems� higher rates of birth defects in their children… and� at first� everyone was stumped�

f PPrrooffeessssoorr:: … and at first everyone was stumped�

difficult to pinpoint the exact causes of their illnesses.

Before we get into that, however, let’s detail some of the

illnesses and problems of what is commonly referred to

as Gulf War Syndrome. Uh, Gulf War Syndrome, or

GWS, is really an amalgamation of various symptoms,

and while most sufferers exhibit more than one symptom,

they rarely exhibit all of them. Obviously, that has

complicated the job of diagnosis. Common symptoms

include chronic skin problems, severe headaches,

memory loss, shortened attention span, muscle pain, and

chronic fatigue. Rarer, but more serious problems include

increased rates of birth defects, cancer, and degenerative

nervous disorders. Uh, the prevalence of GWS, by the

way, is far more widespread in Gulf War veterans than

herbicide related illnesses were in Vietnam vets.

Now, there’s some controversy over exactly what the

cause of GWS is. Uh, my guess is that it is probably

caused by exposure to a number of toxins, not just one.

Here are some of the likely candidates. Depleted

uranium ammunition was used heavily during the Gulf

War, and we have already discussed its potential

hazards. In addition, Iraq had significant stocks of

chemical weapons, uh, nerve gas in particular. Now they

didn’t use those weapons directly against US troops. But

US military leaders were worried about that possibility so

they bombed the storage areas where those chemical

weapons were kept. Some of those chemicals probably

made in into the air after the bombing, and so US troops

may have been exposed to low, non-lethal levels of

nerve agents. Such exposure is known to increase the

risk of cancer and birth defects.

[ f Script ]

Professor (female): OK, today, we’re going to talk about

a kind of unique area in medicine, the uh, illnesses that

have befallen veterans of some modern wars. The basic

premise that we’re working on is that a modern war zone

is absolutely awash in dangerous and toxic substances.

Uh, let me just give you a couple quick examples. Let’s

say you bomb a building, and it burns. Well, buildings are

full of plastic and other substances that are toxic when

burned, so the smoke from the burning building is

poisonous. Uh, plus lots of modern buildings contain

asbestos, a substance that can cause cancer if it gets

into our lungs. If the building is bombed, then, obviously,

asbestos dust is going to be released into the air. Uh …

another quick example would be the use of depleted

uranium ammunition by many modern armies. Depleted

uranium is a form of uranium that is no longer radioactive.

It’s used for ammunition because it is extremely dense

and therefore really good at punching through armor.

Unfortunately, upon impact, depleted uranium rounds

burn and release toxic particles into the air.

So, you get the point, in a war zone there’s all kinds of

toxins floating around in the air, from where they can

enter the bodies of soldiers with relative ease. Now, a lot

of these toxins have delayed affects; that is, the damage

they cause doesn’t show up for months or even years.

The point is that a number of years after a modern war,

you’ll often start having veterans of that war start popping

up with unexplained illnesses. So, then the challenge is

to not only figure out what caused the illness, but also

figure out if there is any way to treat it. Let’s look at two

modern American wars as examples: the Vietnam War

and the 1991 Gulf War against Iraq.

We’ll start with the Vietnam War because it’s the simpler

of the two examples. A few years after the Vietnam War,

veterans started to report a whole slew of medical

problems. Uh, increased rates of cancer, kidney and liver

problems, higher rates of birth defects in their children …

and, at first, everyone was stumped. Well, it turns out

that a herbicide, uh, that’s a chemical that kills plants,

that the army sprayed over large parts of Vietnam during

the war contained a chemical called dioxin. Now, dioxins

are some of the most toxic chemicals known to man, and

they cause many of the same illnesses that Vietnam

veterans were experiencing. Uh, I should also note that

the people of Vietnam were exposed to the same toxins.

So, in that case, it was relatively easy to connect the dots

and realize that this herbicide was probably the cause.

But the next example is far more complex, uh, because

as you’ll see in a minute, soldiers in the Gulf War were

exposed to all sorts of nasty substances. That makes it

Page 147: Insider: The Super Guide Scripts & Answer Key

Scripts & Answer Key A147

9. Supporting Detail Question – (B)

The professor says that there were many toxic substances

that soldiers may have been exposed to during the Gulf

War, and that as a result, it is harder to diagnose and treat.

10. Content-Linking Question – (B)

The professor mentions that the people of Vietnam were

exposed to the same herbicide as Vietnam veterans. It is

therefore logical to assume that they would experience the

same medical problems.

11. Function Meaning Question – (A)

To say that someone is “stumped” means that person

does not know the answer to something.

(Questions 12-17) _ p.641

QQuueessttiioonn ���� –– RReeppllaayy

f PPrrooffeessssoorr:: Looking back on our colonial history� we’llsee that as early as the ���s� there were lawsprescribing mandatory education of some sort forchildren� But they were more like legal theory thanlaws actually put into practice�

Student B (male): So what changed?

Professor: Uh, well ... a number of things. actually. Uh,

the first was that the nation grew more prosperous. At

the founding of the nation, or before, during the colonial

period, the government simply would not have had the

resources to provide free, public education to every child.

You also got to remember that population densities were

pretty low in many places, you know people were spread

out all over the place on farms, in the territories, etc. So,

gathering a sufficient number of children together in one

place to justify the opening of a school was sometimes a

difficult undertaking in itself. That’s also reflected in the

fact that many of the private schools were boarding

schools. The students lived at the schools, often quite far

from their homes. That simply would not have been a

viable option for public schools. So, as the industrial

revolution progressed, and as towns and cities grew,

populations became more centralized and public

education became … well, more “doable.”

Student A: I guess the industrial revolution and the

advance of technology would have made a formal

education more important also, right?

Professor: Sure, that’s a good point. Certainly, there was

a shift in public opinion about the necessity of education.

But up until this point, we have only discussed the social

factors that helped bring about this change. We also

need to talk about the impact of one man in particular.

Horace Mann was the secretary of education for the

state of Massachusetts. He was a free thinker, an

abolitionist, and he was deeply committed to the idea of

the equality of man. He led the charge in establishing

schools for all children, uh, then called common schools,

in his state. In 1837, he helped push through a state law

that established publicly funded grammar schools for

students aged 8-14. His efforts had kind of a cascade

effect, and by 1918 every state in the union had

established public schools to provide compulsory

education to students. Now, that only included primary

school. The growth of what we think of as the American

high school is another story altogether … but that’s going

to have to wait for another day.

[ f Script ]

Professor (male): You know these days, we just take it

for granted that children are going to go to school. I

mean, nearly every nation you could name … uh, even

the poorest ones, have some form of compulsory

education. But this idea is a relatively new one. In fact,

the right to education wasn’t even listed as a basic

human right until 1918. Even if you take what we view to

be a modern, democratic country like the United States,

you’ll see that widespread educational opportunities for

children are a pretty recent thing.

Looking back on our colonial history, we’ll see that as

early as the 1640s, there were laws prescribing

mandatory education of some sort for children. But they

were more like legal theory than laws actually put into

practice. What’s more, the schools that did exist were

almost entirely private schools, and so large portions of

the populace who couldn’t afford tuition were denied any

real educational opportunities.

Student A (female): Wait a minute, professor?

Professor: Uh, yeah, Nancy, you had a question?

Student A: Well, not really a question. I’m just having a

hard time buying that kids got no education back then.

Professor: Well, education in a trade, and what we

consider to be a formal, academic education are different

things. Let’s say you had a kid living on a farm. Well, that

kid would learn the skills he needed to be farmer from his

father, and his parents would probably teach him the

basics of math, uh, maybe a little reading and writing, at

home. But that kid wouldn’t have ever sat down in a

classroom to learn about say… literature, or, uh, world

history. It just wouldn’t have been considered important.

To be sure, some children did receive an academic

education. But they were mostly from upper class

families, and received their educations from private tutors

and in private schools.

Pra

ctic

e T

es

t

Page 148: Insider: The Super Guide Scripts & Answer Key

A148 Insider: The Super Guide

12. Main Idea Question – (B)

The discussion focuses on how public schools developed

in the America. A is too broad, B is too narrow, and D is

not mentioned.

13. Organization Question��� A lack of government funding� A sudden drop in population� The success and availability of private schools in

educating American youth�� The public’s perception of the unimportance of formal

education

The professor says that the early government simply

couldn’t afford public schools, and that since most people

worked on farms, an academic education wasn’t really

seen as being important.

14. Supporting Detail Question – (B)

The professor states that Horace Mann created the first

public schools, called “common schools,” in

Massachusetts.

15. Content-Linking Question(Prediction) – (B)

The professor says that the story of the American high

school will have to wait for another day, suggesting this will

be discussed in the future.

16. Content-Linking Question – (D)

The professor says that laws were more like legal theory,

suggesting that they were not enforced.

17. Function Meaning Question – (C)

In conversational English, if someone does not “buy”

something, it often means they do not believe it.

QQuueessttiioonn ���� –– RReeppllaayy

f SSttuuddeenntt AA:: Wait a minute� professor?

PPrrooffeessssoorr:: Uh� yeah� Nancy� you had a question?

SSttuuddeenntt AA:: Well� not really a question� I’m just having ahard time buying that kids got no education back then�

f SSttuuddeenntt AA:: I’m just having a hard time buying that kidsgot no education back then�

Page 149: Insider: The Super Guide Scripts & Answer Key

Scripts & Answer Key A149

Task 1 _ p.646

[ Sample Response ]

AIDS is a major health problem that greatly affects humans

worldwide. The disease is so problematic because it is a

highly communicative disease that infects without

discrimination and has no cure. No matter where you live

and regardless of your race or sexual orientation, if you

make contact with the bodily fluids of any individual infected

with this immune disease, you put yourself at risk of also

acquiring the fatal illness. Additionally, AIDS kills millions

worldwide, and the rate of infection has steadily increased

since the start of the epidemic in the early 1980s. In sum,

because AIDS is a highly infectious disease without a cure, it

poses a serious risk to human health globally.

Task 2 _ p.647

[ Sample Response ]

Personally, I’d rather spend my weekends taking short trips

than relaxing at home. I think this is a better option because

it gives me a change of scenery and because the weekends

are really my only chance to travel. Everyone needs a

change of environment sometimes. After a long week of

school and work, sometimes I just need to get away for a

while. By taking a short trip, I can leave all my pressures

behind, even if it is just for a day or two. In addition, since I

go to school during the semester and work during the

semester breaks, I don’t really have time to take a true

vacation. The weekends are really my only time to travel,

and I wouldn’t want to waste them just sitting at home.

Task 3 _ p.648

[ Sample Response ]

The man does not approve of the new policy which says

transfer students can’t have on-campus jobs in their first

semester. His girlfriend was going to transfer to his school,

but he says she probably won’t do that now, since she can’t

work on campus. Although students can work off campus, the

man points out that most of the jobs off campus aren’t really

suitable for students. The man also says that he thinks the

university is kind of going overboard. He says that he didn’t

think his first semester was particularly difficult when he

transferred to the school, and that the university is making a

big deal out of something that isn’t really a problem.

Task 4 _ p.649

[ f Script ]

Professor (female): Most of you already know that in a

beehive, all the bees have a single mother, the queen.

The queen mates with a drone, a male bee whose only

purpose is to fertilize the queen’s eggs. The queen lays

her eggs, which will eventually hatch and become worker

bees. Now, worker bees are female also, and therefore

are able to produce eggs. As long as the queen is alive,

however, the worker bees will not lay their eggs. If the

queen dies, however, it threatens the entire beehive,

because all reproduction stops. In this situation, the

workers will begin to lay eggs. The problem with this is

that workers are unable to mate with drones, meaning

their eggs are unfertilized. These unfertilized eggs will

hatch, but they will only produce more drones, not

workers. Basically, this is can be seen as a last attempt

to save the colony. If a new queen can be found, then

the hive might survive. If no new queen is found,

however, the hive will eventually die, since the worker

are waiting tables until 11 at night? The

campus jobs are really the only practical ones

for serious students.

Woman: I guess that’s true. But the university only has

the best interests of the students at heart.

They just want new students to start off on the

right foot, you know?

Man : Maybe, but I think they are making a big deal

out of nothing. I was a transfer student, and I

worked during my first semester here. It wasn’t

particularly stressful. I mean really, what do

you actually have to adjust to? You’re a little

busy for the first two weeks after you transfer

here, and then it’s like any other semester.

Heck, I even made the Dean’s List my first

semester here. I really don’t see what the

university is so worried about.

[ f Script ]

Woman: Hey, Mike. Isn’t your girlfriend transferring herenext semester?

Man : Well, she was. But now she’s not so sure. Woman: Why’s that? Man : Because of that stupid rule they just made

about transfer students not being able to workon campus. She was planning to get a campusjob to help her pay her tuition. Without that,she probably can’t afford to go to school here.

Woman: But she could still get a job off campus, right? Man : Oh, please. You and I both know that’s a

complete joke. The only off campus jobs are atthe local bars and restaurants, and the hoursyou have to work at those places are ridiculous. I mean, how are you supposed to study if you

iBT Practice Test - Speaking

Pra

ctic

e T

es

t

Page 150: Insider: The Super Guide Scripts & Answer Key

A150 Insider: The Super Guide

[ Sample Response ]

In the lecture, the professor explains how worker bees can

reproduce asexually if the queen dies. According to the

professor, the worker bees will only lay eggs if the queen

dies and the survival of the hive is in danger. In this

situation, the workers will lay eggs, but the eggs are

unfertilized because the workers can’t mate with the drones.

Therefore, this is a form of asexual reproduction. According

to the professor, the worker’s eggs will only produce drones,

so if a new queen isn’t found, the hive will eventually die.

This shows that asexual reproduction in bees is not as good

as their regular reproductive method.

Task 5 _ p.650

[ Sample Response ]

The man’s problem is that he isn’t doing well in his biology

class, but the woman doesn’t have time to tutor him. She

suggests that he should either attend a study skills workshop

at the student center or that he should ask his professor to

recommend a different student to tutor him. I think he should

try attending the study skills workshop first. If his problem is

his study skills, as the woman suggests, then attending the

workshop would help him not only in his biology class, but in

his other classes as well. Besides, the professor might not

know a student who is willing to tutor him. The woman

already said that she was too busy to tutor him, and it seems

likely that other students may be just as busy as she is.

Task 6 _ p.651

[ Sample Response ]

In the lecture, the professor talks about the decline in fish

populations in the Chesapeake and the reasons for the

decline. According to the professor, two of the major reasons

for this decline are the growth of chicken farms and the

[ f Script ]

Professor (male): Since the colonial era, the

Chesapeake Bay in eastern Maryland has been one of

the most important fisheries on the American east coast.

Oysters, clams, crabs, and numerous species of fish are

all found in abundance in the Chesapeake, or at least

they were in previous times. In recent decades, many of

the aquatic species found in the Chesapeake have

undergone serious declines in population. While the

reasons for these declines are complex and multi-

faceted, we can point to a few key factors.

One of those factors is the growth of chicken farms along

the shores of the Chesapeake. Over the last few

decades, chicken farming has become a major economic

activity in eastern Maryland, and huge chicken farms,

some with tens of thousands of hens, have sprung up.

Now, the waste from these chicken farms is extremely

high in nitrogen. When this nitrogen-rich waste gets

washed into the Chesapeake, it nourishes bacteria that

feed on nitrogen. Many of these bacteria are toxic to fish

and shellfish, and they kill off large numbers of fish.

Another factor in the decline of aquatic species in the

Chesapeake is the growth of hydrilla. Hydrilla is a type of

aquatic plant … kind of like seaweed. This plant has

spread throughout the Chesapeake. Hydrilla grows in

thick, dense patches that form a kind of underwater

jungle, and it decrease the levels of oxygen in the water.

The combination of the thick underwater growth and lack

of oxygen drives fish away.

[ f Script ]

Student A (male) : Hey, Lisa, do you have a minute?

Student B (female): Sure, but just a minute … I have

biology next.

Student A: Actually, that’s what I wanted to talk to you

about. I was kind of wondering if you could

tutor me. I’m not failing my biology class or

anything like that, but my test scores are

pretty low. At this rate, I’ll probably pass the

class, but just barely, and it’ll drag down my

GPA. So I was hoping you could help me out

a bit.

Student B: Gee, Greg … I’d love to help you out, but I’m

pretty busy myself this semester. I just don’t

think I really have the time to tutor you. What

exactly are you having problems with in the

class?

Student A: Mostly just the tests. I do OK in the labs, and

I always do the assignments, but there’s so

much to remember for the tests.

Student B: Uh-huh. Well, it sounds like maybe you need

to work on your study skills, you know …

work on finding a way to help you remember

the information. One thing you could do is go

down to the student services center. I know

they hold weekly study skills workshops. You

could try attending one of those and see if it

helps.

Student A: Yeah, I guess that’s a thought.

Student B: Or if you really think you need a personal

tutor, you could go talk to your professor. He

might be able to recommend a tutor for you.

Sorry I couldn’t be more help, but I’ve really

got to run.

bees will eventually die, and only male drones, who are

incapable of any form of reproduction, will replace them.

Page 151: Insider: The Super Guide Scripts & Answer Key

Scripts & Answer Key A151

spread of hydrilla. The professor points out that huge

chicken farms have been set up along the shores of the

Chesapeake. The waste from these chicken farms contains

nitrogen, which feeds bacteria that are toxic to fish. The

spread of an aquatic plant called hydrilla has also

contributed to the decline of fish. The professor states that

hydrilla grows in thick patches and decreases the levels of

oxygen in the water. According to the professor, this drives

fish away.

Pra

ctic

e T

es

t

Page 152: Insider: The Super Guide Scripts & Answer Key

A152 Insider: The Super Guide

Task 1 _ p.654

[ Sample Response ]

In the lecture, the professor discusses the reasons why

some astronomers think that Pluto should not be classified

as a planet. These points directly contradict the information

in the reading, which says that it is correct to classify Pluto

as a planet.

First, the professor points out that, while Pluto does orbit the

sun, it lies in the Kuiper belt, which makes it different from

other planets. The professor also notes that other objects

that have been found in the Kuiper belt have not been

classified as planets, even though they are similar to Pluto.

This directly contradicts the claim in the reading that Pluto

should be considered a planet just because it orbits the sun.

The professor also says that while Pluto’s gravity is strong

enough to pull it into the shape of a sphere, this is does not

necessarily classify it as a planet. According to the

professor, Pluto is mostly made up of different kinds of ice,

and should therefore be considered a ball of ice rather than

a planet.

Finally, the professor says that the fact that Pluto has moons

does not necessarily make it a planet. While the reading

says that the presence of moons is a key characteristic of

planets, the professor notes that some asteroids have

moons as well. In fact, the professor says that Pluto might

actually be such an asteroid.

In conclusion, the professor’s lecture largely contradicts the

main points of the reading. While Pluto does orbit the sun, is

spherical in shape, and has orbiting moons, none of these

characteristics justifies its classification as a planet.

Task 2 _ p.658

[ Sample Response ]

Although the number of years a person has served in a

company is important, most promotions should be based on

merit rather than seniority. This is a better policy because it

promotes higher levels of productivity, makes hard working

employees feel more appreciated, and results in a higher

quality workforce over time.

First, basing promotions on merit will lead to higher levels of

productivity. Obviously, if employees know that promotions

are based on merit, they will work as hard as possible. This

is true because they will know that only the hardest working

employees will be promoted. If promotions automatically go

to those who have worked in the company the longest, on

the other hand, there is no incentive to work hard because

one’s level of work will not affect one’s chances of

promotion.

Another important point is that hard working employees will

feel more appreciated if merit-based promotions are used

because their efforts will be recognized and rewarded. This

is important because employees who feel they are

appreciated are more likely to stay with a company over

time. If hard-working employees feel like they are not getting

the recognition they deserve, they may very well move to a

different company.

Finally, basing promotions on merit rather than seniority

eventually leads to a higher quality workforce. This is

because only those who are willing to work hard have any

incentive to remain in the company. In a company where

[ f Script ]

Professor (female): We all grow up learning that thereare nine planets in the Solar System, but manyastronomers want textbooks to be rewritten to indicatethat there are only eight planets. Specifically, they wantto demote Pluto from its status as a planet. Yes, likeother planets, Pluto orbits around the sun and iscompressed into a sphere-like shape by its gravity andhas moons. However, there are significant reasons toquestion its classification as a planet.

First of all, while Pluto does indeed orbit the sun,astronomers have recently discovered that it actually liesin the Kuiper belt, or a section of the solar systemextending from Neptune’s orbit to the sun. This discoveryindicates that Pluto is different than all other planetaryobjects and should be reclassified as a Kuiper belt objector even a former moon of Neptune’s that somehow brokefree of the planet’s orbit. Several other planet-like objectswith similar characteristics as Pluto have been found inNeptune’s orbit and subsequently categorized as Trans-Neptunian or Kuiper belt objects rather than planets.

Secondly, although Pluto’s gravitational pull is strongenough to compress it into a ball, the ball it iscompressed into is made almost entirely of ice – 98%nitrogen ice to be exact. The speculated structure ofPluto is that it is composed of one layer of frozennitrogen, a second layer of water ice, and a third layer ofsilicate and water ice. This further supports the claim thatPluto may be a ball of ice that has been mistakenlyclassified as a planet.

Finally, although Pluto has three moons in its orbit, thisshould not necessarily qualify it as a planet. There aremany asteroid moons, or asteroids that orbit otherasteroids as their natural satellite. It is estimated thatabout 2% of all asteroids have satellite bodies, and Plutomay very well be an asteroid with three other asteroidslocked into its orbit and functioning as asteroid moons.

iBT Practice Test - Writing

Page 153: Insider: The Super Guide Scripts & Answer Key

Scripts & Answer Key A153

seniority is used to decide promotions, even somewhat lazy

employees have a reason to stay with the company because

if they stay long enough, they are likely to be promoted.

However, in a company where all promotions are based on

merit, these employees really have no reason to stay.

In conclusion, deciding promotions based on merit is really

the only logical choice for a company. Not only does it

reward the people who work the hardest, but it also makes

sure that people do not simply sit back and “put in their time”

at the company.

Pra

ctic

e T

es

t